MHE1, AAHE2, AAHE3, AAHE4, AAHE5, AAHE6

Lakukan tugas rumah & ujian kamu dengan baik sekarang menggunakan Quizwiz!

The client diagnosed with schizophrenia spectrum disorder is prescribed an antipsychotic. Which client symptoms, related to the side effects of this medication, should prompt a nurse to intervene immediately? 1. Sore throat and malaise 2. Light-colored urine and bradycardia 3. Anosognosia and avolition 4. Dry mouth and urinary retention

1 The nurse should intervene immediately if the client experiences signs of an infectious process—such as a sore throat, fever, and malaise—when taking antipsychotic drugs.

The client is prescribed alprazolam (Xanax) for acute anxiety. Which client finding should cause a nurse to question this order? 1. History of alcohol use disorder 2. History of personality disorder 3. History of schizophrenia 4. History of hypertension

1 The nurse should question a prescription of alprazolam for acute anxiety if the client has a history of alcohol use disorder. Alprazolam is a benzodiazepine used in the treatment of anxiety and has an increased risk for physiological dependence and tolerance. A client with a history of substance use disorder may be more likely to abuse other addictive substances.

Which action would the nurse take to establish a trusting relationship with a client diagnosed with schizophrenia spectrum disorder? 1. Establish personal contact with family members 2. Be reliable, honest, and consistent during interactions 3. Share limited personal information 4. Sit close to the client to establish rapport

2 The nurse can enhance the establishment of a trusting relationship with a client diagnosed with schizophrenia spectrum disorder by being reliable, honest, and consistent during interactions. The nurse should also convey acceptance of the client's needs and maintain a calm attitude.

The client reports taking an herbal remedy to lose weight. The client states, "I'm taking the recommended dose, but it seems like if two capsules are good, four would be better!" Which response would the nurse make? 1. "Herbal medicines are more likely to cause adverse reactions than prescription medications." 2. "Increasing the amount of herbal medicines can lead to overdose and toxicity." 3. "There are no regulations in place for herbal medicines, so ingredients are often unknown." 4. "Certain companies are better than others. Always buy a reputable brand of herbal medicine."

2 The nurse should advise the client that increasing the amount of herbal preparations can lead to overdose and toxicity. The notion that something designated as being "natural" means that it is completely safe is a myth.

When planning care for a client diagnosed with borderline personality disorder, which self-harm behavior should a nurse expect the client to exhibit? 1. Highly lethal methods to commit suicide 2. Suicidal gestures to elicit a rescue response from others 3. Isolation and starvation as suicidal methods 4. Self-mutilation from decreased endorphins in the body

2 The nurse should expect that a client diagnosed with borderline personality disorder may use suicidal gestures to elicit a rescue response from others. Repetitive, self-mutilating behaviors are common in borderline personality disorders that result from feelings of abandonment following separation from significant others.

The nurse in the emergency department (ED) is assessing a client who with a long history of depression. The nurse finds that the client has gained weight, has dry skin, and has cold sensitivity. The nurse determines the client's depression is exacerbating, further examination and testing reveal the client has hypothyroidism. Which phenomenon occurred? 1. Depression screening 2. Social distancing 3. Trauma-informed caring 4. Diagnostic overshadowing

2 The nurse should refer the client to a psychiatrist. Psychiatrists provide diagnostic, medication management, and counseling services to clients.

During her uncle's wake, a 5-year-old girl runs up to the casket before her mother can stop her. An appointment is made with a nurse practitioner when the child starts twisting and pulling out her hair, resulting in hair loss. Which nursing diagnosis should the nurse assign to this child? 1. Fear 2. Altered family processes 3. Ineffective impulse control 4. Disturbed body image

3 The child is suffering from trichotillomania, which leads to ineffective impulse control. This child is coping with the anxiety generated by viewing her deceased uncle by pulling out her hair.

The nurse is working in an emergency department. With which client should the nurse use the screening, brief intervention, and referral to treatment approach (SBIRT)? 1. Has suicidal thoughts 2. Has nonsuicidal self-injuring behavior 3. Has an opioid addiction 4. Has been raped

3 The client with an opioid addiction should be screened and cared for using the SBIRT approach. The SBIRT approach is an evidence-based approach that can be used in various settings for substance abuse and addiction.

What is the first step the nurse should take to reduce stigma of mental health clients? 1. Increase social contact with mental health clients 2. Attend on-the-job training about mental health clients 3. Have a willingness to interact with mental health clients 4. Understand the person as a mental health client

3 The first step is to have a willingness to interact with mental health clients. The nurse should be willing to engage in meaningful relationships with people who have mental illnesses and addictions.

Which guideline should the nurse use to help differentiate a client diagnosed with panic disorder from a client diagnosed with generalized anxiety disorder (GAD)? 1. GAD is acute in nature, and panic disorder is chronic. 2. Chest pain is a common GAD symptom, whereas this symptom is absent in panic disorders. 3. Depression is a common symptom in GAD and rare in panic disorder. 4. Depersonalization is absent in GAD but is commonly seen in panic disorder.

4 The nurse should recognize that a client diagnosed with panic disorder experiences depersonalization, whereas a client diagnosed with GAD would not. Depersonalization refers to being detached from oneself when experiencing extreme anxiety.

The nurse is teaching about the etiology of illness anxiety disorder (IAD) from a psychodynamic perspective. Which statement by a staff member about clients diagnosed with this disorder indicates that learning has occurred? 1. "When there is a familial predisposition to this disorder, they may develop this disorder." 2. "When the sick role relieves them from stressful situations, their physical symptoms are reinforced." 3. "They misinterpret and cognitively distort their physical symptoms." 4. "They express personal worthlessness through physical symptoms, because physical problems are more acceptable than psychological problems."

4 The nurse should understand that from a psychodynamic perspective, IAD occurs because physical problems are more acceptable than psychological problems.

Which medications would the nurse most likely administer to a client who has a history of opiate withdrawal? 1. Haloperidol (Haldol) and acamprosate (Campral) 2. Naloxone (Narcan) and naltrexone (ReVia) 3. Disulfiram (Antabuse) and lorazepam (Ativan) 4. Methadone (Dolophine) and clonidine (Catapres)

4 The nurse would administer methadone and clonidine for a client who has a history of opiate withdrawal. As the dose of methadone diminishes, renewed abstinence symptoms may be ameliorated by the addition of clonidine.

An involuntarily committed client purposely pushes a dinner tray off the bedside table onto the floor. Which nursing intervention would a nurse implement to address this behavior? 1. Initiate forced medication protocol. 2. Help the client to explore the source of anger. 3. Ignore the act to avoid reinforcing the behavior. 4. With staff support, set firm limits on the behavior.

4 The most appropriate nursing intervention is to set firm limits on the behavior.

A patient has been diagnosed with achalasia based on his history and diagnostic imaging results. The nurse should identify what risk diagnosis when planning the patients care? A) Risk for Aspiration Related to Inhalation of Gastric Contents B) Risk for Imbalanced Nutrition: Less than Body Requirements Related to Impaired Absorption C) Risk for Decreased Cardiac Output Related to Vasovagal Response D) Risk for Impaired Verbal Communication Related to Oral Trauma

A Feedback: Achalasia can result in the aspiration of gastric contents. It is not normally an acute risk to the patients nutritional status and does not affect cardiac output or communication.

A nurse is caring for a patient in the emergent/resuscitative phase of burn injury. During this phase, the nurse should monitor for evidence of what alteration in laboratory values? A) Sodium deficit B) Decreased prothrombin time (PT) C) Potassium deficit D) Decreased hematocrit

A Feedback: Anticipated fluid and electrolyte changes that occur during the emergent/resuscitative phase of burn injury include sodium deficit, potassium excess, base-bicarbonate deficit, and elevated hematocrit. PT does not typically decrease.

An older adult patient has been diagnosed with aortic regurgitation. What change in blood flow should the nurse expect to see on this patients echocardiogram? A) Blood to flow back from the aorta to the left ventricle B) Obstruction of blood flow from the left ventricle C) Blood to flow back from the left atrium to the left ventricle D) Obstruction of blood from the left atrium to left ventricle

A Feedback: Aortic regurgitation occurs when the aortic valve does not completely close, and blood flows back to the left ventricle from the aorta during diastole. Aortic regurgitation does not cause obstruction of blood flow from the left ventricle, blood to flow back from the left atrium to the left ventricle, or obstruction of blood from the left atrium to left ventricle.

A patient is admitted to the burn unit after being transported from a facility 1000 miles away. The patient has burns to the groin area and circumferential burns to both upper thighs. When assessing the patients legs distal to the wound site, the nurse should be cognizant of the risk of what complication? A) Ischemia B) Referred pain C) Cellulitis D) Venous thromboembolism (VTE)

A Feedback: As edema increases, pressure on small blood vessels and nerves in the distal extremities causes an obstruction of blood flow and consequent ischemia. This complication is similar to compartment syndrome. Referred pain, cellulitis, and VTE are not noted complications that occur distal to the injury site.

A patient with a complex cardiac history is scheduled for transthoracic echocardiography. What should the nurse teach the patient in anticipation of this diagnostic procedure? A) The test is noninvasive, and nothing will be inserted into the patients body. B) The patients pain will be managed aggressively during the procedure. C) The test will provide a detailed profile of the hearts electrical activity. D) The patient will remain on bed rest for 1 to 2 hours after the test.

A Feedback: Before transthoracic echocardiography, the nurse informs the patient about the test, explaining that it is painless. The test does not evaluate electrophysiology and bed rest is unnecessary after the procedure.

A cardiac surgery patients new onset of signs and symptoms is suggestive of cardiac tamponade. As a member of the interdisciplinary team, what is the nurses most appropriate action? A) Prepare to assist with pericardiocentesis. B) Reposition the patient into a prone position. C) Administer a dose of metoprolol. D) Administer a bolus of normal saline.

A Feedback: Cardiac tamponade requires immediate pericardiocentesis. Beta-blockers and fluid boluses will not relieve the pressure on the heart and prone positioning would likely exacerbate symptoms.

A patient is being treated in the ED following a terrorist attack. The patient is experiencing visual disturbances, nausea, vomiting, and behavioral changes. The nurse suspects this patient has been exposed to what chemical agent? A) Nerve agent B) Pulmonary agent C) Vesicant D) Blood agent

A Feedback: Nerve agent exposure results in visual disturbances, nausea and vomiting, forgetfulness, irritability, and impaired judgment. This presentation is not suggestive of vesicants, pulmonary agents, or blood agents.

The school nurse is giving a presentation on preventing spinal cord injuries (SCI). What should the nurse identify as prominent risk factors for SCI? Select all that apply. A) Young age B) Frequent travel C) African American race D) Male gender E) Alcohol or drug use

A, D, E Feedback: The predominant risk factors for SCI include young age, male gender, and alcohol and drug use. Ethnicity and travel are not risk factors.

The nurse assesses a client with major depressive disorder. Which assessment finding would the nurse observe? 1. Sadness subsides quickly 2. Promiscuous behaviors 3. Unable to feel any pleasure 4. Excessive spending sprees

ANS: 3

A patient with emphysema is experiencing shortness of breath. To relieve this patients symptoms, the nurse should assist her into what position? A) Sitting upright, leaning forward slightly B) Low Fowlers, with the neck slightly hyperextended C) Prone D) Trendelenburg

Ans: A Feedback: The typical posture of a person with COPD is to lean forward and use the accessory muscles of respiration to breathe. Low Fowlers positioning would be less likely to aid oxygenation. Prone or Trendelenburg positioning would exacerbate shortness of breath.

A perioperative nurse is caring for a postoperative patient. The patient has a shallow respiratory pattern and is reluctant to cough or to begin mobilizing. The nurse should address the patients increased risk for what complication? A) Acute respiratory distress syndrome (ARDS) B) Atelectasis C) Aspiration D) Pulmonary embolism

Ans: B Feedback: A shallow, monotonous respiratory pattern coupled with immobility places the patient at an increased risk of developing atelectasis. These specific factors are less likely to result in pulmonary embolism or aspiration. ARDS involves an exaggerated inflammatory response and does not normally result from factors such as immobility and shallow breathing.

An older adult patient is undergoing diagnostic testing for chronic lymphocytic leukemia (CLL). What assessment finding is certain to be present if the patient has CLL? A) Increased numbers of blast cells B) Increased lymphocyte levels C) Intractable bone pain D) Thrombocytopenia with no evidence of bleeding

B Feedback: An increased lymphocyte count (lymphocytosis) is always present in patients with CLL. Each of the other listed symptoms may or may not be present, and none is definitive for CLL.

A patient diagnosed with acute myelogenous leukemia has just been admitted to the oncology unit. When writing this patients care plan, what potential complication should the nurse address? A) Pancreatitis B) Hemorrhage C) Arteritis D) Liver dysfunction

B Feedback: Pancreatitis, arteritis, and liver dysfunction are generally not complications of leukemia. However, the patient faces a high risk of hemorrhage.

In preparation for cardiac surgery, a patient was taught about measures to prevent venous thromboembolism. What statement indicates that the patient clearly understood this education? A) Ill try to stay in bed for the first few days to allow myself to heal. B) Ill make sure that I dont cross my legs when Im resting in bed. C) Ill keep pillows under my knees to help my blood circulate better. D) Ill put on those compression stockings if I get pain in my calves.

B Feedback: To prevent venous thromboembolism, patients should avoid crossing the legs. Activity is generally begun as soon as possible and pillows should not be placed under the popliteal space. Compression stockings are often used to prevent venous thromboembolism, but they would not be applied when symptoms emerge.

An industrial site has experienced a radiation leak and workers who have been potentially affected are en route to the hospital. To minimize the risks of contaminating the hospital, managers should perform what action? A) Place all potential victims on reverse isolation. B) Establish a triage outside the hospital. C) Have hospital staff put on personal protective equipment. D) Place hospital staff on abbreviated shifts of no more than 4 hours.

B Feedback: Triage outside the hospital is the most effective means of preventing contamination of the facility itself. None of the other listed actions has the potential to prevent the contamination of the hospital itsel

A 58-year-old male patient has been hospitalized for a wedge resection of the left lower lung lobe after a routine chest x-ray shows carcinoma. The patient is anxious and asks if he can smoke. Which statement by the nurse would be most therapeutic? A) Smoking is the reason you are here. B) The doctor left orders for you not to smoke. C) You are anxious about the surgery. Do you see smoking as helping? D) Smoking is OK right now, but after your surgery it is contraindicated.

C

The nurse is assessing a new patient with complaints of overwhelming fatigue and a sore tongue that is visibly smooth and beefy red. This patient is demonstrating signs and symptoms associated with what form of what hematologic disorder? A) Sickle cell anemia B) Hemophilia C) Megaloblastic anemia D) Thrombocytopenia

C Feedback: A red, smooth, sore tongue is a symptom associated with megaloblastic anemia. Sickle cell disease, hemophilia, and thrombocytopenia do not have symptoms involving the tongue.

During the examination of an unconscious patient, the nurse observes that the patients pupils are fixed and dilated. What is the most plausible clinical significance of the nurses finding? A) It suggests onset of metabolic problems. B) It indicates paralysis on the right side of the body. C) It indicates paralysis of cranial nerve X. D) It indicates an injury at the midbrain level.

D Feedback: Pupils that are fixed and dilated indicate injury at the midbrain level. This finding is not suggestive of unilateral paralysis, metabolic deficits, or damage to CN X.

A cardiac care nurse is aware of factors that result in positive chronotropy. These factors would affect a patients cardiac function in what way? A) Exacerbating an existing dysrhythmia B) Initiating a new dysrhythmia C) Resolving ventricular tachycardia D) Increasing the heart rate

D Feedback: Stimulation of the sympathetic system increases heart rate. This phenomenon is known as positive chronotropy. It does not influence dysrhythmias.

A patient on corticosteroid therapy needs to be taught that a course of corticosteroids of 2 weeks duration can suppress the adrenal cortex for how long? A) Up to 4 weeks B) Up to 3 months C) Up to 9 months D) Up to 1 year

D Feedback: Suppression of the adrenal cortex may persist up to 1 year after a course of corticosteroids of only 2 weeks duration.

A nurse is caring for a patient who has been the victim of sexual assault. The nurse documents that the patient appears to be in a state of shock, verbalizing fear, guilt, and humiliation. What phase of rape trauma syndrome is this patient most likely experiencing? A) Reorganization phase B) Denial phase C) Heightened anxiety phase D) Acute disorganization phase

D Feedback: The acute disorganization phase may manifest as an expressed state in which shock, disbelief, fear, guilt, humiliation, anger, and other such emotions are encountered. These varied responses to the assault are not associated with a denial, heightened anxiety, or reorganization phase.

A patient admitted with nephrotic syndrome is being cared for on the medical unit. When writing this patients care plan, based on the major clinical manifestation of nephrotic syndrome, what nursing diagnosis should the nurse include? A) Constipation related to immobility B) Risk for injury related to altered thought processes C) Hyperthermia related to the inflammatory process D) Excess fluid volume related to generalized edema

D Feedback: The major clinical manifestation of nephrotic syndrome is edema, so the appropriate nursing diagnosis is Excess fluid volume related to generalized edema. Edema is usually soft, pitting, and commonly occurs around the eyes, in dependent areas, and in the abdomen.

A patient returns to the floor after a laparoscopic cholecystectomy. The nurse should assess the patient for signs and symptoms of what serious potential complication of this surgery? A) Diabetic coma B) Decubitus ulcer C) Wound evisceration D) Bile duct injury

D Feedback: The most serious complication after laparoscopic cholecystectomy is a bile duct injury. Patients do not face a risk of diabetic coma. A decubitus ulcer is unlikely because immobility is not expected. Evisceration is highly unlikely, due to the laparoscopic approACH

Following cardiac resuscitation, a patient has been placed in a state of mild hypothermia before being transferred to the cardiac intensive care unit. The nurses assessment reveals that the patient is experiencing neuromuscular paralysis. How should the nurse best respond? A) Administer hypertonic IV solution. B) Administer a bolus of warned normal saline. C) Reassess the patient in 15 minutes. D) Document this as an expected assessment finding.

D Feedback: The nurse caring for a patient with hypothermia (passive or induced) needs to monitor for appropriate level of cooling, sedation, and neuromuscular paralysis to prevent seizures; myoclonus; and shivering.

An obtunded patient is admitted to the ED after ingesting bleach. The nurse should prepare to assist with what intervention? A) Prompt administration of an antidote B) Gastric lavage C) Administration of activated charcoal D) Helping the patient drink large amounts of water

D Feedback: The patient who has ingested a corrosive poison, such as bleach, is given water or milk to drink for dilution. Gastric lavage is not used to treat ingestion of corrosives and activated charcoal is ineffective. There is no antidote for a corrosive substance such as bleach.

The nurse is assessing a patient who is known to have right-sided HF. What assessment finding is most consistent with this patients diagnosis? A) Pulmonary edema B) Distended neck veins C) Dry cough D) Orthopnea

B Feedback: Right-sided HF may manifest by distended neck veins, dependent edema, hepatomegaly, weight gain, ascites, anorexia, nausea, nocturia, and weakness. The other answers do not apply.

There has been a radiation-based terrorist attack and a patient is experiencing vomiting, diarrhea, and shock after the attack. How will the patients likelihood of survival be characterized? A) Probable B) Possible C) Improbable D) Extended

C Feedback: Patients who experience vomiting, diarrhea, and shock after radiation exposure are categorized as improbable survival, because they are demonstrating symptoms of exposure levels of more than 800 rads of total body-penetrating irradiation.

____________________ personality disorder is characterized by a lack of self-confidence and excessive need to be taken care of that leads to a passive and submissive role

Dependent

The client has extensive mental health problems that require medication and counseling. To which mental health care professional should the nurse refer the client?

Psychiatrist

Diagnostic testing has been ordered to differentiate between normal anion gap acidosis and high anion gap acidosis in an acutely ill patient. What health problem typically precedes normal anion gap acidosis? A) Metastases B) Excessive potassium intake C) Water intoxication D) Excessive administration of chloride

d

A patients most recent diagnostic imaging has revealed that his lung cancer has metastasized to his bones and liver. What is the most likely mechanism by which the patients cancer cells spread? A) Hematologic spread B) Lymphatic circulation C) Invasion D) Angiogenesis

B

An oncology nurse educator is providing health education to a patient who has been diagnosed with skin cancer. The patients wife has asked about the differences between normal cells and cancer cells. What characteristic of a cancer cell should the educator cite? A) Malignant cells contain more fibronectin than normal body cells. B) Malignant cells contain proteins called tumor-specific antigens. C) Chromosomes contained in cancer cells are more durable and stable than those of normal cells. D) The nuclei of cancer cells are unusually large, but regularly shaped.

B

A patients blood work reveals a platelet level of 17,000/mm3 . When inspecting the patients integumentary system, what finding would be most consistent with this platelet level? A) Dermatitis B) Petechiae C) Urticaria D) Alopecia

B Feedback: When the platelet count drops to less than 20,000/mm3 , petechiae can appear. Low platelet levels do not normally result in dermatitis, urticaria (hives), or alopecia (hair loss).

A patient is admitted to the ED complaining of abdominal pain. Further assessment of the abdomen reveals signs of peritoneal irritation. What assessment findings would corroborate this diagnosis? Select all that apply. A) Ascites B) Rebound tenderness C) Changes in bowel sounds D) Muscular rigidity E) Copious diarrhea

B, C, D Feedback: Signs of peritoneal irritation include abdominal distention, involuntary guarding, tenderness, pain, muscular rigidity, or rebound tenderness along with changes in bowel sounds. Diarrhea and ascites are not signs of peritoneal irritation.

You are called to your patients room by a family member who voices concern about the patients status. On assessment, you find the patient tachypnic, lethargic, weak, and exhibiting a diminished cognitive ability. You also find 3+ pitting edema. What electrolyte imbalance is the most plausible cause of this patients signs and symptoms? A) Hypocalcemia B) Hyponatremia C) Hyperchloremia D) Hypophosphatemia

C

The nurse is performing an initial assessment of an older adult resident who has just relocated to the long-term care facility. During the nurses interview with the patient, she admits that she drinks around 20 ounces of vodka every evening. What types of cancer does this put her at risk for? Select all that apply. A) Malignant melanoma B) Brain cancer C) Breast cancer D) Esophageal cancer E) Liver cancer

C D E

A patient with mitral stenosis exhibits new symptoms of a dysrhythmia. Based on the pathophysiology of this disease process, the nurse would expect the patient to exhibit what heart rhythm? A) Ventricular fibrillation (VF) B) Ventricular tachycardia (VT) C) Atrial fibrillation D) Sinus bradycardia

C Feedback: In patients with mitral valve stenosis, the pulse is weak and often irregular because of atrial fibrillation. Bradycardia, VF, and VT are not characteristic of this valvular disorder.

The nurse is providing a health education workshop to a group of adults focusing on cancer prevention. The nurse should emphasize what action in order to reduce participants risks of renal carcinoma? A) Avoiding heavy alcohol use B) Control of sodium intake C) Smoking cessation D) Adherence to recommended immunization schedules

C Feedback: Tobacco use is a significant risk factor for renal cancer, surpassing the significance of high alcohol and sodium intake. Immunizations do not address an individuals risk of renal cancer.

The nurse is preparing a staff education session about depression in adolescents. Which statement by a staff member indicates teaching has been effective? 1. "Adolescents are not likely to suffer from depression." 2. "Depressed adolescents normally seek immediate treatment." 3. "Many symptoms are attributed to normal adjustments of adolescents." 4. "Suicide is not common among depressed adolescents."

Correct Answer: 3 This statement would indicate effective teaching because many symptoms of depression may be attributed to normal adjustments of adolescents.

An oncology nurse is contributing to the care of a patient who has failed to respond appreciably to conventional cancer treatments. As a result, the care team is considering the possible use of biologic response modifiers (BRFs). The nurse should know that these achieve a therapeutic effect by what means? A) Promoting the synthesis and release of leukocytes B) Focusing the patients immune system exclusively on the tumor C) Potentiating the effects of chemotherapeutic agents and radiation therapy D) Altering the immunologic relationship between the tumor and the patient

D

An oncology patient has begun to experience skin reactions to radiation therapy, prompting the nurse to make the diagnosis Impaired Skin Integrity: erythematous reaction to radiation therapy. What intervention best addresses this nursing diagnosis? A) Apply an ice pack or heating pad PRN to relieve pain and pruritis B) Avoid skin contact with water whenever possible C) Apply phototherapy PRN D) Avoid rubbing or scratching the affected area

D

The ICU nurse is caring for a patient who experienced trauma in a workplace accident. The patient is complaining of having trouble breathing with abdominal pain. An ABG reveals the following results: pH 7.28, PaCO2 50 mm Hg, HCO3 23 mEq/L. The nurse should recognize the likelihood of what acidbase disorder? A) Respiratory acidosis B) Metabolic alkalosis C) Respiratory alkalosis D) Mixed acidbase disorder

D

A patient who has been exposed to anthrax is being treated in the local hospital. The nurse should prioritize what health assessments? A) Integumentary assessment B) Assessment for signs of hemorrhage C) Neurologic assessment D) Assessment of respiratory status

D Feedback: The second stage of anthrax infection by inhalation includes severe respiratory distress, including stridor, cyanosis, hypoxia, diaphoresis, hypotension, and shock. The first stage includes flu-like symptoms. The second stage of infection by inhalation does not include headache, vomiting, or syncope.

____________________ is a pervasive and sustained emotion that may have a major influence on a person's perception of the world.

Mood

The nurse in the medical unit is reviewing the laboratory test results for a client who has been transferred from the intensive care unit. The nurse notes that a cardiac troponin T level assay was performed while the client was in the intensive care unit. The nurse determines that this test was performed to assist in diagnosing which condition? 1.Heart failure 2.Atrial fibrillation 3.Myocardial infarction 4.Ventricular tachycardia

Myocardial infarction

______ who choose to work in nonpsychiatric settings are the frontline responders in recognizing, intervening, and referring clients with neuropsychiatric illness for further treatment.

NURSES

____________________ personality disorder is a pervasive mistrust and suspiciousness of others, such that their motives are interpreted as malevolent.

Paranoid

____________________ personality disorder is characterized by a profound defect in the ability to form personal relationships or to respond to others in any meaningful emotional way

Schizoid

Antianxiety drugs are also called ____________________ and minor tranquilizers

anxiolytics

A sudden event in one's life that disturbs homeostasis, during which usual coping mechanisms cannot resolve the problem, can be defined as a ______.

crisis

The nurse is caring for a client with cardiac disease who has been placed on a cardiac monitor. The nurse notes that the client has developed atrial fibrillation and has a rapid ventricular rate of 150 bpm. The nurse should next assess the client for which of the following? a. hypotension b. flat neck veins c. complaints of nausea d. complaints of headache

hypotension

The nurse is providing care for a patient with chronic obstructive pulmonary disease. When describing the process of respiration the nurse explains how oxygen and carbon dioxide are exchanged between the pulmonary capillaries and the alveoli. The nurse is describing what process? A) Diffusion B) Osmosis C) Active transport D) Filtration

A

Which nursing statements or questions represent appropriate communication to assess an individual in crisis? (Select all that apply.) 1. "Tell me what happened." 2. "Which coping methods have you used, and did they work?" 3. "Describe to me what your life was like before this happened." 4. "Let's focus on the current problem." 5. "I'll assist you in selecting functional coping strategies."

1, 2, 3 This is an appropriate statement to encourage the client to communicate. This statement enables the client to evaluate current coping strategies for effectiveness. This is an appropriate statement to encourage the client to communicate.

Which treatment would a nurse determine is appropriate for a client experiencing frequent migraine headaches? 1. St. John's wort combined with an antidepressant 2. Ginger tea combined with a beta-blocker 3. Feverfew, used according to directions 4. Kava-kava added to a regular diet

3 The nurse should determine that the appropriate treatment for a client experiencing frequent migraine headaches is the herb feverfew. Feverfew is effective in either fresh leaf or freeze-dried form. It is considered to be safe in reasonable doses.

A nurse in the CCU is caring for a patient with HF who has developed an intracardiac thrombus. This creates a high risk for what sequela? A) Stroke B) Myocardial infarction (MI) C) Hemorrhage D) Peripheral edema

A Feedback: Intracardiac thrombi can become lodged in the cerebral vasculature, causing stroke. There is no direct risk of MI, hemorrhage, or peripheral edema.

A nurse is planning discharge education for a patient who underwent a cervical diskectomy. What strategies would the nurse assess that would aid in planning discharge teaching?

A) Care of the cervical collar

A client has heparin-induced thrombocytopenia (HIT). The student nurse asks how this is treated. About what drugs does the nurse instructor teach? (Select all that apply.) a. Argatroban (Argatroban) b. Bivalirudin (Angiomax) c. Clopidogrel (Plavix) d. Lepirudin (Refludan) e. Methylprednisolone (Solu-Medrol)

Argatroban (Argatroban) b. Bivalirudin (Angiomax) d. Lepirudin (Refludan)

A patient is being treated for bites that she suffered during an assault. After the bites have been examined and documented by a forensic examiner, the nurse should perform what action? A) Apply a dressing saturated with chlorhexidine. B) Wash the bites with soap and water. C) Arrange for the patient to receive a hepatitis B vaccination. D) Assess the patients immunization history.

B After forensic evidence has been gathered, cleansing with soap and water is necessary, followed by the administration of antibiotics and tetanus toxoid as prescribed. The patients immunization history does not directly influence the course of treatment and hepatitis B vaccination is not indicated. Chlorhexidine bandages are not recommended.

An elderly woman found with a head injury on the floor of her home is subsequently admitted to the neurologic ICU. What is the best rationale for the following physician orders: elevate the HOB; keep the head in neutral alignment with no neck flexion or head rotation; avoid sharp hip flexion? A) To decrease cerebral arterial pressure B) To avoid impeding venous outflow C) To prevent flexion contractures D) To prevent aspiration of stomach contents

B Feedback: Any activity or position that impedes venous outflow from the head may contribute to increased volume inside the skull and possibly increase ICP. Cerebral arterial pressure will be affected by the balance between oxygen and carbon dioxide. Flexion contractures are not a priority at this time. Stomach contents could still be aspirated in this position.

The triage nurse in the ED is performing a rapid assessment of a man with complaints of severe chest pain and shortness of breath. The patient is diaphoretic, pale, and weak. When the patient collapses, what should the nurse do first? A) Check for a carotid pulse. B) Apply supplemental oxygen. C) Give two full breaths. D) Gently shake and shout, Are you OK?

D Feedback: Assessing responsiveness is the first step in basic life support. Opening the airway and checking for respirations should occur next. If breathing is absent, two breaths should be given, usually accompanied by supplementary oxygen. Circulation is checked by palpating the carotid artery.

A nurse is providing care for a patient whose neck dissection surgery involved the use of a graft. When assessing the graft, the nurse should prioritize data related to what nursing diagnosis? A) Risk for Disuse Syndrome B) Unilateral Neglect C) Risk for Trauma D) Ineffective Tissue Perfusion

D Feedback: Grafted skin is highly vulnerable to inadequate perfusion and subsequent ischemia and necrosis. This is a priority over chronic pain, which is unlikely to be a long-term challenge. Neglect and disuse are not risks related to the graft site.

The nurse is assessing an older adult patient with numerous health problems. What assessment datum indicates an increase in the patients risk for heart failure (HF)? A) The patient takes Lasix (furosemide) 20 mg/day. B) The patients potassium level is 4.7 mEq/L. C) The patient is an African American man. D) The patients age is greater than 65.

D Feedback: HF is the most common reason for hospitalization of people older than 65 years of age and is the second most common reason for visits to a physicians office. A potassium level of 4.7 mEq/L is within reference range and does not indicate an increased risk for HF. The fact that the patient takes Lasix 20 mg/day does not indicate an increased risk for HF, although this drug is often used in the treatment of HF. The patient being an African American man does not indicate an increased risk for HF.

A nurse is planning the care of a patient who has a diagnosis of hemophilia A. When addressing the nursing diagnosis of Acute Pain Related to Joint Hemorrhage, what principle should guide the nurses choice of interventions? A) Gabapentin (Neurontin) is effective because of the neuropathic nature of the patients pain. B) Opioids partially inhibit the patients synthesis of clotting factors. C) Opioids may cause vasodilation and exacerbate bleeding. D) NSAIDs are contraindicated due to the risk for bleeding.

D Feedback: NSAIDs may be contraindicated in patients with hemophilia due to the associated risk of bleeding. Opioids do not have a similar effect and they do not inhibit platelet synthesis. The pain associated with hemophilia is not neuropathic.

A nurse is caring for a client who has DIC. Which of the following medications should the nurse anticipate administering to the client? a. heparin b. vitamin K c. antibiotic d. antilipemic

a. Heparin may be administered to decrease the formation of micro clots, which deplete clotting factors. Vitamin K promotes blood coagulation and is not a medication that is prescribed for DIC.

A client with chronic anemia has had many blood transfusions. What medications does the nurse anticipate teaching the client about adding to the regimen? (Select all that apply.) a. Azacitidine (Vidaza)b. Darbepoetin alfa (Aranesp)c. Decitabine (Dacogen)d. Epoetin alfa (Epogen)e. Methylprednisolone (Solu-Medrol)

b, d

The nurse is caring for a client with a malignant brain tumor. Which medication should the nurse anticipate the HCP ordering?

✓ Phenytoiin (Dilatin), an anticonvulsant

Which client assessment finding would alert the nurse to question a diagnosis of brief psychotic disorder? 1. Has impaired reality testing for a 24-hour period. 2. Has auditory hallucinations for the past 3 hours. 3. Has bizarre behavior for 1 day. 4. Has confusion for 3 weeks.

2 This disorder is identified by the sudden onset of psychotic symptoms that may or may not be preceded by a severe psychosocial stressor. These symptoms last at least 1 day but less than 1 month and there is an eventual full return to the premorbid level of functioning.

A gerontologic nurse is teaching students about the high incidence and prevalence of dehydration in older adults. What factors contribute to this phenomenon? Select all that apply. A) Decreased kidney mass B) Increased conservation of sodium C) Increased total body water D) Decreased renal blood flow E) Decreased excretion of potassium

A D E

An asthma educator is teaching a patient newly diagnosed with asthma and her family about the use of a peak flow meter. The educator should teach the patient that a peak flow meter measures what value? A) Highest airflow during a forced inspiration B) Highest airflow during a forced expiration C) Airflow during a normal inspiration D) Airflow during a normal expiration

Ans: B Feedback: Peak flow meters measure the highest airflow during a forced expiration.

A patient is brought to the ED by ambulance after a motor vehicle accident in which the patient received blunt trauma to the chest. The patient is in acute respiratory failure, is intubated, and is transferred to the ICU. What parameters of care should the nurse monitor most closely? Select all that apply. A) Coping B) Level of consciousness C) Oral intake D) Arterial blood gases E) Vital signs

Ans: B, D, E Feedback: Patients are usually treated in the ICU. The nurse assesses the patients respiratory status by monitoring the level of responsiveness, ABGs, pulse oximetry, and vital signs. Oral intake and coping are not immediate priorities during the acute stage of treatment, but would become more important later during recovery

At 11:30 p.m. the client diagnosed with antisocial personality disorder demands to phone a lawyer to file for a divorce. Unit rules state that no phone calls are permitted after 10 p.m. Which nursing response is most appropriate? 1. "Go ahead and use the phone. I know this pending divorce is stressful." 2. "You know better than to break the rules. I'm surprised at you." 3. "It is after the 10 p.m. phone curfew. You will be able to call tomorrow." 4. "A divorce shouldn't be considered until you have had a good night's sleep."

3 The most appropriate response by the staff is to restate the unit rules in a calm, assertive manner. The nurse can encourage the client to verbalize frustration while maintaining an accepting attitude. The nurse may also help the client to identify the true source of frustration.

The client diagnosed with schizophrenia spectrum disorder tells a nurse about voices commanding him to kill his ex-spouse. Which nursing diagnosis is priority for this client? 1. Disturbed sensory perception 2. Altered thought processes 3. Risk for violence: directed toward others 4. Risk for injury

3 The nurse should prioritize the diagnosis risk for violence: directed toward others. A client who hears voices commanding him to kill someone is at risk for other-directed violence. Other risk factors for violence include aggressive body language, verbal aggression, catatonic excitement, and rage reactions.

Order the stages of Roberts' Seven-stage Crisis Intervention Model. (Enter the number of each step in the proper sequence, using comma and space format, such as: 1, 2, 3, 4.) 1. Deal with feelings and emotions. 2. Generate and explore alternatives. 3. Rapidly establish rapport. 4. Psychosocial and lethality assessment. 5. Identify the major problems or crisis precipitants. 6. Follow up. 7. Implement an action plan

4, 5, 2, 1, 3, 7, 6

While a patient is receiving IV doxorubicin hydrochloride for the treatment of cancer, the nurse observes swelling and pain at the IV site. The nurse should prioritize what action? A) Stopping the administration of the drug immediately B) Notifying the patients physician C) Continuing the infusion but decreasing the rate D) Applying a warm compress to the infusion site

A

You are the nurse caring for a 77-year-old male patient who has been involved in a motor vehicle accident. You and your colleague note that the patients labs indicate minimally elevated serum creatinine levels, which your colleague dismisses. What can this increase in creatinine indicate in older adults? A) Substantially reduced renal function B) Acute kidney injury C) Decreased cardiac output D) Alterations in ratio of body fluids to muscle mass

A

A patients low prothrombin time (PT) was attributed to a vitamin K deficiency and the patients PT normalized after administration of vitamin K. When performing discharge education in an effort to prevent recurrence, what should the nurse emphasize? A) The need for adequate nutrition B) The need to avoid NSAIDs C) The need for constant access to factor concentrate D) The need for meticulous hygiene

A Feedback: Vitamin K deficiency is often the result of a nutritional deficit. NSAIDs do not influence vitamin K synthesis and clotting factors are not necessary to treat or prevent a vitamin K deficiency. Hygiene is not related to the onset or prevention of vitamin K deficiency.

Which action should the nurse take when a depressed client refuses electroconvulsive therapy (ECT)? 1. Accept the client's decision 2. Inform the client that the procedure is mandatory 3. Tell the client that the signature verifies informed consent 4. Call the family to receive approval

ANS: 1

Which statement by a client who is beginning tricyclic antidepressant therapy indicates successful teaching? 1. "I will continue to take this medication even if the symptoms have not subsided." 2. "I will start to see results in about 2 weeks." 3. "I will continue to smoke." 4. "I will start to cut down on my alcohol intake and have only one glass of wine at supper."

ANS: 1

The nurse is caring for a patient whose recent unexplained weight loss and history of smoking have prompted diagnostic testing for cancer. What symptom is most closely associated with the early stages of laryngeal cancer? A) Hoarseness B) Dyspnea C) Dysphagia D) Frequent nosebleeds

Ans: A Feedback: Hoarseness is an early symptom of laryngeal cancer. Dyspnea, dysphagia, and lumps are later signs of laryngeal cancer. Alopecia is not associated with a diagnosis of laryngeal cancer.

The nurse is part of the health care team at an oncology center. A patient has been diagnosed with leukemia and the prognosis is poor, but the patient is not yet aware of the prognosis. How can the bad news best be conveyed to the patient? A) Family should be given the prognosis first. B) The prognosis should be delivered with the patient at eye level. C) The physician should deliver the news to the patient alone. D) The appointment should be scheduled at the end of the day.

B Feedback: Communicating about a life-threatening diagnosis should be done in a team setting at eye level with the patient. The family cannot be notified first because that would breech patient confidentiality. The family may be present at the patients request. The appointment should be scheduled when principles can all be in attendance and unrushed.

A nurse has taken on the care of a patient who had a coronary artery stent placed yesterday. When reviewing the patients daily medication administration record, the nurse should anticipate administering what drug? A) Ibuprofen B) Clopidogrel C) Dipyridamole D) Acetaminophen

B Feedback: Because of the risk of thrombus formation within the stent, the patient receives antiplatelet medications, usually aspirin and clopidogrel. Ibuprofen and acetaminophen are not antiplatelet drugs. Dipyridamole is not the drug of choice following stent placement.

A family member brings the patient to the clinic for a follow-up visit after a stroke. The family member asks the nurse what he can do to decrease his chance of having another stroke. What would be the nurses best answer? A) Have your heart checked regularly. B) Stop smoking as soon as possible. C) Get medication to bring down your sodium levels. D) Eat a nutritious diet.

B Feedback: Smoking is a modifiable and highly significant risk factor for stroke. The significance of smoking, and the potential benefits of quitting, exceed the roles of sodium, diet, and regular medical assessments.

A patient who had a hemiglossectomy earlier in the day is assessed postoperatively, revealing a patent airway, stable vital signs, and no bleeding or drainage from the operative site. The nurse notes the patient is alert. What is the patients priority need at this time? A) Emotional support from visitors and staff B) An effective means of communicating with the nurse C) Referral to a speech therapist D) Dietary teaching focused on consistency of food and frequency of feedings

B Feedback: Verbal communication may be impaired by radical surgery for oral cancer. It is therefore vital to assess the patients ability to communicate in writing before surgery. Emotional support and dietary teaching are critical aspects of the plan of care; however, the patients ability to communicate would be essential for both. Referral to a speech therapist will be required as part of the patients rehabilitation; however, it is not a priority at this particular time. Communication with the nurse is crucial for the delivery of safe and effective care.

The critical care nurse is caring for a patient who is in cardiogenic shock. What assessments must the nurse perform on this patient? Select all that apply. A) Platelet level B) Fluid status C) Cardiac rhythm D) Action of medications E) Sputum volume

B, C, D Feedback: The critical care nurse must carefully assess the patient in cardiogenic shock, observe the cardiac rhythm, monitor hemodynamic parameters, monitor fluid status, and adjust medications and therapies based on the assessment data. Platelet levels and sputum production are not major assessment parameters in a patient who is experiencing cardiogenic shock.

The school nurse is teaching a nutrition class in the local high school. One student states that he has heard that certain foods can increase the incidence of cancer. The nurse responds, Research has shown that certain foods indeed appear to increase the risk of cancer. Which of the following menu selections would be the best choice for potentially reducing the risks of cancer? A) Smoked salmon and green beans B) Pork chops and fried green tomatoes C) Baked apricot chicken and steamed broccoli D) Liver, onions, and steamed peas

C

You are caring for a patient with a secondary diagnosis of hypermagnesemia. What assessment finding would be most consistent with this diagnosis? A) Hypertension B) Kussmaul respirations C) Increased DTRs D) Shallow respirations

D

The nurse is teaching the staff about specific phobias. Which statement from a staff member indicates teaching has been effective? 1. "These clients recognize their fear as excessive and frequently seek treatment." 2. "These clients have a panic level of fear that is overwhelming and unreasonable." 3. "These clients experience symptoms that mirror a cerebrovascular accident." 4. "These clients exhibit symptoms of tachycardia, dysphagia, and diaphoresis."

2 This statement indicates successful teaching. Individuals with specific phobias have a panic level of fear that is overwhelming and unreasonable. Phobia is fear cued by a specific object or situation in which exposure to the stimuli produces an immediate anxiety response.

A high school student has learned that she cannot graduate. Her boyfriend will be attending a college out of state that she planned to attend. She is admitted to a psychiatric unit after overdosing on Tylenol. Which is the priority nursing diagnosis for this client? 1. Ineffective coping R/T situational crisis AEB powerlessness 2. Anxiety R/T fear of failure 3. Risk for self-directed violence R/T hopelessness 4. Risk for low self-esteem R/T loss events AEB suicidal ideations

3 The priority nursing diagnosis for this client is risk for self-directed violence R/T hopelessness. Nurses would prioritize diagnoses and outcomes based on potential safety risk to the client or others.

The nurse asks the client with schizophrenia spectrum disorder, "Do you receive special messages from certain sources, such as the television or radio?" The nurse is assessing which potential symptom of this disorder? 1. Loose associations 2. Paranoid delusions 3. Magical thinking 4. Delusions of reference

4 The nurse is assessing for the potential symptom of delusions of reference. A client who believes he or she receives messages through the radio or TV is experiencing delusions of reference. These delusions involve the client interpreting events within the environment as being directed toward himself or herself. Clients with delusions of reference believe that others are trying to send them messages in various ways, or they must break a code to receive a message.

Prioritize the depressive disorders and their predominant affective symptoms from least to most severe (1-4). (Enter the number of each disorder in the proper sequence, using comma and space format, such as 1, 2, 3, 4.) 1. Dysthymic disorder (helplessness, powerlessness, pessimistic outlook, low self-esteem) 2. Uncomplicated grieving (feelings of anger, anxiety, guilt, helplessness) 3. Major depressive episode (total despair, worthlessness, flat affect, apathy) 4. Transient depression (sadness, dejection, feeling downhearted, having "the blues")

ANS: 4, 2, 1, 3

A nurse and a registered dietitian are assessing clients for partial parenteral nutrition (PPN). For which client would the nurse suggest another route of providing nutrition? a. Client with congestive heart failure b. Older client with dementia c. Client who has multiorgan failure d. Client who is post gastric resection

ANS: A Clients receiving PPN typically get large amounts of fluid volume, making the client with heart failure a poor candidate. The other candidates are appropriate for this type of nutritional support.

The nurse is providing patient teaching to a patient diagnosed with acute rhinosinusitis. For what possible complication should the nurse teach the patient to seek immediate follow-up? A) Periorbital edema B) Headache unrelieved by OTC medications C) Clear drainage from nose D) Blood-tinged mucus when blowing the nose

ANS: A Feedback: Patient teaching is an important aspect of nursing care for the patient with acute rhinosinusitis. The nurse instructs the patient about symptoms of complications that require immediate follow-up. Referral to a physician is indicated if periorbital edema and severe pain on palpation occur. Clear drainage and bloodtinged mucus do not require follow-up if the patient has acute rhinosinusitis. A persistent headache does not necessarily warrant immediate follow-up.

A client is exhibiting symptoms of generalized amnesia. Which of the following questions should the nurse ask to confirm this diagnosis? Select all that apply. A. "Can you tell me your name and where you live?" B. "Have you ever traveled suddenly or unexpectedly away from home?" C. "Have you recently experienced any traumatic event?" D. "Have you ever felt detached from your environment?" E. "Have you had any history of memory problems?"

ANS: A, C, E An individual who has generalized amnesia has amnesia for his or her identity and total life history. The first question assesses orientation to identity. Items C and D assess for awareness of current issues and historical issues, respectively. Affirmative descriptions of either of these issues would rule out generalized amnesia. KEY: Cognitive Level: Analysis | Integrated Processes: Nursing Process: Assessment | Client Need: Psychosocial Integrity

A lethargic client is diagnosed with major depressive disorder. After taking antidepressant therapy for 6 weeks, the symptoms have not resolved. Which nutritional deficiency should a nurse identify as potentially contributing to the client's symptoms?A. Vitamin A deficiency B. Vitamin C deficiency C. Iron deficiency D. Folic acid deficiency

ANS: C The nurse should identify that an iron deficiency could contribute to depression. Iron deficiencies can result in feelings of chronic fatigue. Iron should be consumed by eating meat, fish, green leafy vegetables, nuts, eggs, and enriched bread and pasta.

The nurse responds to the call light of a patient who has had a cervical diskectomy earlier in the day. The patient states that she is having severe pain that had a sudden onset. What is the nurse's most appropriate action?

Call the surgeon to report the patient's pain.

A nurse is caring for a patient with burns who is in the later stages of the acute phase of recovery. The plan of nursing care should include which of the following nursing actions? A) Maintenance of bed rest to aid healing B) Choosing appropriate splints and functional devices C) Administration of beta adrenergic blockers D) Prevention of venous thromboembolism

D Feedback: Prevention of deep vein thrombosis (DVT) is an important factor in care. Early mobilization of the patient is important. The nurse monitors the splints and functional devices, but these are selected by occupational and physical therapists. The hemodynamic changes accompanying burns do not normally require the use of beta blockers.

The nurse caring for a patient who is recovering from full-thickness burns is aware of the patients risk for contracture and hypertrophic scarring. How can the nurse best mitigate this risk? A) Apply skin emollients as ordered after granulation has occurred. B) Keep injured areas immobilized whenever possible to promote healing. C) Administer oral or IV corticosteroids as ordered. D) Encourage physical activity and range of motion exercises.

D Feedback: Exercise and the promotion of mobility can reduce the risk of contracture and hypertrophic scarring. Skin emollients are not normally used in the treatment of burns, and these do not prevent scarring. Steroids are not used to reduce scarring, as they also slow the healing process.

A 62-year-old woman diagnosed with breast cancer is scheduled for a partial mastectomy. The oncology nurse explained that the surgeon will want to take tissue samples to ensure the disease has not spread to adjacent axillary lymph nodes. The patient has asked if she will have her lymph nodes dissected, like her mother did several years ago. What alternative to lymph node dissection will this patient most likely undergo? A) Lymphadenectomy B) Needle biopsy C) Open biopsy D) Sentinel node biopsy

D

A patients most recent laboratory results show a slight decrease in potassium. The physician has opted to forego drug therapy but has suggested increasing the patients dietary intake of potassium. Which of the following would be a good source of potassium? A) Apples B) Asparagus C) Carrots D) Bananas

D

The nurse is caring for a patient in metabolic alkalosis. The patient has an NG tube to low intermittent suction for a diagnosis of bowel obstruction. What drug would the nurse expect to find on the medication orders? A) Cimetidine B) Maalox C) Potassium chloride elixir D) Furosemide

a

A telemetry nurse assessing a client with third-degree heart block has wide QRS complexes and a heart rate of 35 beats per minute of the cardiac monitor. Which assessment should the nurse complete next? a. pulmonary auscultation b. pulse strength and amplitude c. level of consciousness d. mobility and gait stability

c. level of consciousness

A medical nurse who is caring for a patient being discharged home after a radical neck dissection has collaborated with the home health nurse to develop a plan of care for this patient. What is a priority psychosocial outcome for a patient who has had a radical neck dissection? A) Indicates acceptance of altered appearance and demonstrates positive self-image B) Freely expresses needs and concerns related to postoperative pain management C) Compensates effectively for alteration in ability to communicate related to dysarthria D) Demonstrates effective stress management techniques to promote muscle relaxation

A Feedback: Since radical neck dissection involves removal of the sternocleidomastoid muscle, spinal accessory muscles, and cervical lymph nodes on one side of the neck, the patients appearance is visibly altered. The face generally appears asymmetric, with a visible neck depression; shoulder drop also occurs frequently. These changes have the potential to negatively affect self-concept and body image. Facilitating adaptation to these changes is a crucial component of nursing intervention. Patients who have had head and neck surgery generally report less pain as compared with other postoperative patients; however, the nurse must assess each individual patients level of pain and response to analgesics. Patients may experience transient hoarseness following a radical neck dissection; however, their ability to communicate is not permanently altered. Stress management is beneficial but would not be considered the priority in this clinical situation.

A patient is admitted to the critical care unit (CCU) with a diagnosis of cardiomyopathy. When reviewing the patients most recent laboratory results, the nurse should prioritize assessment of which of the following? A) Sodium B) AST, ALT, and bilirubin C) White blood cell differential D) BUN

A Feedback: Sodium is the major electrolyte involved with cardiomyopathy. Cardiomyopathy often leads to heart failure which develops, in part, from fluid overload. Fluid overload is often associated with elevated sodium levels. Consequently, sodium levels are followed more closely than other important laboratory values, including BUN, leukocytes, and liver function tests.

A patient is postoperative day 1 following intracranial surgery. The nurses assessment reveals that the patients LOC is slightly decreased compared with the day of surgery. What is the nurses best response to this assessment finding? A) Recognize that this may represent the peak of post-surgical cerebral edema. B) Alert the surgeon to the possibility of an intracranial hemorrhage. C) Understand that the surgery may have been unsuccessful. D) Recognize the need to refer the patient to the palliative care team.

A Feedback: Some degree of cerebral edema occurs after brain surgery; it tends to peak 24 to 36 hours after surgery, producing decreased responsiveness on the second postoperative day. As such, there is not necessarily any need to deem the surgery unsuccessful or to refer the patient to palliative care. A decrease in LOC is not evidence of an intracranial hemorrhage.

A patient has been diagnosed with a valvular disorder. The patient tells the nurse that he has read about numerous treatment options, including valvuloplasty. What should the nurse teach the patient about valvuloplasty? A) For some patients, valvuloplasty can be done in a cardiac catheterization laboratory. B) Valvuloplasty is a dangerous procedure, but it has excellent potential if it goes well. C) Valvuloplasty is open heart surgery, but this is very safe these days and normally requires only an overnight hospital stay. D) Its prudent to get a second opinion before deciding to have valvuloplasty.

A Feedback: Some valvuloplasty procedures do not require general anesthesia or cardiopulmonary bypass and can be performed in a cardiac catheterization laboratory or hybrid room. Open heart surgery is not required and the procedure does not carry exceptional risks that would designate it as being dangerous. Normally there is no need for the nurse to advocate for a second opinion.

After contributing to the care of several patients who died in the hospital, the nurse has identified some lapses in the care that many of these patients received toward the end of their lives. What have research studies identified as a potential deficiency in the care of the dying in hospital settings? A) Families needs for information and support often go unmet. B) Patients are too sedated to achieve adequate pain control. C) Patients are not given opportunities to communicate with caregivers. D) Patients are ignored by the care team toward the end of life.

A Feedback: Studies have demonstrated that the health care system continues to be challenged when meeting seriously ill patients needs for pain and symptom management and their families needs for information and support. Oversedation, lack of communication, and lack of care are not noted to be deficiencies to the same degree.

A 48-year-old man presents to the ED complaining of severe substernal chest pain radiating down his left arm. He is admitted to the coronary care unit (CCU) with a diagnosis of myocardial infarction (MI). What nursing assessment activity is a priority on admission to the CCU? A) Begin ECG monitoring. B) Obtain information about family history of heart disease. C) Auscultate lung fields. D) Determine if the patient smokes.

A Feedback: The 12-lead ECG provides information that assists in ruling out or diagnosing an acute MI. It should be obtained within 10 minutes from the time a patient reports pain or arrives in the ED. By monitoring serial ECG changes over time, the location, evolution, and resolution of an MI can be identified and monitored; life-threatening arrhythmias are the leading cause of death in the first hours after an MI. Obtaining information about family history of heart disease and whether the patient smokes are not immediate priorities in the acute phase of MI. Data may be obtained from family members later. Lung fields are auscultated after oxygenation and pain control needs are met.

A patient presents to the ED in distress and complaining of crushing chest pain. What is the nurses priority for assessment? A) Prompt initiation of an ECG B) Auscultation of the patients point of maximal impulse (PMI) C) Rapid assessment of the patients peripheral pulses D) Palpation of the patients cardiac apex

A Feedback: The 12-lead ECG provides information that assists in ruling out or diagnosing an acute MI. It should be obtained within 10 minutes from the time a patient reports pain or arrives in the ED. Each of the other listed assessments is valid, but ECG monitoring is the most time dependent priority.

A patient who is a candidate for an implantable cardioverter defibrillator (ICD) asks the nurse about the purpose of this device. What would be the nurses best response? A) To detect and treat dysrhythmias such as ventricular fibrillation and ventricular tachycardia B) To detect and treat bradycardia, which is an excessively slow heart rate C) To detect and treat atrial fibrillation, in which your heart beats too quickly and inefficiently D) To shock your heart if you have a heart attack at home

A Feedback: The ICD is a device that detects and terminates life-threatening episodes of ventricular tachycardia and ventricular fibrillation. It does not treat atrial fibrillation, MI, or bradycardia.

A major earthquake has occurred within the vicinity of the local hospital. The nursing supervisor working the night shift at the hospital receives information that the hospital disaster plan will be activated. The supervisor will need to work with what organization responsible for coordinating interagency relief assistance? A) Office of Emergency Management B) Incident Command System C) Centers for Disease Control and Prevention (CDC) D) American Red Cross

A Feedback: The Office of Emergency Management coordinates the disaster relief efforts at state and local levels. The Incident Command System is a management tool to organize personnel, facilities, equipment, and communication in an emergency situation. The CDC is the agency for disease prevention and control and it supports state and local health departments. The American Red Cross provides additional support.

A nurse is assisting with serving dinner trays on the unit. Upon receiving the dinner tray for a patient admitted with acute gallbladder inflammation, the nurse will question which of the following foods on the tray? A) Fried chicken B) Mashed potatoes C) Dinner roll D) Tapioca pudding

A Feedback: The diet immediately after an episode of acute cholecystitis is initially limited to low-fat liquids. Cooked fruits, rice or tapioca, lean meats, mashed potatoes, bread, and coffee or tea may be added as tolerated. The patient should avoid fried foods such as fried chicken, as fatty foods may bring on an episode of cholecystitis.

Emergency department (ED) staff members have been trained to follow steps that will decrease the risk of secondary exposure to a chemical. When conducting decontamination, staff members should remove the patients clothing and then perform what action? A) Rinse the patient with water. B) Wash the patient with a dilute bleach solution. C) Wash the patient chlorhexidine. D) Rinse the patient with hydrogen peroxide.

A Feedback: The first step in decontamination is removal of the patients clothing and jewelry and then rinsing the patient with water. This is usually followed by a wash with soap and water, not chlorhexidine, bleach, or hydrogen peroxide.

The nurse is planning patient teaching for a patient with ESKD who is scheduled for the creation of a fistula. The nurse would include which of the following in teaching the patient about the fistula? A) A vein and an artery in your arm will be attached surgically. B) The arm should be immobilized for 4 to 6 days. C) One needle will be inserted into the fistula for each dialysis treatment. D) The fistula can be used 2 days after the surgery for dialysis treatment.

A Feedback: The fistula joins an artery and a vein, either side-to-side or end-to-end. This access will need time, usually 2 to 3 months, to mature before it can be used. The patient is encouraged to perform exercises to increase the size of the affected vessels (e.g., squeezing a rubber ball for forearm fistulas). Two needles will be inserted into the fistula for each dialysis treatment.

An emergency department nurse learns from the paramedics that they are transporting a patient who has suffered injury from a scald from a hot kettle. What variables will the nurse consider when determining the depth of burn? A) The causative agent B) The patients preinjury health status C) The patients prognosis for recovery D) The circumstances of the accident

A Feedback: The following factors are considered in determining the depth of a burn: how the injury occurred, causative agent (such as flame or scalding liquid), temperature of the burning agent, duration of contact with the agent, and thickness of the skin. The patients preinjury status, circumstances of the accident, and prognosis for recovery are important, but are not considered when determining the depth of the burn.

A patient who is in the acute phase of recovery from a burn injury has yet to experience adequate pain control. What pain management strategy is most likely to meet this patients needs? A) A patient-controlled analgesia (PCA) system B) Oral opioids supplemented by NSAIDs C) Distraction and relaxation techniques supplemented by NSAIDs D) A combination of benzodiazepines and topical anesthetics

A Feedback: The goal of treatment is to provide a long-acting analgesic that will provide even coverage for this longterm discomfort. It is helpful to use escalating doses when initiating the medication to reach the level of pain control that is acceptable to the patient. The use of patient-controlled analgesia (PCA) gives control to the patient and achieves this goal. Patients cannot normally achieve adequate pain control without the use of opioids, and parenteral administration is usually required.

The ED nurse is planning the care of a patient who has been admitted following a sexual assault. The nurse knows that all of the nursing interventions are aimed at what goal? A) Encouraging the patient to gain a sense of control over his or her life B) Collecting sufficient evidence to secure a criminal conviction C) Helping the patient understand that this will not happen again D) Encouraging the patient to verbalize what happened during the assault

A Feedback: The goals of management are to provide support, to reduce the patients emotional trauma, and to gather available evidence for possible legal proceedings. All of the interventions are aimed at encouraging the patient to gain a sense of control over his or her life. The patients well-being should be considered a priority over criminal proceedings. No health professional can guarantee the patients future safety and having the patient verbalize the event is not a priority.

The nurse is performing a physical assessment on a patient suspected of having HF. The presence of what sound would signal the possibility of impending HF? A) An S3 heart sound B) Pleural friction rub C) Faint breath sounds D) A heart murmur

A Feedback: The heart is auscultated for an S3 heart sound, a sign that the heart is beginning to fail and that increased blood volume fills the ventricle with each beat. HF does not normally cause a pleural friction rub or murmurs. Changes in breath sounds occur, such as the emergence of crackles or wheezes, but faint breath sounds are less characteristic of HF.

The patient has been diagnosed with aphasia after suffering a stroke. What can the nurse do to best make the patients atmosphere more conducive to communication? A) Provide a board of commonly used needs and phrases. B) Have the patient speak to loved ones on the phone daily. C) Help the patient complete his or her sentences. D) Speak in a loud and deliberate voice to the patient.

A Feedback: The inability to talk on the telephone or answer a question or exclusion from conversation causes anger, frustration, fear of the future, and hopelessness. A common pitfall is for the nurse or other health care team member to complete the thoughts or sentences of the patient. This should be avoided because it may cause the patient to feel more frustrated at not being allowed to speak and may deter efforts to practice putting thoughts together and completing a sentence. The patient may also benefit from a communication board, which has pictures of commonly requested needs and phrases. The board may be translated into several languages.

A patient is brought to the emergency department with a burn injury. The nurse knows that the first systemic event after a major burn injury is what? A) Hemodynamic instability B) Gastrointestinal hypermotility C) Respiratory arrest D) Hypokalemia

A Feedback: The initial systemic event after a major burn injury is hemodynamic instability, which results from loss of capillary integrity and a subsequent shift of fluid, sodium, and protein from the intravascular space into the interstitial spaces. This precedes GI changes. Respiratory arrest may or may not occur, largely depending on the presence or absence of smoke inhalation. Hypokalemia does not take place in the initial phase of recovery.

You are caring for a 50-year-old man diagnosed with multiple myeloma; he has just been told by the care team that his prognosis is poor. He is tearful and trying to express his feelings, but he is having difficulty. What should you do first? A) Ask if he would like you to sit with him while he collects his thoughts. B) Tell him that you will leave for now but will be back shortly. C) Offer to call pastoral care or a member of his chosen clergy. D) Reassure him that you can understand how he is feeling.

A Feedback: The most important intervention the nurse can provide is listening empathetically. Seriously ill patients and their families need time and support to cope with the changes brought about by serious illness and the prospect of impending death. The nurse who is able to listen without judging and without trying to solve the patients and familys problems provides an invaluable intervention. The patient needs to feel that people are concerned with his situation. Leaving him does not show acceptance of his feelings. Offering to call pastoral care may be helpful for some patients, but should be done after you have spent time with the patient. Telling the patient that you understand how he is feeling is inappropriate because it does not help him express his feelings.

An office worker takes a cupcake that contains peanut butter. He begins wheezing, with an inspiratory stridor and air hunger and the occupational health nurse is called to the office. The nurse should recognize that the worker is likely suffering from which type of hypersensitivity? A) Anaphylactic (type 1) B) Cytotoxic (type II) C) Immune complex (type III) D) Delayed-type (type IV)

A Feedback: The most severe form of a hypersensitivity reaction is anaphylaxis. An unanticipated severe allergic reaction that is often explosive in onset, anaphylaxis is characterized by edema in many tissues, including the larynx, and is often accompanied by hypotension, bronchospasm, and cardiovascular collapse in severe cases. Type II, or cytotoxic, hypersensitivity occurs when the system mistakenly identifies a normal constituent of the body as foreign. Immune complex (type III) hypersensitivity involves immune complexes formed when antigens bind to antibodies. Type III is associated with systemic lupus erythematosus, rheumatoid arthritis, certain types of nephritis, and bacterial endocarditis. Delayed-type (type IV), also known as cellular hypersensitivity, occurs 24 to 72 hours after exposure to an allergen.

When caring for a patient with increased ICP the nurse knows the importance of monitoring for possible secondary complications, including syndrome of inappropriate antidiuretic hormone (SIADH). What nursing interventions would the nurse most likely initiate if the patient developed SIADH? A) Fluid restriction B) Transfusion of platelets C) Transfusion of fresh frozen plasma (FFP) D) Electrolyte restriction

A Feedback: The nurse also assesses for complications of increased ICP, including diabetes insipidus, and SIADH. SIADH requires fluid restriction and monitoring of serum electrolyte levels. Transfusions are unnecessary.

The nurse is working on the renal transplant unit. To reduce the risk of infection in a patient with a transplanted kidney, it is imperative for the nurse to do what? A) Wash hands carefully and frequently. B) Ensure immediate function of the donated kidney. C) Instruct the patient to wear a face mask. D) Bar visitors from the patients room.

A Feedback: The nurse ensures that the patient is protected from exposure to infection by hospital staff, visitors, and other patients with active infections. Careful handwashing is imperative; face masks may be worn by hospital staff and visitors to reduce the risk for transmitting infectious agents while the patient is receiving high doses of immunosuppressants. Visitors may be limited, but are not normally barred outright. Ensuring kidney function is vital, but does not prevent infection.

The nurse notes that a patient who has undergone skin, tissue, and muscle grafting following a modified radical neck dissection requires suctioning. What is the most important consideration for the nurse when suctioning this patient? A) Avoid applying suction on or near the suture line. B) Position patient on the non operative side with the head of the bed down. C) Assess the patients ability to perform self-suctioning. D) Evaluate the patients ability to swallow saliva and clear fluids

A Feedback: The nurse should avoid positioning the suction catheter on or near the graft suture lines. Application of suction in these areas could damage the graft. Self-sectioning may be unsafe because the patient may damage the suture line. Following a modified radical neck dissection with graft, the patient is usually positioned with the head of the bed elevated to promote drainage and reduce edema. Assessing viability of the graft is important but is not part of the suctioning procedure and may delay initiating suctioning. Maintenance of a patent airway is a nursing priority. Similarly, the patients ability to swallow is an important assessment for the nurse to make; however, it is not directly linked to the patients need for suctioning.

A nurse is caring for a patient with hepatic encephalopathy. While making the initial shift assessment, the nurse notes that the patient has a flapping tremor of the hands. The nurse should document the presence of what sign of liver disease? A) Asterixis B) Constructional apraxia C) Fetor hepaticus D) Palmar erythema

A Feedback: The nurse will document that a patient exhibiting a flapping tremor of the hands is demonstrating asterixis. While constructional apraxia is a motor disturbance, it is the inability to reproduce a simple figure. Fetor hepaticus is a sweet, slightly fecal odor to the breath and not associated with a motor disturbance. Skin changes associated with liver dysfunction may include palmar erythema, which is a reddening of the palms, but is not a flapping tremor.

The nurse is caring for a patient with permanent neurologic impairments resulting from a traumatic head injury. When working with this patient and family, what mutual goal should be prioritized? A) Achieve as high a level of function as possible. B) Enhance the quantity of the patients life. C) Teach the family proper care of the patient. D) Provide community assistance.

A Feedback: The overarching goals of care are to achieve as high a level of function as possible and to enhance the quality of life for the patient with neurologic impairment and his or her family. This goal encompasses family and community participation.

A nurse takes a shift report and finds he is caring for a patient who has been exposed to anthrax by inhalation. What precautions does the nurse know must be put in place when providing care for this patient? A) Standard precautions B) Airborne precautions C) Droplet precautions D) Contact precautions

A Feedback: The patient is not contagious, and anthrax cannot be spread from person to person, so standard precautions are initiated. Airborne, contact, and droplet precautions are not necessary.

Following diagnostic testing, a patient has been admitted to the ICU and placed on cerebral aneurysm precautions. What nursing action should be included in patients plan of care? A) Supervise the patients activities of daily living closely. B) Initiate early ambulation to prevent complications of immobility. C) Provide a high-calorie, low-protein diet. D) Perform all of the patients hygiene and feeding.

A Feedback: The patient is placed on immediate and absolute bed rest in a quiet, nonstressful environment, because activity, pain, and anxiety elevate BP, which increases the risk for bleeding. As such, independent ADLs and ambulation are contraindicated. There is no need for a high-calorie or low-protein diet.

The triage nurse is working in the ED. A homeless person is admitted during a blizzard with complaints of being unable to feel his feet and lower legs. Core temperature is noted at 33.2C (91.8F). The patient is intoxicated with alcohol at the time of admission and is visibly malnourished. What is the triage nurses priority in the care of this patient? A) Addressing the patients hypothermia B) Addressing the patients frostbite in his lower extremities C) Addressing the patients alcohol intoxication D) Addressing the patients malnutrition

A Feedback: The patient may also have frostbite, but hypothermia takes precedence in treatment because it is systemic rather than localized. The alcohol abuse and the alteration in nutrition do not take precedence over the treatment of hypothermia because both problems are a less acute threat to the patients survival.

A patient is recovering from intracranial surgery performed approximately 24 hours ago and is complaining of a headache that the patient rates at 8 on a 10-point pain scale. What nursing action is most appropriate? A) Administer morphine sulfate as ordered. B) Reposition the patient in a prone position. C) Apply a hot pack to the patients scalp. D) Implement distraction techniques.

A Feedback: The patient usually has a headache after a craniotomy as a result of stretching and irritation of nerves in the scalp during surgery. Morphine sulfate may also be used in the management of postoperative pain in patients who have undergone a craniotomy. Prone positioning is contraindicated due to the consequent increase in ICP. Distraction would likely be inadequate to reduce pain and a hot pack may cause vasodilation and increased pain.

Which patient should the nurse prioritize as needing emergent treatment, assuming no other injuries are present except the ones outlined below? A) A patient with a blunt chest trauma with some difficulty breathing B) A patient with a sore neck who was immobilized in the field on a backboard with a cervical collar C) A patient with a possible fractured tibia with adequate pedal pulses D) A patient with an acute onset of confusion

A Feedback: The patient with blunt chest trauma possibly has a compromised airway. Establishment and maintenance of a patent airway and adequate ventilation is prioritized over other health problems, including skeletal injuries and changes in cognition

The paramedics bring a patient who has suffered a sexual assault to the ED. What is important for the sexual assault nurse examiner to do when assessing a sexual assault victim? A) Respect the patients privacy during assessment. B) Shave all pubic hair for laboratory analysis. C) Place items for evidence in plastic bags. D) Bathe the patient before the examination.

A Feedback: The patients privacy and sensitivity must be respected, because the patient will be experiencing a stress response to the assault. Pubic hair is combed or trimmed for sampling. Paper bags are used for evidence collection because plastic bags retain moisture, which promotes mold and mildew that can destroy evidence. Bathing the patient before the examination would destroy or remove key evidence.

The nurse is assessing a patient suspected of having developed acute glomerulonephritis. The nurse should expect to address what clinical manifestation that is characteristic of this health problem? A) Hematuria B) Precipitous decrease in serum creatinine levels C) Hypotension unresolved by fluid administration D) Glucosuria

A Feedback: The primary presenting feature of acute glomerulonephritis is hematuria (blood in the urine), which may be microscopic (identifiable through microscopic examination) or macroscopic or gross (visible to the eye). Proteinuria, primarily albumin, which is present, is due to increased permeability of the glomerular membrane. Blood urea nitrogen (BUN) and serum creatinine levels may rise as urine output drops. Some degree of edema and hypertension is noted in most patients.

The nurse is administering total parenteral nutrition (TPN) to a client who underwent surgery for gastric cancer. Which of the nurses assessments most directly addresses a major complication of TPN? A) Checking the patients capillary blood glucose levels regularly B) Having the patient frequently rate his or her hunger on a 10-point scale C) Measuring the patients heart rhythm at least every 6 hours D) Monitoring the patients level of consciousness each shift

A Feedback: The solution, used as a base for most TPN, consists of a high dextrose concentration and may raise blood glucose levels significantly, resulting in hyperglycemia. This is a more salient threat than hunger, though this should be addressed. Dysrhythmias and decreased LOC are not among the most common complications.

A nurse has asked the nurse educator if there is any way to predict the severity of a patients anaphylactic reaction. What would be the nurses best response? A) The faster the onset of symptoms, the more severe the reaction. B) The reaction will be about one-third more severe than the patients last reaction to the same antigen. C) There is no way to gauge the severity of a patients anaphylaxis, even if it has occurred repeatedly in the past. D) The reaction will generally be slightly less severe than the last reaction to the same antigen.

A Feedback: The time from exposure to the antigen to onset of symptoms is a good indicator of the severity of the reaction: the faster the onset, the more severe the reaction. None of the other statements is an accurate description of the course of anaphylactic reactions.

The nurse is caring for a patient who has terminal lung cancer and is unconscious. Which assessment finding would most clearly indicate to the nurse that the patients death is imminent? A) Mottling of the lower limbs B) Slow, steady pulse C) Bowel incontinence D) Increased swallowing

A Feedback: The time of death is generally preceded by a period of gradual diminishment of bodily functions in which increasing intervals between respirations, weakened and irregular pulse, and skin color changes or mottling may be observed. The patient will not be able to swallow secretions, so suctioning, frequent and gentle mouth care, and, possibly, the administration of a transdermal anticholinergic drug. Bowel incontinence may or may not occur.

A patient with suspected Parkinsons disease is initially being assessed by the nurse. When is the best time to assess for the presence of a tremor? A) When the patient is resting B) When the patient is ambulating C) When the patient is preparing his or her meal tray to eat D) When the patient is participating in occupational therapy

A Feedback: The tremor is present while the patient is at rest; it increases when the patient is walking, concentrating, or feeling anxious. Resting tremor characteristically disappears with purposeful movement, but is evident when the extremities are motionless. Consequently, the nurse should assess for the presence of a tremor when the patient is not performing deliberate actions.

A patients declining cardiac status has been attributed to decreased cardiac action potential. Interventions will be aimed at restoring what aspect of cardiac physiology? A) The cycle of depolarization and repolarization B) The time it takes from the firing of the SA node to the contraction of the ventricles C) The time between the contraction of the atria and the contraction of the ventricles D) The cycle of the firing of the AV node and the contraction of the myocardium

A Feedback: This exchange of ions creates a positively charged intracellular space and a negatively charged extracellular space that characterizes the period known as depolarization. Once depolarization is complete, the exchange of ions reverts to its resting state; this period is known as repolarization. The repeated cycle of depolarization and repolarization is called the cardiac action potential.

A patient is brought by ambulance to the ED after suffering what the family thinks is a stroke. The nurse caring for this patient is aware that an absolute contraindication for thrombolytic therapy is what? A) Evidence of hemorrhagic stroke B) Blood pressure of 180/110 mm Hg C) Evidence of stroke evolution D) Previous thrombolytic therapy within the past 12 months

A Feedback: Thrombolytic therapy would exacerbate a hemorrhagic stroke with potentially fatal consequences. Stroke evolution, high BP, or previous thrombolytic therapy does not contraindicate its safe and effective use.

A hospital patient has experienced a seizure. In the immediate recovery period, what action best protects the patients safety? A) Place the patient in a side-lying position. B) Pad the patients bed rails. C) Administer antianxiety medications as ordered. D) Reassure the patient and family members.

A Feedback: To prevent complications, the patient is placed in the side-lying position to facilitate drainage of oral secretions. Suctioning is performed, if needed, to maintain a patent airway and prevent aspiration. None of the other listed actions promotes safety during the immediate recovery period.

The nurse is caring for a patient admitted with a drug overdose. What is the nurses priority responsibility in caring for this patient? A) Support the patients respiratory and cardiovascular function. B) Provide for the safety of the patient. C) Enhance clearance of the offending agent. D) Ensure the safety of the staff.

A Feedback: Treatment goals for a patient with a drug overdose are to support the respiratory and cardiovascular functions, to enhance clearance of the agent, and to provide for safety of the patient and staff. Of these responsibilities, however, support of vital physiologic function is a priority.

An 82-year-old man is admitted for observation after a fall. Due to his age, the nurse knows that the patient is at increased risk for what complication of his injury? A) Hematoma B) Skull fracture C) Embolus D) Stroke

A Feedback: Two major factors place older adults at increased risk for hematomas. First, the dura becomes more adherent to the skull with increasing age. Second, many older adults take aspirin and anticoagulants as part of routine management of chronic conditions. Because of these factors, the patients risk of a hematoma is likely greater than that of stroke, embolism, or skull fracture.

A patient has sought care, stating that she developed hives overnight. The nurses inspection confirms the presence of urticaria. What type of allergic hypersensitivity reaction has the patient developed? A) Type I B) Type II C) Type III D) Type IV

A Feedback: Urticaria (hives) is a type I hypersensitive allergic reaction

A 55-year-old female patient with hepatocellular carcinoma (HCC) is undergoing radiofrequency ablation. The nurse should recognize what goal of this treatment? A) Destruction of the patients liver tumor B) Restoration of portal vein patency C) Destruction of a liver abscess D) Reversal of metastasis

A Feedback: Using radiofrequency ablation, a tumor up to 5 cm in size can be destroyed in one treatment session. This technique does not address circulatory function or abscess formation. It does not allow for the reversal of metastasis.

A nurse is caring for a patient who has had surgery for oral cancer. When addressing the patients long term needs, the nurse should prioritize interventions and referrals with what goal? A) Enhancement of verbal communication B) Enhancement of immune function C) Maintenance of adequate social support D) Maintenance of fluid balance

A Feedback: Verbal communication may be impaired by radical surgery for oral cancer. Addressing this impairment often requires a long-term commitment. Immune function, social support, and fluid balance are all necessary, but communication is a priority issue for patients recovering from this type of surgery

The nurse is caring for an adult patient who has gone into ventricular fibrillation. When assisting with defibrillating the patient, what must the nurse do? A) Maintain firm contact between paddles and patient skin. B) Apply a layer of water as a conducting agent. C) Call all clear once before discharging the defibrillator. D) Ensure the defibrillator is in the sync mode.

A Feedback: When defibrillating an adult patient, the nurse should maintain good contact between the paddles and the patients skin to prevent arcing, apply an appropriate conducting agent (not water) between the skin and the paddles, and ensure the defibrillator is in the nonsync mode. Clear should be called three times before discharging the paddles.

A patient has been living with dilated cardiomyopathy for several years but has experienced worsening symptoms despite aggressive medical management. The nurse should anticipate what potential treatment? A) Heart transplantation B) Balloon valvuloplasty C) Cardiac catheterization D) Stent placement

A Feedback: When heart failure progresses and medical treatment is no longer effective, surgical intervention, including heart transplantation, is considered. Valvuloplasty, stent placement, and cardiac catheterization will not address the pathophysiology of cardiomyopathy.

The triage nurse in the ED assesses a 66-year-old male patient who presents to the ED with complaints of midsternal chest pain that has lasted for the last 5 hours. If the patients symptoms are due to an MI, what will have happened to the myocardium? A) It may have developed an increased area of infarction during the time without treatment. B) It will probably not have more damage than if he came in immediately. C) It may be responsive to restoration of the area of dead cells with proper treatment. D) It has been irreparably damaged, so immediate treatment is no longer necessary.

A Feedback: When the patient experiences lack of oxygen to myocardium cells during an MI, the sooner treatment is initiated, the more likely the treatment will prevent or minimize myocardial tissue necrosis. Delays in treatment equate with increased myocardial damage. Despite the length of time the symptoms have been present, treatment needs to be initiated immediately to minimize further damage. Dead cells cannot be restored by any means

A cardiac patients resistance to left ventricular filling has caused blood to back up into the patients circulatory system. What health problem is likely to result? A) Acute pulmonary edema B) Right-sided HF C) Right ventricular hypertrophy D) Left-sided HF

A Feedback: With increased resistance to left ventricular filling, blood backs up into the pulmonary circulation. The patient quickly develops pulmonary edema from the blood volume overload in the lungs. When the blood backs up into the pulmonary circulation, right-sided HF, left-sided HF, and right ventricular hypertrophy do not directly occur.

A nurse has reported for a shift at a busy burns and plastics unit in a large university hospital. Which patient is most likely to have life-threatening complications? A) A 4-year-old scald victim burned over 24% of the body B) A 27-year-old male burned over 36% of his body in a car accident C) A 39-year-old female patient burned over 18% of her body D) A 60-year-old male burned over 16% of his body in a brush fire

A Feedback: Young children and the elderly continue to have increased morbidity and mortality when compared to other age groups with similar injuries and present a challenge for burn care. This is an important factor when determining the severity of injury and possible outcome for the patient.

A community health nurse is presenting an educational event and is addressing several health problems, including rheumatic heart disease. What should the nurse describe as the most effective way to prevent rheumatic heart disease? A) Recognizing and promptly treating streptococcal infections B) Prophylactic use of calcium channel blockers in high-risk populations C) Adhering closely to the recommended child immunization schedule D) Smoking cessation

A Group A streptococcus can cause rheumatic heart fever, resulting in rheumatic endocarditis. Being aware of signs and symptoms of streptococcal infections, identifying them quickly, and treating them promptly, are the best preventative techniques for rheumatic endocarditis. Smoking cessation, immunizations, and calcium channel blockers will not prevent rheumatic heart disease.

A school nurse is caring for a child who appears to be having an allergic response. What should be the initial action of the school nurse? A) Assess for signs and symptoms of anaphylaxis. B) Assess for erythema and urticaria. C) Administer an OTC antihistamine. D) Administer epinephrine.

A If a patient is experiencing an allergic response, the nurses initial action is to assess the patient for signs and symptoms of anaphylaxis. Erythema and urticaria may be present, but these are not the most significant or most common signs of anaphylaxis. Assessment must precede interventions, such as administering an antihistamine. Epinephrine is indicated in the treatment of anaphylaxis, not for every allergic reaction.

A patient with leukemia has developed stomatitis and is experiencing a nutritional deficit. An oral anesthetic has consequently been prescribed. What health education should the nurse provide to the patient? A) Chew with care to avoid inadvertently biting the tongue. B) Use the oral anesthetic 1 hour prior to meal time. C) Brush teeth before and after eating. D) Swallow slowly and deliberately.

A If oral anesthetics are used, the patient must be warned to chew with extreme care to avoid inadvertently biting the tongue or buccal mucosa. An oral anesthetic would be metabolized by the time the patient eats if it is used 1 hour prior to meals. There is no specific need to warn the patient about brushing teeth or swallowing slowly because an oral anesthetic has been used.

A patient who is at high risk for developing intracardiac thrombi has been placed on long-term anticoagulation. What aspect of the patients health history creates a heightened risk of intracardiac thrombi? A) Atrial fibrillation B) Infective endocarditis C) Recurrent pneumonia D) Recent surgery

A Intracardiac thrombi are especially common in patients with atrial fibrillation, because the atria do not contract forcefully and blood flows slowly and turbulently, increasing the likelihood of thrombus formation. Endocarditis, pneumonia, and recent surgery do not normally cause an increased risk for intracardiac thrombi formation.

A nurse educator is teaching a group of recent nursing graduates about their occupational risks for contracting hepatitis B. What preventative measures should the educator promote? Select all that apply. A) Immunization B) Use of standard precautions C) Consumption of a vitamin-rich diet D) Annual vitamin K injections E) Annual vitamin B12 injections

A, B Feedback: People who are at high risk, including nurses and other health care personnel exposed to blood or blood products, should receive active immunization. The consistent use of standard precautions is also highly beneficial. Vitamin supplementation is unrelated to an individuals risk of HBV.

A nurse is preparing to discharge a patient home on parenteral nutrition. What should an effective home care teaching program address? Select all that apply. A) Preparing the patient to troubleshoot for problems B) Teaching the patient and family strict aseptic technique C) Teaching the patient and family how to set up the infusion D) Teaching the patient to flush the line with sterile water E) Teaching the patient when it is safe to leave the access site open to air

A, B, C Feedback: An effective home care teaching program prepares the patient to store solutions, set up the infusion, flush the line with heparin, change the dressings, and troubleshoot for problems. The most common complication is sepsis. Strict aseptic technique is taught for hand hygiene, handling equipment, changing the dressing, and preparing the solution. Sterile water is never used for flushes and the access site must never be left open to air.

The student nurse is preparing a teaching plan for a patient being discharged status post MI. What should the student include in the teaching plan? (Mark all that apply.) A) Need for careful monitoring for cardiac symptoms B) Need for carefully regulated exercis C) Need for dietary modifications D) Need for early resumption of prediagnosis activity E) Need for increased fluid intake

A, B, C Feedback: Dietary modifications, exercise, weight loss, and careful monitoring are important strategies for managing three major cardiovascular risk factors: hyperlipidemia, hypertension, and diabetes. There is no need to increase fluid intake and activity should be slowly and deliberately increased.

A patient with spinal cord injury is ready to be discharged home. A family member asks the nurse to review potential complications one more time. What are the potential complications that should be monitored for in this patient? Select all that apply. A) Orthostatic hypotension B) Autonomic dysreflexia C) DVT D) Salt-wasting syndrome E) Increased ICP

A, B, C Feedback: For a spinal cord-injured patient, based on the assessment data, potential complications that may develop include DVT, orthostatic hypotension, and autonomic dysreflexia. Salt-wasting syndrome or increased ICP are not typical complications following the immediate recovery period.

The patient has just returned to the floor after balloon valvuloplasty of the aortic valve and the nurse is planning appropriate assessments. The nurse should know that complications following this procedure include what? Select all that apply. A) Emboli B) Mitral valve damage C) Ventricular dysrhythmia D) Atrial-septal defect E) Plaque formation

A, B, C Feedback: Possible complications include aortic regurgitation, emboli, ventricular perforation, rupture of the aortic valve annulus, ventricular dysrhythmia, mitral valve damage, and bleeding from the catheter insertion sites. Atrial-septal defect and plaque formation are not complications of a balloon valvuloplasty

The nurse is admitting a patient to the unit who is scheduled for removal of an intracranial mass. What diagnostic procedures might be included in this patients admission orders? Select all that apply. A) Transcranial Doppler flow study B) Cerebral angiography C) MRI D) Cranial radiography E) Electromyelography (EMG)

A, B, C Feedback: Preoperative diagnostic procedures may include a CT scan to demonstrate the lesion and show the degree of surrounding brain edema, the ventricular size, and the displacement. An MRI scan provides information similar to that of a CT scan with improved tissue contrast, resolution, and anatomic definition. Cerebral angiography may be used to study a tumors blood supply or to obtain information about vascular lesions. Transcranial Doppler flow studies are used to evaluate the blood flow within intracranial blood vessels. Regular x-rays of the skull would not be diagnostic for an intracranial mass. An EMG would not be ordered prior to intracranial surgery to remove a ma

A patient has been brought to the emergency department by EMS after being found unresponsive. Rapid assessment reveals anaphylaxis as a potential cause of the patients condition. The care team should attempt to assess for what potential causes of anaphylaxis? Select all that apply. A) Foods B) Medications C) Insect stings D) Autoimmunity E) Environmental pollutants

A, B, C Feedback: Substances that most commonly cause anaphylaxis include foods, medications, insect stings, and latex. Pollutants do not commonly cause anaphylaxis and autoimmune processes are more closely associated with types II and III hypersensitivities.

The nurse is caring for a patient who has just returned to the post-surgical unit following renal surgery. When assessing the patients output from surgical drains, the nurse should assess what parameters? Select all that apply. A) Quantity of output B) Color of the output C) Visible characteristics of the output D) Odor of the output E) pH of the output

A, B, C Feedback: Urine output and drainage from tubes inserted during surgery are monitored for amount, color, and type or characteristics. Odor and pH are not normally assessed.

A patient with pancreatic cancer has been scheduled for a pancreaticoduodenectomy (Whipple procedure). During health education, the patient should be informed that this procedure will involve the removal of which of the following? Select all that apply. A) Gallbladder B) Part of the stomach C) Duodenum D) Part of the common bile duct E) Part of the rectum

A, B, C, D Feedback: A pancreaticoduodenectomy (Whipple procedure or resection) is used for potentially resectable cancer of the head of the pancreas (Fig. 50-7). This procedure involves removal of the gallbladder, a portion of the stomach, duodenum, proximal jejunum, head of the pancreas, and distal common bile duct. The rectum is not affected.

A nurse on the renal unit is caring for a patient who will soon begin peritoneal dialysis. The family of the patient asks for education about the peritoneal dialysis catheter that has been placed in the patients peritoneum. The nurse explains the three sections of the catheter and talks about the two cuffs on the dialysis catheter. What would the nurse explain about the cuffs? Select all that apply. A) The cuffs are made of Dacron polyester. B) The cuffs stabilize the catheter. C) The cuffs prevent the dialysate from leaking. D) The cuffs provide a barrier against microorganisms. E) The cuffs absorb dialysate

A, B, C, D Feedback: Most of these catheters have two cuffs, which are made of Dacron polyester. The cuffs stabilize the catheter, limit movement, prevent leaks, and provide a barrier against microorganisms. They do not absorb dialysate.

The nurse is caring for a patient status after a motor vehicle accident. The patient has developed AKI. What is the nurses role in caring for this patient? Select all that apply. A) Providing emotional support for the family B) Monitoring for complications C) Participating in emergency treatment of fluid and electrolyte imbalances D) Providing nursing care for primary disorder (trauma) E) Directing nutritional interventions

A, B, C, D Feedback: The nurse has an important role in caring for the patient with AKI. The nurse monitors for complications, participates in emergency treatment of fluid and electrolyte imbalances, assesses the patients progress and response to treatment, and provides physical and emotional support. Additionally, the nurse keeps family members informed about the patients condition, helps them understand the treatments, and provides psychological support. Although the development of AKI may be the most serious problem, the nurse continues to provide nursing care indicated for the primary disorder (e.g., burns, shock, trauma, obstruction of the urinary tract). The nurse does not direct the patients nutritional status; the dietician and the physician normally collaborate on directing the patients nutritional status.

An adult patient has sought care for the treatment of headaches that have become increasingly severe and frequent over the past several months. Which of the following questions addresses potential etiological factors? Select all that apply? A) Are you exposed to any toxins or chemicals at work? B) How would you describe your ability to cope with stress? C) What medications are you currently taking? D) When was the last time you were hospitalized? E) Does anyone else in your family struggle with headaches?

A, B, C, E Feedback: Headaches are multifactorial, and may involve medications, exposure to toxins, family history, and stress. Hospitalization is an unlikely contributor to headaches.

A nurse is writing the care plan of a patient who has been diagnosed with myelofibrosis. What nursing diagnoses should the nurse address? Select all that apply. A) Disturbed Body Image B) Impaired Mobility C) Imbalanced Nutrition: Less than Body Requirements D) Acute Confusion E) Risk for Infection

A, B, C, E Feedback: The profound splenomegaly that accompanies myelofibrosis can impact the patients body image and mobility. As well, nutritional deficits are common and the patient is at risk for infection. Cognitive effects are less common

A patient has been discharged home on parenteral nutrition (PN). Much of the nurses discharge education focused on coping. What must a patient on PN likely learn to cope with? Select all that apply. A) Changes in lifestyle B) Loss of eating as a social behavior C) Chronic bowel incontinence from GI changes D) Sleep disturbances related to frequent urination during nighttime infusions E) Stress of choosing the correct PN formulation

A, B, D Feedback: Patients must cope with the loss of eating as a social behavior and with changes in lifestyle brought on by sleep disturbances related to frequent urination during night time infusions. PN is not associated with bowel incontinence and the patient does not select or adjust the formulation of PN

A nurse who provides care on an acute medical unit has observed that physicians are frequently reluctant to refer patients to hospice care. What are contributing factors that are known to underlie this tendency? Select all that apply. A) Financial pressures on health care providers B) Patient reluctance to accept this type of care C) Strong association of hospice care with prolonging death D) Advances in curative treatment in late-stage illness E) Ease of making a terminal diagnosis

A, B, D Feedback: Physicians are reluctant to refer patients to hospice, and patients are reluctant to accept this form of care. Reasons include the difficulties in making a terminal prognosis (especially for those patients with noncancer diagnoses), the strong association of hospice with death, advances in curative treatment options in late-stage illness, and financial pressures on health care providers that may cause them to retain rather than refer hospice-eligible patients.

A patient with ESKD is scheduled to begin hemodialysis. The nurse is working with the patient to adapt the patients diet to maximize the therapeutic effect and minimize the risks of complications. The patients diet should include which of the following modifications? Select all that apply. A) Decreased protein intake B) Decreased sodium intake C) Increased potassium intake D) Fluid restriction E) Vitamin D supplementation

A, B, D Feedback: Restricting dietary protein decreases the accumulation of nitrogenous wastes, reduces uremic symptoms, and may even postpone the initiation of dialysis for a few months. Restriction of fluid is also part of the dietary prescription because fluid accumulation may occur. As well, sodium is usually restricted to 2 to 3 g/day. Potassium intake is usually limited, not increased, and there is no particular need for vitamin D supplementation.

The nurse is assessing a patient with acute coronary syndrome (ACS). The nurse includes a careful history in the assessment, especially with regard to signs and symptoms. What signs and symptoms are suggestive of ACS? Select all that apply. A) Dyspnea B) Unusual fatigue C) Hypotension D) Syncope E) Peripheral cyanosis

A, B, D Feedback: Systematic assessment includes a careful history, particularly as it relates to symptoms: chest pain or discomfort, difficulty breathing (dyspnea), palpitations, unusual fatigue, faintness (syncope), or sweating (diaphoresis). Each symptom must be evaluated with regard to time, duration, and the factors that precipitate the symptom and relieve it, and in comparison with previous symptoms. Hypotension and peripheral cyanosis are not typically associated with ACS.

When hemodynamic monitoring is ordered for a patient, a catheter is inserted into the appropriate blood vessel or heart chamber. When assessing a patient who has such a device in place, the nurse should check which of the following components? Select all that apply. A) A transducer B) A flush system C) A leveler D) A pressure bag E) An oscillator

A, B, D Feedback: To perform hemodynamic monitoring, a CVP, pulmonary artery, or arterial catheter is introduced into the appropriate blood vessel or heart chamber. It is connected to a pressure monitoring system that has several components. Included among these are a transducer, a flush system, and a pressure bag. A pressure monitoring system does not have a leveler or an oscillator.

A patient with an inoperable brain tumor has been told that he has a short life expectancy. On what aspects of assessment and care should the home health nurse focus? Select all that apply. A) Pain control B) Management of treatment complications C) Interpretation of diagnostic tests D) Assistance with self-care E) Administration of treatments

A, B, D, E Feedback: Home care needs and interventions focus on four major areas: palliation of symptoms and pain control, assistance in self-care, control of treatment complications, and administration of specific forms of treatment, such as parenteral nutrition. Interpretation of diagnostic tests is normally beyond the purview of the nurse.

A critical care nurse is caring for a patient with a hemodynamic monitoring system in place. For what complications should the nurse assess? Select all that apply. A) Pneumothorax B) Infection C) Atelectasis D) Bronchospasm E) Air embolism

A, B, E Feedback: Complications from use of hemodynamic monitoring systems are uncommon, but can include pneumothorax, infection, and air embolism. Complications of hemodynamic monitoring systems do not include atelectasis or bronchospasm.

Most individuals who have mitral valve prolapse never have any symptoms, although this is not the case for every patient. What symptoms might a patient have with mitral valve prolapse? Select all that apply. A) Anxiety B) Fatigue C) Shoulder pain D) Tachypnea E) Palpitations

A, B, E Feedback: Most people who have mitral valve prolapse never have symptoms. A few have symptoms of fatigue, shortness of breath, lightheadedness, dizziness, syncope, palpitations, chest pain, and anxiety. Hyperpnea and shoulder pain are not characteristic symptoms of mitral valve prolapse.

A patients assessment and diagnostic testing are suggestive of acute pancreatitis. When the nurse is performing the health interview, what assessment questions address likely etiologic factors? Select all that apply. A) How many alcoholic drinks do you typically consume in a week? B) Have you ever been tested for diabetes? C) Have you ever been diagnosed with gallstones? D) Would you say that you eat a particularly high-fat diet? E) Does anyone in your family have cystic fibrosis?

A, C Feedback: Eighty percent of patients with acute pancreatitis have biliary tract disease such as gallstones or a history of long-term alcohol abuse. Diabetes, high-fat consumption, and cystic fibrosis are not noted etiologic factors.

The results of a patients most recent blood work and physical assessment are suggestive of immune thrombocytopenic purpura (ITP). This patient should undergo testing for which of the following potential causes? Select all that apply. A) Hepatitis B) Acute renal failure C) HIV D) Malignant melanoma E) Cholecystitis

A, C Feedback: Viral illnesses have the potential to cause ITP. Renal failure, malignancies, and gall bladder inflammation are not typical causes of ITP.

The patient has a homocysteine level ordered. What aspects of this test should inform the nurses care? Select all that apply. A) A 12-hour fast is necessary before drawing the blood sample. B) Recent inactivity can depress homocysteine levels. C) Genetic factors can elevate homocysteine levels. D) A diet low in folic acid elevates homocysteine levels. E) An ECG should be performed immediately before drawing a sample.

A, C, D Feedback: Genetic factors and a diet low in folic acid, vitamin B6 , and vitamin B12 are associated with elevated homocysteine levels. A 12-hour fast is necessary before drawing a blood sample for an accurate serum measurement. An ECG is unnecessary and recent inactivity does not influence the results of the test.

A patients physician has ordered a liver panel in response to the patients development of jaundice. When reviewing the results of this laboratory testing, the nurse should expect to review what blood tests? Select all that apply. A) Alanine aminotransferase (ALT) B) C-reactive protein (CRP) C) Gamma-glutamyl transferase (GGT) D) Aspartate aminotransferase (AST) E) B-type natriuretic peptide (BNP)

A, C, D Feedback: Liver function testing includes GGT, ALT, and AST. CRP addresses the presence of generalized inflammation and BNP is relevant to heart failure; neither is included in a liver panel.

The nurse is creating a care plan for a patient diagnosed with HF. When addressing the problem of anxiety, what interventions should the nurse include in the care plan? Select all that apply. A) Facilitate the presence of friends and family whenever possible. B) Teach the patient about the harmful effects of anxiety on cardiac function. C) Provide supplemental oxygen, as needed. D) Provide validation of the patients expressions of anxiety. E) Administer benzodiazepines two to three times daily.

A, C, D Feedback: The nurse should empathically validate the patients sensations of anxiety. The presence of friends and family are frequently beneficial and oxygen supplementation promotes comfort. Antianxiety medications may be necessary for some patients, but alternative methods of relief should be prioritized. As well, medications are administered on a PRN basis. Teaching the patient about the potential harms of anxiety is likely to exacerbate, not relieve, the problem.

An oncology nurse recognizes a patients risk for fluid imbalance while the patient is undergoing treatment for leukemia. What relevant assessments should the nurse include in the patients plan of care? Select all that apply. A) Monitoring the patients electrolyte levels B) Monitoring the patients hepatic function C) Measuring the patients weight on a daily basis D) Measuring and recording the patients intake and output E) Auscultating the patients lungs frequently

A, C, D, E Feedback: Assessments that relate to fluid balance include monitoring the patients electrolytes, auscultating the patients chest for adventitious sounds, weighing the patient daily, and closely monitoring intake and output. Liver function is not directly relevant to the patients fluid status in most cases.

The nurse caring for a patient in a persistent vegetative state is regularly assessing for potential complications. Complications of neurologic dysfunction for which the nurse should assess include which of the following? Select all that apply. A) Contractures B) Hemorrhage C) Pressure ulcers D) Venous thromboembolism E) Pneumonia

A, C, D, E Feedback: Based on the assessment data, potential complications may include respiratory distress or failure, pneumonia, aspiration, pressure ulcer, deep vein thrombosis (DVT), and contractures. The pathophysiology of decreased LOC does not normally create a heightened risk for hemorrhage.

The nurse providing care for a patient post PTCA knows to monitor the patient closely. For what complications should the nurse monitor the patient? Select all that apply. A) Abrupt closure of the coronary artery B) Venous insufficiency C) Bleeding at the insertion site D) Retroperitoneal bleeding E) Arterial occlusion

A, C, D, E Feedback: Complications after the procedure may include abrupt closure of the coronary artery and vascular complications, such as bleeding at the insertion site, retroperitoneal bleeding, hematoma, and arterial occlusion, as well as acute renal failure. Venous insufficiency is not a postprocedure complication of a PTCA.

A patient with end-stage liver disease has developed hypervolemia. What nursing interventions would be most appropriate when addressing the patients fluid volume excess? Select all that apply. A) Administering diuretics B) Administering calcium channel blockers C) Implementing fluid restrictions D) Implementing a 1500 kcal/day restriction E) Enhancing patient positioning

A, C, E Feedback: Administering diuretics, implementing fluid restrictions, and enhancing patient positioning can optimize the management of fluid volume excess. Calcium channel blockers and calorie restriction do not address this problem

A nurse is caring for a patient in the late stages of esophageal cancer. The nurse should plan to prevent or address what characteristics of this stage of the disease? Select all that apply. A) Perforation into the mediastinum B) Development of an esophageal lesion C) Erosion into the great vessels D) Painful swallowing E) Obstruction of the esophagus

A, C, E Feedback: In the later stages of esophageal cancer, obstruction of the esophagus is noted, with possible perforation into the mediastinum and erosion into the great vessels. Painful swallowing and the emergence of a lesion are early signs of esophageal cancer.

During a patients recovery from stroke, the nurse should be aware of predictors of stroke outcome in order to help patients and families set realistic goals. What are the predictors of stroke outcome? Select all that apply. A) National Institutes of Health Stroke Scale (NIHSS) score B) Race C) LOC at time of admission D) Gender E) Age

A, C, E Feedback: It is helpful for clinicians to be knowledgeable about the relative importance of predictors of stroke outcome (age, NIHSS score, and LOC at time of admission) to provide stroke survivors and their families with realistic goals. Race and gender are not predictors of stroke outcome.

A patient is in the acute phase of a burn injury. One of the nursing diagnoses in the plan of care is Ineffective Coping Related to Trauma of Burn Injury. What interventions appropriately address this diagnosis? Select all that apply. A) Promote truthful communication. B) Avoid asking the patient to make decisions. C) Teach the patient coping strategies. D) Administer benzodiazepines as ordered. E) Provide positive reinforcement.

A, C, E Feedback: The nurse can assist the patient to develop effective coping strategies by setting specific expectations for behavior, promoting truthful communication to build trust, helping the patient practice appropriate strategies, and giving positive reinforcement when appropriate. The patient may benefit from being able to make decisions regarding his or her care. Benzodiazepines may be needed for short-term management of anxiety, but they are not used to enhance coping.

The nurse is planning the care of a patient with HF. The nurse should identify what overall goals of this patients care? A) Improve functional status B) Prevent endocarditis. C) Extend survival. D) Limit physical activity. E) Relieve patient symptoms.

A, C, E Feedback: The overall goals of management of HF are to relieve the patients symptoms, to improve functional status and quality of life, and to extend survival. Activity limitations should be accommodated, but reducing activity is not a goal. Endocarditis is not a common complication of HF and preventing it is not a major goal of care.

A patient admitted to the ED with severe diarrhea and vomiting is subsequently diagnosed with food poisoning. The nurse caring for this patient assesses for signs and symptoms of fluid and electrolyte imbalances. For what signs and symptoms would this nurse assess? Select all that apply. A) Dysrhythmias B) Hypothermia C) Hypotension D) Hyperglycemia E) Delirium

A, C, E Feedback: The patient is assessed for signs and symptoms of fluid and electrolyte imbalances, including lethargy, rapid pulse rate, fever, oliguria, anuria, hypotension, and delirium. Hyperglycemia and hypothermia are not typically associated with fluid and electrolyte imbalances.

The nurse is performing an admission assessment on a client with a diagnosis of angina pectoris who takes nitroglycerin for chest pain at home. During the assessment, the client complains of chest pain. The nurse should immediately ask the client which question? A. "Where is the pain located?" B. "Are you having any nausea?" C. "Are you allergic to any medications?" D. "Do you have your nitroglycerin with you?"

A. "Where is the pain located?" If a client complains of chest pain, the initial assessment question would be to ask the client about the pain intensity, location, duration, and quality. Although options 2, 3, and 4 all may be components of the assessment, none of these questions would be the initial assessment question with this client.

The nurse is performing an assessment on a client with a diagnosis of left-sided heart failure. Which assessment component would elicit specific information regarding the client's left-sided heart function? A. Listening to lung sounds B. Monitoring for organomegaly C. Assessing for jugular vein distention D. Assessing for peripheral and sacral edema

A. Listening to lung sounds The client with heart failure may present with different symptoms, depending on whether the right or the left side of the heart is failing. Peripheral and sacral edema, jugular vein distention, and organomegaly all are manifestations of problems with right-sided heart function. Lung sounds constitute an accurate indicator of left-sided heart function.

The nurse is preparing a presentation about Beck's cognitive theory. Which cognitive distortion would the nurse include in the teaching session? 1. Negative expectation of the environment 2. Negative expectation of the present 3. Negative expectation of the career 4. Negative expectation of the family

ANS: 1

Which information should the nurse include when teaching parents who have children or adolescents with symptoms of bipolar disorder? (Select all that apply.) 1. First-line treatment is a second-generation antipsychotic. 2. In children and adolescents with bipolar disorder there is a high risk of relapse. 3. There is a direct link between development of bipolar disorder and attention deficit/hyperactivity disorder. 4. The diagnosis of bipolar disorder cannot be assigned prior to the age of 18. 5. Genetic predisposition is not a reliable diagnostic determinant.

ANS: 1, 2

The nurse is teaching about the diagnosis disruptive mood dysregulation disorder (DMDD). Which information should the nurse include? (Select all that apply.) 1. Symptoms include verbal rages or physical aggression toward people or property. 2. Temper outbursts must be present in at least two settings (at home, at school, or with peers). 3. DMDD is characterized by severe recurrent temper outbursts. 4. The temper outbursts are manifested only behaviorally. 5. Symptoms of DMDD must be present for 18 or more months to meet diagnostic criteria.

ANS: 1, 2, 3

Which information would the nurse include in a teaching session about predisposing factors in the development of bipolar disorder, mania episode? (Select all that apply.) 1. There are excessive levels of glutamate. 2. Mania has greater right-side reduction in brain activity. 3. Steroids are the most common medication to trigger a manic response. 4. Expression of gene ANK3 is decreased in manic episodes. 5. Dopamine levels are decreased in mania.

ANS: 1, 2, 3

Which statements represent positive outcomes for clients diagnosed with narcissistic personality disorder? (Select all that apply.) 1. The client will relate one empathetic statement to another client in group by day four. 2. The client will identify one personal limitation by day two. 3. The client will acknowledge one strength that another client possesses by day three. 4. The client will list four personal strengths by day three. 5. The client will discuss two lifetime achievements by discharge.

ANS: 1, 2, 3

The clinic nurse is caring for a client with ulcerative colitis who has signs of depression. Which additional conditions should the nurse assess for in this client? (Select all that apply.) 1. Mania 2. Cardiovascular disease 3. Metabolic syndrome 4. Diabetes 5. Emphysema

ANS: 1, 2, 3, 4

The diagnosis of catatonic disorder due to another medical condition is made when the client's medical history, physical examination, or laboratory findings provide evidence that symptoms are directly attributed to which conditions? (Select all that apply.) 1. Epilepsy 2. Hypothyroidism 3. Hyperadrenalism 4. Encephalitis 5. Hyperaphia

ANS: 1, 2, 3, 4

The nurse is assessing a client for antisocial personality disorder. According to the DSM-5, which symptoms must the client meet in order to be assigned this diagnosis? (Select all that apply.) 1. Age of at least 18 years old 2. Deceitful for personal gain 3. Frequent feelings of being down, remorseful, or hopeless 4. Disregard for and failure to honor financial obligations 5. Avoidance of social events and interaction with others

ANS: 1, 2, 4

Which nursing statements exemplify the process that must be completed by a nurse in the pre-introductory phase prior to caring for clients diagnosed with a substance-related disorder? (Select all that apply.) 1. "I am easily manipulated and need to work on this prior to caring for these clients." 2. "Because of my father's alcoholism, I need to examine my attitude toward these clients." 3. "I need to review the side effects of the medications used in the withdrawal process." 4. "I'll need to set boundaries to maintain a therapeutic relationship." 5. "I need to take charge when dealing with clients diagnosed with substance disorders."

ANS: 1, 2, 4

Which modalities should a nurse recognize as integral parts of a treatment program when planning care for clients diagnosed with schizophrenia spectrum disorder? (Select all that apply.) 1. Group therapy 2. Medication management 3. Deterrent therapy 4. Supportive family therapy 5. Social skills training

ANS: 1, 2, 4, 5

The nurse is caring for a client diagnosed with antisocial personality disorder. Which factors should the nurse consider when planning this client's care? (Select all that apply.) 1. This client has personality traits that are deeply ingrained and difficult to modify. 2. This client needs medication to treat the underlying physiological pathology. 3. This client uses manipulation, making the implementation of treatment problematic. 4. This client wants instant gratification, which hinders compliance with a plan of care. 5. This client is likely to have secondary diagnoses of substance abuse and depression.

ANS: 1, 3, 4, 5

A nurse is about to meet with a client suffering from codependency. Which data would the nurse expect to find during the assessment of this client? (Select all that apply.) 1. Has a long history of focusing thoughts and behaviors on other people 2. As a child, experienced overindulgent and overprotective parents 3. Is a people pleaser and does almost anything to gain approval 4. Exhibits helpless behaviors but actually feels very competent 5. Can achieve a sense of control through fulfilling the needs of others

ANS: 1, 3, 5

A nursing supervisor is offering an impaired staff member information regarding a peer assistance program. Which facts should the supervisor include? (Select all that apply.) 1. A hot-line number will be available in order to call for help. 2. A verbal contract detailing the method of treatment will be initiated prior to the program. 3. Peer support is provided through regular contact with the impaired nurse. 4. Contact to provide peer support will last for one year. 5. One of the program goals is to intervene early in order to reduce hazards to clients.

ANS: 1, 3, 5

The client is diagnosed with illness anxiety disorder (IAD). Which symptoms would the client most likely exhibit? (Select all that apply.) 1. Doctor shopping 2. Pseudocyesis 3. Anxiety 4. Flat affect 5. Avoids hospitals

ANS: 1, 3, 5

The client is exhibiting symptoms of generalized amnesia. Which questions should the nurse ask to help confirm this diagnosis? (Select all that apply.) 1. "Have you taken any new medications recently?" 2. "Have you recently traveled away from home?" 3. "Have you recently experienced any traumatic event?" 4. "Have you ever felt detached from your environment?" 5. "Have you had any history of memory problems?"

ANS: 1, 3, 5

The nurse is admitting a client with a diagnosis of schizotypal personality disorder. Which client findings would make the nurse question this diagnosis? (Select all that apply.) 1. Is the center of attention 2. Has unusual perceptual experiences 3. Has a bipolar disorder 4. Is odd and eccentric but not delusional 5. Has autism spectrum disorder

ANS: 1, 3, 5

Which practices should a nurse describe to a client as being incorporated during yoga therapy? (Select all that apply.) 1. Deep breathing 2. Meridian flow 3. Balanced body postures 4. Massage 5. Meditation

ANS: 1, 3, 5

An older adult client has a diagnosis of dysthymic disorder. Which signs and symptoms should the nurse expect the client to exhibit? (Select all that apply.) 1. Sad mood on most days 2. Mood rating of 2 out of 10 for the past 6 months 3. Labile mood 4. Sad mood for the past 3 years after spouse's death 5. Pressured speech when communicating

ANS: 1, 4

The client has been diagnosed with generalized anxiety disorder (GAD). Which symptoms would the nurse observe upon assessment? (Select all that apply.) 1. Muscle tension 2. Paresthesia 3. Hyperventilation 4. Restlessness 5. Procrastination

ANS: 1, 4, 5

Which guidelines from the U.S. Departments of Agriculture and Health and Human Services would the nurse use to promote health and prevent disease in clients? (Select all that apply.) 1. Consume less than 300 mg per day of dietary cholesterol. 2. Limit total calorie intake to 2,000 calories per day. 3. Reduce daily sodium intake to 3,000 mg per day. 4. Consume less than 10 percent of calories from saturated fatty acids. 5. Limit alcohol consumption to one drink per day for women and two drinks per day for men.

ANS: 1, 4, 5

After 5 months of taking nortriptyline (Aventyl) for depressive symptoms, a client reports that the medication doesn't seem as effective as before. Which question should the nurse ask to determine the cause of this problem? 1. "Are you consuming foods high in tyramine?" 2. "How many packs of cigarettes do you smoke daily?" 3. "Do you drink any alcohol?" 4. "When did you last eat yogurt?"

ANS: 2

The depressed client is receiving light therapy. Which instruction would the nurse share with the client? 1. "White LED lights will be used with protective glasses to block ultraviolet rays." 2. "You will sit in front of the light box with your eyes open." 3. "The light sessions will start out at 5 minutes and work up to 30 minute intervals." 4. "Vagal stimulation from the light waves will help release melatonin in the brain."

ANS: 2

The nurse is caring for a client with a postpartum emotional disorder. Which postpartum disorder is correctly matched with its presenting symptoms? 1. Baby blues (lack of concentration, agitation, guilt, and an abnormal attitude toward bodily functions) 2. Moderate postpartum depression (irritability, loss of libido, sleep disturbances, expresses concern about inability to care for baby) 3. Maternity blues (overprotection of infant, severe guilt, depressed mood, lack of concentration) 4. Postpartum depression with psychotic features (transient depressed mood, decisive, abnormal fear of child abduction, suicidal ideations)

ANS: 2

The nurse is caring for a client with major depressive disorder who is withdrawn, uncommunicative, and secludes self in room. Which nursing diagnosis should the nurse add to the plan of care? 1. Spiritual distress 2. Social isolation 3. Low self-esteem 4. Powerlessness

ANS: 2

The nurse is teaching the depressed client about bupropion (Wellbutrin). Which statement by the client indicates effective teaching? 1. "I will begin to wear short sleeves when outdoors." 2. "I will not take two pills if I miss a dose." 3. "I will discontinue the medication when my depression is gone." 4. "I will stand up smoothly and quickly to keep my balance."

ANS: 2

The nurse performs a full physical health assessment on an older adult client admitted with a diagnosis of major depressive disorder. What is the rationale for the nurse's assessment? 1. The attention during the assessment is beneficial in decreasing social isolation in the elderly. 2. Depression can generate somatic symptoms that can mask actual physical disorders. 3. Physical health complications are likely to arise from antidepressant therapy. 4. Depressed geriatric clients avoid addressing physical health and ignore medical problems.

ANS: 2

Which characteristic would help a nurse distinguish between dysthymia and major depressive disorder (MDD)? 1. Dysthymia is associated with the menstrual cycle. 2. Dysthymia is a chronically depressed mood. 3. MDD lasts for at least 2 years. 4. MDD does not have delusions or hallucinations.

ANS: 2

Which scale would a nurse practitioner use to assess a depressed client? 1. Zung Depression Scale 2. Hamilton Depression Rating Scale 3. Beck Depression Inventory 4. AIMS Depression Rating Scale

ANS: 2

The client who prefers to use St. John's wort and psychotherapy in lieu of medication therapy asks for tips on using St. John's wort. Which teaching points should the nurse provide? (Select all that apply.) 1. Select a reputable brand that provides 1200 mg/day. 2. Take with food to lessen the GI irritation. 3. Monitor for adverse reactions. 4. Do not take with an anticoagulant. 5. Stay in the sun for at least 30 minutes.

ANS: 2, 3

The nurse is discussing treatment options with a client who has arachnophobia. Which commonly used behavioral therapies for phobias should the nurse explain to the client? (Select all that apply.) 1. Benzodiazepine therapy 2. Systematic desensitization 3. Imploding (flooding) 4. Competing response training 5. Habit reversal training

ANS: 2, 3

A nursing supervisor is about to meet with a staff nurse suspected of diverting clients' pain medications. Which assessment data would lead the supervisor to suspect that the staff nurse is impaired? (Select all that apply.) 1. Is frequently absent from work 2. Experiences mood swings 3. Makes elaborate excuses for behavior 4. Frequently uses the restroom 5. Has a flushed face

ANS: 2, 3, 4, 5

The client experiences sadness and melancholia in September continuing through November. Which factors should a nurse identify as most likely to contribute to the etiology of these symptoms? (Select all that apply.) 1. Gender differences in social opportunities 2. Increased production of melatonin 3. Hyposecretion of cortisol 4. Less exposure to natural sunlight 5. Blockade of histamine reuptake

ANS: 2, 4

A nursing instructor is teaching nursing students about cirrhosis of the liver. Which statements by nursing students about hepatic encephalopathy indicate successful teaching? (Select all that apply.) 1. "A diet rich in protein will promote hepatic healing." 2. "This condition results from a rise in serum ammonia, leading to impaired mental functioning." 3. "In this condition, an excessive amount of serous fluid accumulates in the abdominal cavity." 4. "Neomycin and lactulose are used in the treatment of this condition." 5. "This condition is caused by the inability of the liver to convert ammonia to urea."

ANS: 2, 4, 5

A client is prescribed transdermal selegiline (Emsam) for depressive symptoms. Which action would the nurse take to administer this medication? 1. Apply new patch to the lower abdomen. 2. Apply new patch to inner surface of upper arm. 3. Place new patch on dry, intact skin. 4. Place direct heat to new patch for a tight seal.

ANS: 3

The client with major depressive episode is experiencing command hallucination for self-harm. Which intervention should be the nurse's priority at this time? 1. Obtaining an order for locked seclusion until client is no longer suicidal 2. Conducting 15-minute checks to ensure safety 3. Placing the client on one-to-one observation while continuing to monitor suicidal ideations 4. Encouraging client to express feelings related to suicide

ANS: 3

The nurse discovers a client has a history of divorce, job loss, family estrangement, and cocaine abuse. Which theory explains the etiology of this client's depressive symptoms? 1. Psychoanalytic theory 2. Object loss theory 3. Learning theory 4. Cognitive theory

ANS: 3

The nurse is assisting with electroconvulsive therapy (ECT). What is the rationale for administering 100% oxygen to a client during and after ECT? 1. To prevent brain damage from the electrical impulse of the procedure 2. To prevent decreased blood pressure, pulse, and respiration owing to electrical stimulation 3. To prevent anoxia resulting from medication-induced paralysis of respiratory muscles 4. To prevent blocked airway, resulting from seizure activity

ANS: 3

A client is taking phenelzine (Nardil). Which statement by the client should cause the nurse to intervene? 1. "I cannot use over-the-counter medications for my colds." 2. "I have to cut out eating my raisin bran every morning." 3. "I will have to avoid pepperoni pizza when eating with my friends." 4. "I am taking diet pills to lose weight for my friend's wedding."

ANS: 4

The nurse determines that a depressed client is using the cognitive distortion of "automatic thoughts." Which client statement is evidence of the "automatic thought" of discounting positives? 1. "It's all my fault for trusting him." 2. "I don't play games. I never win." 3. "She never visits, because she thinks I don't care." 4. "Growing plants is so easy. Any old fool can grow a rose."

ANS: 4

The nurse is preparing an antidepressant medication for a 13-year-old client who is experiencing major depressive disorder. Which FDA-approved medication should the nurse administer? 1. Paroxetine (Paxil) 2. Sertraline (Zoloft) 3. Citalopram (Celexa) 4. Escitalopram (Lexapro)

ANS: 4

A 70-kg adult with chronic renal failure is on a 40-g protein diet. The client has a reduced glomerular filtration rate and is not undergoing dialysis. Which result would give the nurse the most concern? a. Albumin level of 2.5 g/dL b. Phosphorus level of 5 mg/dL c. Sodium level of 135 mmol/L d. Potassium level of 5.5 mmol/L

ANS: A Protein restriction is necessary with chronic renal failure due to the buildup of waste products from protein breakdown. The nurse would be concerned with the low albumin level since this indicates that the protein in the diet is not enough for the client's metabolic needs. The electrolyte values are not related to the protein-restricted diet.

A client has a serum potassium level of 6.5 mmol/L, a serum creatinine level of 2 mg/dL, and a urine output of 350 mL/day. What is the best action by the nurse? a. Place the client on a cardiac monitor immediately. b. Teach the client to limit high-potassium foods .c. Continue to monitor the client's intake and output. d. Ask to have the laboratory redraw the blood specimen.

ANS: A The priority action by the nurse should be to check the cardiac status with a monitor. High potassium levels can lead to dysrhythmias. The other choices are logical nursing interventions for acute kidney injury but not the best immediate action.

A client with chronic kidney disease (CKD) is experiencing nausea, vomiting, visual changes, and anorexia. Which action by the nurse is best? a. Check the client's digoxin (Lanoxin) level. b. Administer an anti-nausea medication. c. Ask if the client is able to eat crackers. d. Get a referral to a gastrointestinal provider.

ANS: A These signs and symptoms are indications of digoxin (Lanoxin) toxicity. The nurse should check the level of this medication. Administering antiemetics, asking if the client can eat, and obtaining a referral to a specialist all address the client's symptoms but do not lead to the cause of the symptoms.

A marathon runner comes into the clinic and states "I have not urinated very much in the last few days." The nurse notes a heart rate of 110 beats/min and a blood pressure of 86/58 mm Hg. Which action by the nurse is the priority? a. Give the client a bottle of water immediately. b. Start an intravenous line for fluids. c. Teach the client to drink 2 to 3 liters of water daily. d. Perform an electrocardiogram.

ANS: A This athlete is mildly dehydrated as evidenced by the higher heart rate and lower blood pressure. The nurse can start hydrating the client with a bottle of water first, followed by teaching the client to drink 2 to 3 liters of water each day. An intravenous line may be ordered later, after the client's degree of dehydration is assessed. An electrocardiogram is not necessary at this time.

A client is receiving total parenteral nutrition (TPN). On assessment, the nurse notes the clients pulse is 128 beats/min, blood pressure is 98/56 mm Hg, and skin turgor is dry. What action should the nurse perform next? a. Assess the 24-hour fluid balance. b. Assess the clients oral cavity. c. Prepare to hang a normal saline bolus. d. Turn up the infusion rate of the TPN.

ANS: AThis client has clinical indicators of dehydration, so the nurse calculates the clients 24-hour intake, output, and fluid balance. This information is then reported to the provider. The clients oral cavity assessment may or may not be consistent with dehydration. The nurse may need to give the client a fluid bolus, but not as an independent action. The clients dehydration is most likely due to fluid shifts from the TPN, so turning up the infusion rate would make the problem worse, and is not done as an independent action.

A nurse assesses a client with atrial fibrillation. Which manifestation should alert the nurse to the possibility of a serious complication from this condition? a. Sinus tachycardia b. Speech alterations c. Fatigue d. Dyspnea with activity

ANS: B Clients with atrial fibrillation are at risk for embolic stroke. Evidence of embolic events includes changes in mentation, speech, sensory function, and motor function. Clients with atrial fibrillation often have a rapid ventricular response as a result. Fatigue is a nonspecific complaint. Clients with atrial fibrillation often have dyspnea as a result of the decreased cardiac output caused by the rhythm disturbance.DIF: Applying/Application REF: 666KEY: Cardiac electrical conduction| vascular perfusionMSC: IntegratedProcess:NursingProcess:AssessmentNOT: Client Needs Category: Physiological Integrity: Reduction of Risk Potential

The charge nurse of the medical-surgical unit is making staff assignments. Which staff member should be assigned to a client with chronic kidney disease who is exhibiting a low-grade fever and a pericardial friction rub? a. Registered nurse who just floated from the surgical unit b. Registered nurse who just floated from the dialysis unit c. Registered nurse who was assigned the same client yesterday d. Licensed practical nurse with 5 years' experience on this floor

ANS: C The client is exhibiting symptoms of pericarditis, which can occur with chronic kidney disease. Continuity of care is important to assess subtle differences in clients. Therefore, the registered nurse (RN) who was assigned to this client previously should again give care to this client. The float nurses would not be as knowledgeable about the unit and its clients. The licensed practical nurse may not have the education level of the RN to assess for pericarditis.

A nurse reviews these laboratory values of a client who returned from kidney transplantation 12 hours ago:Sodium 136 mEq/LPotassium 5 mEq/LBlood urea nitrogen (BUN) 44 mg/dLSerum creatinine 2.5 mg/dL What initial intervention would the nurse anticipate? a. Start hemodialysis immediately. b. Discuss the need for peritoneal dialysis. c. Increase the dose of immunosuppression. d. Return the client to surgery for exploration

ANS: C The client may need a higher dose of immunosuppressive medication as evidenced by the elevated BUN and serum creatinine levels. This increased dose may reverse the possible acute rejection of the transplanted kidney. The client does not need hemodialysis, peritoneal dialysis, or further surgery at this point.

A client has been treated for a deep vein thrombus and today presents to the clinic with petechiae. Laboratory results show a platelet count of 42,000/mm3. The nurse reviews the clients medication list to determine if the client is taking which drug? a. Enoxaparin (Lovenox) b. Salicylates (aspirin) c. Unfractionated heparin d. Warfarin (Coumadin)

ANS: C This client has manifestations of heparin-induced thrombocytopenia. Enoxaparin, salicylates, and warfarin do not cause this condition.

A client is brought to an emergency department by police after threatening to jump off a bridge several hours ago. To assess for suicide potential, which question would a nurse ask first? 1. "Are you currently thinking about harming yourself?" 2. "Why do you want to harm yourself?" 3. "Have you thought about the consequences of your actions?" 4. "Who is your emergency contact person?"

1 The nurse would first assess the client for current harmful or suicidal thoughts to minimize risk of harm to the client and provide appropriate interventions. A suicidal client is experiencing a psychiatric emergency. The crisis team's priority is to assess client safety.

Which of the following interventions would a nurse use when caring for an inpatient client who expresses anger inappropriately? (Select all that apply.) 1. Maintain a calm demeanor. 2. Clearly delineate the consequences of the behavior. 3. Use therapeutic touch to convey empathy. 4. Set limits on the behavior. 5. Teach the client to avoid "I" statements related to expression of feelings.

1, 2, 4

The client diagnosed with borderline personality disorder brings up a conflict with the staff in a community meeting and develops a following of clients who unreasonably demand modification of unit rules. How can the nursing staff best handle this situation? 1. Allow the clients to apply the autocratic process when developing unit rules. 2. Maintain consistency of care and open communication to avoid staff manipulation. 3. Allow the client spokesman to verbalize concerns during a unit staff meeting. 4. Maintain unit order by the application of punitive leadership.

2 The nursing staff can best handle this situation by maintaining consistency of care by open communication to avoid staff manipulation. Clients with borderline personality disorder can exhibit negative patterns of interaction, such as clinging and distancing, splitting, manipulation, and self-destructive behaviors.

A nurse is caring for a client who is scheduled for a dose of cefazolin and vitamins at this time. Hemodialysis for this client is also scheduled in 60 minutes. Which action by the nurse is best? a. Administer cefazolin since the level of the antibiotic must be maintained b. Hold the vitamins but administer the cefazolin. c. Hold the cefazolin but administer the vitamins. d. Hold all medications since both cefazolin and vitamins are dialyzable.

ANS: D Both the cefazolin and the vitamins should be held until after the hemodialysis is completed because they would otherwise be removed by the dialysis process.

A nurse understands that when a practitioner corrects subluxation by manipulating the vertebrae of the spinal column the practitioner is employing which therapy? A. Allopathic therapy B. Therapeutic touch therapy C. Massage therapy D. Chiropractic therapy

ANS: D Chiropractic therapy involves the correction of subluxations by manipulating the vertebrae of the spinal column. The theory behind chiropractic medicine is that energy flows from the brain to all parts of the body through the spinal cord and spinal nerves.

A despondent client who has recently lost her husband of 30 years tearfully states, "I'll feel a lot better if I sell my house and move away." Which nursing response is most appropriate? 1. "I'm confident you know what's best for you." 2. "This may not be the best time for you to make such an important decision." 3. "Your children will be terribly disappointed." 4. "Tell me why you want to make this change."

2 During crisis intervention, the nurse would guide the client through a problem-solving process. The nurse would help the individual confront the source of the problem, encourage the individual to discuss changes he or she would like to make, and encourage exploration of feelings about aspects of the crisis that cannot be changed.

A wife brings her husband to an emergency department (ED) after an attempt to hang himself. He is a full-time student and works 8 hours at night to support his family. He states, "I can't function any longer under all this stress." Which type of crisis is the client experiencing? 1. Maturational/developmental crisis 2. Psychiatric emergency crisis 3. Anticipated life transition crisis 4. Traumatic stress crisis

2 The nurse would determine that the client is experiencing a psychiatric emergency crisis. Psychiatric emergencies occur when crisis situations result in severe impairment, incompetence, or an inability to assume personal responsibility.

The nurse is assessing a client diagnosed with somatic symptom disorder (SSD). Which findings would the nurse expect to observe? 1. Presence of multiple personalities, depersonalization, derealization, and "gaps" in memory 2. Aphonia, la belle indifference, paralysis with no physical reason, and possible hallucinations 3. Anxious, seeing several health-care providers simultaneously, overmedicates, and vague symptoms 4. Pretends to be ill, aggravates existing symptoms, inflicts self-injury and has many hospitalizations

3 These findings indicate somatic symptom disorder. Clients often receive medical care from several health-care providers, sometimes concurrently, leading to the possibility of dangerous combinations of treatments. They tend to seek relief through overmedicating with prescribed analgesics or antianxiety agents. Symptoms may be vague, dramatized, or exaggerated in their presentation.

Which statement by the client indicates successful teaching about taking lithium carbonate (Lithobid) for the treatment of bipolar disorder? 1. "I should decrease my intake of sodium." 2. "Drinking ten large glasses of water a day is good for me." 3. "Weight gain is a common, but troubling, side effect." 4. "Diarrhea should be expected while using this drug."

3 This statement indicates successful teaching. Weight gain is a common side effect of lithium carbonate.

The health-care provider prescribes lithium carbonate (Eskalith) and olanzapine (Zyprexa) for a client with severe manic episodes. The client's spouse asks the nurse how Zyprexa works. Which response would the nurse make? 1. "Zyprexa in combination with Eskalith cures bipolar disorder symptoms." 2. "Zyprexa prevents extrapyramidal side effects." 3. "Zyprexa increases the effectiveness of the immune system." 4. "Zyprexa calms the agitation associated with mania."

4 The nurse should explain to the client's spouse that olanzapine can calm agitation associated with mania.

A college student, who was nearly raped while out jogging, completes a series of appointments with a rape crisis nurse. At the final session, which client statement most clearly suggests that the goals of crisis intervention have been met? 1. "You've really been helpful. Can I count on you for continued support?" 2. "I work out in the college gym rather than jogging outdoors." 3. "I'm really glad I didn't go home. It would have been hard to come back." 4. "I carry mace when I jog. It makes me feel safe and secure."

4 The nurse would evaluate that the client who has developed adaptive coping strategies has achieved the goals of crisis intervention. The final phase of crisis intervention involves evaluating the outcome of the crisis intervention and anticipatory planning.

A client comes to a psychiatric clinic experiencing sudden extreme fatigue, decreased sleep, and decreased appetite. The client works 12 hours a day and rates anxiety as 8/10 on a numeric scale. Which long-term outcome is realistic in addressing this client's crisis? 1. The client will change his type-A personality traits to more adaptive ones within one week. 2. The client will list five positive self-attributes. 3. The client will examine how childhood events led to his overachieving orientation. 4. The client will return to previous adaptive levels of functioning by week six.

4 The nurse would identify that a realistic long-term outcome for this client is to return to previous adaptive levels of functioning. The nurse would work with the client to develop attainable outcomes that reflect immediacy of the situation.

A nurse is assessing a patient who has been diagnosed with cholecystitis, and is experiencing localized abdominal pain. When assessing the characteristics of the patients pain, the nurse should anticipate that it may radiate to what region? A) Left upper chest B) Inguinal region C) Neck or jaw D) Right shoulder

D Feedback: The patient may have biliary colic with excruciating upper right abdominal pain that radiates to the back or right shoulder. Pain from cholecystitis does not typically radiate to the left upper chest, inguinal area, neck, or jaw.

which practices should a nurse prescribe to a client as being incorporated during yoga therapy (SATA) 1. Meridian therapy 2. Deep breathing 3. Balanced body postures 4. Massage therapy 5. Meditation

Deep breathing .balanced body postures Meditation

____________________ disorder is manifested by signs and symptoms of schizophrenia, along with a strong element of symptomatology associated with the mood disorders (depression or mania).

Schizoaffective

A nurse assists with the cardioversion of a client experiencing acute atrial fibrillation. Which action should the nurse take prior to the initiation of cardioversion? a. postion the client on the left side b. administer intravenous adenosine c. turn off oxygen d. ensure tongue blade is available

TURN OFF OXYGEN

A client with a diagnosis of angina pectoris is hospitalized for an angioplasty. The client returns to the nursing unit after the procedure and the nurse provides instructions to the client regarding home measures. Which statement if made by the client indicates an understanding of the instructions? a. i need to adhere to my dietary restrictions b. i need to cut down on cigarette smoking c. i am so relieved that my heart is repaired d. i am so relieved I can eat anything I want to now

a. i need to adhere to my dietary restrictions

The concept of ____________________ arose out of a need to define the dysfunctional behaviors that are evident among members of the family of a chemically dependent person.

codependency

Interventions that are different from, but used in conjunction with, traditional or conventional medical treatment are termed ____________________ medicine

complementary

The clinic nurse is triaging clients. The nurse should require which client with nonsuicidal self-injuring behavior to be seen immediately? 1. Is self-cutting in response to command hallucinations 2. Has a history of borderline personality disorder 3. Is on leave from the military 4. Has thoughts of being detached from the body

1 A client who exhibits nonsuicidal self-injuring behavior (self-cutting) in response to command hallucinations should be considered in need of immediate medical attention.

Neurological tests have ruled out pathology in a client's sudden lower-extremity paralysis. Which action would the nurse take? 1. Deal with physical symptoms in a detached manner. 2. Challenge the validity of physical symptoms. 3. Meet dependency needs until the physical limitations subside. 4. Encourage a discussion of feelings about the lower-extremity problem.

1 The nurse should assist the client in dealing with physical symptoms in a detached manner. The nurse should minimize time given in response to physical complaints. Lack of reinforcement may help to extinguish the maladaptive response.

A nurse is assessing a pathological gambler. What would differentiate this client's behaviors from the behaviors of a non-pathological gambler? 1. Pathological gamblers have abnormal levels of neurotransmitters, whereas non-pathological gamblers do not. 2. Pathological gambling occurs more commonly among women, whereas non-pathological gambling occurs more commonly among men. 3. Pathological gambling generally runs an acute course, whereas non-pathological gambling runs a chronic course. 4. Pathological gambling is not related to stress relief, whereas non-pathological gambling is related to stress relief.

1 There is a correlation between pathological gambling and abnormalities in the neurotransmitter, dopamine. This is not the case with non-pathological gambling.

The client diagnosed with antisocial personality disorder spits, curses, and refuses to answer questions during the assessment interview. Which response would the nurse make? 1. "You are very disrespectful. You need to learn to control yourself." 2. "I understand that you are angry, but this behavior will not be tolerated." 3. "What behaviors could you modify to improve this situation?" 4. "Which antipersonality disorder medications have helped you in the past?"

2 The appropriate nursing response is to reflect the client's feeling while setting firm limits on behavior. Clients diagnosed with antisocial personality disorder have a low tolerance for frustration, see themselves as victims, and use projection as a primary ego defense mechanism.

An inpatient client with a known history of violence suddenly begins to pace. Which additional client behavior would alert a nurse to escalating anger and aggression? 1. The client requests prn medications. 2. The client has a tense facial expression. 3. The client refuses to eat lunch. 4. The client sits in group with back to peers.

2 The nurse would assess that tense facial expressions may indicate that a client's anger is escalating.

The nurse is preparing a presentation about the onset of symptoms for agoraphobia. Which information should the nurse include in the teaching session? 1. Occurs in early adolescence and persists until midlife 2. Occurs in the 20s and 30s and persists for many years 3. Occurs in the 40s and 50s and persists until death 4. Occurs after the age of 60 and persists for at least 6 years

2 This information should be included in the teaching session. The onset of the symptoms of agoraphobia most commonly occurs in the 20s and 30s and persists for many years.

The psychiatrist prescribes haloperidol (Haldol) 50 mg bid, benztropine (Cogentin) 1 mg prn, and zolpidem (Ambien) 10 mg at bedtime for a client with schizophrenia spectrum disorder. Which client behavior would warrant the nurse to administer benztropine? 1. Tactile hallucinations 2. Tardive dyskinesia 3. Muscle rigidity 4. Reports of hearing disturbing voices

3 An anticholinergic medication such as benztropine would be used to treat the extrapyramidal symptom of muscle rigidity.

Which data in the history would the nurse expect to find in a client diagnosed with substance-induced psychotic disorder? 1. Had delirium 2. Had less severe withdrawal symptoms 3. Has an opioid use disorder 4. Has a fluid and electrolyte imbalance

3 The prominent hallucinations and delusions associated with substance-induced or medication-induced disorder are found to be directly attributable to substance intoxication or withdrawal, like opioid use disorder.

Place the following stages of the codependency recovery process according to Cermak beginning with the first stage (1-4). (Enter the number of each stage in the proper sequence, using comma and space format, such as: 1, 2, 3, 4) 1. The Core Issues Stage 2. The Reintegration Stage 3. The Survival Stage 4. The Reidentification Stage

3, 4, 1, 2

The nurse is working with a patient who had an MI and is now active in rehabilitation. The nurse should teach this patient to cease activity if which of the following occurs? A) The patient experiences chest pain, palpitations, or dyspnea. B) The patient experiences a noticeable increase in heart rate during activity. C) The patients oxygen saturation level drops below 96%. D) The patients respiratory rate exceeds 30 breaths/min.

A Feedback: Any activity or exercise that causes dyspnea and chest pain should be stopped in the patient with CAD. Heart rate must not exceed the target rate, but an increase above resting rate is expected and is therapeutic. In most patients, a respiratory rate that exceeds 30 breaths/min is not problematic. Similarly, oxygen saturation slightly below 96% does not necessitate cessation of activity

The nurse is reviewing the medication administration record of a patient diagnosed with systolic HF. What medication should the nurse anticipate administering to this patient? A) A beta-adrenergic blocker B) An antiplatelet aggregator C) A calcium channel blocker D) A nonsteroidal anti-inflammatory drug (NSAID)

A Feedback: Several medications are routinely prescribed for systolic HF, including ACE inhibitors, beta-blockers, diuretics, and digitalis. Calcium channel blockers, antiplatelet aggregators, and NSAIDs are not commonly prescribed.

A nurse is caring for a patient with Hodgkin lymphoma at the oncology clinic. The nurse should be aware of what main goal of care? A) Cure of the disease B) Enhancing quality of life C) Controlling symptoms D) Palliation

A Feedback: The goal in the treatment of Hodgkin lymphoma is cure. Palliation is thus not normally necessary. Quality of life and symptom control are vital, but the overarching goal is the cure th

The nurse is preparing a staff development presentation to improve the screening, intervention, and referral process for clients in the geriatric community center. Which information should the nurse identify as barriers to this initiative? (Select all that apply.) 1. Patient concerns about privacy 2. Competing workload demands 3. New nurses 4. The staff's attitude 5. Changing screening requirements

ANS: 1, 2, 3, 4

The client asks the nurse about pet therapy. Which responses by the nurse provide the client with accurate information? (Select all that apply.) 1. "Pet therapy reduces depression." 2. "Pet therapy decreases blood pressure." 3. "Pet therapy enhances client mood." 4. "Pet therapy improves sensory functioning." 5. "Pet therapy mitigates the effects of loneliness."

ANS: 1, 2, 3, 5

The depressed client is prescribed a monoamine oxidase inhibitor (MAOI). Which statements by the client should indicate to a nurse that the discharge teaching about this medication has been successful? (Select all that apply.) 1. "I'll have to let my surgeon know about this medication before I have my cholecystectomy." 2. "I guess I will have to give up my glass of red wine with dinner." 3. "I'll have to be very careful about reading food labels." 4. "I'm going to drink my caffeinated coffee in the morning." 5. "I'll be sure not to stop this medication abruptly."

ANS: 1, 2, 3, 5

A clinic nurse is about to meet with a client diagnosed with a gambling disorder. The nurse would assess which symptoms and behaviors? (Select all that apply.) 1. Stressful situations precipitate gambling behaviors. 2. Anticipation and restlessness can only be relieved by placing a bet. 3. Winning brings about feelings of sexual satisfaction. 4. Gambling is used as a coping strategy. 5. Compulsive gambling began in early adolescence.

ANS: 1, 2, 4

The nurse is a manager of a pediatric unit. Which actions should the nurse manager take to equip staff to address neuropsychiatric symptoms in pediatric clients? (Select all that apply.) 1. Encourage use of screening tools 2. Provide education of staff members 3. Keep referrals to a minimum 4. Increase social contact with individuals with mental illness 5. Promote defensive medicine

ANS: 1, 2, 4

An attractive female client with a diagnosis of body dysmorphic disorder (BDD) presents with high anxiety levels because of her belief that her facial features are large and grotesque. Which additional symptoms would support this diagnosis? (Select all that apply.) 1. Mirror checking 2. Excessive grooming 3. Stereotypic movement 4. History of delusional thinking 5. Skin picking

ANS: 1, 2, 5

The client is diagnosed with functional neurological symptom disorder. Which symptoms would the nurse most likely observe? (Select all that apply.) 1. Anosmia 2. Abreaction 3. Akinesia 4. Aphonia 5. Amnesia

ANS: 1, 3, 4

Which statements reflect current attitudes toward complementary and alternative therapies? (Select all that apply.) 1. Some health insurance companies are beginning to cover treatments such as acupuncture and massage therapy. 2. The majority of third-party payers do not cover chiropractic client treatments. 3. A large number of U.S. medical schools, among them Harvard and Yale, now offer coursework in holistic methods. 4. The American Medical Association supports the inclusion of complementary and alternative medicine in medical education. 5. Interest in dietary supplements is decreasing worldwide.

ANS: 1, 3, 4

The nurse is caring for a client diagnosed with generalized anxiety disorder. Which activities would the nurse encourage for this client? (Select all that apply.) 1. Recognize the signs of escalating anxiety. 2. Avoid any situation that causes stress. 3. Employ newly learned relaxation techniques. 4. Cognitively reframe thoughts about situations that generate anxiety. 5. Avoid caffeinated products.

ANS: 1, 3, 4, 5

Which nursing diagnosis is priority when providing nursing care to a client diagnosed with avoidant personality disorder? 1. Risk for violence: directed toward others R/T suspicious thinking 2. Risk for suicide R/T altered thought 3. Altered sensory perception R/T increased levels of anxiety 4. Social isolation R/T fear of rejection

ANS: 4 Rationale: The priority nursing diagnosis for a client diagnosed with avoidant personality disorder should be social isolation R/T inability to relate to others. These clients avoid close or romantic relationships, interpersonal attachments, and intimate sexual relationships.

Which highest priority outcome would the nurse add to the plan of care for a depressed client? 1. The client will promise to remain safe. 2. The client will discuss feelings with staff and family by day three. 3. The client will establish a trusting relationship with the nurse. 4. The client will not harm self during hospital stay.

ANS: 4 The nurse's highest priority should be that the client will not harm self during the hospital stay. Client safety should always be the nurse's highest priority.

The nurse is assessing a teenaged client diagnosed with cyclothymic disorder. Which DSM-5 diagnostic criteria would the nurse expect this client to meet? (Select all that apply.) 1. Symptoms lasting for a minimum of two years 2. Numerous periods with manic symptoms 3. Possible comorbid diagnosis of a delusional disorder 4. Symptoms cause clinically significant impairment in important areas of functioning 5. Depressive symptoms that do not meet the criteria for major depressive episode

ANS: 4, 5

The nurse is teaching the main principles of hemodialysis to a client with chronic kidney disease. Which statement by the client indicates a need for further teaching by the nurse? a. "My sodium level changes by movement from the blood into the dialysate." b. "Dialysis works by movement of wastes from lower to higher concentration." c. "Extra fluid can be pulled from the blood by osmosis." d. "The dialysate is similar to blood but without any toxins."

ANS: B Dialysis works using the passive transfer of toxins by diffusion. Diffusion is the movement of molecules from an area of higher concentration to an area of lower concentration. The other statements show a correct understanding about hemodialysis.

A client is assessed by the nurse after a hemodialysis session. The nurse notes bleeding from the client's nose and around the intravenous catheter. What action by the nurse is the priority? a. Hold pressure over the client's nose for 10 minutes. b. Take the client's pulse, blood pressure, and temperature. c. Assess for a bruit or thrill over the arteriovenous fistula. d. Prepare protamine sulfate for administration.

ANS: D Heparin is used with hemodialysis treatments. The bleeding alerts the nurse that too much anticoagulant is in the client's system and protamine sulfate should be administered. Pressure, taking vital signs, and assessing for a bruit or thrill are not as important as medication administration.

A nurse assesses a client who is prescribed an infusion of vasopressin (Pitressin) for bleeding esophageal varices. Which clinical manifestation should alert the nurse to a serious adverse effect? a. Nausea and vomiting b. Frontal headache c. Vertigo and syncope d. Mid-sternal chest pain

ANS: D Mid-sternal chest pain is indicative of acute angina or myocardial infarction, which can be precipitated by vasopressin. Nausea and vomiting, headache, and vertigo and syncope are not side effects of vasopressin.DIF: Applying/Application REF: 1200

A client with chronic kidney disease states, "I feel chained to the hemodialysis machine." What is the nurse's best response to the client's statement? a. "That feeling will gradually go away as you get used to the treatment." b. "You probably need to see a psychiatrist to see if you are depressed." c. "Do you need help from social services to discuss financial aid?" d. "Tell me more about your feelings regarding hemodialysis treatment."

ANS: D The nurse needs to explore the client's feelings in order to help the client cope and enter a phase of acceptance or resignation. It is common for clients to be discouraged because of the dependency of the treatment, especially during the first year. Referrals to a mental health provider or social services are possibilities, but only after exploring the client's feelings first. Telling the client his or her feelings will go away is dismissive of the client's concerns.

A client with acute kidney injury has a blood pressure of 76/55 mm Hg. The health care provider ordered 1000 mL of normal saline to be infused over 1 hour to maintain perfusion. The client is starting to develop shortness of breath. What is the nurse's priority action? a. Calculate the mean arterial pressure (MAP). b. Ask for insertion of a pulmonary artery catheter. c. Take the client's pulse. d. Slow down the normal saline infusion.

ANS: D The nurse should assess that the client could be developing fluid overload and respiratory distress and slow down the normal saline infusion. The calculation of the MAP also reflects perfusion. The insertion of a pulmonary artery catheter would evaluate the client's hemodynamic status, but this should not be the initial action by the nurse. Vital signs are also important after adjusting the intravenous infusion.

The nurse working on a cardiac care unit is caring for a patient whose stroke volume has increased. The nurse is aware that afterload influences a patients stroke volume. The nurse recognizes that afterload is increased when there is what? A) Arterial vasoconstriction B) Venous vasoconstriction C) Arterial vasodilation D) Venous vasodilation

Ans: A Feedback: Arterial vasoconstriction increases the systemic vascular resistance, which increases the afterload. Venous vasoconstriction decreases preload thereby decreasing stroke volume. Venous vasodilation increases preload.

A patient is responding poorly to interventions aimed at treating shock and appears to be transitioning to the irreversible stage of shock. What action should the intensive care nurse include during this phase of the patients care? A) Communicate clearly and frequently with the patients family. B) Taper down interventions slowly when the prognosis worsens. C) Transfer the patient to a subacute unit when recovery appears unlikely. D) Ask the patients family how they would prefer treatment to proceed.

Ans: A Feedback: As it becomes obvious that the patient is unlikely to survive, the family must be informed about the prognosis and likely outcome. Opportunities should be provided, throughout the patients care, for the family to see, touch, and talk to the patient. The onus should not be placed on the family to guide care, however. Interventions are not normally reduced gradually when they are deemed ineffective; instead, they are discontinued when they appear futile. The patient would not be transferred to a subacute unit.

The nurse is caring for a patient admitted with angina who is scheduled for cardiac catheterization. The patient is anxious and asks the reason for this test. What is the best response? A) Cardiac catheterization is usually done to assess how blocked or open a patients coronary arteries are. B) Cardiac catheterization is most commonly done to detect how efficiently a patients heart muscle contracts. C) Cardiac catheterization is usually done to evaluate cardiovascular response to stress. D) Cardiac catheterization is most commonly done to evaluate cardiac electrical activity.

Ans: A Feedback: Cardiac catheterization is usually used to assess coronary artery patency to determine if revascularization procedures are necessary. A thallium stress test shows myocardial ischemia after stress. An ECG shows the electrical activity of the heart.

The nurse in the ED is caring for a patient recently admitted with a likely myocardial infarction. The nurse understands that the patients heart is pumping an inadequate supply of oxygen to the tissues. For what health problem should the nurse assess? A) Dysrhythmias B) Increase in blood pressure C) Increase in heart rate D) Decrease in oxygen demands

Ans: A Feedback: Cardiogenic shock occurs when the hearts ability to pump blood is impaired and the supply of oxygen is inadequate for the heart and tissues. Symptoms of cardiogenic shock include angina pain and dysrhythmias. Cardiogenic shock does not cause increased blood pressure, increased heart rate, or a decrease in oxygen demands.

A pediatric nurse practitioner is caring for a child who has just been diagnosed with asthma. The nurse has provided the parents with information that includes potential causative agents for an asthmatic reaction. What potential causative agent should the nurse describe? A) Pets B) Lack of sleep C) Psychosocial stress D) Bacteria

Ans: A Feedback: Common causative agents that may trigger an asthma attack are as follows: dust, dust mites, pets, soap, certain foods, molds, and pollens. Lack of sleep, stress, and bacteria are not common triggers for asthma attacks.

The nurse is caring for a patient who needs education on his medication therapy for allergic rhinitis. The patient is to take cromolyn (Nasalcrom) daily. In providing education for this patient, how should the nurse describe the action of the medication? A) It inhibits the release of histamine and other chemicals. B) It inhibits the action of proton pumps. C) It inhibits the action of the sodium-potassium pump in the nasal epithelium. D) It causes bronchodilation and relaxes smooth muscle in the bronchi.

Ans: A Feedback: Cromolyn (Nasalcrom) inhibits the release of histamine and other chemicals. It is prescribed to treat allergic rhinitis. Beta-adrenergic agents lead to bronchodilation and stimulate beta-2adrenergic receptors in the smooth muscle of the bronchi and bronchioles. It does not affect proton pump action or the sodium-potassium pump in the nasal cells.

The nurse is caring for a patient who is exhibiting signs and symptoms of hypovolemic shock following injuries suffered in a motor vehicle accident. The nurse anticipates that the physician will promptly order the administration of a crystalloid IV solution to restore intravascular volume. In addition to normal saline, which crystalloid fluid is commonly used to treat hypovolemic shock? A) Lactated Ringers B) Albumin C) Dextran D) 3% NaCl

Ans: A Feedback: Crystalloids are electrolyte solutions used for the treatment of hypovolemic shock. Lactated Ringers and 0.9% sodium chloride are isotonic crystalloid fluids commonly used to manage hypovolemic shock. Dextran and albumin are colloids, but Dextran, even as a colloid, is not indicated for the treatment of hypovolemic shock. 3% NaCl is a hypertonic solution and is not isotonic.

The nurse is caring for a patient who has had an ECG. The nurse notes that leads I, II, and III differ from one another on the cardiac rhythm strip. How should the nurse best respond? A) Recognize that the view of the electrical current changes in relation to the lead placement. B) Recognize that the electrophysiological conduction of the heart differs with lead placement. C) Inform the technician that the ECG equipment has malfunctioned. D) Inform the physician that the patient is experiencing a new onset of dysrhythmia.

Ans: A Feedback: Each lead offers a different reference point to view the electrical activity of the heart. The lead displays the configuration of electrical activity of the heart. Differences between leads are not necessarily attributable to equipment malfunction or dysrhythmias.

The physician has placed a central venous pressure (CVP) monitoring line in an acutely ill patient so right ventricular function and venous blood return can be closely monitored. The results show decreased CVP. What does this indicate? A) Possible hypovolemia B) Possible myocardial infarction (MI) C) Left-sided heart failure D) Aortic valve regurgitation

Ans: A Feedback: Hypovolemia may cause a decreased CVP. MI, valve regurgitation and heart failure are less likely causes of decreased CVP.

A nurse is caring for a 6-year-old patient with cystic fibrosis. In order to enhance the childs nutritional status, what intervention should most likely be included in the plan of care? A) Pancreatic enzyme supplementation with meals B) Provision of five to six small meals per day rather than three larger meals C) Total parenteral nutrition (TPN) D) Magnesium, thiamine, and iron supplementation

Ans: A Feedback: Nearly 90% of patients with CF have pancreatic exocrine insufficiency and require oral pancreatic enzyme supplementation with meals. Frequent, small meals or TPN are not normally indicated. Vitamin supplements are required, but specific replacement of magnesium, thiamine, and iron is not typical.

The nurse has admitted a patient who is scheduled for a thoracic resection. The nurse is providing preoperative teaching and is discussing several diagnostic studies that will be required prior to surgery. Which study will be performed to determine whether the planned resection will leave sufficient functioning lung tissue? A) Pulmonary function studies B) Exercise tolerance tests C) Arterial blood gas values D) Chest x-ray

Ans: A Feedback: Pulmonary function studies are performed to determine whether the planned resection will leave sufficient functioning lung tissue. ABG values are assessed to provide a more complete picture of the functional capacity of the lung. Exercise tolerance tests are useful to determine if the patient who is a candidate for pneumonectomy can tolerate removal of one of the lungs. Preoperative studies, such as a chest x-ray, are performed to provide a baseline for comparison during the postoperative period and to detect any unsuspected abnormalities.

The nurse is calculating a cardiac patients pulse pressure. If the patients blood pressure is 122/76 mm Hg, what is the patients pulse pressure? A) 46 mm Hg B) 99 mm Hg C) 198 mm Hg D) 76 mm

Ans: A Feedback: Pulse pressure is the difference between the systolic and diastolic pressure. In this case, this value is 46 mm Hg

The school nurse is presenting a class on smoking cessation at the local high school. A participant in the class asks the nurse about the risk of lung cancer in those who smoke. What response related to risk for lung cancer in smokers is most accurate? A) The younger you are when you start smoking, the higher your risk of lung cancer. B) The risk for lung cancer never decreases once you have smoked, which is why smokers need annual chest x-rays. C) The risk for lung cancer is determined mostly by what type of cigarettes you smoke. D) The risk for lung cancer depends primarily on the other risk factors for cancer that you have

Ans: A Feedback: Risk is determined by the pack-year history (number of packs of cigarettes used each day, multiplied by the number of years smoked), the age of initiation of smoking, the depth of inhalation, and the tar and nicotine levels in the cigarettes smoked. The younger a person is when he or she starts smoking, the greater the risk of developing lung cancer. Risk declines after smoking cessation. The type of cigarettes is a significant variable, but this is not the most important factor.

A nurse has been asked to give a workshop on COPD for a local community group. The nurse emphasizes the importance of smoking cessation because smoking has what pathophysiologic effect? A) Increases the amount of mucus production B) Destabilizes hemoglobin C) Shrinks the alveoli in the lungs D) Collapses the alveoli in the lungs

Ans: A Feedback: Smoking irritates the goblet cells and mucous glands, causing an increased accumulation of mucus, which, in turn, produces more irritation, infection, and damage to the lung.

The nurse is caring for a patient who is ready to be weaned from the ventilator. In preparing to assist in the collaborative process of weaning the patient from a ventilator, the nurse is aware that the weaning of the patient will progress in what order? A) Removal from the ventilator, tube, and then oxygen B) Removal from oxygen, ventilator, and then tube C) Removal of the tube, oxygen, and then ventilator D) Removal from oxygen, tube, and then ventilator

Ans: A Feedback: The process of withdrawing the patient from dependence on the ventilator takes place in three stages: the patient is gradually removed from the ventilator, then from the tube, and, finally, oxygen.

A patient has been discharged home after thoracic surgery. The home care nurse performs the initial visit and finds the patient discouraged and saddened. The client states, I am recovering so slowly. I really thought I would be better by now. What nursing action should the nurse prioritize? A) Provide emotional support to the patient and family. B) Schedule a visit to the patients primary physician within 24 hours. C) Notify the physician that the patient needs a referral to a psychiatrist. D) Place a referral for a social worker to visit the patient.

Ans: A Feedback: The recovery process may take longer than the patient had expected, and providing support to the patient is an important task for the home care nurse. It is not necessary, based on this scenario, to schedule a visit with the physician within 24 hours, or to get a referral to a psychiatrist or a social worker.

A hospital has been the site of an increased incidence of hospital-acquired pneumonia (HAP). What is an important measure for the prevention of HAP? A) Administration of prophylactic antibiotics B) Administration of pneumococcal vaccine to vulnerable individuals C) Obtaining culture and sensitivity swabs from all newly admitted patients D) Administration of antiretroviral medications to patients over age 65

Ans: B Feedback: Pneumococcal vaccination reduces the incidence of pneumonia, hospitalizations for cardiac conditions, and deaths in the general older adult population. A onetime vaccination of pneumococcal polysaccharide vaccine (PPSV) is recommended for all patients 65 years of age or older and those with chronic diseases. Antibiotics are not given on a preventative basis and antiretroviral medications do not affect the most common causative microorganisms. Culture and sensitivity testing by swabbing is not performed for pneumonia since the microorganisms are found in sputum

A new employee asks the occupational health nurse about measures to prevent inhalation exposure of the substances. Which statement by the nurse will decrease the patients exposure risk to toxic substances? A) Position a fan blowing on the toxic substances to prevent the substance from becoming stagnant in the air. B) Wear protective attire and devices when working with a toxic substance. C) Make sure that you keep your immunizations up to date to prevent respiratory diseases resulting from toxins. D) Always wear a disposable paper face mask when you are working with inhalable toxins.

Ans: B Feedback: When working with toxic substances, the employee must wear or use protective devices such as face masks, hoods, or industrial respirators. Immunizations do not confer protection from toxins and a paper mask is normally insufficient protection. Never position a fan directly blowing on the toxic substance as it will disperse the fumes throughout the area.

A patient has just been diagnosed with squamous cell carcinoma of the neck. While the nurse is doing health education, the patient asks, Does this kind of cancer tend to spread to other parts of the body? What is the nurses best response? A) In many cases, this type of cancer spreads to other parts of the body. B) This cancer usually does not spread to distant sites in the body. C) You will have to speak to your oncologist about that. D) Squamous cell carcinoma is nothing to be concerned about, so try to focus on your health

Ans: B The incidence of distant metastasis with squamous cell carcinoma of the head and neck (including larynx cancer) is relatively low. The patients prognosis is determined by the oncologist, but the patient has asked a general question and it would be inappropriate to refuse a response. The nurse must not downplay the patients concerns.

A nurse is developing a teaching plan for a patient with COPD. What should the nurse include as the most important area of teaching? A) Avoiding extremes of heat and cold B) Setting and accepting realistic short- and long-range goals C) Adopting a lifestyle of moderate activity D) Avoiding emotional disturbances and stressful situations

Ans: B Feedback: A major area of teaching involves setting and accepting realistic short-term and long-range goals. The other options should also be included in the teaching plan, but they are not areas that are as high a priority as setting and accepting realistic goals.

The nurse is caring for a patient who is experiencing mild shortness of breath during the immediate postoperative period, with oxygen saturation readings between 89% and 91%. What method of oxygen delivery is most appropriate for the patients needs? A) Non-rebreathing mask B) Nasal cannula C) Simple mask D) Partial-rebreathing mask

Ans: B Feedback: A nasal cannula is used when the patient requires a low to medium concentration of oxygen for which precise accuracy is not essential. The Venturi mask is used primarily for patients with COPD because it can accurately provide an appropriate level of supplemental oxygen, thus avoiding the risk of suppressing the hypoxic drive. The patients respiratory status does not require a partial- or nonrebreathing mask.

A mother calls the clinic asking for a prescription for Amoxicillin for her 2-year-old son who has what the nurse suspects to be viral rhinitis. What should the nurse explain to this mother? A) I will relay your request promptly to the doctor, but I suspect that she wont get back to you if its a cold. B) Ill certainly inform the doctor, but if it is a cold, antibiotics wont be used because they do not affect the virus. C) Ill phone in the prescription for you since it can be prescribed by the pharmacist. D) Amoxicillin is not likely the best antibiotic, but Ill call in the right prescription for you.

Ans: B Feedback: Antimicrobial agents (antibiotics) should not be used because they do not affect the virus or reduce the incidence of bacterial complications. In addition, their inappropriate use has been implicated in development of organisms resistant to therapy. It would be inappropriate to tell the patient that the physician will not respond to her request

An asthma nurse educator is working with a group of adolescent asthma patients. What intervention is most likely to prevent asthma exacerbations among these patients? A) Encouraging patients to carry a corticosteroid rescue inhaler at all times B) Educating patients about recognizing and avoiding asthma triggers C) Teaching patients to utilize alternative therapies in asthma management D) Ensuring that patients keep their immunizations up to date

Ans: B Feedback: Asthma exacerbations are best managed by early treatment and education, including the use of written action plans as part of any overall effort to educate patients about self-management techniques, especially those with moderate or severe persistent asthma or with a history of severe exacerbations. Corticosteroids are not used as rescue inhalers. Alternative therapies are not normally a high priority, though their use may be appropriate in some cases. Immunizations should be kept up to date, but this does not necessarily prevent asthma exacerbations.

A nurse is caring for a patient who has been hospitalized with an acute asthma exacerbation. What drugs should the nurse expect to be ordered for this patient to gain underlying control of persistent asthma? A) Rescue inhalers B) Anti-inflammatory drugs C) Antibiotics D) Antitussives

Ans: B Feedback: Because the underlying pathology of asthma is inflammation, control of persistent asthma is accomplished primarily with regular use of anti-inflammatory medications. Rescue inhalers, antibiotics, and antitussives do not aid in the first-line control of persistent asthma.

The nurse recognizes that aspiration is a potential complication of a laryngectomy. How should the nurse best manage this risk? A) Facilitate total parenteral nutrition (TPN). B) Keep a complete suction setup at the bedside. C) Feed the patient several small meals daily. D) Refer the patient for occupational therapy

Ans: B Feedback: Due to the risk for aspiration, the nurse keeps a suction setup available in the hospital and instructs the family to do so at home for use if needed. TPN is not indicated and small meals do not necessarily reduce the risk of aspiration. Physical therapists do not address swallowing ability

The nurse is transferring a patient who is in the progressive stage of shock into ICU from the medical unit. The medical nurse is aware that shock affects many organ systems and that nursing management of the patient will focus on what intervention? A) Reviewing the cause of shock and prioritizing the patients psychosocial needs B) Assessing and understanding shock and the significant changes in assessment data to guide the plan of care C) Giving the prescribed treatment, but shifting focus to providing family time as the patient is unlikely to survive D) Promoting the patients coping skills in an effort to better deal with the physiologic changes accompanying shock

Ans: B Feedback: Nursing care of patients in the progressive stage of shock requires expertise in assessing and understanding shock and the significance of changes in assessment data. Early interventions are essential to the survival of patients in shock; thus, suspecting that a patient may be in shock and reporting subtle changes in assessment are imperative. Psychosocial needs, such as coping, are important considerations, but they are not prioritized over physiologic health

The critical care nurse is caring for a patient who has had an MI. The nurse should expect to assist with establishing what hemodynamic monitoring procedure to assess the patients left ventricular function? A) Central venous pressure (CVP) monitoring B) Pulmonary artery pressure monitoring (PAPM) C) Systemic arterial pressure monitoring (SAPM) D) Arterial blood gases (ABG)

Ans: B Feedback: PAPM is used to assess left ventricular function. CVP is used to assess right ventricular function; SAPM is used for continual assessment of BP. ABG are used to assess for acidic and alkalotic levels in the blood.

While auscultating a patients heart sounds, the nurse hears an extra heart sound immediately after the second heart sound (S2 ). An audible S3 would be considered an expected finding in what patient? A) An older adult B) A 20-year-old patient C) A patient who has undergone valve replacement D) A patient who takes a beta-adrenergic blocker

Ans: B Feedback: S3 represents a normal finding in children and adults up to 35 or 40 years of age. In these cases, it is called a physiologic S3 . It is an abnormal finding in a patient with an artificial valve, an older adult, or a patient who takes a beta blocker.

An older adult patient has been diagnosed with COPD. What characteristic of the patients current health status would preclude the safe and effective use of a metered-dose inhaler (MDI)? A) The patient has not yet quit smoking. B) The patient has severe arthritis in her hands. C) The patient requires both corticosteroids and beta2 -agonists. D) The patient has cataracts.

Ans: B Feedback: Safe and effective MDI use requires the patient to be able to manipulate the device independently, which may be difficult if the patient has arthritis. Smoking does not preclude MDI use. A modest loss of vision does not preclude the use of an MDI and a patient can safely use more than one MDI.

A patient is receiving thrombolytic therapy for the treatment of pulmonary emboli. What is the best way for the nurse to assess the patients oxygenation status at the bedside? A) Obtain serial ABG samples. B) Monitor pulse oximetry readings. C) Test pulmonary function. D) Monitor incentive spirometry volumes.

Ans: B Feedback: The nurse assesses the patient with pulmonary emboli frequently for signs of hypoxemia and monitors the pulse oximetry values to evaluate the effectiveness of the oxygen therapy. ABGs are accurate indicators of oxygenation status, but are not analyzed at the bedside. PFTs and incentive spirometry volumes do not accurately reveal oxygenation status.

A patient who is in shock is receiving dopamine in addition to IV fluids. What principle should inform the nurses care planning during the administration of a vasoactive drug? A) The drug should be discontinued immediately after blood pressure increases. B) The drug dose should be tapered down once vital signs improve. C) The patient should have arterial blood gases drawn every 10 minutes during treatment. D) The infusion rate should be titrated according the patients subjective sensation of adequate perfusion.

Ans: B Feedback: When vasoactive medications are discontinued, they should never be stopped abruptly because this could cause severe hemodynamic instability, perpetuating the shock state. Subjective assessment data are secondary to objective data. Arterial blood gases should be carefully monitored, but every10-minute draws are not the norm.

A critical care nurse is planning assessments in the knowledge that patients in shock are vulnerable to developing fluid replacement complications. For what signs and symptoms should the nurse monitor the patient? Select all that apply. A) Hypovolemia B) Difficulty breathing C) Cardiovascular overload D) Pulmonary edema E) Hypoglycemia

Ans: B, C, D Feedback: Fluid replacement complications can occur, often when large volumes are administered rapidly. Therefore, the nurse monitors the patient closely for cardiovascular overload, signs of difficulty breathing, and pulmonary edema. Hypovolemia is what necessitates fluid replacement, and hypoglycemia is not a central concern with fluid replacement.

The nurse is performing an assessment on a patient who has been diagnosed with cancer of the larynx. Part of the nurses assessment addresses the patients general state of nutrition. Which laboratory values would be assessed when determining the nutritional status of the patient? Select all that apply. A) White blood cell count B) Protein level C) Albumin level D) Platelet count E) Glucose level

Ans: B, C, E Feedback: The nurse also assesses the patients general state of nutrition, including height and weight and body mass index, and reviews laboratory values that assist in determining the patients nutritional status (albumin, protein, glucose, and electrolyte levels). The white blood cell count and the platelet count would not normally assist in determining the patients nutritional status.

It is cold season and the school nurse been asked to provide an educational event for the parent teacher organization of the local elementary school. What should the nurse include in teaching about the treatment of pharyngitis? A) Pharyngitis is more common in children whose immunizations are not up to date. B) There are no effective, evidence-based treatments for pharyngitis. C) Use of warm saline gargles or throat irrigations can relieve symptoms. D) Heat may increase the spasms in pharyngeal muscles

Ans: C Feedback: Depending on the severity of the pharyngitis and the degree of pain, warm saline gargles or throatirrigations are used. The benefits of this treatment depend on the degree of heat that is applied. The nurse teaches about these procedures and about the recommended temperature of the solution: high enough to be effective and as warm as the patient can tolerate, usually 105F to 110F (40.6C to 43.3C). Irrigating the throat may reduce spasm in the pharyngeal muscles and relieve soreness of the throat. You would not tell the parent teacher organization that there is no real treatment of pharyngitis.

The nurse is caring for a patient whose progressing infection places her at high risk for shock. What assessment finding would the nurse consider a potential sign of shock? A) Elevated systolic blood pressure B) Elevated mean arterial pressure (MAP) C) Shallow, rapid respirations D) Bradycardia

Ans: C Feedback: A symptom of shock is shallow, rapid respirations. Systolic blood pressure drops in shock, and MAP is less than 65 mm Hg. Bradycardia occurs in neurogenic shock; other states of shock have tachycardia as a symptom. Infection can lead to septic shock.

The ICU nurse is caring for a patient in hypovolemic shock following a postpartum hemorrhage. For what serious complication of treatment should the nurse monitor the patient? A) Anaphylaxis B) Decreased oxygen consumption C) Abdominal compartment syndrome D) Decreased serum osmolality

Ans: C Feedback: Abdominal compartment syndrome (ACS) is a serious complication that may occur when large volumes of fluid are administered. The scenario does not describe an antigenantibody reaction of any type. Decreased oxygen consumption by the body is not a concern in hypovolemic shock. With a decrease in fluids in the intravascular space, increased serum osmolality would occur.

When assessing for substances that are known to harm workers lungs, the occupational health nurse should assess their potential exposure to which of the following? A) Organic acids B) Propane C) Asbestos D) Gypsum

Ans: C Feedback: Asbestos is among the more common causes of pneumoconiosis. Organic acids, propane, and gypsum do not have this effect.

A nurse is creating a health promotion intervention focused on chronic obstructive pulmonary disease (COPD). What should the nurse identify as a complication of COPD? A) Lung cancer B) Cystic fibrosis C) Respiratory failure D) Hemothorax

Ans: C Feedback: Complications of COPD include respiratory failure, pneumothorax, atelectasis, pneumonia, and pulmonary hypertension (corpulmonale). Lung cancer, cystic fibrosis, and hemothorax are not common complications.

While assessing the patient, the nurse observes constant bubbling in the water-seal chamber of the patients closed chest-drainage system. What should the nurse conclude? A) The system is functioning normally. B) The patient has a pneumothorax. C) The system has an air leak. D) The chest tube is obstructed.

Ans: C Feedback: Constant bubbling in the chamber often indicates an air leak and requires immediate assessment and intervention. The patient with a pneumothorax will have intermittent bubbling in the water-seal chamber. If the tube is obstructed, the nurse should notice that the fluid has stopped fluctuating in the water-seal chamber.

A nurse is preparing a patient for scheduled transesophageal echocardiography. What action should the nurse perform? A) Instruct the patient to drink 1 liter of water before the test. B) Administer IV benzodiazepines and opioids. C) Inform the patient that she will remain on bed rest following the procedure. D) Inform the patient that an access line will be initiated in her femoral artery.

Ans: C Feedback: During the recovery period, the patient must maintain bed rest with the head of the bed elevated to 45 degrees. The patient must be NPO 6 hours preprocedure. The patient is sedated to make him or her comfortable, but will not be heavily sedated, and opioids are not necessary. Also, the patient will have a peripheral IV line initiated preprocedure.

The nurse is caring for a patient in the ICU admitted with ARDS after exposure to toxic fumes from a hazardous spill at work. The patient has become hypotensive. What is the cause of this complication to the ARDS treatment? A) Pulmonary hypotension due to decreased cardiac output B) Severe and progressive pulmonary hypertension C) Hypovolemia secondary to leakage of fluid into the interstitial spaces D) Increased cardiac output from high levels of PEEP therapy

Ans: C Feedback: Systemic hypotension may occur in ARDS as a result of hypovolemia secondary to leakage of fluid into the interstitial spaces and depressed cardiac output from high levels of PEEP therapy. Pulmonary hypertension, not pulmonary hypotension, sometimes is a complication of ARDS, but it is not the cause of the patient becoming hypotensive.

A patient has had a myocardial infarction and has been diagnosed as having damage to the layer of the heart responsible for the pumping action. You are aware that the damage occurred where? A) Endocardium B) Pericardium C) Myocardium D) Visceral pericardium

Ans: C Feedback: The myocardium is the layer of the heart responsible for the pumping action.

When circulatory shock occurs, there is massive vasodilation causing pooling of the blood in the periphery of the body. An ICU nurse caring for a patient in circulatory shock should know that the pooling of blood in the periphery leads to what pathophysiological effect? A) Increased stroke volume B) Increased cardiac output C) Decreased heart rate D) Decreased venous return

Ans: D Feedback: Pooling of blood in the periphery results in decreased venous return. Decreased venous return results in decreased stroke volume and decreased cardiac output. Decreased cardiac output, in turn, causes decreased blood pressure and, ultimately, decreased tissue perfusion. Heart rate increases in an attempt to meet the demands of the body.

The decision has been made to discharge a ventilator-dependent patient home. The nurse is developing a teaching plan for this patient and his family. What would be most important to include in this teaching plan? A) Administration of inhaled corticosteroids B) Assessment of neurologic status C) Turning and coughing D) Signs of pulmonary infection

Ans: D Feedback: The nurse teaches the patient and family about the ventilator, suctioning, tracheostomy care, signs of pulmonary infection, cuff inflation and deflation, and assessment of vital signs. Neurologic assessment and turning and coughing are less important than signs and symptoms of infection. Inhaled corticosteroids may or may not be prescribed

A patient newly diagnosed with cancer is scheduled to begin chemotherapy treatment and the nurse is providing anticipatory guidance about potential adverse effects. When addressing the most common adverse effect, what should the nurse describe? A) Pruritis (itching) B) Nausea and vomiting C) Altered glucose metabolism D) Confusion

B

The nurse is admitting an oncology patient to the unit prior to surgery. The nurse reads in the electronic health record that the patient has just finished radiation therapy. With knowledge of the consequent health risks, the nurse should prioritize assessments related to what health problem? A) Cognitive deficits B) Impaired wound healing C) Cardiac tamponade D) Tumor lysis syndrome

B

The public health nurse is presenting a health-promotion class to a group at a local community center. Which intervention most directly addresses the leading cause of cancer deaths in North America? A) Monthly self-breast exams B) Smoking cessation C) Annual colonoscopies D) Monthly testicular exams

B

The nurse is addressing exercise and physical activity during discharge education with a patient diagnosed with HF. What should the nurse teach this patient about exercise? A) Do not exercise unsupervised. B) Eventually aim to work up to 30 minutes of exercise each day. C) Slow down if you get dizzy or short of breath. D) Start your exercise program with high-impact activities.

B Feedback: Eventually, a total of 30 minutes of physical activity every day should be encouraged. Supervision is not necessarily required and the emergence of symptoms should prompt the patient to stop exercising, not simply to slow the pace. Low-impact activities should be prioritized.

An adult patient has presented to the health clinic with a complaint of a firm, painless cervical lymph node. The patient denies any recent infectious diseases. What is the nurses most appropriate response to the patients complaint? A) Call 911. B) Promptly refer the patient for medical assessment. C) Facilitate a radiograph of the patients neck and have the results forwarded to the patients primary care provider. D) Encourage the patient to track the size of the lymph node and seek care in 1 week.

B Feedback: Hodgkin lymphoma usually begins as an enlargement of one or more lymph nodes on one side of the neck. The individual nodes are painless and firm but not hard. Prompt medical assessment is necessary if a patient has this presentation. However, there is no acute need to call 911. Delaying care for 1 week could have serious consequences and x-rays are not among the common diagnostic tests.

A group of military nurses are reviewing the care of victims of biochemical terrorist attacks. The nurses should identify what agents as having the shortest latency? A) Viral agents B) Nerve agents C) Pulmonary agents D) Blood agents

B Feedback: Latency is the time from absorption to the appearance of signs and symptoms. Sulfur mustards and pulmonary agents have the longest latency, whereas vesicants, nerve agents, and cyanide produce signs and symptoms within seconds.

The nurse is caring for a patient who is scheduled to undergo mechanical valve replacement. Patient education should include which of the following? A) Use of patient-controlled analgesia B) Long-term anticoagulant therapy C) Steroid therapy D) Use of IV diuretics

B Feedback: Mechanical valves necessitate long-term use of required anticoagulants. Diuretics and steroids are not indicated and patient-controlled analgesia may or may be not be used in the immediate postoperative period.

A neurologic flow chart is often used to document the care of a patient with a traumatic brain injury. At what point in the patients care should the nurse begin to use a neurologic flow chart? A) When the patients condition begins to deteriorate B) As soon as the initial assessment is made C) At the beginning of each shift D) When there is a clinically significant change in the patients condition

B Feedback: Neurologic parameters are assessed initially and as frequently as the patients condition requires. As soon as the initial assessment is made, the use of a neurologic flowchart is started and maintained. A new chart is not begun at the start of every shift.

A home care nurse is performing a visit to a patients home to perform wound care following the patients hospital treatment for severe burns. While interacting with the patient, the nurse should assess for evidence of what complication? A) Psychosis B) Post-traumatic stress disorder C) Delirium D) Vascular dementia

B Feedback: Post-traumatic stress disorder (PTSD) is the most common psychiatric disorder in burn survivors, with a prevalence that may be as high as 45%. As a result, it is important for the nurse to assess for this complication of burn injuries. Psychosis, delirium, and dementia are not among the noted psychiatric and psychosocial complications of burns.

A school nurse is called to the playground where a 6-year-old girl has been found unresponsive and staring into space, according to the playground supervisor. How would the nurse document the girls activity in her chart at school? A) Generalized seizure B) Absence seizure C) Focal seizure D) Unclassified seizure

B Feedback: Staring episodes characterize an absence seizure, whereas focal seizures, generalized seizures, and unclassified seizures involve uncontrolled motor activity.

A nurse is caring for a patient who is receiving parenteral nutrition. When writing this patients plan of care, which of the following nursing diagnoses should be included? A) Risk for Peripheral Neurovascular Dysfunction Related to Catheter Placement B) Ineffective Role Performance Related to Parenteral Nutrition C) Bowel Incontinence Related to Parenteral Nutrition D) Chronic Pain Related to Catheter Placement

B Feedback: The limitations associated with PN can make it difficult for patients to maintain their usual roles. PN does not normally cause bowel incontinence and catheters are not associated with chronic pain or neurovascular dysfunction.

The nurses assessment of a patient with thyroidectomy suggests tetany and a review of the most recent blood work corroborate this finding. The nurse should prepare to administer what intervention? A) Oral calcium chloride and vitamin D B) IV calcium gluconate C) STAT levothyroxine D) Administration of parathyroid hormone (PTH)

B Feedback: When hypocalcemia and tetany occur after a thyroidectomy, the immediate treatment is administration of IV calcium gluconate. This has a much faster therapeutic effect than PO calcium or vitamin D supplements. PTH and levothyroxine are not used to treat this complication.

A patient with increased ICP has a ventriculostomy for monitoring ICP. The nurses most recent assessment reveals that the patient is now exhibiting nuchal rigidity and photophobia. The nurse would be correct in suspecting the presence of what complication? A) Encephalitis B) CSF leak C) Meningitis D) Catheter occlusion

C Complications of a ventriculostomy include ventricular infectious meningitis and problems with the monitoring system. Nuchal rigidity and photophobia are clinical manifestations of meningitis, but are not suggestive of encephalitis, a CSF leak, or an occluded catheter

The nurse is caring for a patient who has just had an implantable cardioverter defibrillator (ICD) placed. What is the priority area for the nurses assessment? A) Assessing the patients activity level B) Facilitating transthoracic echocardiography C) Vigilant monitoring of the patients ECG D) Close monitoring of the patients peripheral perfusion

C Feedback: After a permanent electronic device (pacemaker or ICD) is inserted, the patients heart rate and rhythm are monitored by ECG. This is a priority over peripheral circulation and activity. Echocardiography is not indicated.

The triage nurse in the ED is assessing a patient with chronic HF who has presented with worsening symptoms. In reviewing the patients medical history, what is a potential primary cause of the patients heart failure? A) Endocarditis B) Pleural effusion C) Atherosclerosis D) Atrial-septal defect

C Feedback: Atherosclerosis of the coronary arteries is the primary cause of HF. Pleural effusion, endocarditis, and an atrial-septal defect are not health problems that contribute to the etiology of HF.

A patient with a brain tumor has begun to exhibit signs of cachexia. What subsequent assessment should the nurse prioritize? A) Assessment of peripheral nervous function B) Assessment of cranial nerve function C) Assessment of nutritional status D) Assessment of respiratory statu

C Feedback: Cachexia is a wasting syndrome of weight loss, muscle atrophy, fatigue, weakness, and significant loss of appetite. Consequently, nutritional assessment is paramount.

A patient with hypertrophic cardiomyopathy (HCM) has been admitted to the medical unit. During the nurses admission interview, the patient states that she takes over-the-counter water pills on a regular basis. How should the nurse best respond to the fact that the patient has been taking diuretics? A) Encourage the patient to drink at least 2 liters of fluid daily. B) Increase the patients oral sodium intake. C) Inform the care provider because diuretics are contraindicated. D) Ensure that the patients fluid balance is monitored vigilantly.

C Feedback: Diuretics are contraindicated in patients with HCM, so the primary care provider should be made aware. Adjusting the patients sodium or fluid intake or fluid monitoring does not address this important contraindication.

A patient has experienced a seizure in which she became rigid and then experienced alternating muscle relaxation and contraction. What type of seizure does the nurse recognize? A) Unclassified seizure B) Absence seizure C) Generalized seizure D) Focal seizure

C Feedback: Generalized seizures often involve both hemispheres of the brain, causing both sides of the body to react. Intense rigidity of the entire body may occur, followed by alternating muscle relaxation and contraction (generalized tonicclonic contraction). This pattern of rigidity does not occur in patients who experience unclassified, absence, or focal seizures.

The family of a patient in the ICU diagnosed with acute pancreatitis asks the nurse why the patient has been moved to an air bed. What would be the nurses best response? A) Air beds allow the care team to reposition her more easily while shes on bed rest. B) Air beds are far more comfortable than regular beds and shell likely have to be on bed rest a long time. C) The bed automatically moves, so shes less likely to develop pressure sores while shes in bed. D) The bed automatically moves, so she is likely to have less pain.

C Feedback: It is important to turn the patient every 2 hours; use of specialty beds may be indicated to prevent skin breakdown. The rationale for a specialty bed is not related to repositioning, comfort, or ease of movement

A nurse is giving an educational class to members of the local disaster team. What should the nurse instruct members of the disaster team to do in a chemical bioterrorist attack? A) Cover their eyes. B) Put on a personal protective equipment mask. C) Stand up. D) Crawl to an exit.

C Feedback: Most chemicals are heavier than air, except for hydrogen cyanide. Therefore, in the presence of most chemicals, people should stand up to avoid heavy exposure because the chemical will sink toward the floor or ground. For this reason, covering their eyes, putting on a PPE mask, or crawling to an exit will not decrease exposure.

The nurse is reviewing the medication administration record of a female patient who possesses numerous risk factors for stroke. Which of the womans medications carries the greatest potential for reducing her risk of stroke? A) Naproxen 250 PO b.i.d. B) Calcium carbonate 1,000 mg PO b.i.d. C) Aspirin 81 mg PO o.d. D) Lorazepam 1 mg SL b.i.d. PR

C Feedback: Research findings suggest that low-dose aspirin may lower the risk of stroke in women who are at risk. Naproxen, lorazepam, and calcium supplements do not have this effect.

A patient has undergone diagnostic testing and received a diagnosis of sinus bradycardia attributable to sinus node dysfunction. When planning this patients care, what nursing diagnosis is most appropriate? A) Acute pain B) Risk for unilateral neglect C) Risk for activity intolerance D) Risk for fluid volume excess

C Feedback: Sinus bradycardia causes decreased cardiac output that is likely to cause activity intolerance. It does not typically cause pain, fluid imbalances, or neglect of a unilateral nature.

A 77-year-old male is admitted to a unit with a suspected diagnosis of acute myeloid leukemia (AML). When planning this patients care, the nurse should be aware of what epidemiologic fact? A) Early diagnosis is associated with good outcomes. B) Five-year survival for older adults is approximately 50%. C) Five-year survival for patients over 75 years old is less than 2%. D) Survival rates are wholly dependent on the patients pre-illness level of health.

C Feedback: The 5-year survival rate for patients with AML who are 50 years of age or younger is 43%; it drops to 19% for those between 50 and 64 years, and drops to1.6% for those older than 75 years. Early diagnosis is beneficial, but is nonetheless not associated with good outcomes or high survival rates. Preillness health is significant, but not the most important variable.

The nursing educator is presenting a case study of an adult patient who has abnormal ventricular depolarization. This pathologic change would be most evident in what component of the ECG? A) P wave B) T wave C) QRS complex D) U wave

C Feedback: The QRS complex represents the depolarization of the ventricles and, as such, the electrical activity of that ventricle.

The current phase of a patients treatment for a burn injury prioritizes wound care, nutritional support, and prevention of complications such as infection. Based on these care priorities, the patient is in what phase of burn care? A) Emergent B) Immediate resuscitative C) Acute D) Rehabilitation

C Feedback: The acute or intermediate phase of burn care follows the emergent/resuscitative phase and begins 48 to 72 hours after the burn injury. During this phase, attention is directed toward continued assessment and maintenance of respiratory and circulatory status, fluid and electrolyte balance, and gastrointestinal function. Infection prevention, burn wound care (i.e., wound cleaning, topical antibacterial therapy, wound dressing, dressing changes, wound dbridement, and wound grafting), pain management, and nutritional support are priorities at this stage. Priorities during the emergent or immediate resuscitative phase include first aid, prevention of shock and respiratory distress, detection and treatment of concomitant injuries, and initial wound assessment and care. The priorities during the rehabilitation phase include prevention of scars and contractures, rehabilitation, functional and cosmetic reconstruction, and psychosocial counseling

A nurse is creating a care plan for a patient with acute pancreatitis. The care plan includes reduced activity. What rationale for this intervention should be cited in the care plan? A) Bed rest reduces the patients metabolism and reduces the risk of metabolic acidosis. B) Reduced activity protects the physical integrity of pancreatic cells. C) Bed rest lowers the metabolic rate and reduces enzyme production. D) Inactivity reduces caloric need and gastrointestinal motility.

C Feedback: The acutely ill patient is maintained on bed rest to decrease the metabolic rate and reduce the secretion of pancreatic and gastric enzymes. Staying in bed does not release energy from the body to fight the disease.

The nurse is caring for a patient in the ICU who has a brain stem herniation and who is exhibiting an altered level of consciousness. Monitoring reveals that the patients mean arterial pressure (MAP) is 60 mm Hg with an intracranial pressure (ICP) reading of 5 mm Hg. What is the nurses most appropriate action? A) Position the patient in the high Fowlers position as tolerated. B) Administer osmotic diuretics as ordered. C) Participate in interventions to increase cerebral perfusion pressure. D) Prepare the patient for craniotomy.

C Feedback: The cerebral perfusion pressure (CPP) is 55 mm Hg, which is considered low. The normal CPP is 70 to 100 mm Hg. Patients with a CPP of less than 50 mm Hg experience irreversible neurologic damage. As a result, interventions are necessary. A craniotomy is not directly indicated. Diuretics and increased height of bed would exacerbate the patients condition

A patient has been admitted to the postsurgical unit following the creation of an ileal conduit. What should the nurse measure to determine the size of the appliance needed? A) The circumference of the stoma B) The narrowest part of the stoma C) The widest part of the stoma D) Half the width of the stoma

C Feedback: The correct appliance size is determined by measuring the widest part of the stoma with a ruler. The permanent appliance should be no more than 1.6 mm (1/8 inch) larger than the diameter of the stoma and the same shape as the stoma to prevent contact of the skin with drainage.

A nurse is caring for a patient who is in the diuresis phase of AKI. The nurse should closely monitor the patient for what complication during this phase? A) Hypokalemia B) Hypocalcemia C) Dehydration D) Acute flank pain

C Feedback: The diuresis period is marked by a gradual increase in urine output, which signals that glomerular filtration has started to recover. The patient must be observed closely for dehydration during this phase; if dehydration occurs, the uremic symptoms are likely to increase. Excessive losses of potassium and calcium are not typical during this phase, and diuresis does not normally result in pain.

The nurse is admitting a patient with complaints of dyspnea on exertion and fatigue. The patients ECG shows dysrhythmias that are sometimes associated with left ventricular hypertrophy. What diagnostic tool would be most helpful in diagnosing cardiomyopathy? A) Cardiac catheterization B) Arterial blood gases C) Echocardiogram D) Exercise stress test

C Feedback: The echocardiogram is one of the most helpful diagnostic tools because the structure and function of the ventricles can be observed easily. The ECG is also important, and can demonstrate dysrhythmias and changes consistent with left ventricular hypertrophy. Cardiac catheterization specifically addresses coronary artery function and arterial blood gases evaluate gas exchange and acid balance. Stress testing is not normally used to differentiate cardiomyopathy from other cardiac pathologies.

The nurse is caring for a recent immigrant who has been diagnosed with mitral valve regurgitation. The nurse should know that in developing countries the most common cause of mitral valve regurgitation is what? A) A decrease in gamma globulins B) An insect bite C) Rheumatic heart disease and its sequelae D) Sepsis and its sequelae

C Feedback: The most common cause of mitral valve regurgitation in developing countries is rheumatic heart disease and its sequelae.

A nurse is providing discharge education to a patient who has undergone a laparoscopic cholecystectomy. During the immediate recovery period, the nurse should recommend what foods? A) High-fiber foods B) Low-purine, nutrient-dense foods C) Low-fat foods high in proteins and carbohydrates D) Foods that are low-residue and low in fat

C Feedback: The nurse encourages the patient to eat a diet that is low in fats and high in carbohydrates and proteins immediately after surgery. There is no specific need to increase fiber or avoid purines. A low-residue diet is not indicated.

A patient is brought to the ED by family members who tell the nurse that the patient has been exhibiting paranoid, agitated behavior. What should the nurse do when interacting with this patient? A) Keep the patient in a confined space. B) Use therapeutic touch appropriately. C) Give the patient honest answers about likely treatment. D) Attempt to convince the patient that his or her fears are unfounded.

C Feedback: The nurse should offer appropriate and honest explanations in order to foster rapport and trust. Confinement is likely to cause escalation, as is touching the patient. The nurse should not normally engage in trying to convince the patient that his or her fears are unjustified, as this can also cause escalation.

A patient is being admitted to the neurologic ICU following an acute head injury that has resulted in cerebral edema. When planning this patients care, the nurse would expect to administer what priority medication? A) Hydrochlorothiazide (HydroDIURIL) B) Furosemide (Lasix) C) Mannitol (Osmitrol) D) Spirolactone (Aldactone)

C Feedback: The osmotic diuretic mannitol is given to dehydrate the brain tissue and reduce cerebral edema. This drug acts by reducing the volume of brain and extracellular fluid. Spirolactone, furosemide, and hydrochlorothiazide are diuretics that are not typically used in the treatment of increased ICP resulting from cerebral edema.

You are a floor nurse caring for a patient with alcohol withdrawal syndrome. What would be an appropriate nursing action to minimize the potential for hallucinations? A) Engage the patient in a process of health education. B) Administer opioid analgesics as ordered. C) Place the patient in a private, well-lit room. D) Provide television or a radio as therapeutic distraction

C Feedback: The patient should be placed in a quiet single room with lights on and in a calm nonstressful environment. TV and radio stimulation should be avoided. Analgesics are not normally necessary, and would potentially contribute to hallucinations. Health education would be inappropriate while the patient is experiencing acute withdrawal.

The ED nurse is caring for a patient with a suspected MI. What drug should the nurse anticipate administering to this patient? A) Oxycodone B) Warfarin C) Morphine D) Acetaminophen

C Feedback: The patient with suspected MI is given aspirin, nitroglycerin, morphine, an IV beta- blocker, and other medications, as indicated, while the diagnosis is being confirmed. Tylenol, warfarin, and oxycodone are not typically used.

The nurse is creating a plan of care for a patient with a cardiomyopathy. What priority goal should underlie most of the assessments and interventions that are selected for this patient? A) Absence of complications B) Adherence to the self-care program C) Improved cardiac output D) Increased activity tolerance

C Feedback: The priority nursing diagnosis of a patient with cardiomyopathy would include improved or maintained cardiac output. Regardless of the category and cause, cardiomyopathy may lead to severe heart failure, lethal dysrhythmias, and death. The pathophysiology of all cardiomyopathies is a series of progressive events that culminate in impaired cardiac output. Absence of complications, adherence to the self-care program, and increased activity tolerance should be included in the care plan, but they do not have the priority of improved cardiac output.

The nurse is providing an educational workshop about coronary artery disease (CAD) and its risk factors. The nurse explains to participants that CAD has many risk factors, some that can be controlled and some that cannot. What risk factors would the nurse list that can be controlled or modified? A) Gender, obesity, family history, and smoking B) Inactivity, stress, gender, and smoking C) Obesity, inactivity, diet, and smoking D) Stress, family history, and obesity

C Feedback: The risk factors for CAD that can be controlled or modified include obesity, inactivity, diet, stress, and smoking. Gender and family history are risk factors that cannot be controlled.

The clinical nurse educator is presenting health promotion education to a patient who will be treated for non-Hodgkin lymphoma on an outpatient basis. The nurse should recommend which of the following actions? A) Avoiding direct sun exposure in excess of 15 minutes daily B) Avoiding grapefruit juice and fresh grapefruit C) Avoiding highly crowded public places D) Using an electric shaver rather than a razor

C Feedback: The risk of infection is significant for these patients, not only from treatment-related myelosuppression but also from the defective immune response that results from the disease itself. Limiting infection exposure is thus necessary. The need to avoid grapefruit is dependent on the patients medication regimen. Sun exposure and the use of razors are not necessarily contraindicated.

A patient with a diagnosis of cirrhosis has developed variceal bleeding and will imminently undergo variceal banding. What psychosocial nursing diagnosis should the nurse most likely prioritize during this phase of the patients treatment? A) Decisional Conflict B) Deficient Knowledge C) Death Anxiety D) Disturbed Thought Processes

C Feedback: The sudden hemorrhage that accompanies variceal bleeding is intensely anxiety-provoking. The nurse must address the patients likely fear of death, which is a realistic possibility. For most patients, anxiety is likely to be a more acute concern than lack of knowledge or decisional conflict. The patient may or may not experience disturbances in thought processes.

A neurologic nurse is reviewing seizures with a group of staff nurses. How should this nurse best describe the cause of a seizure? A) Sudden electrolyte changes throughout the brain B) A dysrhythmia in the peripheral nervous system C) A dysrhythmia in the nerve cells in one section of the brain D) Sudden disruptions in the blood flow throughout the brain

C Feedback: The underlying cause of a seizure is an electrical disturbance (dysrhythmia) in the nerve cells in one section of the brain; these cells emit abnormal, recurring, uncontrolled electrical discharges. Seizures are not caused by changes in blood flow or electrolytes.

The nurse planning the care of a patient with head injuries is addressing the patients nursing diagnosis of sleep deprivation. What action should the nurse implement? A) Administer a benzodiazepine at bedtime each night. B) Do not disturb the patient between 2200 and 0600. C) Cluster overnight nursing activities to minimize disturbances. D) Ensure that the patient does not sleep during the day.

C Feedback: To allow the patient longer times of uninterrupted sleep and rest, the nurse can group nursing care activities so that the patient is disturbed less frequently. However, it is impractical and unsafe to provide no care for an 8-hour period. The use of benzodiazepines should be avoided.

A nurse is performing an admission assessment of a patient with a diagnosis of cirrhosis. What technique should the nurse use to palpate the patients liver? A) Place hand under the right lower abdominal quadrant and press down lightly with the other hand. B) Place the left hand over the abdomen and behind the left side at the 11th rib. C) Place hand under right lower rib cage and press down lightly with the other hand. D) Hold hand 90 degrees to right side of the abdomen and push down firmly.

C Feedback: To palpate the liver, the examiner places one hand under the right lower rib cage and presses downward with light pressure with the other hand. The liver is not on the left side or in the right lower abdominal quadrant.

A nurse is a long-term care facility is admitting a new resident who has a bleeding disorder. When planning this residents care, the nurse should include which of the following? A) Housing the resident in a private room B) Implementing a passive ROM program to compensate for activity limitation C) Implementing of a plan for fall prevention D) Providing the patient with a high-fiber diet

C Feedback: To prevent bleeding episodes, the nurse should ensure that an older adult with a bleeding disorder does not suffer a fall. Activity limitation is not necessarily required, however. A private room is not necessary and there is no reason to increase fiber intake.

A nurse is admitting a patient with a severe migraine headache and a history of acute coronary syndrome. What migraine medication would the nurse question for this patient? A) Rizatriptan (Maxalt) B) Naratriptan (Amerge) C) Sumatriptan succinate (Imitrex) D) Zolmitriptan (Zomig)

C Feedback: Triptans can cause chest pain and are contraindicated in patients with ischemic heart disease. Maxalt, Amerge, and Zomig are triptans used in routine clinical use for the treatment of migraine headaches

A patient who has been experiencing numerous episodes of unexplained headaches and vomiting has subsequently been referred for testing to rule out a brain tumor. What characteristic of the patients vomiting is most consistent with a brain tumor? A) The patients vomiting is accompanied by epistaxis. B) The patients vomiting does not relieve his nausea. C) The patients vomiting is unrelated to food intake. D) The patients emesis is blood-tinged.

C Feedback: Vomiting is often unrelated to food intake if caused by a brain tumor. The presence or absence of blood is not related to the possible etiology and vomiting may or may not relieve the patients nausea.

A hospice nurse is well aware of how difficult it is to deal with others pain on a daily basis. This nurse should put healthy practices into place to guard against what outcome? A) Inefficiency in the provision of care B) Excessive weight gain C) Emotional exhaustion D) Social withdrawal

C Feedback: Well before the nurse exhibits symptoms of stress or burnout, he or she should acknowledge the difficulty of coping with others pain on a daily basis and put healthy practices in place that guard against emotional exhaustion. Emotional exhaustion is more likely to have deleterious effects than inefficiency, social withdrawal, or weight gain, though these may signal emotional exhaustion.

The nurse is caring for a patient receiving hemodialysis three times weekly. The patient has had surgery to form an arteriovenous fistula. What is most important for the nurse to be aware of when providing care for this patient? A) Using a stethoscope for auscultating the fistula is contraindicated. B) The patient feels best immediately after the dialysis treatment. C) Taking a BP reading on the affected arm can damage the fistula. D) The patient should not feel pain during initiation of dialysis.

C Feedback: When blood flow is reduced through the access for any reason (hypotension, application of BP cuff/tourniquet), the access site can clot. Auscultation of a bruit in the fistula is one way to determine patency. Typically, patients feel fatigued immediately after hemodialysis because of the rapid change in fluid and electrolyte status. Although the area over the fistula may have some decreased sensation, a needle stick is still painful.

A nurse is admitting a patient with immune thrombocytopenic purpura to the unit. In completing the admission assessment, the nurse must be alert for what medications that potentially alter platelet function? Select all that apply. A) Antihypertensives B) Penicillins C) Sulfa-containing medications D) Aspirin-based drugs E) NSAIDs

C, D, E Feedback: The nurse must be alert for sulfa-containing medications and others that alter platelet function (e.g., aspirin-based or other NSAIDs). Antihypertensive drugs and the penicillins do not alter platelet function.

A patient has been assessed for aldosteronism and has recently begun treatment. What are priority areas for assessment that the nurse should frequently address? Select all that apply. A) Pupillary response B) Creatinine and BUN levels C) Potassium level D) Peripheral pulses E) BP

C, E Feedback: Patients with aldosteronism exhibit a profound decline in the serum levels of potassium, and hypertension is the most prominent and almost universal sign of aldosteronism. Pupillary response, peripheral pulses, and renal function are not directly affected.

A nurse teaches a client who is prescribed digoxin (Lanoxin) therapy. Which statement should the nurse include in this client's teaching? a. Avoid taking aspirin or aspirin containing products b. increase your intake of foods that are high in potassium c. hold this medication if your pulse rate is below 80 bpm d. do not take this medication within 1 hour of taking an antacid

D

A nurse is caring for a patient who has sustained a deep partial-thickness burn injury. In prioritizing the nursing diagnoses for the plan of care, the nurse will give the highest priority to what nursing diagnosis? A) Activity Intolerance B) Anxiety C) Ineffective Coping D) Acute Pain

D Pain is inevitable during recovery from any burn injury. Pain in the burn patient has been described as one of the most severe causes of acute pain. Management of the often-severe pain is one of the most difficult challenges facing the burn team. While the other nursing diagnoses listed are valid, the presence of pain may contribute to these diagnoses. Management of the patients pain is the priority, as it may have a direct correlation to the other listed nursing diagnoses.

A 15-year-old is admitted to the renal unit with a diagnosis of postinfectious glomerular disease. The nurse should recognize that this form of kidney disease may have been precipitated by what event? A) Psychosocial stress B) Hypersensitivity to an immunization C) Menarche D) Streptococcal infection

D Postinfectious causes of postinfectious glomerular disease are group A beta-hemolytic streptococcal infection of the throat that precedes the onset of glomerulonephritis by 2 to 3 weeks. Menarche, stress, and hypersensitivity are not typical causes

A patient has been taking prednisone for several weeks after experiencing a hypersensitivity reaction. To prevent adrenal insufficiency, the nurse should ensure that the patient knows to do which of the following? A) Take the drug concurrent with levothyroxine (Synthroid). B) Take each dose of prednisone with a dose of calcium chloride. C) Gradually replace the prednisone with an OTC alternative. D) Slowly taper down the dose of prednisone, as ordered.

D Corticosteroid dosages are reduced gradually (tapered) to allow normal adrenal function to return and to prevent steroid-induced adrenal insufficiency. There are no OTC substitutes for prednisone and neither calcium chloride nor levothyroxine addresses the risk of adrenal insufficiency.

A nurse is caring for patient whose diagnosis of multiple myeloma is being treated with bortezomib. The nurse should assess for what adverse effect of this treatment? A) Stomatitis B) Nephropathy C) Cognitive changes D) Peripheral neuropathy

D Feedback: A significant toxicity associated with the use of bortezomib for multiple myeloma is peripheral neuropathy. Stomatitis, cognitive changes, and nephropathy are not noted to be adverse effects of this medication.

A triage nurse is talking to a patient when the patient begins choking on his lunch. The patient is coughing forcefully. What should the nurse do? A) Stand him up and perform the abdominal thrust maneuver from behind. B) Lay him down, straddle him, and perform the abdominal thrust maneuver. C) Leave him to get assistance. D) Stay with him and encourage him, but not intervene at this time.

D Feedback: If the patient is coughing, he should be able to dislodge the object or cause a complete obstruction. If complete obstruction occurs, the nurse should perform the abdominal thrust maneuver with the patient standing. If the patient is unconscious, the nurse should lay the patient down. A nurse should never leave a choking patient alone.

A patient calls his cardiologists office and talks to the nurse. He is concerned because he feels he is being defibrillated too often. The nurse tells the patient to come to the office to be evaluated because the nurse knows that the most frequent complication of ICD therapy is what? A) Infection B) Failure to capture C) Premature battery depletion D) Oversensing of dysrhythmias

D Feedback: Inappropriate delivery of ICD therapy, usually due to oversensing of atrial and sinus tachycardias with a rapid ventricular rate response, is the most frequent complication of ICD. Infections, failure to capture, and premature battery failure are less common.

An emergency department nurse has just admitted a patient with a burn. What characteristic of the burn will primarily determine whether the patient experiences a systemic response to this injury? A) The length of time since the burn B) The location of burned skin surfaces C) The source of the burn D) The total body surface area (TBSA) affected by the burn

D Feedback: Systemic effects are a result of several variables. However, TBSA and wound severity are considered the major factors that affect the presence or absence of systemic effects.

A patient with chronic kidney disease has been hospitalized and is receiving hemodialysis on a scheduled basis. The nurse should include which of the following actions in the plan of care? A) Ensure that the patient moves the extremity with the vascular access site as little as possible. B) Change the dressing over the vascular access site at least every 12 hours. C) Utilize the vascular access site for infusion of IV fluids. D) Assess for a thrill or bruit over the vascular access site each shift.

D Feedback: The bruit, or thrill, over the venous access site must be evaluated at least every shift. Frequent dressing changes are unnecessary and the patient does not normally need to immobilize the site. The site must not be used for purposes other than dialysis.

A patient with a cerebral aneurysm exhibits signs and symptoms of an increase in intracranial pressure (ICP). What nursing intervention would be most appropriate for this patient? A) Range-of-motion exercises to prevent contractures B) Encouraging independence with ADLs to promote recovery C) Early initiation of physical therapy D) Absolute bed rest in a quiet, nonstimulating environment

D Feedback: The patient is placed on immediate and absolute bed rest in a quiet, nonstressful environment because activity, pain, and anxiety elevate BP, which increases the risk for bleeding. Visitors are restricted. The nurse administers all personal care. The patient is fed and bathed to prevent any exertion that might raise BP.

A workplace explosion has left a 40-year-old man burned over 65% of his body. His burns are second- and third-degree burns, but he is conscious. How would this person be triaged? A) Green B) Yellow C) Red D) Black

D Feedback: The purpose of triaging in a disaster is to do the greatest good for the greatest number of people. The patient would be triaged as black due to the unlikelihood of survival. Persons triaged as green, yellow, or red have a higher chance of recovery.

A patient arrives in the emergency department after being burned in a house fire. The patients burns cover the face and the left forearm. What extent of burns does the patient most likely have? A) 13% B) 25% C) 9% D) 18%

D Feedback: When estimating the percentage of body area or burn surface area that has been burned, the Rule of Nines is used: the face is 9%, and the forearm is 9% for a total of 18% in this patient.

A nurse teaches a client who experiences occasional premature atrial contractions (PACs) accompanied by palpitations that resolve spontaneously without treatment. Which statement should the nurse include in this clients teaching? a. minimize or abstain from caffeine b. lie of your side until the attack subsides c. use your oxygen when you experience PACs d. take amiodarone daily to prevent PACs

a. minimize or abstain from caffeine

A nurse is describing the process by which blood is ejected into circulation as the chambers of the heart become smaller. The instructor categorizes this action of the heart as what? A) Systole B) Diastole C) Repolarization D) Ejection fraction

ns: A Feedback: Systole is the action of the chambers of the heart becoming smaller and ejecting blood. This action of the heart is not diastole (relaxations), ejection fraction (the amount of blood expelled), or repolarization (electrical charging).

The nurse is working with a client diagnosed with somatic symptom disorder (SSD). Which distinguishing criterion is present in SSD but absent in illness anxiety disorder (IAD)? 1. Experiences significant physical symptoms 2. Has a change in the quality of self-awareness 3. Has a perceived disturbance in body image or appearance 4. Experiences anxiety about acquiring an illness

1 Individuals experiencing somatic symptoms without corroborating pathology are considered to have SSD, and those with minimal or no somatic symptoms would be diagnosed with IAD, a diagnosis new to the DSM-5.

Which approach should the nurse use to maintain a therapeutic relationship with a client diagnosed with borderline personality disorder (BPD)? 1. Being firm, consistent, and empathic while addressing specific client behaviors 2. Promoting client self-expression by implementing laissez-faire leadership 3. Using authoritative leadership to help clients learn to conform to society norms 4. Overlooking inappropriate behaviors to avoid providing secondary gains

1 The best nursing approach when working with a client diagnosed with borderline personality disorder is to be firm, consistent, and empathetic while addressing specific client behaviors. Individuals with borderline personality disorder always seem to be in a state of crisis and can often have negative patterns of interaction, such as manipulation and splitting.

A client diagnosed with chronic alcohol addiction is being discharged from an inpatient treatment facility after detoxification. Which client outcome, related to Alcoholics Anonymous (AA), would be most appropriate for a nurse to discuss with the client during discharge teaching? 1. To immediately attend AA meetings at least weekly. 2. To rely on an AA sponsor to help control alcohol cravings. 3. To incorporate family in AA attendance. 4. To seek appropriate deterrent medications through AA.

1 The most appropriate client outcome for the nurse to discuss during discharge teaching is attending AA meetings at least weekly. AA is a major self-help organization for the treatment of alcohol addiction. It accepts alcohol addiction as an illness and promotes total abstinence as the only cure.

The client with panic disorder says, "When an attack happens, I feel like I am going to die." Which response should the nurse make? 1. "I know it's frightening, but try to remind yourself that this will only last a short time." 2. "Death from a panic attack happens so infrequently that there is no need to worry." 3. "Most people who experience panic attacks have feelings of impending doom." 4. "Tell me why you think you are going to die every time you have a panic attack."

1 The most appropriate nursing response to the client's concerns is to empathize with the client and provide encouragement that panic attacks only last a short period. Panic attacks usually last minutes but can, rarely, last hours.

The client diagnosed with chronic low back pain is considering acupuncture. Which response would the nurse make when the client asks about how this type of treatment works? 1. "Western medicine believes that acupuncture stimulates the body's release of pain-fighting chemicals called endorphins." 2. "I'm not sure if that would be a good idea. There are a lot of risks, including HIV." 3. "Acupuncture works by encouraging the body to increase its development of serotonin and norepinephrine." 4. "Your acupuncturist is your best resource for answering your specific questions."

1 The most appropriate response by the nurse is to educate the client on the medical philosophy that acupuncture stimulates the body's release of endorphins. The Western medical philosophy regarding acupressure and acupuncture is that they stimulate the body's own painkilling chemicals—the morphine-like substances known as endorphins.

The nursing student, having no knowledge of alternative treatments, states, "Aren't these therapies like a fad? Won't they eventually fade away?" Which response would the nurse make? 1. "Like nursing, complementary therapies take a holistic approach to healing." 2. "The American Nurses Association researches the effectiveness of these therapies." 3. "It is important to remain nonjudgmental about these therapies." 4. "Alternative therapy concepts are rooted in psychoanalysis."

1 The nurse is accurate when comparing complementary therapies to the holistic approach of nursing. Complementary medicine is viewed as holistic health-care, which deals not only with the physical perspective but also the emotional and spiritual components of the individual. The complementary therapies, as well as nursing process, view the person as consisting of multiple, integrated elements. Diagnostic measures are not based on one aspect, but include a holistic assessment of the person.

Family members of a client ask the nurse to explain the difference between schizoid and avoidant personality disorders. Which nursing response is appropriate? 1. Clients diagnosed with avoidant personality disorder desire intimacy but fear it, and clients diagnosed with schizoid personality disorder prefer to be alone. 2. Clients diagnosed with schizoid personality disorder exhibit delusions and hallucinations, while clients diagnosed with avoidant personality disorder do not. 3. Clients diagnosed with avoidant personality disorder are eccentric, and clients diagnosed with schizoid personality disorder are dull and vacant. 4. Clients diagnosed with schizoid personality disorder have a history of psychosis, while clients diagnosed with avoidant personality disorder remain based in reality.

1 The nurse should educate the family that clients diagnosed with avoidant personality disorder desire intimacy but fear it, while clients diagnosed with schizoid personality disorder prefer to be alone.

An adolescent diagnosed with schizophrenia spectrum disorder experiences command hallucinations to harm others. The client's parents ask a nurse, "Where do the voices come from?" Which response should the nurse make? 1. "Your child has a chemical imbalance of the brain, which leads to altered perceptions." 2. "Your child's hallucinations are caused by medication interactions." 3. "Your child has too little serotonin in the brain, causing delusions and hallucinations." 4. "Your child's abnormal hormonal changes have precipitated auditory hallucinations."

1 The nurse should explain that a chemical imbalance of the brain leads to altered perceptions. The current position on the dopamine hypothesis is that positive symptoms (like command hallucinations) may be related to increased numbers of dopamine receptors in the brain causing an imbalance.

The client diagnosed with bipolar disorder: manic episode has a goal of gaining 2 lbs. (0.91 kg.) by the end of the week. Which action would the nurse take to achieve this outcome? 1. Provide client with high-calorie finger foods throughout the day. 2. Accompany client to cafeteria to encourage adequate dietary consumption. 3. Initiate total parenteral nutrition to meet dietary needs. 4. Teach the importance of a varied diet to meet nutritional needs.

1 The nurse should provide the client with high-calorie finger foods throughout the day to help the client achieve the outcome of gaining 2 lbs. (0.91 kg.) by the end of the week. Because of the hyperactive state, the client will have difficulty sitting still to consume large meals. Foods and drinks that can be carried around and eaten with little effort will be more effective.

A client diagnosed with major depression and substance use disorder has an altered sleep pattern and demands a psychiatrist prescribe a sedative. Which rationale explains why the nurse should encourage the client to first try nonpharmacological interventions? 1. Sedative-hypnotics are potentially addictive, and their effectiveness will be compromised owing to tolerance. 2. Sedative-hypnotics decrease the production of needed liver enzymes. 3. Sedative-hypnotics lengthen necessary REM (rapid eye movement, dream) sleep. 4. Sedative-hypnotics are known not to be as effective in promoting sleep as antidepressant medications.

1 The nurse should recommend nonpharmacological interventions to this client because sedative-hypnotics are potentially addictive, and their effectiveness will be compromised owing to tolerance and cross-tolerance. The effects of central nervous system depressants are additive with one another, capable of producing physiological and psychological addiction.

The client with a myocardial infarction tells the intensive care nurse, "You won't have to care for me pretty soon. I will not be a burden to you or others." Which initial action should the nurse take? 1. Screen the client for suicide 2. Transfer the client to the medical unit 3. Allow the client some private, quiet time 4. Teach the client that he or she will be able to care for himself or herself

1 The nurse should screen the client for suicide as the initial action. Even though the client has a myocardial infarction and not a mental health disorder, the client's statement indicates possible depression and suicide. The nurse must remember that clients in all medical settings may not be forthcoming with thoughts of suicide unless specifically asked.

A lonely, depressed divorcée has been self-medicating with small amounts of cocaine for the past year to feel better. Which term should the nurse use in report to best describe this individual's situation? 1. Psychological addiction 2. Codependence 3. Substance induced disorder 4. Intoxication

1 The nurse should use the term psychological addiction to best describe the client's situation. A client is considered to be psychologically addicted to a substance when there is an overwhelming desire to use a drug in order to produce pleasure (feel better) or avoid discomfort.

The nurse is assessing a client diagnosed with hoarding disorder. Which statement would the nurse expect to hear from the client? 1. "I am a perfectionist." 2. "I get obsessive about cleaning my counter tops." 3. "I donate my clothing to charities." 4. "I prefer to have wide walkways in my home."

1 The nurse would expect the client to say this. Associated symptoms of hoarding disorder include perfectionism, indecisiveness, anxiety, depression, distractibility, and difficulty planning and organizing tasks.

Which nursing diagnosis is the priority when providing nursing care to a client diagnosed with paranoid personality disorder? 1. Risk for violence: directed toward others R/T suspicious thinking 2. Risk for suicide R/T altered thought 3. Altered sensory perception R/T increased levels of anxiety 4. Social isolation R/T inability to relate to others

1 The priority nursing diagnosis for a client diagnosed with paranoid personality disorder should be risk for violence: directed toward others R/T suspicious thinking. Clients diagnosed with paranoid personality disorder are often tense and irritable, which increases the likelihood of violent behavior. The desire for reprisal and vindication is so intense that a possible loss of control can result in aggression and violence.

Which nursing diagnosis is the priority for a client experiencing alcohol withdrawal? 1. Risk for injury R/T central nervous system stimulation 2. Disturbed thought processes R/T tactile hallucinations 3. Ineffective coping R/T powerlessness over alcohol use 4. Ineffective denial R/T continued alcohol use despite negative consequences

1 The priority nursing diagnosis for a client experiencing alcohol withdrawal should be risk for injury R/T central nervous system stimulation. Alcohol withdrawal may include the following symptoms: coarse tremors of hands, tongue, or eyelids; nausea or vomiting; malaise or weakness; tachycardia; sweating; elevated blood pressure; anxiety; depressed mood; hallucinations; headache; and insomnia.

The client diagnosed with bipolar disorder intentionally overdoses on antidepressants. Family members report that the client has experienced a relationship break-up, anorexia, and a recent job loss. Which nursing diagnosis is the priority based upon the client's signs and symptoms? 1. Risk for self-directed violence R/T multiple losses 2. Anxiety: severe R/T hyperactivity 3. Imbalanced nutrition: less than body requirements R/T refusal to eat 4. Dysfunctional grieving R/T hopelessness

1 The priority nursing diagnosis for this client should be risk for self-directed violence R/T multiple losses. The nurse should always prioritize client safety. This client is at risk for suicide because of the recent suicide attempt and losses of a relationship and job.

The nursing instructor is teaching about medications used to treat clients diagnosed with panic disorder. Which student statement indicates teaching has been effective? 1. "Clonazepam (Klonopin) is particularly effective in the treatment of panic disorder." 2. "Clonidine (Catapres) is used off-label in long-term treatment of panic disorder." 3. "Atenolol (Tenormin) can be used in low doses to relieve symptoms of panic attacks." 4. "Buspirone (BuSpar) is used for its immediate effect to lower anxiety during panic attacks."

1 The student indicates learning has occurred when he or she states that clonazepam is a particularly effective treatment for panic disorder. Clonazepam is a type of benzodiazepine in which the major risk is physical dependence and tolerance, which may encourage abuse. It can be used on an as-needed basis to reduce anxiety and its related symptoms.

Which statement by the nurse indicates a correct understanding of psychopharmacology for somatic disorders? 1. "Somatization disorders with depression can be treated with selective serotonin reuptake inhibitors." 2. "Anxiety associated with these disorders can be treated long-term with benzodiazepines." 3. "Conversion disorder can be treated with intravenous administration of antidepressants." 4. "First-line treatment for depersonalization-derealization disorder is antianxiety agents."

1 This statement indicates a correct understanding. Psychopharmacology is not effective unless it is being used to treat underlying depression or anxiety. When antidepressant therapy is warranted, selective serotonin reuptake inhibitors are generally preferred.

The nurse is providing discharge teaching to a client about benzodiazepines. Which client statement would indicate a need for further follow-up instructions? 1. "I will need scheduled blood work in order to monitor for toxic levels of this drug." 2. "I won't stop taking this medication abruptly because there could be serious complications." 3. "I will not drink alcohol while taking this medication." 4. "I won't take extra doses of this drug because I can become addicted."

1 This statement indicates a need for additional information about taking benzodiazepines when stating the need for blood work to monitor for toxic levels. Benzodiazepines do not require blood work.

Which of the following are behavior assessment categories in the Broset Violence Checklist? (Select all that apply.) 1. Confusion 2. Paranoia 3. Boisterousness 4. Panic 5. Irritability

1, 3, 5

Place the spectrum of schizophrenic and other psychotic disorders as described by the DSM-5 on a gradient of psychopathology from least to most severe (1-4). (Enter the number of each disorder in the proper sequence, using comma and space format, such as: 1, 2, 3, 4) 1. Delusional disorder 2. Schizophrenia 3. Schizophreniform disorder 4. Substance-induced psychotic disorder

1, 4, 3, 2

the nurse evaluates a client's patient controlled analgesia pump and notices 50 attempts within a 15 minute. What is the best rationale for assessing this patients for substance abuse? 1. Narcotic pain medication is contraindicated for all clients with active substance use disorders. 2. Clients who are addicted to alcohol or benzodiazepines may develop cross-tolerance to analgesics and require increased doses to achieve effective pain control. 3. There is no need to assess the client for substance addiction. There is an obvious PCA malfunction, because these clients have a higher pain tolerance. 4. The client is experiencing alcohol withdrawal symptoms and needs accurate assessment.

2

The client diagnosed with obsessive-compulsive disorder has an elaborate routine for toileting activities. Which would be an appropriate initial client outcome during the first week of hospitalization? 1. The client will refrain from ritualistic behaviors during daylight hours. 2. The client will wake early enough to complete rituals prior to breakfast. 3. The client will participate in three unit activities by day three. 4. The client will substitute a productive activity for rituals by day one.

2 An appropriate initial client outcome is for the client to wake early enough to complete rituals prior to breakfast.

The nurse discovers the client purposefully inserted a contaminated catheter into the urethra, leading to a urinary tract infection. The nurse recognizes this behavior as characteristic of which mental disorder? 1. Illness anxiety disorder 2. Factitious disorder 3. Functional neurological symptom disorder 4. Depersonalization-derealization disorder

2 Factitious disorders involve conscious, intentional feigning of physical or psychological symptoms. Individuals with factitious disorder pretend to be ill in order to receive emotional care and support commonly associated with the role of client. Individuals become very inventive in their quest to produce symptoms.

Which finding would alert the nurse that a client is exhibiting selective amnesia? 1. Cannot relate any lifetime memories. 2. Can describe driving to Iowa but cannot remember the car accident that occurred. 3. Can explain abstract concepts. 4. Cannot provide personal demographic information during admission assessment.

2 In selective amnesia, the individual can recall only certain incidents associated with a stressful event for a specific period after the event, like driving to Iowa but not remembering the car accident.

Which premise is basic to the recovery model used to treat clients diagnosed with bipolar disorder? 1. Medication compliance 2. Independent management 3. Total absence of symptoms 4. Improved psychosocial relationships

2 Independent management is a basic premise for the recovery model. The recovery model has been used primarily in caring for individuals with serious mental illness, such as schizophrenia and bipolar disorder. However, concepts of the model are amenable to use with all individuals experiencing emotional conditions with which they require assistance and who have a desire to take control and manage their lives more independently.

A client who had cardiac surgery 24 hours ago has a urine output averaging 20 mL/hr for 2 hours. The client received a single bolus of 500 mL of intravenous fluid. Urine output for the subsequent hour was 25 mL. Daily laboratory results indicate that the blood urea nitrogen level is 45 mg/dL and the serum creatinine level is 2.2 mg/dL. Based on these findings, the nurse would anticipate that the client is at risk for which of the following? 1. Hypovolemia 2. Acute renal failure 3. Glomerulonephritis 4. Urinary tract infection

2 Rationale:The client who undergoes cardiac surgery is at risk for renal injury from poor perfusion, hemolysis, low cardiac output, or vasopressor medication therapy. Renal insult is signaled by decreased urine output and increased blood urea nitrogen and creatinine levels. The client may need medications to increase renal perfusion and possibly could need peritoneal dialysis or hemodialysis. No data in the question indicate the presence of hypovolemia, urinary tract infection, or glomerulonephritis.

Which nursing intervention is appropriate when caring for a client diagnosed with borderline personality disorder? 1. Seclude the client when inappropriate behaviors are exhibited. 2. Rotate staff members who work with the client. 3. Teach about antianxiety medications to improve medication compliance. 4. Offer sympathy when client engages in self-mutilation.

2 Rotate staff members who work with the client in order to avoid client's developing dependence on particular individuals. These interventions are intended to help the individual understand that staff splitting will not be tolerated, and to work toward diminishing clinging and distancing behaviors.

The client expresses low self-worth, has much difficulty making decisions, avoids positions of responsibility, and has a behavioral pattern of "suffering" in silence. Which information best explains the childhood nurturance of this client's personality disorder? 1. Was provided from many sources, and independent behaviors were encouraged 2. Was provided exclusively from one source, and independent behaviors were discouraged 3. Was provided exclusively from one source, and independent behaviors were encouraged 4. Was provided from many sources, and independent behaviors were discouraged

2 The behaviors presented in the question represent symptoms of dependent personality disorder. Nurturance provided from one source and discouragement of independent behaviors can contribute to the development of this personality disorder. Dependent behaviors may be rewarded by a parent who is overprotective and discourages autonomy.

A nurse questions the charge nurse about an order for fluvoxamine (Luvox) 300 mg daily in two divided doses for a client diagnosed with obsessive-compulsive disorder (OCD). Which charge nurse response is most accurate? 1. "High doses of tricyclic medications will be required for effective treatment of OCD." 2. "High doses of selective serotonin reuptake inhibitor (SSRI), above what is effective for depression, may be required for OCD." 3. "The dose of Luvox is low because of the side effect of daytime drowsiness." 4. "The dose of this SSRI is outside the therapeutic range and needs to be brought to the psychiatrist's attention."

2 The most accurate charge nurse response is that SSRI doses in excess of what is effective for treating depression may be required in the treatment of OCD. SSRIs have been approved by the Food and Drug Administration for the treatment of OCD. Fluvoxamine is an SSRI.

A son is seeking advice about his mother who seems to worry unnecessarily about everything. The son states, "Should I seek psychiatric help for my mother?" Which is an appropriate nursing response? 1. "My mother also worries unnecessarily. I think it is part of the aging process." 2. "Anxiety is considered abnormal when it is out of proportion to the stimulus causing it and when it impairs functioning." 3. "From what you have told me, you should get her to a psychiatrist as soon as possible." 4. "Worry and anxiety are complex phenomena and are effectively treated only with psychotropic medications."

2 The most appropriate response by the nurse is to explain to the son that anxiety is considered abnormal when it is out of proportion and impairs functioning. Anxiety is a normal reaction to a realistic danger or threat to biological integrity or self-concept.

The nurse is working in a long-term care facility. Which action by the nurse demonstrates attention to a priority issue for screening for all clients in the facility? 1. Performing crisis intervention 2. Assessing a client for trauma 3. Determining presence of hallucinations 4. Monitoring for anxiety disorders

2 The nurse should assess for trauma. Three of the most important issue to screen for are trauma, suicide risk, and substance use disorders.

Which nursing diagnosis is the priority for a client experiencing alcohol withdrawal? 1. Risk for injury R/T central nervous system stimulation 2. Disturbed thought processes R/T tactile hallucinations 3. Ineffective coping R/T powerlessness over alcohol use 4. Ineffective denial R/T continued alcohol use despite negative consequences

2 The nurse should assess the client for substance addiction, because clients who are addicted to alcohol or benzodiazepines may have developed cross-tolerance to analgesics and require increased doses to achieve effective pain control. Cross-tolerance is exhibited when one drug results in a lessened response to another drug.

The client presents at an outpatient clinic appointment and states, "My dead husband returned to me during a séance." Which personality disorder should the nurse associate with this behavior? 1. Obsessive-compulsive 2. Schizotypal 3. Narcissistic 4. Borderline

2 The nurse should associate schizotypal personality disorder with this behavior. Magical thinking, ideas of reference, illusions, and depersonalization are part of their everyday world. Examples include superstitiousness; belief in clairvoyance, telepathy, or "sixth sense"; and beliefs that "others can feel my feelings."

Which outcome would the nurse add to the plan of care for an inpatient client diagnosed with somatic symptom disorder (SSD)? 1. The client will admit to fabricating physical symptoms to gain benefits by day three. 2. The client will list three potential adaptive coping strategies to deal with stress by day two. 3. The client will identify the connection between function loss and severe stress by day three. 4. The client will maintain a sense of reality during stressful situations by day four.

2 The nurse should determine that an appropriate outcome for a client diagnosed with SSD would be for the client to list three potential adaptive coping strategies to deal with stress by day two.

The nurse is teaching staff about clients diagnosed with histrionic personality disorder and the quality of their relationships. Which statement made by a staff member indicates learning has occurred? 1. "Their dramatic style tends to make their interpersonal relationships interesting and fulfilling." 2. "Their interpersonal relationships tend to be shallow and fleeting, serving their dependency needs." 3. "They tend to develop few relationships because they are strongly independent but generally maintain deep affection." 4. "They pay particular attention to details, which can interfere with the development of relationships."

2 The nurse should evaluate that learning has occurred when the staff member describes clients diagnosed with histrionic personality disorder as having relationships that are shallow and fleeting. These types of relationships tend to serve their dependency needs.

A nurse is reviewing the stat laboratory data of a client in the emergency department. At what minimum blood alcohol level should a nurse expect intoxication to occur? 1. 50 mg/dL 2. 100 mg/dL 3. 250 mg/dL 4. 300 mg/dL

2 The nurse should expect that 100 mg/dL is the minimum blood alcohol level at which intoxication occurs. Intoxication usually occurs between 100 and 200 mg/dL.

Which client is most likely to be admitted to an inpatient facility for self-destructive behaviors? 1. One with antisocial personality disorder 2. One with borderline personality disorder 3. One with schizoid personality disorder 4. One with paranoid personality disorder

2 The nurse should expect that a client diagnosed with borderline personality disorder would most likely be admitted to an inpatient facility for self-destructive behaviors. The behavior of clients with borderline personality disorder is unstable, and hospitalization is often required as a result of attempts at self-injury, persistent suicide risk, substance abuse and dependence, or a combination of these behaviors.

The client questions the nurse about therapeutic touch. Which nursing response best explains the goal of this therapy? 1. "This enhances circulation to the body by deep, circular massage." 2. "This re-patterns the body's energy field by the use of rhythmic hand motions." 3. "This improves breathing by increasing oxygen to the brain and body tissues." 4. "This decreases blood pressure by releasing body toxins."

2 The nurse should explain that the goal of the practice of therapeutic touch is to re-pattern the body's energy field by the use of rhythmic hand motions. Therapeutic touch is based on the philosophy that the human body projects fields of energy that become blocked when pain or illness occurs. These energy fields extend beyond the surface of the body, meaning the practitioner need not actually touch the client's skin. Instead, the therapist's hands are passed over the client's body, remaining 2 to 4 inches from the skin.

The client diagnosed with bipolar disorder is distraught over insomnia experienced over the last 2 nights and a 13-lb. (5.9 kg.) weight loss over the past 2 weeks. Which nursing diagnosis is priority? 1. Knowledge deficit R/T bipolar disorder 2. Imbalanced nutrition: less than body requirements R/T hyperactivity 3. Risk for suicide R/T powerlessness 4. Altered sleep patterns R/T mania

2 The nurse should identify that the priority nursing diagnosis for this client is altered nutrition: less than body requirements R/T hyperactivity. Because of the client's rapid weight loss, the nurse should prioritize interventions to ensure proper nutrition and physical health.

A client with schizophrenia spectrum disorder presents with bizarre behaviors and delusions. Which nursing action should be prioritized to maintain this client's safety? 1. Monitor for medication nonadherence. 2. Note escalating behaviors immediately. 3. Interpret attempts at communication. 4. Assess triggers for bizarre, inappropriate behaviors.

2 The nurse should note escalating behaviors immediately, to maintain this client's safety. Early intervention may prevent an aggressive response and keep the client and others safe.

A client with a history of alcohol use disorder is brought to an emergency department (ED) by family members who state that the client has had nothing to drink in the last 48 hours. When the nurse reports to the ED physician, which client sign or symptom should be the nurse's first priority? 1. Hearing and visual impairment 2. Blood pressure of 180/100 mm Hg 3. Mood rating of 2/10 on numeric scale 4. Dehydration

2 The nurse should recognize that high blood pressure is a symptom of alcohol withdrawal syndrome and should promptly report this finding to the physician. Complications associated with alcohol withdrawal syndrome may progress to alcohol withdrawal delirium in about the second or third day following cessation of prolonged alcohol use.

The nurse is caring for a client with schizophrenia spectrum disorder who is experiencing a flat affect, paranoid delusions, anhedonia, anergia, neologisms, and echolalia. Which statement correctly differentiates the client's positive and negative symptoms of schizophrenia? 1. Paranoid delusions, anhedonia, and anergia are positive symptoms of schizophrenia. 2. Paranoid delusions, neologisms, and echolalia are positive symptoms of schizophrenia. 3. Paranoid delusions, anergia, and echolalia are negative symptoms of schizophrenia. 4. Paranoid delusions, flat affect, and anhedonia are negative symptoms of schizophrenia.

2 The nurse should recognize that positive symptoms of schizophrenia include, but are not limited to, paranoid delusions, neologisms, and echolalia.

Which information would the nurse include in a teaching session about alternative therapies? 1. These therapies view all humans as being biologically similar. 2. These therapies view a person as a combination of multiple, integrated elements. 3. These therapies focus primarily on the structure and functions of the body. 4. These therapies focus on disease from a purely scientific method perspective.

2 The nurse should understand that alternative therapies view a person as a combination of multiple, integrated elements.

A client has been diagnosed with major depressive episode. After treatment with an antidepressant, the client exhibits pressured speech and flight of ideas. Based on this symptom change, which information should the nurse share with the health-care provider? 1. Ask to increase the dosage of the antidepressant. 2. Ask that the client be reevaluated. 3. Ask to order benztropine (Cogentin) for the extrapyramidal symptoms. 4. Ask that another class of antidepressants be used.

2 The nurse would ask the health-care provider to reevaluate the client because the client switched from depression to mania. Antidepressants carry as high as a 40% risk of potentially triggering a switch from depression to mania in individuals with bipolar disorder. A full manic episode that emerges during antidepressant treatment (e.g., medication, electroconvulsive therapy) but persists beyond the physiological effect of that treatment is sufficient evidence for a manic episode and, therefore, a Bipolar I diagnosis.

The client with bipolar disorder refuses to take lithium carbonate (Lithobid) because of excessive weight gain. In order to increase adherence, which anticonvulsant medication should the nurse be prepared to administer? 1. Sertraline (Zoloft) 2. Valproic acid (Depakote) 3. Verapamil (Calan) 4. Lurasidone (Latuda)

2 Valproic acid is an anticonvulsant that helps stabilize mood. Anticonvulsant drugs used either alone or in combination with lithium have produced satisfactory results in stabilizing moods in bipolar disorder.

Which factor differentiates a client diagnosed with schizotypal personality disorder from a client diagnosed with schizoid personality disorder? 1. Clients diagnosed with schizotypal personality disorder are treated with cognitive behavioral therapy, whereas clients diagnosed with schizoid personality disorder need medications. 2. Clients diagnosed with schizoid personality disorder experience anxiety only in social settings, whereas clients diagnosed with schizotypal personality disorder experience generalized anxiety. 3. Clients diagnosed with schizotypal personality disorder experience social anxiety from paranoid fears, whereas clients diagnosed with schizoid personality disorder would isolate themselves on a continual basis. 4. Clients diagnosed with schizoid personality disorder have magical thinking and depersonalization, whereas clients diagnosed with schizotypal personality disorder do not.

3 A client diagnosed with schizoid personality disorder prefers being alone to being with others. However, clients with schizotypal personality disorder have excessive social anxiety that does not diminish with familiarity and tends to be associated with paranoid fears rather than negative judgments about self.

The nurse is providing care to a depressed, introverted client who is receiving outpatient surgery for a fractured hip. Which action should the nurse take to provide patient-centered care? 1. Refer the client for involuntary hospitalization 2. Allow the client plenty of solitude time to prepare for surgery 3. Involve the client in choosing a blue or green gown to wear 4. Develop a partnership with the spouse who is not withdrawn

3 Allowing the client to make decisions about their care (choosing a blue or green gown) is an indication of patient-centered care. Just because a client has a mental illness does not necessarily mean that they are incapable of making decisions.

Which physically healthy adult client should a nurse identify as exhibiting the characteristics of a dependent personality disorder? 1. Meets social needs by contact with 15 cats 2. Has a history of depending on intense relationships to meet basic needs 3. Lives with parents and relies totally on public transportation 4. Is serious, inflexible, and lacks spontaneity

3 An adult client who lives with parents and totally relies upon public transportation exhibits signs of dependent personality disorder. Dependent personality disorder is characterized by a pervasive and excessive need to be taken care of that leads to submissive behaviors.

Which information would a nurse include in client teaching about social anxiety disorder? 1. Obsessions are the underlying reason for clients to avoid social situations. 2. These people avoid social interactions because of a perceived physical flaw. 3. Individuals with social anxiety disorder avoid performing in front of others. 4. People with this disorder avoid social gatherings because of fear of separation.

3 Clients diagnosed with social anxiety disorder have a marked fear or anxiety about one or more social situations in which the individual is exposed to possible scrutiny by others. Examples include social interactions (e.g., having a conversation, meeting unfamiliar people), being observed (e.g., eating or drinking), and performing in front of others (e.g., giving a speech).

In which setting should the nurse be aware that the client with a substance use disorder would most likely seek initial treatment? 1. Psychiatric hospital 2. Addiction treatment center 3. Urgent care clinic 4. Inpatient psychiatric unit

3 Clients with substance use disorders commonly seek care first in general medical and community practice settings, like an urgent care clinic, before being treated in psychiatric or substance abuse treatment settings.

Which criteria according to the DSM-5 would need to be present for a client to be diagnosed with dissociative fugue? 1. An inability to recall important autobiographical information 2. Clinically significant distress in social and occupational functioning 3. Sudden unexpected travel or bewildered wandering 4. "Blackouts" related to alcohol toxicity

3 Dissociative fugue is characterized by a sudden, unexpected travel away from customary place of daily activities, or by bewildered wandering, with the inability to recall some or all of one's past.

Which nursing action would be identified with Stage IV of Roberts' Seven-stage Crisis Intervention Model? 1. Collaboratively implement an action plan. 2. Help the client identify the major problems or crisis precipitants. 3. Help the client deal with feelings and emotions. 4. Collaboratively generate and explore alternatives.

3 Helping a client deal with feelings and emotions aligns with Stage IV: Deal with Feelings and Emotions.

The nurse is caring for clients in a free community clinic. Which technique should the nurse use to conduct a trauma screening? 1. Quickly assessing the overall situation 2. Implementing a thorough head-to-toe assessment 3. Interviewing in a secluded area 4. Using empathy with the family members

3 Interviewing in a secluded area is the technique the nurse should use during a trauma screening. It is critical that nurses conduct trauma screenings in private and communicate with a compassionate, nonjudgmental attitude.

A client diagnosed with a gambling disorder asks the nurse about medications that may be ordered by the physician to treat this disorder. The nurse would give the client information on which medications? 1. Escitalopram (Lexapro) and clozapine (Clozaril) 2. Citalopram (Celexa) and olanzapine (Zyprexa) 3. Lithium carbonate (Lithobid) and naltrexone (ReVia) 4. Haloperidol (Haldol) and ziprasidone (Geodon)

3 Lithium carbonate (Lithobid) and naltrexone (ReVia) have demonstrated some effectiveness for gambling disorder.

The client with migraine headaches says, "My nurse practitioner told me that acupuncture may enhance the effects of my medications." Which type of therapy is the nurse practitioner recommending? 1. Alternative 2. Physiological 3. Complementary 4. Psychosocial

3 Since the acupuncture is being used in combination with medication, the nurse practitioner is recommending a type of complementary therapy. It is complementing the conventional methods (medication) with the addition of an alternative strategy (acupuncture).

A nurse is preparing a staff education session about the impaired nurse and the consequences of this impairment. Which statement by a staff member indicates successful teaching? 1. "The state board of nursing must be notified with subjective documentation of impairment." 2. "All state boards of nursing have passed laws that, under any circumstances, do not allow impaired nurses to practice." 3. "Some state boards of nursing administer the treatment programs themselves, while others refer the nurse to other resources." 4. "After a return to practice, a recovering nurse may be closely monitored for several days."

3 Some of these state boards administer the treatment programs themselves, and others refer the nurse to community resources or state nurses' association assistance programs. This may require successful completion of inpatient, outpatient, group, or individual counseling treatment program(s); evidence of regular attendance at nurse support groups or 12-step program; random negative drug screens; and employment or volunteer activities during the suspension period.

A client's wife has been making excuses for her alcoholic husband's work absences. In family therapy, she states, "His problems at work are my fault." Which response by the nurse is therapeutic? 1. "Why do you assume responsibility for his behaviors?" 2. "I think you should start to confront his behavior." 3. "Your husband needs to deal with the consequences of his drinking." 4. "Do you understand what the term enabler means?"

3 The appropriate nursing response is to use confrontation with caring. The nurse should understand that the client's wife may be in denial and enabling the husband's behavior. Codependency is a typical behavior of spouses of alcoholics. The nurse must help the wife through the stages of recovery beginning with Stage I: The Survival Stage in which the partner begins to let go of the denial that problems exist.

When planning care for clients diagnosed with personality disorders, which treatment goal is appropriate? 1. To stabilize the client's pathology by using the correct combination of psychotropic medications 2. To change the characteristics of the dysfunctional personality 3. To reduce personality trait inflexibility that interferes with functioning and relationships 4. To decrease the prevalence of neurotransmitters at receptor sites

3 The goal of treatment for clients diagnosed with personality disorders should be to reduce inflexibility of personality traits that interfere with functioning and relationships.

The nursing instructor is teaching about bipolar disorders. Which symptom should the instructor include that is present during a manic episode but is absent in hypomania? 1. Inflated self-esteem or grandiosity 2. Decreased need for sleep 3. Psychosis 4. Flight of ideas or racing thoughts

3 The instructor would include psychosis in the teaching session. In hypomania the episode is not severe enough to cause marked impairment in social or occupational functioning or to necessitate hospitalization. If there are psychotic features, the episode is, by definition, manic. Therefore, psychotic features occur with mania but not with hypomania.

The client diagnosed with paranoid personality disorder becomes aggressive on the unit. Which nursing intervention is most appropriate? 1. Provide objective evidence that reasons for violence are unwarranted. 2. Initially restrain the client to maintain safety. 3. Use clear, calm statements with a confident physical stance. 4. Empathize with the client's paranoid perceptions.

3 The most appropriate nursing intervention is to use clear, calm statements with a confident physical stance. A calm attitude provides the client with a feeling of safety and security.

A client diagnosed with schizophrenia spectrum disorder states, "Can't you hear him? It's the devil. He's telling me I'm going to hell." Which is the most appropriate nursing response? 1. "Did you take your medicine this morning?" 2. "You are not going to hell. You are a good person." 3. "The voices must sound scary, but I do not hear any voices." 4. "The devil only talks to people who are receptive to his influence."

3 The most appropriate nursing response is to reassure the client while not reinforcing the hallucination. Reminding the client that "the voices" are not real will prevent validation of the hallucination. It is also important for the nurse to connect with the client's fears and inner feelings.

Parents ask a nurse how they should reply when their son, diagnosed with schizophrenia spectrum disorder, tells them that voices command him to harm others. Which nursing response is appropriate? 1. "Tell him to stop discussing the voices." 2. "Ignore what he is saying, while attempting to discover the underlying cause." 3. "Focus on the feelings generated by the hallucinations and present reality." 4. "Present objective evidence that the voices are not real."

3 The most appropriate response by the nurse is to instruct the parents to focus on the feelings generated by the hallucinations and present reality. The parents should accept that their child is experiencing the hallucination but should not reinforce this unreal sensory perception.

The aging client takes an antipsychotic for schizophrenia spectrum disorder and a beta-adrenergic blocking agent for hypertension. Based on an understanding of the combined side effects of these drugs, which statement by a nurse is most appropriate? 1. "Make sure you concentrate on taking slow, deep, cleansing breaths." 2. "Watch your diet and try to engage in some regular physical activity." 3. "Rise slowly when you change position from lying to sitting or sitting to standing." 4. "Wear sunscreen and try to avoid midday sun exposure."

3 The most appropriate statement by the nurse is to instruct the client to rise slowly when changing positions. Antipsychotic medications and beta blockers cause a decrease in blood pressure. When given in combination, the additive effect of these drugs places the client at risk for developing orthostatic hypotension.

The client is experiencing a severe panic attack. Which nursing intervention would meet this client's physiological need? 1. Teach deep breathing relaxation exercises. 2. Place the client in a brightly lit room. 3. Have the client breathe into a paper bag. 4. Administer the ordered prn buspirone (BuSpar).

3 The nurse can meet this client's physiological need by having the client breathe into a paper bag. Hyperventilation may occur during periods of extreme anxiety. Hyperventilation causes the amount of carbon dioxide (CO2) in the blood to decrease, possibly resulting in lightheadedness, rapid heart rate, shortness of breath, numbness or tingling in the hands or feet, and syncope. If hyperventilation occurs, assist the client to breathe into a small paper bag held over the mouth and nose. Six to 12 natural breaths should be taken, alternating with short periods of diaphragmatic breathing.

The client diagnosed with dissociative identity disorder (DID) switches personalities when confronted by the nurse about inappropriate actions. The nurse recognizes that this dissociation serves which function? 1. It is a means to attain secondary gain. 2. It is a means to explore feelings of excessive and inappropriate guilt. 3. It serves to isolate painful events so the person's awareness and anxiety are decreased. 4. It serves to establish personality boundaries and limit inappropriate impulses.

3 The nurse should anticipate that a client who switches personalities when confronted about inappropriate actions is dissociating in order to isolate painful events to reduce a client's awareness and anxiety associated with events that are perceived as extremely stressful. The transition between personalities is usually sudden, dramatic, and precipitated by stress.

The client presents with symptoms of alcohol withdrawal and states, "I haven't eaten in three days." The nurse's assessment reveals BP 170/100 mm Hg, P 110, R 28, and T 97°F (36°C) with dry mucous membranes and poor skin turgor. What should be the priority nursing diagnosis? 1. Denial 2. Fluid volume excess 3. Imbalanced nutrition: less than body requirements 4. Ineffective individual coping

3 The nurse should assess that the priority nursing diagnosis is imbalanced nutrition: less than body requirements based upon the client's statement regarding lack of nutritional intake for three days. The client is exhibiting signs and symptoms of malnutrition, as well as alcohol withdrawal. The nurse should consult a dietitian, restrict sodium intake to minimize fluid retention, and provide small, frequent feedings of nonirritating foods.

The client diagnosed with schizophrenia spectrum disorder tells the nurse, "I'm sad that the voice is telling me to stop seeing my psychiatrist." Which symptom is the client exhibiting? 1. Magical thinking 2. Persecutory delusions 3. Command hallucinations 4. Altered thought processes

3 The nurse should determine that the client is exhibiting command hallucinations. Clients with command hallucinations could potentially be physically, emotionally, and/or sexually harmful to others or to self.

The client living in a riverfront community is seeking help with an extreme fear of bridges, which is interfering with daily functioning. The psychiatric nurse practitioner decides to try systematic desensitization. Which best explanation of this treatment should the nurse provide? 1. "Using your imagination, we will attempt to achieve a state of relaxation." 2. "Because anxiety and relaxation are mutually exclusive states, we can attempt to substitute a relaxation response for the anxiety response." 3. "Through a series of increasingly anxiety-provoking steps, we will gradually increase your tolerance to anxiety while in a relaxed state." 4. "In one intense session, you will be exposed to a maximum level of anxiety that you will learn to tolerate."

3 The nurse should explain to the client that when participating in systematic desensitization, he or she will go through a series of increasingly anxiety-provoking steps that will gradually increase tolerance while in a relaxed state. Systematic desensitization was introduced by Joseph Wolpe in 1958 and is based on behavioral conditioning principles.

Which reaction to a compliment from a staff member should the nurse identify as a typical response from a client diagnosed with avoidant personality disorder? 1. Interpreting the compliment as a secret code used to increase personal power 2. Feeling the compliment was well deserved 3. Being grateful for the compliment but fearing later rejection and humiliation 4. Wondering what deep meaning and purpose is attached to the compliment

3 The nurse should identify that a client diagnosed with avoidant personality disorder would be grateful for the compliment but would fear later rejection and humiliation. Individuals diagnosed with avoidant personality disorder are extremely sensitive to rejection and are often awkward and uncomfortable in social situations.

Which medication treatment should the nurse administer to clients diagnosed with generalized anxiety disorder (GAD)? 1. Long-term treatment with diazepam (Valium) 2. Acute symptom control with propranolol (Inderal) 3. Long-term treatment with buspirone (BuSpar) 4. Acute symptom control with ziprasidone (Geodon)

3 The nurse should identify that an appropriate treatment for clients diagnosed with GAD is long-term treatment with buspirone. Buspirone is an anxiolytic medication that is effective in 60% to 80% of clients diagnosed with GAD. Buspirone takes 10 to 14 days for alleviation of symptoms but does not have the dependency concerns of other anxiolytics.

The chronically fatigued client is diagnosed with major depressive disorder. After taking antidepressant medication for 6 weeks, the client's symptoms have not resolved. Which nutritional deficiency should a nurse identify as potentially contributing to the client's symptoms? 1. Vitamin A 2. Vitamin C 3. Iron 4. Calcium

3 The nurse should identify that an iron deficiency could contribute to feelings of chronic fatigue because iron is needed to produce hemoglobin. Iron should be consumed by eating meat, fish, green leafy vegetables, nuts, eggs, and enriched bread and pasta.

Looking at a slightly bleeding paper cut, the client screams, "Somebody help me quick! I'm bleeding. Call 911!" The nurse should identify this behavior as characteristic of which personality disorder? 1. Schizoid 2. Obsessive-compulsive 3. Histrionic 4. Paranoid

3 The nurse should identify this behavior as characteristic of histrionic personality disorder. Individuals with this disorder tend to be self-dramatizing, attention seeking, over-gregarious, and seductive.

The nurse develops the following outcomes for a client diagnosed with bipolar disorder: manic episode: 1. Maintain nutritional status. 2. Interact appropriately with peers. 3. Remain free from injury. 4. Sleep 6 to 8 hours a night. In which order would the nurse prioritize the outcomes, starting with the highest and ending with the lowest priority outcome? 1. 2, 1, 3, 4 2. 4, 1, 2, 3 3. 3, 1, 4, 2 4. 1, 4, 2, 3

3 The nurse should order client outcomes based on priority in the following order: Remain free of injury, maintain nutritional status, sleep 6 to 8 hours a night, and interact appropriately with peers. The nurse should prioritize the client's safety (remain free of injury) first, followed by physical health (nutrition and then sleep), and ending with social needs, interacting with others.

A client with schizophrenia spectrum disorder presents with bizarre behaviors and delusions. Which nursing action should be prioritized to maintain this client's safety? 1. Monitor for medication nonadherence. 2. Note escalating behaviors immediately. 3. Interpret attempts at communication. 4. Assess triggers for bizarre, inappropriate behaviors.

3 The nurse should plan to teach the client how to make eye contact when communicating. Social skills, such as making eye contact, can assist clients to communicate needs and to establish relationships.

The client is newly diagnosed with dissociative identity disorder (DID) stemming from severe childhood sexual abuse. Which nursing intervention takes priority? 1. Encourage exploration of sexual abuse. 2. Suggest guided imagery. 3. Establish trust and rapport. 4. Administer antianxiety medications.

3 The nurse should prioritize establishing trust and rapport when beginning to work with a client diagnosed with DID. DID was formerly called multiple personality disorder. Trust is the basis of every therapeutic relationship. Each personality views itself as a separate entity and must be treated as such to establish rapport.

Which symptom should the nurse observe in a client diagnosed with obsessive-compulsive personality disorder? 1. Intrusive and persistent thoughts 2. Unwanted, repetitive ritualistic behavior 3. Lack of spontaneity when dealing with others 4. Feelings of "sixth sense" that are externally imposed

3 The nurse should suspect a diagnosis of obsessive-compulsive personality disorder when a client experiences lack of spontaneity. Individuals with this disorder are very serious, formal, and have difficulty expressing emotions. They are perfectionistic and preoccupied with rules.

The college student has severe test anxiety. Instead of studying for finals, the student relieves stress by attending a movie. Which priority nursing diagnosis should the campus nurse assign for this student? 1. Non-adherence R/T test taking 2. Ineffective role performance R/T helplessness 3. Ineffective coping R/T anxiety 4. Powerlessness R/T fear

3 The priority nursing diagnosis for this student is ineffective coping R/T anxiety. The nurse should assist in implementing interventions that will improve the student's healthy coping skills and reduce anxiety.

On the first day of a client's alcohol detoxification, which nursing intervention should take priority? 1. Strongly encourage the client to attend 90 Alcoholics Anonymous (AA) meetings in 90 days. 2. Educate the client about the biopsychosocial consequences of alcohol abuse. 3. Administer ordered chlordiazepoxide (Librium) in a dosage according to protocol. 4. Provide thiamin supplements to prevent Wernicke-Korsakoff syndrome.

3 The priority nursing intervention for this client should be to administer ordered chlordiazepoxide in a dosage according to protocol. Chlordiazepoxide is a benzodiazepine and is often used for medication-assisted therapy in alcohol withdrawal to reduce life-threatening complications.

A client has experienced an episode of pulmonary edema. The nurse determines that the client's respiratory status is improving if which breath sounds are noted? 1.Rhonchi 2.Wheezes 3.Crackles in the lung bases 4.Crackles throughout the lung fields

3.Crackles in the lung bases

A home care nurse notices the client who startles easily is exhibiting signs of posttraumatic stress disorder. The nurse asks, "Have you ever made a suicide attempt?" to which the client responds, "Yes, I have." Which response should the nurse make next? 1. Notify the primary care provider 2. Gently touch the client's arm 3. Say, "Why would you do that? I am here to help you." 4. Ask, "Are you having thoughts of suicide right now?"

4 A "yes" response to either question ("In the past month, have you had thoughts about suicide?" and "Have you ever made a suicide attempt?") should prompt the nurse to ask a third question, "Are you having thoughts of suicide right now?"

Which client statement demonstrates positive progress toward recovery from a substance use disorder? 1. "I have completed detox and therefore am in control of my drug use." 2. "When I can't control my cravings, I will faithfully attend Narcotic Anonymous." 3. "As a church deacon, my focus will now be on spiritual renewal." 4. "Taking those pills got out of control. It cost me my job, marriage, and children."

4 A client who takes responsibility for the consequences of substance use disorder or substance addiction is making positive progress toward recovery. This would indicate completion of the first step of a 12-step program (AA).

Which nursing diagnosis should a nurse identify as appropriate when working with a client diagnosed with schizoid personality disorder? 1. Altered thought processes R/T increased stress 2. Risk for suicide R/T loneliness 3. Risk for violence: directed toward others R/T paranoid thinking 4. Social isolation R/T inability to relate to others

4 An appropriate nursing diagnosis when working with a client diagnosed with schizoid personality disorder is social isolation R/T inability to relate to others. These clients avoid close or romantic relationships, interpersonal attachments, and intimate sexual relationships.

A highly agitated client paces the unit and states, "I could buy and sell this place." The client's mood fluctuates from fits of laughter to outbursts of anger. Which chart entry would the nurse document for this client's behavior? 1. "Rates mood 8/10. Exhibiting looseness of association. Euphoric." 2. "Mood irritability. Exhibiting magical thinking. Nervousness." 3. "Blunted affect. Exhibiting delusions of reference. Hyperactive." 4. "Agitated and pacing. Exhibiting grandiosity. Mood labile."

4 Based upon the client's behaviors and statements, the nurse would chart "Agitated and pacing. Exhibiting grandiosity. Mood labile." The client is exhibiting mood labile from fits of laughter to outbursts of anger. Grandiosity refers to the attitude that one's abilities are better than everyone else's (I could buy and sell this place).

When a practitioner corrects subluxation by manipulating the vertebrae of the spinal column, which therapy is the practitioner employing? 1. Allopathic 2. Therapeutic touch 3. Massage 4. Chiropractic

4 Chiropractic therapy involves the correction of subluxations by manipulating the vertebrae of the spinal column. The theory behind chiropractic medicine is that energy flows from the brain to all parts of the body through the spinal cord and spinal nerves.

The nurse presents a staff development session about the various challenges in the treatment of clients diagnosed with bipolar disorder. Which factor should the nurse include that is an essential component of relapse prevention? 1. Methods to treat insomnia 2. Inability to control irritability 3. Lack of insight 4. Medication adherence

4 Medication adherence must be emphasized as an essential component of relapse prevention. Discontinuation can result in return of bipolar symptoms.

Which client response would reflect the impulsive self-destructive behavior that is commonly associated with borderline personality disorder when the day-shift nurse leaves the unit? 1. The client suddenly leans on the nurse's arm and whispers, "The night nurse is evil. You have to stay." 2. The client suddenly hugs the nurse's arm and states, "I will be up all night if you don't stay with me." 3. The client suddenly grabs the nurse's arm, yelling, "Please don't go! I can't sleep without you being here." 4. The client suddenly shows the nurse a bloody arm and states, "I cut myself because you are leaving me."

4 The client who states, "I cut myself because you are leaving me," reflects impulsive self-destructive behavior that is commonly associated with borderline personality disorder. Repetitive, self-mutilating behaviors are common in clients diagnosed with borderline personality disorders that result from feelings of abandonment following separation from significant others.

Which nursing intervention would be most appropriate when caring for an agitated, suspicious client diagnosed with schizophrenia spectrum disorder? 1. Supply neon lights and soft music. 2. Maintain continual eye contact throughout the interview. 3. Use therapeutic touch to increase trust and rapport. 4. Provide personal space to respect the client's boundaries.

4 The most appropriate nursing intervention is to provide personal space to respect the client's boundaries. Providing personal space may serve to reduce anxiety and thus reduce the client's risk for violence. The nurse should observe the client while carrying out routine tasks.

The nurse holds the hand of a client who is experiencing alcohol withdrawal. The nurse is assessing for which condition? 1. Emotional strength 2. Wernicke-Korsakoff syndrome 3. Tachycardia 4. Coarse tremors

4 The nurse is most likely assessing the client for coarse tremors secondary to alcohol withdrawal. Withdrawal from alcohol can also cause headache, insomnia, transient hallucinations, depression, irritability, anxiety, elevated blood pressure, sweating, tachycardia, malaise, and coarse tremors.

The nurse is caring for a client diagnosed with dissociative identity disorder (DID). What is the primary goal of therapy for this client? 1. To recover memories while improving thinking patterns 2. To prevent social isolation 3. To decrease anxiety and need for secondary gain 4. To collaborate among subpersonalities to improve functioning

4 The nurse should anticipate that the primary therapeutic goal for a client diagnosed with DID is to collaborate among subpersonalities to improve functioning. Some clients choose to pursue a lengthy therapeutic regimen to achieve integration, a blending of all the personalities into one. The goal is to optimize the client's functioning and potential.

During an interview, which client statement should alert the nurse to a potential diagnosis of schizotypal personality disorder? 1. "I don't have a problem. My family is inflexible, and my relatives are out to get me." 2. "I am so excited about working with you. Have you noticed my new nail polish, 'Ruby Red Roses'?" 3. "I spend all my time tending my bees. I know a whole lot of information about bees." 4. "I am getting a message from the beyond that we have been involved with each other in a previous life."

4 The nurse should assess that a client who states that he or she is getting a message from beyond indicates a potential diagnosis of schizotypal personality disorder. The person experiences magical thinking, ideas of reference, illusions, and depersonalization as part of daily life. Examples include superstitiousness; belief in clairvoyance; telepathy, or "sixth sense"; and beliefs that "others can feel my feelings."

Which client statement indicates a knowledge deficit related to a substance use disorder? 1. "Although it's legal, alcohol is one of the most widely abused drugs in our society." 2. "Tolerance to heroin develops quickly." 3. "Flashbacks from lysergic acid diethylamide (LSD) use may reoccur spontaneously." 4. "Marijuana is like smoking cigarettes. Everyone does it. It's essentially harmless."

4 The nurse should determine that the client has a knowledge deficit related to substance use disorders when the client compares marijuana to smoking cigarettes and claims it to be harmless. The evidence of research indicates that smoked marijuana is harmful.

The client is newly diagnosed with obsessive-compulsive disorder and spends 50 minutes folding clothes and rearranging them in drawers. Which nursing action would best address this client's problem? 1. Distract the client with other activities whenever ritual behaviors begin. 2. Report the behavior to the psychiatrist to obtain an order for medication dosage increase. 3. Lock the room to discourage ritualistic behavior. 4. Discuss the anxiety-provoking triggers that precipitate the ritualistic behaviors.

4 The nurse should discuss with the client the anxiety-provoking triggers that precipitate the ritualistic behavior. If the client is going to be able to control interrupting anxiety, he or she must first learn to recognize precipitating factors.

The client asks a nurse to explain the difference between alternative and complementary medicine. Which is an appropriate nursing response? 1. "Alternative medicine is a more acceptable practice than complementary medicine." 2. "Alternative and complementary medicine are terms that essentially mean the same thing." 3. "Complementary medicine disregards traditional medical approaches." 4. "Complementary therapies partner alternative approaches with traditional medical practice."

4 The nurse should explain to the client that complementary therapies partner alternative approaches with traditional medicine. About 34 percent of adults in the US use some form of complementary or alternative therapy.

Which are examples of primary and secondary gains that a client diagnosed with SSD: predominately pain, may experience? 1. Primary: chooses to seek a new health-care provider; Secondary: euphoric feeling from new medications 2. Primary: euphoric feeling from new medications; Secondary: chooses to seek a new health-care provider 3. Primary: receives get-well cards; Secondary: pain prevents attending stressful family reunion 4. Primary: pain prevents attending stressful family reunion; Secondary: receives get-well cards

4 The nurse should identify that primary gains are those that allow the client to avoid an unpleasant activity (stressful family reunion) and that secondary gains are those in which the client receives emotional support or attention (get-well cards).

The client taking lithium carbonate (Lithobid) presents to an emergency department with a temperature of 101°F (38°C), severe diarrhea, blurred vision, and tinnitus. Which interpretation should the nurse make regarding these symptoms? 1. Symptoms indicate spending too much time outdoors without sunblock. 2. Symptoms indicate antipsychotic withdrawal syndrome. 3. Symptoms indicate the development of neutropenia. 4. Symptoms indicate lithium carbonate toxicity.

4 The nurse should interpret that the client's symptoms indicate lithium carbonate toxicity. The signs and symptoms of toxicity include ataxia, blurred vision, severe diarrhea, persistent nausea and vomiting, and tinnitus. Lithium levels should be monitored monthly with maintenance therapy to ensure therapeutic blood levels.

A cab driver, stuck in traffic, becomes lightheaded, tremulous, diaphoretic, tachycardic, and dyspneic. A work-up in an emergency department reveals no pathology. Which nursing diagnosis should be the nurse's first priority? 1. Fear 2. Powerlessness 3. Altered role performance 4. Anxiety

4 The nurse should suspect that the client has exhibited signs and symptoms of a panic disorder. The priority nursing diagnosis should be anxiety. Panic disorder is characterized by recurrent, sudden-onset panic attacks in which the person feels intense fear, apprehension, or terror.

The nurse in the emergency department (ED) is assessing a client who with a long history of depression. The nurse finds that the client has gained weight, has dry skin, and has cold sensitivity. The nurse determines the client's depression is exacerbating, further examination and testing reveal the client has hypothyroidism. Which phenomenon occurred? 1. Depression screening 2. Social distancing 3. Trauma-informed caring 4. Diagnostic overshadowing

4 The nurse used diagnostic overshadowing, a phenomenon in which clients' physical symptoms are attributed to their mental illness. The nurse attributed the weight gain, dry skin, and cold sensitivity to the depression rather than to hypothyroidism.

Which describes the rationale for holding a debriefing session with clients and staff after a take-down intervention has taken place in an inpatient unit? 1. Reinforce unit rules with the client population. 2. Create protocols for the future release of tensions associated with anger. 3. Process client feelings and alleviate fears of undeserved seclusion and restraint. 4. Discuss the situation that led to inappropriate expressions of anger.

4 The nurse would determine that the purpose for holding a debriefing session with clients and staff after a take-down intervention is to discuss the situation that led to inappropriate behavior. It is important to determine the factors leading to the inappropriate behavior in order to develop future intervention strategies. It is also important to help clients and staff process feelings about the situation.

A new mother is concerned about her ability to perform her parental role. She is quite anxious and ambivalent about leaving the postpartum unit. To offer effective client care, a nurse would note that this type of crisis is precipitated by 1. Unexpected external stressors. 2. Preexisting psychopathology. 3. An acute response to an external situational stressor. 4. Normal life-cycle transitions that overwhelm the client.

4 The nurse would understand that this type of crisis is precipitated by normal life-cycle transitions that overwhelm the client. Reassurance and guidance would be provided as needed, and the client should be referred to services that can provide assistance.

The nurse tells a client diagnosed with obsessive-compulsive personality disorder that the nursing staff will start alternating weekend shifts. Which response should the nurse expect from this client? 1. "You really don't have to go by that schedule. I'd just stay home sick." 2. "There has got to be a hidden agenda behind this schedule change." 3. "Who do you think you are? I expect to interact with the same nurse every Saturday." 4. "You can't make these kinds of changes! Isn't there a rule that governs this decision?"

4 The statement, "You can't make these kinds of changes! Isn't there a rule that governs this decision?" is typical of a client with obsessive-compulsive disorder. The nurse should identify that a client with obsessive-compulsive personality disorder would have a difficult time accepting changes. This disorder is characterized by inflexibility and lack of spontaneity. Individuals with this disorder are very serious, formal, over-disciplined, perfectionistic, and preoccupied with rules.

The nursing instructor is teaching about the DSM-5 diagnostic criteria for depersonalization-derealization disorder. Which student statement indicates a need for follow-up instruction? 1. "Clients with this disorder can experience emotional and/or physical numbing and a distorted sense of time." 2. "Clients with this disorder can experience unreality or detachment with respect to their surroundings." 3. "During the course of this disorder, individuals or objects are experienced as dreamlike, foggy, lifeless, or visually distorted." 4. "During the course of this disorder, the client is out of touch with reality and is impaired in social, occupational, or other areas of functioning."

4 This statement indicates a need for further follow-up instruction. The DSM-5 states that during the depersonalization and/or derealization experiences, reality testing remains intact, but the symptoms cause significant distress or impairment in social, occupation, or other important areas of functioning. Depersonalization (a disturbance in the perception of oneself) is differentiated from derealization, which describes an alteration in the perception of the external environment.

The nurse is teaching a client diagnosed with anxiety about treatment options. Which statement by the client indicates effective teaching? 1. "There is nothing that I can do to that will reduce anxiety." 2. "Medication is available, but only for those who have had anxiety for a year or more." 3. "If I ignore the symptoms of anxiety, it will go away." 4. "Practicing yoga or meditation may help reduce my anxiety."

4 This statement indicates effective teaching. Practicing yoga or meditation may help reduce the symptoms of anxiety. These are examples of stress management.

Which term should a nurse use to describe the administration of a central nervous system (CNS) depressant to a client with alcohol withdrawal? 1. Antagonist treatment 2. Deterrent therapy 3. Codependency therapy 4. Medication-assisted treatment

4 Various medications have been used to decrease the intensity of symptoms in an individual who is withdrawing from, or who is experiencing the effects of excessive use of, alcohol and other drugs. This is called medication-assisted treatment.

A 16-year-old female patient experiences alopecia resulting from chemotherapy, prompting the nursing diagnoses of disturbed body image and situational low self-esteem. What action by the patient would best indicate that she is meeting the goal of improved body image and self-esteem? A) The patient requests that her family bring her makeup and wig. B) The patient begins to discuss the future with her family. C) The patient reports less disruption from pain and discomfort. D) The patient cries openly when discussing her disease.

A

A 50-year-old man diagnosed with leukemia will begin chemotherapy. What would the nurse do to combat the most common adverse effects of chemotherapy? A) Administer an antiemetic. B) Administer an antimetabolite. C) Administer a tumor antibiotic. D) Administer an anticoagulant.

A

A 60-year-old patient with a diagnosis of prostate cancer is scheduled to have an interstitial implant for high-dose radiation (HDR). What safety measure should the nurse include in this patients subsequent plan of care? A) Limit the time that visitors spend at the patients bedside. B) Teach the patient to perform all aspects of basic care independently. C) Assign male nurses to the patients care whenever possible. D) Situate the patient in a shared room with other patients receiving brachytherapy

A

A nurse who works in an oncology clinic is assessing a patient who has arrived for a 2-month follow-up appointment following chemotherapy. The nurse notes that the patients skin appears yellow. Which blood tests should be done to further explore this clinical sign? A) Liver function tests (LFTs) B) Complete blood count (CBC) C) Platelet count D) Blood urea nitrogen and creatinine

A

A patient has questioned the nurses administration of IV normal saline, asking whether sterile water would be a more appropriate choice than saltwater. Under what circumstances would the nurse administer electrolyte-free water intravenously? A) Never, because it rapidly enters red blood cells, causing them to rupture. B) When the patient is severely dehydrated resulting in neurologic signs and symptoms C) When the patient is in excess of calcium and/or magnesium ions D) When a patients fluid volume deficit is due to acute or chronic renal failure

A

The clinic nurse is caring for a 42-year-old male oncology patient. He complains of extreme fatigue and weakness after his first week of radiation therapy. Which response by the nurse would best reassure this patient? A) These symptoms usually result from radiation therapy; however, we will continue to monitor your laboratory and x-ray studies. B) These symptoms are part of your disease and are an unfortunately inevitable part of living with cancer. C) Try not to be concerned about these symptoms. Every patient feels this way after having radiation therapy. D) Even though it is uncomfortable, this is a good sign. It means that only the cancer cells are dying

A

The home health nurse is performing a home visit for an oncology patient discharged 3 days ago after completing treatment for non-Hodgkin lymphoma. The nurses assessment should include examination for the signs and symptoms of what complication? A) Tumor lysis syndrome (TLS) B) Syndrome of inappropriate antiduretic hormone (SIADH) C) Disseminated intravascular coagulation (DIC) D) Hypercalcemia

A

The hospice nurse has just admitted a new patient to the program. What principle guides hospice care? A) Care addresses the needs of the patient as well as the needs of the family. B) Care is focused on the patient centrally and the family peripherally. C) The focus of all aspects of care is solely on the patient. D) The care team prioritizes the patients physical needs and the family is responsible for the patients emotional needs

A

The nurse is caring for a patient who is to begin receiving external radiation for a malignant tumor of the neck. While providing patient education, what potential adverse effects should the nurse discuss with the patient? A) Impaired nutritional status B) Cognitive changes C) Diarrhea D) Alopecia

A

You are caring for a patient who has just been told that her stage IV colon cancer has recurred and metastasized to the liver. The oncologist offers the patient the option of surgery to treat the progression of this disease. What type of surgery does the oncologist offer? A) Palliative B) Reconstructive C) Salvage D) Prophylactic

A

You are the nurse caring for a patient who is to receive IV daunorubicin, a chemotherapeutic agent. You start the infusion and check the insertion site as per protocol. During your most recent check, you note that the IV has infiltrated so you stop the infusion. What is your main concern with this infiltration? A) Extravasation of the medication B) Discomfort to the patient C) Blanching at the site D) Hypersensitivity reaction to the medication

A

You are the nurse evaluating a newly admitted patients laboratory results, which include several values that are outside of reference ranges. Which of the following would cause the release of antidiuretic hormone (ADH)? A) Increased serum sodium B) Decreased serum potassium C) Decreased hemoglobin D) Increased platelets

A

The nurse is caring for a patient who has returned to the postsurgical suite after post-anesthetic recovery from a nephrectomy. The nurses most recent hourly assessment reveals a significant drop in level of consciousness and BP as well as scant urine output over the past hour. What is the nurses best response? A) Assess the patient for signs of bleeding and inform the physician. B) Monitor the patients vital signs every 15 minutes for the next hour. C) Reposition the patient and reassess vital signs. D) Palpate the patients flanks for pain and inform the physician.

A Bleeding may be suspected when the patient experiences fatigue and when urine output is less than 30 mL/h. The physician must be made aware of this finding promptly. Palpating the patients flanks would cause intense pain that is of no benefit to assessment.

A nurse is caring for a patient with severe anemia. The patient is tachycardic and complains of dizziness and exertional dyspnea. The nurse knows that in an effort to deliver more blood to hypoxic tissue, the workload on the heart is increased. What signs and symptoms might develop if this patient goes into heart failure? A) Peripheral edema B) Nausea and vomiting C) Migraine D) Fever

A Cardiac status should be carefully assessed in patients with anemia. When the hemoglobin level is low, the heart attempts to compensate by pumping faster and harder in an effort to deliver more blood to hypoxic tissue. This increased cardiac workload can result in such symptoms as tachycardia, palpitations, dyspnea, dizziness, orthopnea, and exertional dyspnea. Heart failure may eventually develop, as evidenced by an enlarged heart (cardiomegaly) and liver (hepatomegaly), and by peripheral edema. Nausea, migraine, and fever are not associated with heart failure.

A public health nurse has reviewed local data about the incidence and prevalence of burn injuries in the community. These data are likely to support what health promotion effort? A) Education about home safety B) Education about safe storage of chemicals C) Education about workplace health threats D) Education about safe driving

A Feedback: A large majority of burns occur in the home setting; educational interventions should address this epidemiologic trend.

An adult patient has been admitted to the medical unit for the treatment of acute pancreatitis. What nursing action should be included in this patients plan of care? A) Measure the patients abdominal girth daily. B) Limit the use of opioid analgesics. C) Monitor the patient for signs of dysphagia. D) Encourage activity as tolerated.

A Due to the risk of ascites, the nurse should monitor the patients abdominal girth. There is no specific need to avoid the use of opioids or to monitor for dysphagia, and activity is usually limited.

A nurse has been called for duty during a response to a natural disaster. In this context of care, the nurse should expect to do which of the following? A) Practice outside of her normal area of clinical expertise. B) Perform interventions that are not based on assessment data. C) Prioritize psychosocial needs over physiologic needs. D) Prioritize the interests of older adults over younger patients.

A During a disaster, nurses may be asked to perform duties outside their areas of expertise and may take on responsibilities normally held by physicians or advanced practice nurses.

A patient is admitted to the ED with suspected alcohol intoxication. The ED nurse is aware of the need to assess for conditions that can mimic acute alcohol intoxication. In light of this need, the nurse should perform what action? A) Check the patients blood glucose level. B) Assess for a documented history of major depression. C) Determine whether the patient has ingested a corrosive substance. D) Arrange for assessment of serum potassium levels.

A Feedback: Hypoglycemia can mimic alcohol intoxication and should be assessed in a patient suspected of alcohol intoxication. Potassium imbalances, depression, and poison ingestion are not noted to mimic the characteristic signs and symptoms of alcohol intoxication.

The nurse is providing health education to a patient who has a C6 spinal cord injury. The patient asks why autonomic dysreflexia is considered an emergency. What would be the nurses best answer? A) The sudden increase in BP can raise the ICP or rupture a cerebral blood vessel. B) The suddenness of the onset of the syndrome tells us the body is struggling to maintain its normal state. C) Autonomic dysreflexia causes permanent damage to delicate nerve fibers that are healing. D) The sudden, severe headache increases muscle tone and can cause further nerve damage.

A Feedback: The sudden increase in BP may cause a rupture of one or more cerebral blood vessels or lead to increased ICP. Autonomic dysreflexia does not directly cause nerve damage.

A patient is undergoing testing for suspected adrenocortical insufficiency. The care team should ensure that the patient has been assessed for the most common cause of adrenocortical insufficiency. What is the most common cause of this health problem? A) Therapeutic use of corticosteroids B) Pheochromocytoma C) Inadequate secretion of ACTH D) Adrenal tumor

A Feedback: Therapeutic use of corticosteroids is the most common cause of adrenocortical insufficiency. The other options also cause adrenocortical insufficiency, but they are not the most common causes.

The announcement is made that the facility may return to normal functioning after a local disaster. In the emergency operations plan, what is this referred to as? A) Demobilization response B) Post-incident response C) Crisis diffusion D) Reversion

A The demobilization response occurs when it is deemed that the facility may return to normal daily functioning. This is not known as the post-incident response, crisis diffusion or reversion.

The ICU nurse caring for a patient in shock is administering vasoactive medications as per orders. The nurse should know that vasoactive medications should be administered in what way? A) Through a central venous line B) By a gravity infusion IV set C) By IV push for rapid onset of action D) Mixed with parenteral feedings to balance osmosis

A Whenever possible, vasoactive medications should be administered through a central venous line because infiltration and extravasation of some vasoactive medications can cause tissue necrosis and sloughing. An IV pump or controller must be used to ensure that t

The nurse is describing some of the major characteristics of cancer to a patient who has recently received a diagnosis of malignant melanoma. When differentiating between benign and malignant cancer cells, the nurse should explain differences in which of the following aspects? Select all that apply. A) Rate of growth B) Ability to cause death C) Size of cells D) Cell contents E) Ability to spread

A B E

You are caring for an adult patient who has developed a mild oral yeast infection following chemotherapy. What actions should you encourage the patient to perform? Select all that apply. A) Use a lip lubricant. B) Scrub the tongue with a firm-bristled toothbrush. C) Use dental floss every 24 hours. D) Rinse the mouth with normal saline. E) Eat spicy food to aid in eradicating the yeast.

A C D

You are doing discharge teaching with a patient who has hypophosphatemia during his time in hospital. The patient has a diet ordered that is high in phosphate. What foods would you teach this patient to include in his diet? Select all that apply. A) Milk B) Beef C) Poultry D) Green vegetables E) Liver

A C E

The nurse is planning the care of a patient who has been recently diagnosed with a cerebellar tumor. Due to the location of this patients tumor, the nurse should implement measures to prevent what complication? A) Falls B) Audio hallucinations C) Respiratory depression D) Labile BP

A Feedback: A cerebellar tumor causes dizziness, an ataxic or staggering gait with a tendency to fall toward the side of the lesion, and marked muscle incoordination. Because of this, the patient faces a high risk of falls. Hallucinations and unstable vital signs are not closely associated with cerebellar tumo

A medical nurse is providing end-of-life care for a patient with metastatic bone cancer. The nurse notes that the patient has been receiving oral analgesics for her pain with adequate effect, but is now having difficulty swallowing the medication. What should the nurse do? A) Request the physician to order analgesics by an alternative route. B) Crush the medication in order to aid swallowing and absorption. C) Administer the patients medication with the meal tray. D) Administer the medication rectally.

A Feedback: A change in medication route is indicated and must be made by a physicians order. Many pain medications cannot be crushed and given to a patient. Giving the medication with a meal is not going to make it any easier to swallow. Rectal administration may or may not be an option.

A patient has a recent diagnosis of chronic pancreatitis and is undergoing diagnostic testing to determine pancreatic islet cell function. The nurse should anticipate what diagnostic test? A) Glucose tolerance test B) ERCP C) Pancreatic biopsy D) Abdominal ultrasonography

A Feedback: A glucose tolerance test evaluates pancreatic islet cell function and provides necessary information for making decisions about surgical resection of the pancreas. This specific clinical information is not provided by ERCP, biopsy, or ultrasound.

The nurse is caring for an adult patient with HF who is prescribed digoxin. When assessing the patient for adverse effects, the nurse should assess for which of the following signs and symptoms? A) Confusion and bradycardia B) Uncontrolled diuresis and tachycardia C) Numbness and tingling in the extremities D) Chest pain and shortness of breath

A Feedback: A key concern associated with digitalis therapy is digitalis toxicity. Symptoms include anorexia, nausea, visual disturbances, confusion, and bradycardia. The other listed signs and symptoms are not characteristic of digitalis toxicity.

A patient comes to the clinic complaining of fatigue and the health interview is suggestive of pica. Laboratory findings reveal a low serum iron level and a low ferritin level. With what would the nurse suspect that the patient will be diagnosed? A) Iron deficiency anemia B) Pernicious anemia C) Sickle cell anemia D) Hemolytic anemia

A Feedback: A low serum iron level, a low ferritin level, and symptoms of pica are associated with iron deficiency anemia. TIBC may also be elevated. None of the other anemias are associated with pica.

A patient with a C5 spinal cord injury is tetraplegic. After being moved out of the ICU, the patient complains of a severe throbbing headache. What should the nurse do first? A) Check the patients indwelling urinary catheter for kinks to ensure patency. B) Lower the HOB to improve perfusion. C) Administer analgesia. D) Reassure the patient that headaches are expected after spinal cord injuries.

A Feedback: A severe throbbing headache is a common symptom of autonomic dysreflexia, which occurs after injuries to the spinal cord above T6. The syndrome is usually brought on by sympathetic stimulation, such as bowel and bladder distention. Lowering the HOB can increase ICP. Before administering analgesia, the nurse should check the patients catheter, record vital signs, and perform an abdominal assessment. A severe throbbing headache is a dangerous symptom in this patient and is not expected.

A patient with cancer of the bladder has just returned to the unit from the PACU after surgery to create an ileal conduit. The nurse is monitoring the patients urine output hourly and notifies the physician when the hourly output is less than what? A) 30 mL B) 50 mL C) 100 mL D) 125 mL

A Feedback: A urine output below 30 mL/hr may indicate dehydration or an obstruction in the ileal conduit, with possible backflow or leakage from the ureteroileal anastomosis.

A patient who just suffered a suspected ischemic stroke is brought to the ED by ambulance. On what should the nurses primary assessment focus? A) Cardiac and respiratory status B) Seizure activity C) Pain D) Fluid and electrolyte balance

A Feedback: Acute care begins with managing ABCs. Patients may have difficulty keeping an open and clear airway secondary to decreased LOC. Neurologic assessment with close monitoring for signs of increased neurologic deficit and seizure activity occurs next. Fluid and electrolyte balance must be controlled carefully with the goal of adequate hydration to promote perfusion and decrease further brain activity.

A nurse is caring for a patient who has an order to discontinue the administration of parenteral nutrition. What should the nurse do to prevent the occurrence of rebound hypoglycemia in the patient? A) Administer an isotonic dextrose solution for 1 to 2 hours after discontinuing the PN. B) Administer a hypertonic dextrose solution for 1 to 2 hours after discontinuing the PN. C) Administer 3 ampules of dextrose 50% immediately prior to discontinuing the PN. D) Administer 3 ampules of dextrose 50% 1 hour after discontinuing the PN.

A Feedback: After administration of the PN solution is gradually discontinued, an isotonic dextrose solution is administered for 1 to 2 hours to protect against rebound hypoglycemia. The other listed actions would likely cause hyperglycemia.

A patient is admitted to the ED after being involved in a motor vehicle accident. The patient has multiple injuries. After establishing an airway and adequate ventilation, the ED team should prioritize what aspect of care? A) Control the patients hemorrhage. B) Assess for cognitive effects of the injury. C) Splint the patients fractures. D) Assess the patients neurologic status

A Feedback: After establishing airway and ventilation, the team should evaluate and restore cardiac output by controlling hemorrhage. This must precede neurologic assessments and treatment of skeletal injuries.

The nurse is caring for a patient postoperative day 4 following a kidney transplant. When assessing for potential signs and symptoms of rejection, what assessment should the nurse prioritize? A) Assessment of the quantity of the patients urine output B) Assessment of the patients incision C) Assessment of the patients abdominal girth D) Assessment for flank or abdominal pain

A Feedback: After kidney transplantation, the nurse should perform all of the listed assessments. However, oliguria is considered to be more suggestive of rejection than changes to the patients abdomen or incision.

A nurse on a burn unit is caring for a patient in the acute phase of burn care. While performing an assessment during this phase of burn care, the nurse recognizes that airway obstruction related to upper airway edema may occur up to how long after the burn injury? A) 2 days B) 3 days C) 5 days D) 1 week

A Feedback: Airway obstruction caused by upper airway edema can take as long as 48 hours to develop. Changes detected by x-ray and arterial blood gases may occur as the effects of resuscitative fluid and the chemical reaction of smoke ingredients with lung tissues become apparent.

A nurse is assessing a patient with an acoustic neuroma who has been recently admitted to an oncology unit. What symptoms is the nurse likely to find during the initial assessment? A) Loss of hearing, tinnitus, and vertigo B) Loss of vision, change in mental status, and hyperthermia C) Loss of hearing, increased sodium retention, and hypertension D) Loss of vision, headache, and tachycardia

A Feedback: An acoustic neuroma is a tumor of the eighth cranial nerve, the cranial nerve most responsible for hearing and balance. The patient with an acoustic neuroma usually experiences loss of hearing, tinnitus, and episodes of vertigo and staggering gait. Acoustic neuromas do not cause loss of vision, increased sodium retention, or tachycardia.

The nurse has just admitted a 66-year-old patient for cardiac surgery. The patient tearfully admits to the nurse that she is afraid of dying while undergoing the surgery. What is the nurses best response? A) Explore the factors underlying the patients anxiety. B) Teach the patient guided imagery techniques. C) Obtain an order for a PRN benzodiazepine. D) Describe the procedure in greater detail.

A Feedback: An assessment of anxiety levels is required in the patient to assist the patient in identifying fears and developing coping mechanisms for those fears. The nurse must further assess and explore the patients anxiety before providing interventions such as education or medications.

A patient with a diagnosis of HF is started on a beta-blocker. What is the nurses priority role during gradual increases in the patients dose? A) Educating the patient that symptom relief may not occur for several weeks B) Stressing that symptom relief may take up to 4 months to occur C) Making adjustments to each days dose based on the blood pressure trends D) Educating the patient about the potential changes in LOC that may result from the drug

A Feedback: An important nursing role during titration is educating the patient about the potential worsening of symptoms during the early phase of treatment and stressing that improvement may take several weeks. Relief does not take 4 months, however. The nurse monitors blood pressure, but changes are not made based on short-term assessment results. Beta-blockers rarely affect LOC

A patient has been scheduled for an ultrasound of the gallbladder the following morning. What should the nurse do in preparation for this diagnostic study? A) Have the patient refrain from food and fluids after midnight. B) Administer the contrast agent orally 10 to 12 hours before the study. C) Administer the radioactive agent intravenously the evening before the study. D) Encourage the intake of 64 ounces of water 8 hours before the study.

A Feedback: An ultrasound of the gallbladder is most accurate if the patient fasts overnight, so that the gallbladder is distended. Contrast and radioactive agents are not used when performing ultrasonography of the gallbladder, as an ultrasound is based on reflected sound waves.

The nurse is providing care for a patient who has experienced a type I hypersensitivity reaction. What condition is an example of such a reaction? A) Anaphylactic reaction after a bee sting B) Skin reaction resulting from adhesive tape C) Myasthenia gravis D) Rheumatoid arthritis

A Feedback: Anaphylactic (type I) hypersensitivity is an immediate reaction mediated by IgE antibodies and requires previous exposure to the specific antigen. Skin reactions are more commonly type IV and myasthenia gravis is thought to be a type II reaction. Rheumatoid arthritis is not a type I hypersensitivity reaction.

A patient with mitral valve prolapse is admitted for a scheduled bronchoscopy to investigate recent hemoptysis. The physician has ordered gentamicin to be taken before the procedure. What is the rationale for this? A) To prevent bacterial endocarditis B) To prevent hospital-acquired pneumonia C) To minimize the need for antibiotic use during the procedure D) To decrease the need for surgical asepsis

A Feedback: Antibiotic prophylaxis is recommended for high-risk patients immediately before and sometimes after the following invasive procedures, such as bronchoscopy. Gentamicin would not be given to prevent pneumonia, to avoid antibiotic use during the procedure, or to decrease the need for surgical asepsis.

A nurse is caring for a patient who is exhibiting ventricular tachycardia (VT). Because the patient is pulseless, the nurse should prepare for what intervention? A) Defibrillation B) ECG monitoring C) Implantation of a cardioverter defibrillator D) Angioplasty

A Feedback: Any type of VT in a patient who is unconscious and without a pulse is treated in the same manner as ventricular fibrillation: Immediate defibrillation is the action of choice. ECG monitoring is appropriate, but this is an assessment, not an intervention, and will not resolve the problem. An ICD and angioplasty do not address the dysrhythmia.

The nurse is caring for a patient who has been diagnosed with an elevated cholesterol level. The nurse is aware that plaque on the inner lumen of arteries is composed chiefly of what? A) Lipids and fibrous tissue B) White blood cells C) Lipoproteins D) High-density cholesterol

A Feedback: As T-lymphocytes and monocytes infiltrate to ingest lipids on the arterial wall and then die, a fibrous tissue develops. This causes plaques to form on the inner lumen of arterial walls. These plaques do not consist of white cells, lipoproteins, or high-density cholesterol.

A patient who has experienced an ischemic stroke has been admitted to the medical unit. The patients family in adamant that she remain on bed rest to hasten her recovery and to conserve energy. What principle of care should inform the nurses response to the family? A) The patient should mobilize as soon as she is physically able. B) To prevent contractures and muscle atrophy, bed rest should not exceed 4 weeks. C) The patient should remain on bed rest until she expresses a desire to mobilize. D) Lack of mobility will greatly increase the patients risk of stroke recurrence.

A Feedback: As soon as possible, the patient is assisted out of bed and an active rehabilitation program is started. Delaying mobility causes complications, but not necessarily stroke recurrence. Mobility should not be withheld until the patient initiates.

A patient with thyroid cancer has undergone surgery and a significant amount of parathyroid tissue has been removed. The nurse caring for the patient should prioritize what question when addressing potential complications? A) Do you feel any muscle twitches or spasms? B) Do you feel flushed or sweaty? C) Are you experiencing any dizziness or lightheadedness? D) Are you having any pain that seems to be radiating from your bones?

A Feedback: As the blood calcium level falls, hyperirritability of the nerves occurs, with spasms of the hands and feet and muscle twitching. This is characteristic of hypoparathyroidism. Flushing, diaphoresis, dizziness, and pain are atypical signs of the resulting hypocalcemia.

The nurse is performing an initial assessment of a client diagnosed with HF. The nurse also assesses the patients sensorium and LOC. Why is the assessment of the patients sensorium and LOC important in patients with HF? A) HF ultimately affects oxygen transportation to the brain. B) Patients with HF are susceptible to overstimulation of the sympathetic nervous system. C) Decreased LOC causes an exacerbation of the signs and symptoms of HF. D) The most significant adverse effect of medications used for HF treatment is altered LOC.

A Feedback: As the volume of blood ejected by the heart decreases, so does the amount of oxygen transported to the brain. Sympathetic stimulation is not a primary concern in patients with HF, although it is a possibility. HF affects LOC but the reverse is not usually true. Medications used to treat HF carry many adverse effects, but the most common and significant effects are cardiovascular

The nurse has identified the nursing diagnosis of risk for infection in a patient who undergoes peritoneal dialysis. What nursing action best addresses this risk? A) Maintain aseptic technique when administering dialysate. B) Wash the skin surrounding the catheter site with soap and water prior to each exchange. C) Add antibiotics to the dialysate as ordered. D) Administer prophylactic antibiotics by mouth or IV as ordered.

A Feedback: Aseptic technique is used to prevent peritonitis and other infectious complications of peritoneal dialysis. It is not necessary to cleanse the skin with soap and water prior to each exchange. Antibiotics may be added to dialysate to treat infection, but they are not used to prevent infection.

The nurse is caring for a patient whose recent health history includes an altered LOC. What should be the nurses first action when assessing this patient? A) Assessing the patients verbal response B) Assessing the patients ability to follow complex commands C) Assessing the patients judgment D) Assessing the patients response to pain

A Feedback: Assessment of the patient with an altered LOC often starts with assessing the verbal response through determining the patients orientation to time, person, and place. In most cases, this assessment will precede each of the other listed assessments, even though each may be indicated.

The school nurse has been called to the football field where player is immobile on the field after landing awkwardly on his head during a play. While awaiting an ambulance, what action should the nurse perform? A) Ensure that the player is not moved. B) Obtain the players vital signs, if possible. C) Perform a rapid assessment of the players range of motion. D) Assess the players reflexes.

A Feedback: At the scene of the injury, the patient must be immobilized on a spinal (back) board, with the head and neck maintained in a neutral position, to prevent an incomplete injury from becoming complete. This is a priority over determining the patients vital signs. It would be inappropriate to test ROM or reflexes.

A patient is brought to the ED and determined to be experiencing symptomatic sinus bradycardia. The nurse caring for this patient is aware the medication of choice for treatment of this dysrhythmia is the administration of atropine. What guidelines will the nurse follow when administering atropine? A) Administer atropine 0.5 mg as an IV bolus every 3 to 5 minutes to a maximum of 3.0 mg. B) Administer atropine as a continuous infusion until symptoms resolve. C) Administer atropine as a continuous infusion to a maximum of 30 mg in 24 hours. D) Administer atropine 1.0 mg sublingually.

A Feedback: Atropine 0.5 mg given rapidly as an intravenous (IV) bolus every 3 to 5 minutes to a maximum total dose of 3.0 mg is the medication of choice in treating symptomatic sinus bradycardia. By this guideline, the other listed options are inappropriate.

During a health education session, a participant has asked about the hepatitis E virus. What prevention measure should the nurse recommend for preventing infection with this virus? A) Following proper hand-washing techniques B) Avoiding chemicals that are toxic to the liver C) Wearing a condom during sexual contact D) Limiting alcohol intake

A Feedback: Avoiding contact with the hepatitis E virus through good hygiene, including hand-washing, is the major method of prevention. Hepatitis E is transmitted by the fecaloral route, principally through contaminated water in areas with poor sanitation. Consequently, none of the other listed preventative measures is indicated.

The nurse is caring for a patient whose spinal cord injury has caused recent muscle spasticity. What medication should the nurse expect to be ordered to control this? A) Baclofen (Lioresal) B) Dexamethasone (Decadron) C) Mannitol (Osmitrol) D) Phenobarbital (Luminal)

A Feedback: Baclofen is classified as an antispasmodic agent in the treatment of muscles spasms related to spinal cord injury. Decadron is an anti-inflammatory medication used to decrease inflammation in both SCI and head injury. Mannitol is used to decrease cerebral edema in patients with head injury. Phenobarbital is an anticonvulsant that is used in the treatment of seizure activity.

The nurse has tested the pH of urine from a patients newly created ileal conduit and obtained a result of 6.8. What is the nurses best response to this assessment finding? A) Obtain an order to increase the patients dose of ascorbic acid. B) Administer IV sodium bicarbonate as ordered. C) Encourage the patient to drink at least 500 mL of water and retest in 3 hours. D) Irrigate the ileal conduit with a dilute citric acid solution as ordered.

A Feedback: Because severe alkaline encrustation can accumulate rapidly around the stoma, the urine pH is kept below 6.5 by administration of ascorbic acid by mouth. An increased pH may suggest a need to increase ascorbic acid dosing. This is not treated by administering bicarbonate or citric acid, nor by increasing fluid intake.

A nurse is caring for a patient with cancer of the liver whose condition has required the insertion of a percutaneous biliary drainage system. The nurses most recent assessment reveals the presence of dark green fluid in the collection container. What is the nurses best response to this assessment finding? A) Document the presence of normal bile output. B) Irrigate the drainage system with normal saline as ordered. C) Aspirate a sample of the drainage for culture. D) Promptly report this assessment finding to the primary care provider.

A Feedback: Bile is usually a dark green or brownish-yellow color, so this would constitute an expected assessment finding, with no other action necessary

The nurse is caring for a patient who has had a biventricular pacemaker implanted. When planning the patients care, the nurse should recognize what goal of this intervention? A) Resynchronization B) Defibrillation C) Angioplasty D) Ablation

A Feedback: Biventricular (both ventricles) pacing, also called resynchronization therapy, may be used to treat advanced heart failure that does not respond to medication. This type of pacing therapy is not called defibrillation, angioplasty, or ablation therapy.

A patient who is being treated in the hospital for a spinal cord injury is advocating for the removal of his urinary catheter, stating that he wants to try to resume normal elimination. What principle should guide the care teams decision regarding this intervention? A) Urinary retention can have serious consequences in patients with SCIs. B) Urinary function is permanently lost following an SCI. C) Urinary catheters should not remain in place for more than 7 days. D) Overuse of urinary catheters can exacerbate nerve damage.

A Feedback: Bladder distention, a major cause of autonomic dysreflexia, can also cause trauma. For this reason, removal of a urinary catheter must be considered with caution. Extended use of urinary catheterization is often necessary following SCI. The effect of a spinal cord lesion on urinary function depends on the level of the injury. Catheter use does not cause nerve damage, although it is a major risk factor for UTIs.

A burn patient is transitioning from the acute phase of the injury to the rehabilitation phase. The patient tells the nurse, I cant wait to have surgery to reconstruct my face so I look normal again. What would be the nurses best response? A) Thats something that you and your doctor will likely talk about after your scars mature. B) That is something for you to talk to your doctor about because its not a nursing responsibility. C) I know this is really important to you, but you have to realize that no one can make you look like you used to. D) Unfortunately, its likely that you will have most of these scars for the rest of your life.

A Feedback: Burn reconstruction is a treatment option after all scars have matured and is discussed within the first few years after injury. Even though this is not a nursing responsibility, the nurse should still respond appropriately to the patients query. It is true that the patient will not realistically look like he or she used to, but this does not instill hope.

Renal failure can have prerenal, renal, or postrenal causes. A patient with acute kidney injury is being assessed to determine where, physiologically, the cause is. If the cause is found to be prerenal, which condition most likely caused it? A) Heart failure B) Glomerulonephritis C) Ureterolithiasis D) Aminoglycoside toxicity

A Feedback: By causing inadequate renal perfusion, heart failure can lead to prerenal failure. Glomerulonephritis and aminoglycoside toxicity are renal causes, and ureterolithiasis is a postrenal cause.

A patient is admitted to the neurologic ICU with a suspected diffuse axonal injury. What would be the primary neuroimaging diagnostic tool used on this patient to evaluate the brain structure? A) MRI B) PET scan C) X-ray D) Ultrasound

A Feedback: CT and MRI scans, the primary neuroimaging diagnostic tools, are useful in evaluating the brain structure. Ultrasound would not show the brain nor would an x-ray. A PET scan shows brain function, not brain structure.

A patient with cardiovascular disease is being treated with amlodipine (Norvasc), a calcium channel blocking agent. The therapeutic effects of calcium channel blockers include which of the following? A) Reducing the hearts workload by decreasing heart rate and myocardial contraction B) Preventing platelet aggregation and subsequent thrombosis C) Reducing myocardial oxygen consumption by blocking adrenergic stimulation to the heart D) Increasing the efficiency of myocardial oxygen consumption, thus decreasing ischemia and relieving pain

A Feedback: Calcium channel blocking agents decrease sinoatrial node automaticity and atrioventricular node conduction, resulting in a slower heart rate and a decrease in the strength of the heart muscle contraction. These effects decrease the workload of the heart. Antiplatelet and anticoagulation medications are administered to prevent platelet aggregation and subsequent thrombosis, which impedes blood flow. Beta-blockers reduce myocardial consumption by blocking beta-adrenergic sympathetic stimulation to the heart. The result is reduced myocardial contractility (force of contraction) to balance the myocardium oxygen needs and supply. Nitrates reduce myocardial oxygen consumption, which decreases ischemia and relieves pain by dilating the veins and, in higher doses, the arteries.

The cardiac nurse is caring for a patient who has been diagnosed with dilated cardiomyopathy (DCM). Echocardiography is likely to reveal what pathophysiological finding? A) Decreased ejection fraction B) Decreased heart rate C) Ventricular hypertrophy D) Mitral valve regurgitation

A Feedback: DCM is distinguished by significant dilation of the ventricles without simultaneous hypertrophy. The ventricles have elevated systolic and diastolic volumes, but a decreased ejection fraction. Bradycardia and mitral valve regurgitation do not typically occur in patients with DCM.

A man survived a workplace accident that claimed the lives of many of his colleagues several months ago. The man has recently sought care for the treatment of depression. How should the nurse best understand the mans current mental health problem? A) The man is experiencing a common response following a disaster. B) The man fails to appreciate the fact that he survived the disaster. C) The man most likely feels guilty about his actions during the disaster. D) The mans depression most likely predated the disaster.

A Feedback: Depression is a common response to disaster. It does not suggest that the patient feels guilty about his actions or that he does not appreciate the fact that he survived. It is possible, but less likely, that the patient was depressed prior to the disaster.

A patient with a recent diagnosis of ITP has asked the nurse why the care team has not chosen to administer platelets, stating, I have low platelets, so why not give me a transfusion of exactly what Im missing? How should the nurse best respond? A) Transfused platelets usually arent beneficial because theyre rapidly destroyed in the body. B) A platelet transfusion often blunts your bodys own production of platelets even further. C) Finding a matching donor for a platelet transfusion is exceedingly difficult. D) A very small percentage of the platelets in a transfusion are actually functional.

A Feedback: Despite extremely low platelet counts, platelet transfusions are usually avoided. Transfusions tend to be ineffective not because the platelets are nonfunctional but because the patients antiplatelet antibodies bind with the transfused platelets, causing them to be destroyed. Matching the patients blood type is not usually necessary for a platelet transfusion. Platelet transfusions do not exacerbate low platelet production.

A patient has developed diabetes insipidus after having increased ICP following head trauma. What nursing assessment best addresses this complication? A) Vigilant monitoring of fluid balance B) Continuous BP monitoring C) Serial arterial blood gases (ABGs) D) Monitoring of the patients airway for patency

A Feedback: Diabetes insipidus requires fluid and electrolyte replacement, along with the administration of vasopressin, to replace and slow the urine output. Because of these alterations in fluid balance, careful monitoring is necessary. None of the other listed assessments directly addresses the major manifestations of diabetes insipidus.

A patient with Huntington disease has just been admitted to a long-term care facility. The charge nurse is creating a care plan for this patient. Nutritional management for a patient with Huntington disease should be informed by what principle? A) The patient is likely to have an increased appetite. B) The patient is likely to required enzyme supplements. C) The patient will likely require a clear liquid diet. D) The patient will benefit from a low-protein diet.

A Feedback: Due to the continuous involuntary movements, patients will have a ravenous appetite. Despite this ravenous appetite, patients usually become emaciated and exhausted. As the disease progresses, patients experience difficulty in swallowing and thin liquids should be avoided. Protein will not be limited with this disease. Enzyme supplements are not normally required.

Cardiopulmonary resuscitation has been initiated on a patient who was found unresponsive. When performing chest compressions, the nurse should do which of the following? A) Perform at least 100 chest compressions per minute. B) Pause to allow a colleague to provide a breath every 10 compressions. C) Pause chest compressions to allow for vital signs monitoring every 4 to 5 minutes. D) Perform high-quality chest compressions as rapidly as possible.

A Feedback: During CPR, the chest is compressed 2 inches at a rate of at least 100 compressions per minute. This rate is the resuscitators goal; the aim is not to give compressions as rapidly as possible. Compressions are not stopped after 10 compressions to allow for a breath or for full vital signs monitoring.

A patient has been admitted to a burn intensive care unit with extensive full-thickness burns over 25% of the body. After ensuring cardiopulmonary stability, what would be the nurses immediate, priority concern when planning this patients care? A) Fluid status B) Risk of infection C) Nutritional status D) Psychosocial coping

A Feedback: During the early phase of burn care, the nurse is most concerned with fluid resuscitation, to correct large-volume fluid loss through the damaged skin. Infection control and early nutritional support are important, but fluid resuscitation is an immediate priority. Coping is a higher priority later in the recovery period.

A patient with a T2 injury is in spinal shock. The nurse will expect to observe what assessment finding? A) Absence of reflexes along with flaccid extremities B) Positive Babinskis reflex along with spastic extremities C) Hyperreflexia along with spastic extremities D) Spasticity of all four extremities

A Feedback: During the period immediately following a spinal cord injury, spinal shock occurs. In spinal shock, all reflexes are absent and the extremities are flaccid. When spinal shock subsides, the patient demonstrates a positive Babinskis reflex, hyperreflexia, and spasticity of all four extremities.

The nurse has created a plan of care for a patient who is at risk for increased ICP. The patients care plan should specify monitoring for what early sign of increased ICP? A) Disorientation and restlessness B) Decreased pulse and respirations C) Projectile vomiting D) Loss of corneal reflex

A Feedback: Early indicators of ICP include disorientation and restlessness. Later signs include decreased pulse and respirations, projectile vomiting, and loss of brain stem reflexes, such as the corneal reflex.

An 83-year-old patient is brought in by ambulance from a long-term care facility. The patients symptoms are weakness, lethargy, incontinence, and a change in mental status. The nurse knows that emergencies in older adults may be more difficult to manage. Why would this be true? A) Older adults may have an altered response to treatment. B) Older adults are often reluctant to adhere to prescribed treatment. C) Older adults have difficulty giving a health history. D) Older adults often stigmatize their peers who use the ED.

A Feedback: Emergencies in this age group may be more difficult to manage because elderly patients may have an atypical presentation, an altered response to treatment, a greater risk of developing complications, or a combination of these factors. The elderly patient may perceive the emergency as a crisis signaling the end of an independent lifestyle or even resulting in death. Stigmatization and nonadherence to treatment are not commonly noted. Older adults do not necessarily have difficulty giving a health history.

When preparing to discharge a patient home, the nurse has met with the family and warned them that the patient may exhibit unexpected emotional responses. The nurse should teach the family that these responses are typically a result of what cause? A) Frustration around changes in function and communication B) Unmet physiologic needs C) Changes in brain activity during sleep and wakefulness D) Temporary changes in metabolism

A Feedback: Emotional problems associated with stroke are often related to the new challenges around ADLs and communication. These challenges are more likely than metabolic changes, unmet physiologic needs, or changes in brain activity, each of which should be ruled out.

The nurse and the other members of the team are caring for a patient who converted to ventricular fibrillation (VF). The patient was defibrillated unsuccessfully and the patient remains in VF. According to national standards, the nurse should anticipate the administration of what medication? A) Epinephrine 1 mg IV push B) Lidocaine 100 mg IV push C) Amiodarone 300 mg IV push D) Sodium bicarbonate 1 amp IV push

A Feedback: Epinephrine should be administered as soon as possible after the first unsuccessful defibrillation and then every 3 to 5 minutes. Antiarrhythmic medications such as amiodarone and licocaine are given if ventricular dysrhythmia persists.

The nurse is assessing a patient with a suspected stroke. What assessment finding is most suggestive of a stroke? A) Facial droop B) Dysrhythmias C) Periorbital edema D) Projectile vomiting

A Feedback: Facial drooping or asymmetry is a classic abnormal finding on a physical assessment that may be associated with a stroke. Facial edema is not suggestive of a stroke and patients less commonly experience dysrhythmias or vomiting.

A nurse is caring for a patient who is postoperative from a neck dissection. What would be the most appropriate nursing action to enhance the patients appetite? A) Encourage the family to bring in the patients favored foods. B) Limit visitors at mealtimes so that the patient is not distracted. C) Avoid offering food unless the patient initiates. D) Provide thorough oral care immediately after the patient eats.

A Feedback: Family involvement and home-cooked favorite foods may help the patient to eat. Having visitors at mealtimes may make eating more pleasant and increase the patients appetite. The nurse should not place the complete onus for initiating meals on the patient. Oral care after meals is necessary, but does not influence appetite.

A patient in the emergent/resuscitative phase of a burn injury has had blood work and arterial blood gases drawn. Upon analysis of the patients laboratory studies, the nurse will expect the results to indicate what? A) Hyperkalemia, hyponatremia, elevated hematocrit, and metabolic acidosis B) Hypokalemia, hypernatremia, decreased hematocrit, and metabolic acidosis C) Hyperkalemia, hypernatremia, decreased hematocrit, and metabolic alkalosis D) Hypokalemia, hyponatremia, elevated hematocrit, and metabolic alkalosis

A Feedback: Fluid and electrolyte changes in the emergent/resuscitative phase of a burn injury include hyperkalemia related to the release of potassium into the extracellular fluid, hyponatremia from large amounts of sodium lost in trapped edema fluid, hemoconcentration that leads to an increased hematocrit, and loss of bicarbonate ions that results in metabolic acidosis.

The nurse is caring for a patient who has just undergone catheter ablation therapy. The nurse in the step- down unit should prioritize what assessment? A) Cardiac monitoring B) Monitoring the implanted device signal C) Pain assessment D) Monitoring the patients level of consciousness (LOC)

A Feedback: Following catheter ablation therapy, the patient is closely monitored to ensure the dysrhythmia does not reemerge. This is a priority over monitoring of LOC and pain, although these are valid and important assessments. Ablation does not involve the implantation of a device.

A patients health decline necessitates the use of total parenteral nutrition. The patient has questioned the need for insertion of a central venous catheter, expressing a preference for a normal IV. The nurse should know that peripheral administration of high-concentration PN formulas is contraindicated because of the risk for what complication? A) Chemical phlebitis B) Hyperglycemia C) Dumping syndrome D) Line sepsis

A Feedback: Formulations with dextrose concentrations of more than 10% should not be administered through peripheral veins because they irritate the intima (innermost walls) of small veins, causing chemical phlebitis. Hyperglycemia and line sepsis are risks with both peripheral and central administration of PN. PN is not associated with dumping syndrome.

The nurse is planning the care of a patient with a T1 spinal cord injury. The nurse has identified the diagnosis of risk for impaired skin integrity. How can the nurse best address this risk? A) Change the patients position frequently. B) Provide a high-protein diet. C) Provide light massage at least daily. D) Teach the patient deep breathing and coughing exercises.

A Feedback: Frequent position changes are among the best preventative measures against pressure ulcers. A high- protein diet can benefit wound healing, but does not necessarily prevent skin breakdown. Light massage and deep breathing do not protect or restore skin integrity.

A backcountry skier has been airlifted to the ED after becoming lost and developing hypothermia and frostbite. How should the nurse best manage the patients frostbite? A) Immerse affected extremities in water slightly above normal body temperature. B) Immerse the patients frostbitten extremities in the warmest water the patient can tolerate. C) Gently massage the patients frozen extremities in between water baths. D) Perform passive range-of-motion exercises of the affected extremities to promote circulation.

A Feedback: Frozen extremities are usually placed in a 37C to 40C (98.6F to 104F) circulating bath for 30- to 40- minute spans. To avoid further mechanical injury, the body part is not handled. Massage is contraindicated.

A group of nurses have attended an inservice on the prevention of occupationally acquired diseases that affect healthcare providers. What action has the greatest potential to reduce a nurses risk of acquiring hepatitis C in the workplace? A) Disposing of sharps appropriately and not recapping needles B) Performing meticulous hand hygiene at the appropriate moments in care C) Adhering to the recommended schedule of immunizations D) Wearing an N95 mask when providing care for patients on airborne precautions

A Feedback: HCV is bloodborne. Consequently, prevention of needlestick injuries is paramount. Hand hygiene, immunizations and appropriate use of masks are important aspects of overall infection control, but these actions do not directly mitigate the risk of HCV.

A football player is thought to have sustained an injury to his kidneys from being tackled from behind. The ER nurse caring for the patient reviews the initial orders written by the physician and notes that an order to collect all voided urine and send it to the laboratory for analysis. The nurse understands that this nursing intervention is important for what reason? A) Hematuria is the most common manifestation of renal trauma and blood losses may be microscopic, so laboratory analysis is essential. B) Intake and output calculations are essential and the laboratory will calculate the precise urine output produced by this patient. C) A creatinine clearance study may be ordered at a later time and the laboratory will hold all urine until it is determined if the test will be necessary. D) There is great concern about electrolyte imbalances and the laboratory will monitor the urine for changes in potassium and sodium concentrations

A Feedback: Hematuria is the most common manifestation of renal trauma; its presence after trauma suggests renal injury. Hematuria may not occur, or it may be detectable only on microscopic examination. All urine should be saved and sent to the laboratory for analysis to detect RBCs and to evaluate the course of bleeding. Measuring intake and output is not a function of the laboratory. The laboratory does not save urine to test creatinine clearance at a later time. The laboratory does not monitor the urine for sodium or potassium concentrations.

A 45-year-old man with diabetic nephropathy has ESKD and is starting dialysis. What should the nurse teach the patient about hemodialysis? A) Hemodialysis is a treatment option that is usually required three times a week. B) Hemodialysis is a program that will require you to commit to daily treatment. C) This will require you to have surgery and a catheter will need to be inserted into your abdomen. D) Hemodialysis is a treatment that is used for a few months until your kidney heals and starts to produce urine again.

A Feedback: Hemodialysis is the most commonly used method of dialysis. Patients receiving hemodialysis must undergo treatment for the rest of their lives or until they undergo successful kidney transplantation. Treatments usually occur three times a week for at least 3 to 4 hours per treatment.

A patient has been admitted to the emergency department with signs of anaphylaxis following a bee sting. The nurse knows that if this is a true allergic reaction the patient will present with what alteration in laboratory values? A) Increased eosinophils B) Increased neutrophils C) Increased serum albumin D) Decreased blood glucose

A Feedback: Higher percentages of eosinophils are considered moderate to severe eosinophilia. Moderate eosinophilia is defined as 15% to 40% eosinophils and is found in patients with allergic disorders. Hypersensitivity does not result in hypoglycemia or increased albumin and neu

One aspect of the nurses comprehensive assessment when caring for the terminally ill is the assessment of hope. The nurse is assessing a patient with liver failure for the presence of hope. What would the nurse identify as a hope-fostering category? A) Uplifting memories B) Ignoring negative outcomes C) Envisioning one specific outcome D) Avoiding an actual or potential threat

A Feedback: Hope is a multidimensional construct that provides comfort as a person endures life threats and personal challenges. Uplifting memories are noted as a hope-fostering category, whereas the other listed options are not identified as such.

An adult oncology patient has a diagnosis of bladder cancer with metastasis and the patient has asked the nurse about the possibility of hospice care. Which principle is central to a hospice setting? A) The patient and family should be viewed as a single unit of care. B) Persistent symptoms of terminal illness should not be treated. C) Each member of the interdisciplinary team should develop an individual plan of care. D) Terminally ill patients should die in the hospital whenever possible.

A Feedback: Hospice care requires that the patient and family be viewed as a single unit of care. The other listed principles are wholly inconsistent with the principles of hospice care

The nurse on the hospitals infection control committee is looking into two cases of hospital-acquired infective endocarditis among a specific classification of patients. What classification of patients would be at greatest risk for hospital-acquired endocarditis? A) Hemodialysis patients B) Patients on immunoglobulins C) Patients who undergo intermittent urinary catheterization D) Children under the age of 12

A Feedback: Hospital-acquired infective endocarditis occurs most often in patients with debilitating disease or indwelling catheters and in patients who are receiving hemodialysis or prolonged IV fluid or antibiotic therapy. Patients taking immunosuppressive medications or corticosteroids are more susceptible to fungal endocarditis. Patients on immunoglobulins, those who need in and out catheterization, and children are not at increased risk for nosocomial infective endocarditis.

An oncology nurse is caring for a patient with multiple myeloma who is experiencing bone destruction. When reviewing the patients most recent blood tests, the nurse should anticipate what imbalance? A) Hypercalcemia B) Hyperproteinemia C) Elevated serum viscosity D) Elevated RBC count

A Feedback: Hypercalcemia may result when bone destruction occurs due to the disease process. Elevated serum viscosity occurs because plasma cells excrete excess immunoglobulin. RBC count will be decreased. Hyperproteinemia would not be present.

A patients abdominal ultrasound indicates cholelithiasis. When the nurse is reviewing the patients laboratory studies, what finding is most closely associated with this diagnosis? A) Increased bilirubin B) Decreased serum cholesterol C) Increased blood urea nitrogen (BUN) D) Decreased serum alkaline phosphatase level

A Feedback: If the flow of blood is impeded, bilirubin, a pigment derived from the breakdown of red blood cells, does not enter the intestines. As a result, bilirubin levels in the blood increase. Cholesterol, BUN, and alkaline phosphatase levels are not typically affected.

A patient has sustained a severe burn injury and is thought to have an impaired intestinal mucosal barrier. Since this patient is considered at an increased risk for infection, what intervention will best assist in avoiding increased intestinal permeability and prevent early endotoxin translocation? A) Early enteral feeding B) Administration of prophylactic antibiotics C) Bowel cleansing procedures D) Administration of stool softeners

A Feedback: If the intestinal mucosa receives some type of protection against permeability change, infection could be avoided. Early enteral feeding is one step to help avoid this increased intestinal permeability and prevent early endotoxin translocation. Antibiotics are seldom prescribed prophylactically because of the risk of promoting resistant strains of bacteria. A bowel cleansing procedure would not be ordered for this patient. The administration of stool softeners would not assist in avoiding increased intestinal permeability and prevent early endotoxin translocation.

The nurse is caring for a patient who is in the recovery room following the implantation of an ICD. The patient has developed ventricular tachycardia (VT). What should the nurse assess and document? A) ECG to compare time of onset of VT and onset of devices shock B) ECG so physician can see what type of dysrhythmia the patient has C) Patients level of consciousness (LOC) at the time of the dysrhythmia D) Patients activity at time of dysrhythmia

A Feedback: If the patient has an ICD implanted and develops VT or ventricular fibrillation, the ECG should be recorded to note the time between the onset of the dysrhythmia and the onset of the devices shock or antitachycardia pacing. This is a priority over LOC or activity at the time of onset.

A patients neck dissection surgery resulted in damage to the patients superior laryngeal nerve. What area of assessment should the nurse consequently prioritize? A) The patients swallowing ability B) The patients ability to speak C) The patients management of secretions D) The patients airway patency

A Feedback: If the superior laryngeal nerve is damaged, the patient may have difficulty swallowing liquids and food because of the partial lack of sensation of the glottis. Damage to this particular nerve does not inhibit speech and only affects management of secretions and airway patency indirectly

A patient with a liver mass is undergoing a percutaneous liver biopsy. What action should the nurse perform when assisting with this procedure? A) Position the patient on the right side with a pillow under the costal margin after the procedure. B) Administer 1 unit of albumin 90 minutes before the procedure as ordered. C) Administer at least 1 unit of packed red blood cells as ordered the day before the scheduled procedure. D) Confirm that the patients electrolyte levels have been assessed prior to the procedure.

A Feedback: Immediately after a percutaneous liver biopsy, assist the patient to turn onto the right side and place a pillow under the costal margin. Prior administration of albumin or PRBCs is unnecessary. Coagulation tests should be performed, but electrolyte analysis is not necessary.

A hospitals emergency operations plan has been enacted following an industrial accident. While one nurse performs the initial triage, what should other emergency medical services personnel do? A) Perform life-saving measures. B) Classify patients according to acuity. C) Provide health promotion education. D) Modify the emergency operations plan.

A Feedback: In an emergency, patients are immediately tagged and transported or given life-saving interventions. One person performs the initial triage while other emergency medical services (EMS) personnel perform life- saving measures and transport patients. Health promotion is not a priority during the acute stage of the crisis. Classifying patients is the task of the triage nurse. EMS personnel prioritize life-saving measures; they do not modify the operations plan.

A gerontologic nurse is advocating for diagnostic testing of an 81-year-old patient who is experiencing personality changes. The nurse is aware of what factor that is known to affect the diagnosis and treatment of brain tumors in older adults? A) The effects of brain tumors are often attributed to the cognitive effects of aging. B) Brain tumors in older adults do not normally produce focal effects. C) Older adults typically have numerous benign brain tumors by the eighth decade of life. D) Brain tumors cannot normally be treated in patient over age 75.

A Feedback: In older adult patients, early signs and symptoms of intracranial tumors can be easily overlooked or incorrectly attributed to cognitive and neurologic changes associated with normal aging. Brain tumors are not normally benign and they produce focal effects in all patients. Treatment options are not dependent primarily on age.

A nurse is caring for a patient who has a diagnosis of acute leukemia. What assessment most directly addresses the most common cause of death among patients with leukemia? A) Monitoring for infection B) Monitoring nutritional status C) Monitor electrolyte levels D) Monitoring liver function

A Feedback: In patients with acute leukemia, death typically occurs from infection or bleeding. Compromised nutrition, electrolyte imbalances, and impaired liver function are all plausible, but none is among the most common causes of death in this patient population.

The nurse is caring for a patient with severe left ventricular dysfunction who has been identified as being at risk for sudden cardiac death. What medical intervention can be performed that may extend the survival of the patient? A) Insertion of an implantable cardioverter defibrillator B) Insertion of an implantable pacemaker C) Administration of a calcium channel blocker D) Administration of a beta-blocker

A Feedback: In patients with severe left ventricular dysfunction and the possibility of life-threatening dysrhythmias, placement of an implantable cardioverter defibrillator (ICD) can prevent sudden cardiac death and extend survival. A pacemaker, a calcium channel blocker, and a beta-blocker are not medical interventions that may extend the survival of the patient with left ventricular dysfunction.

The nurse is caring for a patient who sustained a moderate head injury following a bicycle accident. The nurses most recent assessment reveals that the patients respiratory effort has increased. What is the nurses most appropriate response? A) Inform the care team and assess for further signs of possible increased ICP. B) Administer bronchodilators as ordered and monitor the patients LOC. C) Increase the patients bed height and reassess in 30 minutes. D) Administer a bolus of normal saline as ordered.

A Feedback: Increased respiratory effort can be suggestive of increasing ICP, and the care team should be promptly informed. A bolus of IV fluid will not address the problem. Repositioning the patient and administering bronchodilators are insufficient responses, even though these actions may later be ordered.

A patient has been found to have an indolent neoplasm. The nurse should recognize what implication of this condition? A) The patient faces a significant risk of malignancy. B) The patient has a myeloid form of leukemia. C) The patient has a lymphocytic form of leukemia. D) The patient has a major risk factor for hemophilia.

A Feedback: Indolent neoplasms have the potential to develop into a neoplasm, but this is not always the case. The patient does not necessary have, or go on to develop, leukemia. Indolent neoplasms are unrelated to the pathophysiology of hemophilia.

A patient with a diagnosis of acute myeloid leukemia (AML) is being treated with induction therapy on the oncology unit. What nursing action should be prioritized in the patients care plan? A) Protective isolation and vigilant use of standard precautions B) Provision of a high-calorie, low-texture diet and appropriate oral hygiene C) Including the family in planning the patients activities of daily living D) Monitoring and treating the patients pain

A Feedback: Induction therapy causes neutropenia and a severe risk of infection. This risk must be addressed directly in order to ensure the patients survival. For this reason, infection control would be prioritized over nutritional interventions, family care, and pain, even though each of these are important aspects of nursing care.

A nurse on a solid organ transplant unit is planning the care of a patient who will soon be admitted upon immediate recovery following liver transplantation. What aspect of nursing care is the nurses priority? A) Implementation of infection-control measures B) Close monitoring of skin integrity and color C) Frequent assessment of the patients psychosocial status D) Administration of antiretroviral medication

A Feedback: Infection control is paramount following liver transplantation. This is a priority over skin integrity and psychosocial status, even though these are valid areas of assessment and intervention. Antiretrovirals are not indicated.

A patient is being discharged after a liver transplant and the nurse is performing discharge education. When planning this patients continuing care, the nurse should prioritize which of the following risk diagnoses? A) Risk for Infection Related to Immunosuppressant Use B) Risk for Injury Related to Decreased Hemostasis C) Risk for Unstable Blood Glucose Related to Impaired Gluconeogenesis D) Risk for Contamination Related to Accumulation of Ammonia

A Feedback: Infection is the leading cause of death after liver transplantation. Pulmonary and fungal infections are common; susceptibility to infection is increased by the immunosuppressive therapy that is needed to prevent rejection. This risk exceeds the threats of injury and unstable blood glucose. The diagnosis of Risk for Contamination relates to environmental toxin exposure

After a major ischemic stroke, a possible complication is cerebral edema. Nursing care during the immediate recovery period from an ischemic stroke should include which of the following? A) Positioning to avoid hypoxia B) Maximizing PaCO2 C) Administering hypertonic IV solution D) Initiating early mobilization

A Feedback: Interventions during this period include measures to reduce ICP, such as administering an osmotic diuretic (e.g., mannitol), maintaining the partial pressure of carbon dioxide (PaCO2 ) within the range of 30 to 35 mm Hg, and positioning to avoid hypoxia. Hypertonic IV solutions are not used unless sodium depletion is evident. Mobilization would take place after the immediate threat of increased ICP has past.

A nurse is preparing to administer a patients intravenous fat emulsion simultaneously with parenteral nutrition (PN). Which of the following principles should guide the nurses action? A) Intravenous fat emulsions may be infused simultaneously with PN through a Y-connector close to the infusion site and should not be filtered. B) The nurse should prepare for placement of another intravenous line, as intravenous fat emulsions may not be infused simultaneously through the line used for PN. C) Intravenous fat emulsions may be infused simultaneously with PN through a Y-connector close to the infusion site after running the emulsion through a filter. D) The intravenous fat emulsions can be piggy-backed into any existing IV solution that is infusing

A Feedback: Intravenous fat emulsions may be infused simultaneously with PN through a Y-connector close to the infusion site and should not be filtered. The patient does not need another intravenous line for the fat emulsion. The IVFE cannot be piggy-backed into any existing IV solution that is infusing.

A patient is brought to the ED by two police officers. The patient was found unconscious on the sidewalk, with his face and hands covered in blood. At present, the patient is verbally abusive and is fighting the staff in the ED, but appears medically stable. The decision is made to place the patient in restraints. What action should the nurse perform when the patient is restrained? A) Frequently assess the patients skin integrity. B) Inform the patient that he is likely to be charged with assault. C) Avoid interacting with the patient until the restraints are removed. D) Take the opportunity to perform a full physical assessment.

A Feedback: It is important to assess skin integrity when physical restraints are used. Criminal charges are not the responsibility of the nurse and the nurse should still interact with the patient. A full physical assessment, however, would likely be delayed until the patient is not combative.

A patient has developed hepatic encephalopathy secondary to cirrhosis and is receiving care on the medical unit. The patients current medication regimen includes lactulose (Cephulac) four times daily. What desired outcome should the nurse relate to this pharmacologic intervention? A) Two to 3 soft bowel movements daily B) Significant increase in appetite and food intake C) Absence of nausea and vomiting D) Absence of blood or mucus in stool

A Feedback: Lactulose (Cephulac) is administered to reduce serum ammonia levels. Two or three soft stools per day are desirable; this indicates that lactulose is performing as intended. Lactulose does not address the patients appetite, symptoms of nausea and vomiting, or the development of blood and mucus in the stool

An oncology nurse is providing health education for a patient who has recently been diagnosed with leukemia. What should the nurse explain about commonalities between all of the different subtypes of leukemia? A) The different leukemias all involve unregulated proliferation of white blood cells. B) The different leukemias all have unregulated proliferation of red blood cells and decreased bone marrow function. C) The different leukemias all result in a decrease in the production of white blood cells. D) The different leukemias all involve the development of cancer in the lymphatic system.

A Feedback: Leukemia commonly involves unregulated proliferation of white blood cells. Decreased production of red blood cells is associated with anemias. Decreased production of white blood cells is associated with leukopenia. The leukemias are not characterized by their involvement with the lymphatic system.

A patient has received treatment for oral cancer. The combination of medications and radiotherapy has resulted in leukopenia. Which of the following is an appropriate response to this change in health status? A) Ensure that none of the patients visitors has an infection. B) Arrange for a diet that is high in protein and low in fat. C) Administer colony stimulating factors (CSFs) as ordered. D) Prepare to administer chemotherapeutics as ordered.

A Feedback: Leukopenia reduces defense mechanisms, increasing the risk of infections. Visitors who might transmit microorganisms are prohibited if the patients immunologic system is depressed. Changes in diet, CSFs, and the use of chemotherapy do not resolve leukopenia.

The nurse in a pediatric ICU is caring for a child who is dying of sickle cell anemia. The childs mother has been unable to eat or sleep and can talk only about her impending loss and the guilt she feels about the childs pain and suffering. What intervention has the highest priority? A) Allowing the patient to express her feelings without judging her B) Helping the patient to understand the phases of the grieving process C) Reassuring the patient that the childs death is not her fault D) Arranging for genetic counseling to inform the patient of her chances of having another child with the disease

A Feedback: Listening to the patient express her feelings openly without judging her is the highest priority. The nurse should not impose his or her own values on the patient. The nurse should also help the patient to understand the grieving process and use all the support systems that are available to assist her in coping with this situation. Genetic counseling may be appropriate at a later time.

The nurse is assessing a 73-year-old patient who was diagnosed with metastatic prostate cancer. The nurse notes that the patient is exhibiting signs of loss, grief, and intense sadness. Based on this assessment data, the nurse will document that the patient is most likely in what stage of death and dying? A) Depression B) Denial C) Anger D) Resignation

A Feedback: Loss, grief, and intense sadness indicate depression. Denial is indicated by the refusal to admit the truth or reality. Anger is indicated by rage and resentment. Acceptance is indicated by a gradual, peaceful withdrawal from life.

A male patient presents at the free clinic with complaints of impotency. Upon physical examination, the nurse practitioner notes the presence of hypogonadism. What diagnosis should the nurse suspect? A) Prolactinoma B) Angioma C) Glioma D) Adrenocorticotropic hormone (ACTH)producing adenoma

A Feedback: Male patients with prolactinomas may present with impotence and hypogonadism. An ACTH-producing adenoma would cause acromegaly. The scenario contains insufficient information to know if the tumor is an angioma, glioma, or neuroma

A patient on the medical unit is dying and the nurse has determined that the familys psychosocial needs during the dying process need to be addressed. What is a cause of many patient care dilemmas at the end of life? A) Poor communication between the family and the care team B) Denial of imminent death on the part of the family or the patient C) Limited visitation opportunities for friends and family D) Conflict between family members

A Feedback: Many dilemmas in patient care at the end of life are related to poor communication between team members and the patient and family, as well as to failure of team members to communicate with each other effectively. Regardless of the care setting, the nurse can ensure a proactive approach to the psychosocial care of the patient and family. Denial of death may be a response to the situation, but it is not classified as a need. Visitation should accommodate wishes of the family member as long as patient care is not compromised.

A patient is undergoing preoperative teaching before his cardiac surgery and the nurse is aware that a temporary pacemaker will be placed later that day. What is the nurses responsibility in the care of the patients pacemaker? A) Monitoring for pacemaker malfunction or battery failure B) Determining when it is appropriate to remove the pacemaker C) Making necessary changes to the pacemaker settings D) Selecting alternatives to future pacemaker use

A Feedback: Monitoring for pacemaker malfunctioning and battery failure is a nursing responsibility. The other listed actions are physician responsibilities.

A patient with ongoing back pain, nausea, and abdominal bloating has been diagnosed with cholecystitis secondary to gallstones. The nurse should anticipate that the patient will undergo what intervention? A) Laparoscopic cholecystectomy B) Methyl tertiary butyl ether (MTBE) infusion C) Intracorporeal lithotripsy D) Extracorporeal shock wave therapy (ESWL)

A Feedback: Most of the nonsurgical approaches, including lithotripsy and dissolution of gallstones, provide only temporary solutions to gallstone problems and are infrequently used in the United States. Cholecystectomy is the preferred treatment.

A patient has been exposed to a nerve agent in a biochemical terrorist attack. This type of agent bonds with acetylcholinesterase, so that acetylcholine is not inactivated. What is the pathologic effect of this type of agent? A) Hyperstimulation of the nerve endings B) Temporary deactivation of the nerve endings C) Binding of the nerve endings D) Destruction of the nerve endings

A Feedback: Nerve agents can be inhaled or absorbed percutaneously or subcutaneously. These agents bond with acetylcholinesterase, so that acetylcholine is not inactivated; the adverse result is continuous stimulation (hyperstimulation) of the nerve endings. Nerve endings are not deactivated, bound, or destroyed.

A nurse is participating in the planning of a hospitals emergency operations plan. The nurse is aware of the potential for ethical dilemmas during a disaster or other emergency. Ethical dilemmas in these contexts are best addressed by which of the following actions? A) Having an ethical framework in place prior to an emergency B) Allowing staff to provide care anonymously during an emergency C) Assuring staff that they are not legally accountable for care provided during an emergency D) Teaching staff that principles of ethics do not apply in an emergency situation

A Feedback: Nurses can plan for the ethical dilemmas they may face during disasters by establishing a framework for evaluating ethical questions before they arise and by identifying and exploring possible responses to difficult clinical situations. Ethical principles do not become wholly irrelevant in emergencies. Care cannot be given anonymously and accountability for practice always exists, even in an emergency.

A 60-year-old patient with chronic myeloid leukemia will be treated in the home setting and the nurse is preparing appropriate health education. What topic should the nurse emphasize? A) The importance of adhering to the prescribed drug regimen B) The need to ensure that vaccinations are up to date C) The importance of daily physical activity D) The need to avoid shellfish and raw foods

A Feedback: Nurses need to understand that the effectiveness of the drugs used to treat CML is based on the ability of the patient to adhere to the medication regimen as prescribed. Adherence is often incomplete, thus this must be a focus of health education. Vaccinations normally would not be administered during treatment and daily physical activity may be impossible for the patient. Dietary restrictions are not normally necessary.

A patient has been brought to the ED with multiple trauma after a motor vehicle accident. After immediate threats to life have been addressed, the nurse and trauma team should take what action? A) Perform a rapid physical assessment. B) Initiate health education. C) Perform diagnostic imaging. D) Establish the circumstances of the accident.

A Feedback: Once immediate threats to life have been corrected, a rapid physical examination is done to identify injuries and priorities of treatment. Health education is initiated later in the care process and diagnostic imaging would take place after a rapid physical assessment. It is not the care teams responsibility to determine the circumstances of the accident.

. A patient with a cholelithiasis has been scheduled for a laparoscopic cholecystectomy. Why is laparoscopic cholecystectomy preferred by surgeons over an open procedure? A) Laparoscopic cholecystectomy poses fewer surgical risks than an open procedure. B) Laparoscopic cholecystectomy can be performed in a clinic setting, while an open procedure requires an OR. C) A laparoscopic approach allows for the removal of the entire gallbladder. D) A laparoscopic approach can be performed under conscious sedation.

A Feedback: Open surgery has largely been replaced by laparoscopic cholecystectomy (removal of the gallbladder through a small incision through the umbilicus). As a result, surgical risks have decreased, along with the length of hospital stay and the long recovery period required after standard surgical cholecystectomy. Both approaches allow for removal of the entire gallbladder and must be performed under general anesthetic in an operating theater.

A nurse who provides care in an ambulatory clinic integrates basic cancer screening into admission assessments. What patient most likely faces the highest immediate risk of oral cancer? A) A 65-year-old man with alcoholism who smokes B) A 45-year-old woman who has type 1 diabetes and who wears dentures C) A 32-year-old man who is obese and uses smokeless tobacco D) A 57-year-old man with GERD and dental caries

A Feedback: Oral cancers are often associated with the use of alcohol and tobacco, which when used together have a synergistic carcinogenic effect. Most cases of oral cancers occur in people over the age of 60 and a disproportionate number of cases occur in men. Diabetes, dentures, dental caries, and GERD are not risk factors for oral cancer.

The nurse is preparing a patient for cardiac surgery. During the procedure, the patients heart will be removed and a donor heart implanted at the vena cava and pulmonary veins. What procedure will this patient undergo? A) Orthotopic transplant B) Xenograft C) Heterotropic transplant D) Homograft

A Feedback: Orthotopic transplantation is the most common surgical procedure for cardiac transplantation. The recipients heart is removed, and the donor heart is implanted at the vena cava and pulmonary veins. Some surgeons still prefer to remove the recipients heart, leaving a portion of the recipients atria (with the vena cava and pulmonary veins) in place. Homografts, or allografts (i.e., human valves), are obtained from cadaver tissue donations and are used for aortic and pulmonic valve replacement. Xenografts and heterotropic transplantation are not terms used to describe heart transplantation.

The nurse is caring for a patient who has undergone percutaneous transluminal coronary angioplasty (PTCA). What is the major indicator of success for this procedure? A) Increase in the size of the arterys lumen B) Decrease in arterial blood flow in relation to venous flow C) Increase in the patients resting heart rate D) Increase in the patients level of consciousness (LOC)

A Feedback: PTCA is used to open blocked coronary vessels and resolve ischemia. The procedure may result in beneficial changes to the patients LOC or heart rate, but these are not the overarching goals of PTCA. Increased arterial flow is the focus of the procedures.

A patient who is receiving care for osteosarcoma has been experiencing severe pain since being diagnosed. As a result, the patient has been receiving analgesics on both a scheduled and PRN basis. For the past several hours, however, the patients level of consciousness has declined and she is now unresponsive. How should the patients pain control regimen be affected? A) The patients pain control regimen should be continued. B) The pain control regimen should be placed on hold until the patients level of consciousness improves. C) IV analgesics should be withheld and replaced with transdermal analgesics. D) The patients analgesic dosages should be reduced by approximately one half.

A Feedback: Pain should be aggressively treated, even if dying patients become unable to verbally report their pain. There is no need to forego the IV route. There is no specific need to discontinue the pain control regiment or to reduce it

An emergency department nurse has just received a patient with burn injuries brought in by ambulance. The paramedics have started a large-bore IV and covered the burn in cool towels. The burn is estimated as covering 24% of the patients body. How should the nurse best address the pathophysiologic changes resulting from major burns during the initial burn-shock period? A) Administer IV fluids B) Administer broad-spectrum antibiotics C) Administer IV potassium chloride D) Administer packed red blood cells

A Feedback: Pathophysiologic changes resulting from major burns during the initial burn-shock period include massive fluid losses. Addressing these losses is a major priority in the initial phase of treatment. Antibiotics and PRBCs are not normally administered. Potassium chloride would exacerbate the patients hyperkalemia.

A cardiovascular patient with a previous history of pulmonary embolism (PE) is experiencing a sudden onset of dyspnea, rapid breathing, and chest pain. The nurse recognizes the characteristic signs and symptoms of a PE. What is the nurses best action? A) Rapidly assess the patients cardiopulmonary status. B) Arrange for an ECG. C) Increase the height of the patients bed. D) Manage the patients anxiety.

A Feedback: Patient management in the event of a PE begins with cardiopulmonary assessment and intervention. This is a priority over ECG monitoring, management of anxiety, or repositioning of the patient, even though each of these actions may be appropriate and necessary.

A patient with HF has met with his primary care provider and begun treatment with an angiotensin- converting enzyme (ACE) inhibitor. When the patient begins treatment, the nurse should prioritize what assessment? A) Blood pressure B) Level of consciousness (LOC) C) Assessment for nausea D) Oxygen saturation

A Feedback: Patients receiving ACE inhibitors are monitored for hypotension, hyperkalemia (increased potassium in the blood), and alterations in renal function. ACE inhibitors do not typically cause alterations in LOC. Oxygen saturation must be monitored in patients with HF, but this is not particular to ACE inhibitor therapy. ACE inhibitors do not normally cause nausea.

A patient who has undergone valve replacement surgery is being prepared for discharge home. Because the patient will be discharged with a prescription for warfarin (Coumadin), the nurse should educate the patient about which of the following? A) The need for regularly scheduled testing of the patients International Normalized Ratio (INR) B) The need to learn to sleep in a semi-Fowlers position for the first 6 to 8 weeks to prevent emboli C) The need to avoid foods that contain vitamin K D) The need to take enteric-coated ASA on a daily basis

A Feedback: Patients who take warfarin (Coumadin) after valve replacement have individualized target INRs; usually between 2 and 3.5 for mitral valve replacement and 1.8 and 2.2 for aortic valve replacement. Natural sources of vitamin K do not normally need to be avoided and ASA is not indicated. Sleeping upright is unnecessary.

A nurse is planning the care of a patient who has been diagnosed with essential thrombocythemia (ET). What nursing diagnosis should the nurse prioritize when choosing interventions? A) Risk for Ineffective Tissue Perfusion B) Risk for Imbalanced Fluid Volume C) Risk for Ineffective Breathing Pattern D) Risk for Ineffective Thermoregulation

A Feedback: Patients with ET are at risk for hypercoagulation and consequent ineffective tissue perfusion. Fluid volume, breathing, and thermoregulation are not normally affected.

The nurse is providing patient education prior to a patients discharge home after treatment for HF. The nurse gives the patient a home care checklist as part of the discharge teaching. What should be included on this checklist? A) Know how to recognize and prevent orthostatic hypotension. B) Weigh yourself weekly at a consistent time of day. C) Measure everything you eat and drink until otherwise instructed. D) Limit physical activity to only those tasks that are absolutely necessary.

A Feedback: Patients with HF should be aware of the risks of orthostatic hypotension. Weight should be measured daily; detailed documentation of all forms of intake is not usually required. Activity should be gradually increased within the parameters of safety and comfort.

A patient who suffered an ischemic stroke now has disturbed sensory perception. What principle should guide the nurses care of this patient? A) The patient should be approached on the side where visual perception is intact. B) Attention to the affected side should be minimized in order to decrease anxiety. C) The patient should avoid turning in the direction of the defective visual field to minimize shoulder subluxation. D) The patient should be approached on the opposite side of where the visual perception is intact to promote recovery.

A Feedback: Patients with decreased field of vision should first be approached on the side where visual perception is intact. All visual stimuli should be placed on this side. The patient can and should be taught to turn the head in the direction of the defective visual field to compensate for this loss. The nurse should constantly remind the patient of the other side of the body and should later stand at a position that encourages the patient to move or turn to visualize who and what is in the room.

A patient with a history of cirrhosis is admitted to the ICU with a diagnosis of bleeding esophageal varices; an attempt to stop the bleeding has been only partially successful. What would the critical care nurse expect the care team to order for this patient? A) Packed red blood cells (PRBCs) B) Vitamin K C) Oral anticoagulants D) Heparin infusion

A Feedback: Patients with liver dysfunction may have life-threatening hemorrhage from peptic ulcers or esophageal varices. In these cases, replacement with fresh frozen plasma, PRBCs, and platelets is usually required. Vitamin K may be ordered once the bleeding is stopped, but that is not what is needed to stop the bleeding of the varices. Anticoagulants would exacerbate the patients bleeding.

The nurse who is leading a wellness workshop has been asked about actions to reduce the risk of bladder cancer. What health promotion action most directly addresses a major risk factor for bladder cancer? A) Smoking cessation B) Reduction of alcohol intake C) Maintenance of a diet high in vitamins and nutrients D) Vitamin D supplementation

A Feedback: People who smoke develop bladder cancer twice as often as those who do not smoke. High alcohol intake and low vitamin intake are not noted to contribute to bladder cancer.

A patient with ESKD receives continuous ambulatory peritoneal dialysis. The nurse observes that the dialysate drainage fluid is cloudy. What is the nurses most appropriate action? A) Inform the physician and assess the patient for signs of infection. B) Flush the peritoneal catheter with normal saline. C) Remove the catheter promptly and have the catheter tip cultured. D) Administer a bolus of IV normal saline as ordered.

A Feedback: Peritonitis is the most common and serious complication of peritoneal dialysis. The first sign of peritonitis is cloudy dialysate drainage fluid, so prompt reporting to the primary care provider and rapid assessment for other signs of infection are warranted. Administration of an IV bolus is not necessary or appropriate and the physician would determine whether removal of the catheter is required. Flushing the catheter does not address the risk for infection.

When discussing angina pectoris secondary to atherosclerotic disease with a patient, the patient asks why he tends to experience chest pain when he exerts himself. The nurse should describe which of the following phenomena? A) Exercise increases the hearts oxygen demands. B) Exercise causes vasoconstriction of the coronary arteries. C) Exercise shunts blood flow from the heart to the mesenteric area. D) Exercise increases the metabolism of cardiac medications.

A Feedback: Physical exertion increases the myocardial oxygen demand. If the patient has arteriosclerosis of the coronary arteries, then blood supply is diminished to the myocardium. Exercise does not cause vasoconstriction or interfere with drug metabolism. Exercise does not shunt blood flow away from the heart.

The nurse is caring for a patient who is scheduled for cardiac surgery. What should the nurse include in preoperative care? A) With the patient, clarify the surgical procedure that will be performed. B) Withhold the patients scheduled medications for at least 12 hours preoperatively. C) Inform the patient that health teaching will begin as soon as possible after surgery. D) Avoid discussing the patients fears as not to exacerbate them.

A Feedback: Preoperatively, it is necessary to evaluate the patients understanding of the surgical procedure, informed consent, and adherence to treatment protocols. Teaching would begin on admission or even prior to admission. The physician would write orders to alter the patients medication regimen if necessary; this will vary from patient to patient. Fears should be addressed directly and empathically.

A patient has been admitted to the neurologic ICU with a diagnosis of a brain tumor. The patient is scheduled to have a tumor resection/removal in the morning. Which of the following assessment parameters should the nurse include in the initial assessment? A) Gag reflex B) Deep tendon reflexes C) Abdominal girth D) Hearing acuity

A Feedback: Preoperatively, the gag reflex and ability to swallow are evaluated. In patients with diminished gag response, care includes teaching the patient to direct food and fluids toward the unaffected side, having the patient sit upright to eat, offering a semisoft diet, and having suction readily available. Deep tendon reflexes, abdominal girth, and hearing acuity are less commonly affected by brain tumors and do not affect the risk for aspiration.

A patient admitted to the medical unit with HF is exhibiting signs and symptoms of pulmonary edema. The nurse is aware that positioning will promote circulation. How should the nurse best position the patient? A) In a high Fowlers position B) On the left side-lying position C) In a flat, supine position D) In the Trendelenburg position

A Feedback: Proper positioning can help reduce venous return to the heart. The patient is positioned upright. If the patient is unable to sit with the lower extremities dependent, the patient may be placed in an upright position in bed. The supine position and Trendelenburg positions will not reduce venous return, lower the output of the right ventricle, or decrease lung congestion. Similarly, side-lying does not promote circulation.

A patients absolute neutrophil count (ANC) is 440/mm3 . But the nurses assessment reveals no apparent signs or symptoms of infection. What action should the nurse prioritize when providing care for this patient? A) Meticulous hand hygiene B) Timely administration of antibiotics C) Provision of a nutrient-dense diet D) Maintaining a sterile care environment

A Feedback: Providing care for a patient with neutropenia requires that the nurse adhere closely to standard precautions and infection control procedures. Hand hygiene is central to such efforts. Prophylactic antibiotics are rarely used and it is not possible to provide a sterile environment for care. Nutrition is highly beneficial, but hand hygiene is the central aspect of care.

The ED staff has been notified of the imminent arrival of a patient who has been exposed to chlorine. The nurse should anticipate the need to address what nursing diagnosis? A) Impaired gas exchange B) Decreased cardiac output C) Chronic pain D) Excess fluid volume

A Feedback: Pulmonary agents, such as phosgene and chlorine, destroy the pulmonary membrane that separates the alveolus from the capillary bed, disrupting alveolarcapillary oxygen transport mechanisms. Capillary leakage results in fluid-filled alveoli and gas exchange ceases to occur. Pain is likely, but is acute rather than chronic. Fluid volume excess is unlikely to be a priority diagnosis and cardiac output will be secondarily affected by the pulmonary effects.

The nursing supervisor at the local hospital is advised that your hospital will be receiving multiple trauma victims from a blast that occurred at a local manufacturing plant. The paramedics call in a victim of the blast with injuries including a head injury and hemorrhage. What phase of blast injury should the nurse expect to treat in this patient? A) Primary phase B) Secondary phase C) Tertiary phase D) Quaternary phase

A Feedback: Pulmonary barotraumas, including pulmonary contusions; head injuries, including concussion, other severe brain injuries; tympanic membrane rupture, middle ear injury; abdominal hollow organ perforation; and hemorrhage are all injuries that can occur in the primary phase of a blast. These particular injuries are not characteristic of the subsequent phases.

Following an extensive diagnostic workup, an older adult patient has been diagnosed with a secondary myelodysplastic syndrome (MDS). What assessment question most directly addresses the potential etiology of this patients health problem? A) Were you ever exposed to toxic chemicals in any of the jobs that you held? B) When you were younger, did you tend to have recurrent infections of any kind? C) Have your parents or siblings had any disease like this? D) Would you say that youve had a lot of sun exposure in your lifetime?

A Feedback: Secondary MDS can occur at any age and results from prior toxic exposure to chemicals, including chemotherapeutic medications. Family history, sun exposure, and previous infections are unrelated to the pathophysiology of secondary MDS.

A 37-year-old male patient presents at the emergency department (ED) complaining of nausea and vomiting and severe abdominal pain. The patients abdomen is rigid, and there is bruising to the patients flank. The patients wife states that he was on a drinking binge for the past 2 days. The ED nurse should assist in assessing the patient for what health problem? A) Severe pancreatitis with possible peritonitis B) Acute cholecystitis C) Chronic pancreatitis D) Acute appendicitis with possible perforatio

A Feedback: Severe abdominal pain is the major symptom of pancreatitis that causes the patient to seek medical care. Pain in pancreatitis is accompanied by nausea and vomiting that does not relieve the pain or nausea. Abdominal guarding is present and a rigid or board-like abdomen may be a sign of peritonitis. Ecchymosis (bruising) to the flank or around the umbilicus may indicate severe peritonitis. Pain generally occurs 24 to 48 hours after a heavy meal or alcohol ingestion. The link with alcohol intake makes pancreatitis a more likely possibility than appendicitis or cholecystitis.

The nurse is caring for a patient with increased intracranial pressure (ICP) caused by a traumatic brain injury. Which of the following clinical manifestations would suggest that the patient may be experiencing increased brain compression causing brain stem damage? A) Hyperthermia B) Tachycardia C) Hypertension D) Bradypnea

A Feedback: Signs of increasing ICP include slowing of the heart rate (bradycardia), increasing systolic BP, and widening pulse pressure. As brain compression increases, respirations become rapid, BP may decrease, and the pulse slows further. A rapid rise in body temperature is regarded as unfavorable. Hyperthermia increases the metabolic demands of the brain and may indicate brain stem damage.

A triage nurse in the emergency department is assessing a patient who presented with complaints of general malaise. Assessment reveals the presence of jaundice and increased abdominal girth. What assessment question best addresses the possible etiology of this patients presentation? A) How many alcoholic drinks do you typically consume in a week? B) To the best of your knowledge, are your immunizations up to date? C) Have you ever worked in an occupation where you might have been exposed to toxins? D) Has anyone in your family ever experienced symptoms similar to yours?

A Feedback: Signs or symptoms of hepatic dysfunction indicate a need to assess for alcohol use. Immunization status, occupational risks, and family history are also relevant considerations, but alcohol use is a more common etiologic factor in liver disease.

You are making initial shift assessments on your patients. While assessing one patients peripheral IV site, you note edema around the insertion site. How should you document this complication related to IV therapy? A) Air emboli B) Phlebitis C) Infiltration D) Fluid overload

Ans C

The client diagnosed with a brain tumor is being admitted to the medical oncology unit at 2000. Which health-care provider's order should be implemented first? 1. Regular soft diet with between-meal snacks. 2. Dexamethasone (Decadron), a steroid, every 6 hours IVP. 3. Prochlorperazine (Compazine), an antiemetic, a.c. 4. CBC and chemistry panel laboratory tests

Ans: 2 The diet is not a priority over preventing increased intracranial pressure resulting from the tumor. It is 2000, or 8 P.M., and meals are usually served in hospitals around 0800, 1200, and 1700. The next meal will be served at 0800.2. Dexamethasone is the glucocorticoid of choice for brain swelling. The client is at risk for increased intracranial pressure as a result of the tumor and edema caused by the tumor. The nurse should administer the steroid first to initiate the positive effects of the medication.3. This medication is ordered a.c., which means "before meals." The next dose of this medication is not until 0730.4. These are routine laboratory tests and will not be drawn until the next morning.

A patient who is being treated for pneumonia starts complaining of sudden shortness of breath. An arterial blood gas (ABG) is drawn. The ABG has the following values: pH 7.21, PaCO2 64 mm Hg, HCO3 = 24 mm Hg. What does the ABG reflect? A) Respiratory acidosis B) Metabolic alkalosis C) Respiratory alkalosis D) Metabolic acidosis

Ans: A

Postural drainage has been ordered for a patient who is having difficulty mobilizing her bronchial secretions. Before repositioning the patient and beginning treatment, the nurse should perform what health assessment? A) Chest auscultation B) Pulmonary function testing C) Chest percussion D) Thoracic palpation

Ans: A Feedback: Chest auscultation should be performed before and after postural drainage in order to evaluate the effectiveness of the therapy. Percussion and palpation are less likely to provide clinically meaningful data for the nurse. PFTs are normally beyond the scope of the nurse and are not necessary immediately before postural drainage.

A nurse practitioner has provided care for three different patients with chronic pharyngitis over the past several months. Which patients are at greatest risk for developing chronic pharyngitis? A) Patients who are habitual users of alcohol and tobacco B) Patients who are habitual users of caffeine and other stimulants C) Patients who eat a diet high in spicy foods D) Patients who have gastrointestinal reflux disease (GERD)

Ans: A Feedback: Chronic pharyngitis is common in adults who live and work in dusty surroundings, use the voice to excess, suffer from chronic chough, and habitually use alcohol and tobacco. Caffeine and spicy foods have not been linked to chronic pharyngitis. GERD is not a noted risk factor.

You are caring for a patient admitted with a diagnosis of acute kidney injury. When you review your patients most recent laboratory reports, you note that the patients magnesium levels are high. You should prioritize assessment for which of the following health problems? A) Diminished deep tendon reflexes B) Tachycardia C) Cool, clammy skin D) Acute flank pain

Ans: A Feedback: To gauge a patients magnesium status, the nurse should check deep tendon reflexes. If the reflex is absent, this may indicate high serum magnesium. Tachycardia, flank pain, and cool, clammy skin are not typically associated with hypermagnesemia.

A patient the nurse is caring for has a permanent pacemaker implanted with the identification code beginning with VVI. What does this indicate? A) Ventricular paced, ventricular sensed, inhibited B) Variable paced, ventricular sensed, inhibited C) Ventricular sensed, ventricular situated, implanted D) Variable sensed, variable paced, inhibited

Ans: A Feedback: The identification of VVI indicates ventricular paced, ventricular sensed, inhibited.

The nurse, a member of the health care team in the ED, is caring for a patient who is determined to be in the irreversible stage of shock. What would be the most appropriate nursing intervention? A) Provide opportunities for the family to spend time with the patient, and help them to understand the irreversible stage of shock. B) Inform the patients family immediately that the patient will likely not survive to allow the family time to make plans and move forward. C) Closely monitor fluid replacement therapy, and inform the family that the patient will probably survive and return to normal life. D) Protect the patients airway, optimize intravascular volume, and initiate the early rehabilitation process.

Ans: A Feedback: The irreversible (or refractory) stage of shock represents the point along the shock continuum at which organ damage is so severe that the patient does not respond to treatment and cannot survive. Providing opportunities for the family to spend time with the patient and helping them to understand the irreversible stage of shock is the best intervention. Informing the patients family early that the patient will likely not survive does allow the family to make plans and move forward, but informing the family too early will rob the family of hope and interrupt the grieving process. The chance of surviving the irreversible (or refractory) stage of shock is very small, and the nurse needs to help the family cope with the reality of the situation. With the chances of survival so small, the priorities shift from aggressive treatment and safety to addressing the end-of-life issues.

A patient has been brought to the ED by the paramedics. The patient is suspected of having ARDS. What intervention should the nurse first anticipate? A) Preparing to assist with intubating the patient B) Setting up oxygen at 5 L/minute by nasal cannula C) Performing deep suctioning D) Setting up a nebulizer to administer corticosteroids

Ans: A Feedback: A patient who has ARDS usually requires intubation and mechanical ventilation. Oxygen by nasal cannula would likely be insufficient. Deep suctioning and nebulizers may be indicated, but the priority is to secure the airway.

The critical care nurse and the other members of the care team are assessing the patient to see if he is ready to be weaned from the ventilator. What are the most important predictors of successful weaning that the nurse should identify? A) Stable vital signs and ABGs B) Pulse oximetry above 80% and stable vital signs C) Stable nutritional status and ABGs D) Normal orientation and level of consciousness

Ans: A Feedback: Among many other predictors, stable vital signs and ABGs are important predictors of successful weaning. Pulse oximetry must greatly exceed 80%. Nutritional status is important, but vital signs and ABGs are even more significant. Patients who are weaned may or may not have full level of consciousness.

An x-ray of a trauma patient reveals rib fractures and the patient is diagnosed with a small flail chest injury. Which intervention should the nurse include in the patients plan of care? A) Suction the patients airway secretions. B) Immobilize the ribs with an abdominal binder. C) Prepare the patient for surgery. D) Immediately sedate and intubate the patient.

Ans: A Feedback: As with rib fracture, treatment of flail chest is usually supportive. Management includes clearing secretions from the lungs, and controlling pain. If only a small segment of the chest is involved, it is important to clear the airway through positioning, coughing, deep breathing, and suctioning. Intubation is required for severe flail chest injuries, and surgery is required only in rare circumstances to stabilize the flail segment

The nurse at a long-term care facility is assessing each of the residents. Which resident most likely faces the greatest risk for aspiration? A) A resident who suffered a severe stroke several weeks ago B) A resident with mid-stage Alzheimers disease C) A 92-year-old resident who needs extensive help with ADLs D) A resident with severe and deforming rheumatoid arthritis

Ans: A Feedback: Aspiration may occur if the patient cannot adequately coordinate protective glottic, laryngeal, and cough reflexes. These reflexes are often affected by stroke. A patient with mid-stage Alzheimers disease does not likely have the voluntary muscle problems that occur later in the disease. Clients that need help with ADLs or have severe arthritis should not have difficulty swallowing unless it exists secondary to another problem

A nurse who sits on the hospitals ethics committee is reviewing a complex case that has many of the hallmarks of assisted suicide. Which of the following would be an example of assisted suicide? A) Administering a lethal dose of medication to a patient whose death is imminent B) Administering a morphine infusion without assessing for respiratory depression C) Granting a patients request not to initiate enteral feeding when the patient is unable to eat D) Neglecting to resuscitate a patient with a do not resuscitate order

Ans: A Feedback: Assisted suicide refers to providing another person the means to end his or her own life. This is not to be confused with the ethically and legally supported practices of withholding or withdrawing medical treatment in accordance with the wishes of the terminally ill individual. The other listed options do not fit this accepted definition of assisted suicide.

A patient is exhibiting signs of a pneumothorax following tracheostomy. The surgeon inserts a chest tube into the anterior chest wall. What should the nurse tell the family is the primary purpose of this chest tube? A) To remove air from the pleural space B) To drain copious sputum secretions C) To monitor bleeding around the lungs D) To assist with mechanical ventilation

Ans: A Feedback: Chest tubes and closed drainage systems are used to re-expand the lung involved and to remove excess air, fluid, and blood. The primary purpose of a chest tube is not to drain sputum secretions, monitor bleeding, or assist with mechanical ventilation.

The ED nurse is assessing a young gymnast who fell from a balance beam. The gymnast presents with a clear fluid leaking from her nose. What should the ED nurse suspect? A) Fracture of the cribriform plate B) Rupture of an ethmoid sinus C) Abrasion of the soft tissue D) Fracture of the nasal septum

Ans: A Feedback: Clear fluid from either nostril suggests a fracture of the cribriform plate with leakage of cerebrospinal fluid. The symptoms are not indicative of an abrasion of the soft tissue or rupture of a sinus. Clear fluid leakage from the nose would not be indicative of a fracture of the nasal septum.

A nurse is caring for a patient who has been admitted with an exacerbation of chronic bronchiectasis. The nurse should expect to assess the patient for which of the following clinical manifestations? A) Copious sputum production B) Pain on inspiration C) Pigeon chest D) Dry cough

Ans: A Feedback: Clinical manifestations of bronchiectasis include hemoptysis, chronic cough, copious purulent sputum, and clubbing of the fingers. Because of the copious production of sputum, the cough is rarely dry. A pigeon chest is not associated with the disease and patients do not normally experience pain on inspiration.

The nurse is conducting patient teaching about cholesterol levels. When discussing the patients elevated LDL and lowered HDL levels, the patient shows an understanding of the significance of these levels by stating what? A) Increased LDL and decreased HDL increase my risk of coronary artery disease. B) Increased LDL has the potential to decrease my risk of heart disease. C) The decreased HDL level will increase the amount of cholesterol moved away from the artery walls. D) The increased LDL will decrease the amount of cholesterol deposited on the artery walls.

Ans: A Feedback: Elevated LDL levels and decreased HDL levels are associated with a greater incidence of coronary artery disease.

The perioperative nurse is writing a care plan for a patient who has returned from surgery 2 hours prior. Which measure should the nurse implement to most decrease the patients risk of developing pulmonary emboli (PE)? A) Early ambulation B) Increased dietary intake of protein C) Maintaining the patient in a supine position D) Administering aspirin with warfarin

Ans: A Feedback: For patients at risk for PE, the most effective approach for prevention is to prevent deep vein thrombosis. Active leg exercises to avoid venous stasis, early ambulation, and use of elastic compression stocking are general preventive measures. The patient does not require increased dietary intake of protein directly related to prevention of PE, although it will assist in wound healing during the postoperative period. The patient should not be maintained in one position, but frequently repositioned, unless contraindicated by the surgical procedure. Aspirin should never be administered with warfarin because it will increase the patients risk for bleeding.

A nurse is evaluating the diagnostic study data of a patient with suspected cystic fibrosis (CF). Which of the following test results is associated with a diagnosis of cystic fibrosis? A) Elevated sweat chloride concentration B) Presence of protein in the urine C) Positive phenylketonuria D) Malignancy on lung biopsy

Ans: A Feedback: Gene mutations affect transport of chloride ions, leading to CF, which is characterized by thick, viscous secretions in the lungs, pancreas, liver, intestine, and reproductive tract as well as increased salt content in sweat gland secretions. Proteinuria, positive phenylketonuria, and malignancy are not diagnostic for CF.

The nurse is assessing an adult patient following a motor vehicle accident. The nurse observes that the patient has an increased use of accessory muscles and is complaining of chest pain and shortness of breath. The nurse should recognize the possibility of what condition? A) Pneumothorax B) Anxiety C) Acute bronchitis D) Aspiration

Ans: A Feedback: If the pneumothorax is large and the lung collapses totally, acute respiratory distress occurs. The patient is anxious, has dyspnea and air hunger, has increased use of the accessory muscles, and may develop central cyanosis from severe hypoxemia. These symptoms are not definitive of pneumothorax, but because of the patients recent trauma they are inconsistent with anxiety, bronchitis, or aspiration.

The nurse is caring for a patient who is scheduled for a lobectomy for a diagnosis of lung cancer. While assisting with a subclavian vein central line insertion, the nurse notes the clients oxygen saturation rapidly dropping. The patient complains of shortness of breath and becomes tachypneic. The nurse suspects a pneumothorax has developed. Further assessment findings supporting the presence of a pneumothorax include what? A) Diminished or absent breath sounds on the affected side B) Paradoxical chest wall movement with respirations C) Sudden loss of consciousness D) Muffled heart sounds

Ans: A Feedback: In the case of a simple pneumothorax, auscultating the breath sounds will reveal absent or diminished breath sounds on the affected side. Paradoxical chest wall movements occur in flail chest conditions. Sudden loss of consciousness does not typically occur. Muffled or distant heart sounds occur in pericardial tamponade.

The nurse in the ICU is admitting a 57-year-old man with a diagnosis of possible septic shock. The nurses assessment reveals that the patient has a normal blood pressure, increased heart rate, decreased bowel sounds, and cold, clammy skin. The nurses analysis of these data should lead to what preliminary conclusion? A) The patient is in the compensatory stage of shock. B) The patient is in the progressive stage of shock. C) The patient will stabilize and be released by tomorrow. D) The patient is in the irreversible stage of shock.

Ans: A Feedback: In the compensatory stage of shock, the blood pressure remains within normal limits. Vasoconstriction, increased heart rate, and increased contractility of the heart contribute to maintaining adequate cardiac output. Patients display the often-described fight or flight response. The body shunts blood from organs such as the skin, kidneys, and GI tract to the brain and heart to ensure adequate blood supply to these vital organs. As a result, the skin is cool and clammy, and bowel sounds are hypoactive. In progressive shock, the blood pressure drops. In septic shock, the patients chance of survival is low and he will certainly not be released within 24 hours. If the patient were in the irreversible stage of shock, his blood pressure would be very low and his organs would be failing.

A critical care nurse is caring for a client with an endotracheal tube who is on a ventilator. The nurse knows that meticulous airway management of this patient is necessary. What is the main rationale for this? A) Maintaining a patent airway B) Preventing the need for suctioning C) Maintaining the sterility of the patients airway D) Increasing the patients lung compliance

Ans: A Feedback: Maintaining a patent (open) airway is achieved through meticulous airway management, whether in an emergency situation such as airway obstruction or in long-term management, as in caring for a patient with an endotracheal or a tracheostomy tube. The other answers are incorrect.

A critical care nurse is aware of the high incidence of ventilator-associated pneumonia (VAP) in patients who are being treated for shock. What intervention should be specified in the patients plan of care while the patient is ventilated? A) Performing frequent oral care B) Maintaining the patient in a supine position C) Suctioning the patient every 15 minutes unless contraindicated D) Administering prophylactic antibiotics, as ordered

Ans: A Feedback: Nursing interventions that reduce the incidence of VAP must also be implemented. These include frequent oral care, aseptic suction technique, turning, and elevating the head of the bed at least 30 degrees to prevent aspiration. Suctioning should not be excessively frequent and prophylactic antibiotics are not normally indicated.

A nurse is planning the care of a client with bronchiectasis. What goal of care should the nurse prioritize? A) The patient will successfully mobilize pulmonary secretions. B) The patient will maintain an oxygen saturation level of 98%. C) The patients pulmonary blood pressure will decrease to within reference ranges. D) The patient will resume prediagnosis level of function within 72 hours.

Ans: A Feedback: Nursing management focuses on alleviating symptoms and helping patients clear pulmonary secretions. Pulmonary pressures are not a central focus in the care of the patient with bronchiectasis. Rapid resumption of prediagnosis function and oxygen saturation above 98% are unrealistic goals

The nurse is caring for a patient who has been recently diagnosed with late stage pancreatic cancer. The patient refuses to accept the diagnosis and refuses to adhere to treatment. What is the most likely psychosocial purpose of this patients strategy? A) The patient may be trying to protect loved ones from the emotional effects of the illness. B) The patient is being noncompliant in order to assert power over caregivers. C) The patient may be skeptical of the benefits of the Western biomedical model of health. D) The patient thinks that treatment does not provide him comfort.

Ans: A Feedback: Patients who are characterized as being in denial may be using this strategy to preserve important interpersonal relationships, to protect others from the emotional effects of their illness, and to protect themselves because of fears of abandonment. Each of the other listed options is plausible, but less likely.

A patient has had a nasogastric tube in place for 6 days due to the development of paralytic ileus after surgery. In light of the prolonged presence of the nasogastric tube, the nurse should prioritize assessments related to what complication? A) Sinus infections B) Esophageal strictures C) Pharyngitis D) Laryngitis

Ans: A Feedback: Patients with nasotracheal and nasogastric tubes in place are at risk for development of sinus infections. Thus, accurate assessment of patients with these tubes is critical. Use of a nasogastric tube is not associated with the development of the other listed pathologies.

The ICU nurse is caring for a patient with multiple organ dysfunction syndrome (MODS) due to shock. What nursing action should be prioritized at this point during care? A) Providing information and support to family members B) Preparing the family for a long recovery process C) Educating the patient regarding the use of supportive fluids D) Facilitating the rehabilitation phase of treatment

Ans: A Feedback: Providing information and support to family members is a critical role of the nurse. Most patients with MODS do not recover, so the rehabilitation phase of recovery is not a short-term priority. Educating the patient about the use of supportive fluids is not a high priority.

The nurse is caring for a patient at risk for atelectasis. The nurse implements a first-line measure to prevent atelectasis development in the patient. What is an example of a first-line measure to minimize atelectasis? A) Incentive spirometry B) Intermittent positive-pressure breathing (IPPB) C) Positive end-expiratory pressure (PEEP) D) Bronchoscopy

Ans: A Feedback: Strategies to prevent atelectasis, which include frequent turning, early ambulation, lung-volume expansion maneuvers (deep breathing exercises, incentive spirometry), and coughing, serve as the firstline measures to minimize or treat atelectasis by improving ventilation. In patients who do not respond to first-line measures or who cannot perform deep-breathing exercises, other treatments such as positive end-expiratory pressure (PEEP), continuous or intermittent positive-pressure breathing (IPPB), or bronchoscopy may be used.

An adult patient has tested positive for tuberculosis (TB). While providing patient teaching, what information should the nurse prioritize? A) The importance of adhering closely to the prescribed medication regimen B) The fact that the disease is a lifelong, chronic condition that will affect ADLs C) The fact that TB is self-limiting, but can take up to 2 years to resolve D) The need to work closely with the occupational and physical therapists

Ans: A Feedback: Successful treatment of TB is highly dependent on careful adherence to the medication regimen. The disease is not self-limiting; occupational and physical therapy are not necessarily indicated. TB is curable.

You are precepting a new graduate nurse in the ICU. You are collaborating in the care of a patient who is receiving large volumes of crystalloid fluid to treat hypovolemic shock. In light of this intervention, for what sign would you teach the new nurse to monitor the patient? A) Hypothermia B) Bradycardia C) Coffee ground emesis D) Pain

Ans: A Feedback: Temperature should be monitored closely to ensure that rapid fluid resuscitation does not precipitate hypothermia. IV fluids may need to be warmed during the administration of large volumes. The nurse should monitor the patient for cardiovascular overload and pulmonary edema when large volumes of IV solution are administered. Coffee ground emesis is an indication of a GI bleed, not shock. Pain is related to cardiogenic shock.

The OR nurse is setting up a water-seal chest drainage system for a patient who has just had a thoracotomy. The nurse knows that the amount of suction in the system is determined by the water level. At what suction level should the nurse set the system? A) 20 cm H2O B) 15 cm H2O C) 10 cm H2O D) 5 cm H2O

Ans: A Feedback: The amount of suction is determined by the water level. It is usually set at 20 cm H2O; adding more fluid results in more suction.

A nurse is admitting a new patient who has been admitted with a diagnosis of COPD exacerbation. How can the nurse best help the patient achieve the goal of maintaining effective oxygenation? A) Teach the patient strategies for promoting diaphragmatic breathing. B) Administer supplementary oxygen by simple face mask. C) Teach the patient to perform airway suctioning. D) Assist the patient in developing an appropriate exercise program.

Ans: A Feedback: The breathing pattern of most people with COPD is shallow, rapid, and inefficient; the more severe the disease, the more inefficient the breathing pattern. With practice, this type of upper chest breathing can be changed to diaphragmatic breathing, which reduces the respiratory rate, increases alveolar ventilation, and sometimes helps expel as much air as possible during expiration. Suctioning is not normally necessary in patients with COPD. Supplementary oxygen is not normally delivered by simple face mask and exercise may or may not be appropriate

A medical nurse is providing palliative care to a patient with a diagnosis of end-stage chronic obstructive pulmonary disease (COPD). What is the primary goal of this nurses care? A) To improve the patients and familys quality of life B) To support aggressive and innovative treatments for cure C) To provide physical support for the patient D) To help the patient develop a separate plan with each discipline of the health care team

Ans: A Feedback: The goal of palliative care is to improve the patients and the familys quality of life. The support should include the patients physical, emotional, and spiritual well-being. Each discipline should contribute to a single care plan that addresses the needs of the patient and family. The goal of palliative care is not aggressive support for curing the patient. Providing physical support for the patient is also not the goal of palliative care. Palliative care does not strive to achieve separate plans of care developed by the patient with each discipline of the health care team

The critical care nurse is caring for a patient who has been experiencing bradycardia after cardiovascular surgery. The nurse knows that the heart rate is determined by myocardial cells with the fastest inherent firing rate. Under normal circumstances where are these cells located? A) SA node B) AV node C) Bundle of His D) Purkinje cell

Ans: A Feedback: The heart rate is determined by the myocardial cells with the fastest inherent firing rate. Under normal circumstances, the SA node has the highest inherent rate (60 to 100 impulses per minute).

While planning a patients care, the nurse identifies nursing actions to minimize the patients pleuritic pain. Which intervention should the nurse include in the plan of care? A) Avoid actions that will cause the patient to breathe deeply. B) Ambulate the patient at least three times daily. C) Arrange for a soft-textured diet and increased fluid intake. D) Encourage the patient to speak as little as possible

Ans: A Feedback: The key characteristic of pleuritic pain is its relationship to respiratory movement. Taking a deep breath, coughing, or sneezing worsens the pain. A soft diet is not necessarily indicated and there is no need for the patient to avoid speaking. Ambulation has multiple benefits, but pain management is not among them.

The nurse is assessing a patient who has a 35 pack-year history of cigarette smoking. In light of this known risk factor for lung cancer, what statement should prompt the nurse to refer the patient for further assessment? A) Lately, I have this cough that just never seems to go away. B) I find that I dont have nearly the stamina that I used to. C) I seem to get nearly every cold and flu that goes around my workplace. D) I never used to have any allergies, but now I think Im developing allergies to dust and pet hair.

Ans: A Feedback: The most frequent symptom of lung cancer is cough or change in a chronic cough. People frequently ignore this symptom and attribute it to smoking or a respiratory infection. A new onset of allergies, frequent respiratory infections and fatigue are not characteristic early signs of lung cancer.

A critical-care nurse is caring for a patient diagnosed with pneumonia as a surgical complication. The nurses assessment reveals that the patient has an increased work of breathing due to copious tracheobronchial secretions. What should the nurse encourage the patient to do? A) Increase oral fluids unless contraindicated. B) Call the nurse for oral suctioning, as needed. C) Lie in a low Fowlers or supine position. D) Increase activity

Ans: A Feedback: The nurse should encourage hydration because adequate hydration thins and loosens pulmonary secretions. Oral suctioning is not sufficiently deep to remove tracheobronchial secretions. The patient should have the head of the bed raised, and rest should be promoted to avoid exacerbation of symptoms

The home care nurse is assessing the home environment of a patient who will be discharged from the hospital shortly after his laryngectomy. The nurse should inform the patient that he may need to arrange for the installation of which system in his home? A) A humidification system B) An air conditioning system C) A water purification system D) A radiant heating system

Ans: A Feedback: The nurse stresses the importance of humidification at home and instructs the family to obtain and set up a humidification system before the patient returns home. Air-conditioning may be too cool and too drying for the patient. A water purification system or a radiant heating system is not necessary.

The public health nurse is administering Mantoux tests to children who are being registered for kindergarten in the community. How should the nurse administer this test? A) Administer intradermal injections into the childrens inner forearms. B) Administer intramuscular injections into each childs vastus lateralis. C) Administer a subcutaneous injection into each childs umbilical area. D) Administer a subcutaneous injection at a 45-degree angle into each childs deltoid.

Ans: A Feedback: The purified protein derivative (PPD) is always injected into the intradermal layer of the inner aspect of the forearm. The subcutaneous and intramuscular routes are not utilized.

The nurse is caring for a patient in the ICU whose condition is deteriorating. The nurse receives orders to initiate an infusion of dopamine. What would be the priority assessment and interventions specific to the administration of vasoactive medications? A) Frequent monitoring of vital signs, monitoring the central line site, and providing accurate drug titration B) Reviewing medications, performing a focused cardiovascular assessment, and providing patient education C) Reviewing the laboratory findings, monitoring urine output, and assessing for peripheral edema D) Routine monitoring of vital signs, monitoring the peripheral IV site, and providing early discharge instructions

Ans: A Feedback: When vasoactive medications are administered, vital signs must be monitored frequently (at least every 15 minutes until stable, or more often if indicated). Vasoactive medications should be administered through a central venous line because infiltration and extravasation of some vasoactive medications can cause tissue necrosis and sloughing. An IV pump should be used to ensure that the medications are delivered safely and accurately. Individual medication dosages are usually titrated by the nurse, who adjusts drip rates based on the prescribed dose and the patients response. Reviewing medications, performing a focused cardiovascular assessment, and providing patient education are important nursing tasks, but they are not specific to the administration of IV vasoactive drugs. Reviewing the laboratory findings, monitoring urine output, and assessing for peripheral edema are not the priorities for administration of IV vasoactive drugs. Vital signs are taken on a frequent basis when monitoring administration of IV vasoactive drugs, vasoactive medications should be administered through a central venous line, and early discharge instructions would be inappropriate in this time of crisis.

A patient is being treated in the ICU for neurogenic shock secondary to a spinal cord injury. Despite aggressive interventions, the patients mean arterial pressure (MAP) has fallen to 55 mm Hg. The nurse should gauge the onset of acute kidney injury by referring to what laboratory findings? Select all that apply. A) Blood urea nitrogen (BUN) level B) Urine specific gravity C) Alkaline phosphatase level D) Creatinine level E) Serum albumin level

Ans: A, B, D Feedback: Acute kidney injury (AKI) is characterized by an increase in BUN and serum creatinine levels, fluid and electrolyte shifts, acidbase imbalances, and a loss of the renalhormonal regulation of BP. Urine specific gravity is also affected. Alkaline phosphatase and albumin levels are related to hepatic function.

A nurse educator is reviewing the indications for chest drainage systems with a group of medical nurses. What indications should the nurses identify? Select all that apply. A) Post thoracotomy B) Spontaneous pneumothorax C) Need for postural drainage D) Chest trauma resulting in pneumothorax E) Pleurisy

Ans: A, B, D Feedback: Chest drainage systems are used in treatment of spontaneous pneumothorax and trauma resulting in pneumothorax. Postural drainage and pleurisy are not criteria for use of a chest drainage system.

The occupational health nurse is assessing new employees at a company. What would be important to assess in employees with a potential occupational respiratory exposure to a toxin? Select all that apply. A) Time frame of exposure B) Type of respiratory protection used C) Immunization status D) Breath sounds E) Intensity of exposure

Ans: A, B, D, E Feedback: Key aspects of any assessment of patients with a potential occupational respiratory history include job and job activities, exposure levels, general hygiene, time frame of exposure, effectiveness of respiratory protection used, and direct versus indirect exposures. The patients current respiratory status would also be a priority. Occupational lung hazards are not normally influenced by immunizations.

A nurse is working with a child who is undergoing a diagnostic workup for suspected asthma. What are the signs and symptoms that are consistent with a diagnosis of asthma? Select all that apply. A) Chest tightness B) Crackles C) Bradypnea D) Wheezing E) Cough

Ans: A, D, E Feedback: Asthma is a chronic inflammatory disease of the airways that causes airway hyperresponsiveness, mucosal edema, and mucus production. This inflammation ultimately leads to recurrent episodes of asthma symptoms: cough, chest tightness, wheezing, and dyspnea. Crackles and bradypnea are not typical symptoms of asthma.

The intensive care nurse is responsible for the care of a patient with shock. What cardiac signs or symptoms would suggest to the nurse that the patient may be experiencing acute organ dysfunction? Select all that apply. A) Drop in systolic blood pressure of 40 mm Hg from baselines B) Hypotension that responds to bolus fluid resuscitation C) Exaggerated response to vasoactive medications D) Serum lactate >4 mmol/L E) Mean arterial pressure (MAP) of 65 mm Hg

Ans: A, D, E Feedback: Signs of acute organ dysfunction in the cardiovascular system include systolic blood pressure <90 mm Hg or mean arterial pressure (MAP) <65 mm Hg, drop in systolic blood pressure >40 mm Hg from baselines or serum lactate >4 mmol/L. An exaggerated response to vasoactive medications and an adequate response to fluid resuscitation would not be noted.

A team of nurses are reviewing the similarities and differences between the different classifications of shock. Which subclassifications of circulatory shock should the nurses identify? Select all that apply. A) Anaphylactic B) Hypovolemic C) Cardiogenic D) Septic E) Neurogenic

Ans: A, D, E Feedback: The varied mechanisms leading to the initial vasodilation in circulatory shock provide the basis for the further subclassification of shock into three types: septic shock, neurogenic shock, and anaphylactic shock. Hypovolemic and cardiogenic shock are not subclassifications of circulatory shock.

The physician has ordered a peripheral IV to be inserted before the patient goes for computed tomography. What should the nurse do when selecting a site on the hand or arm for insertion of an IV catheter? A) Choose a hairless site if available. B) Consider potential effects on the patients mobility when selecting a site. C) Have the patient briefly hold his arm over his head before insertion. D) Leave the tourniquet on for at least 3 minutes

Ans: B Feedback: Ideally, both arms and hands are carefully inspected before choosing a specific venipuncture site that does not interfere with mobility. Instruct the patient to hold his arm in a dependent position to increase blood flow. Never leave a tourniquet in place longer than 2 minutes. The site does not necessarily need to be devoid of hair.

You are the surgical nurse caring for a 65-year-old female patient who is postoperative day 1 following a thyroidectomy. During your shift assessment, the patient complains of tingling in her lips and fingers. She tells you that she has an intermittent spasm in her wrist and hand and she exhibits increased muscle tone. What electrolyte imbalance should you first suspect? A) Hypophosphatemia B) Hypocalcemia C) Hypermagnesemia D) Hyperkalemia

Ans: B Feedback: Tetany is the most characteristic manifestation of hypocalcemia and hypomagnesemia. Sensations of tingling may occur in the tips of the fingers, around the mouth, and, less commonly, in the feet. Hypophosphatemia creates central nervous dysfunction, resulting in seizures and coma. Hypermagnesemia creates hypoactive reflexes and somnolence. Signs of hyperkalemia include paresthesias and anxiety.

A patient with a longstanding diagnosis of generalized anxiety disorder presents to the emergency room. The triage nurse notes upon assessment that the patient is hyperventilating. The triage nurse is aware that hyperventilation is the most common cause of which acidbase imbalance? A) Respiratory acidosis B) Respiratory alkalosis C) Increased PaCO2 D) CNS disturbances

Ans: B Feedback: The most common cause of acute respiratory alkalosis is hyperventilation. Extreme anxiety can lead to hyperventilation. Acute respiratory acidosis occurs in emergency situations, such as pulmonary edema, and is exhibited by hypoventilation and decreased PaCO2 . CNS disturbances are found in extreme hyponatremia and fluid overload.

The nurse is caring for a patient with a severe nosebleed. The physician inserts a nasal sponge and tells the patient it may have to remain in place up to 6 days before it is removed. The nurse should identify that this patient is at increased risk for what? A) Viral sinusitis B) Toxic shock syndrome C) Pharyngitis D) Adenoiditis

Ans: B Feedback: A compressed nasal sponge may be used. Once the sponge becomes saturated with blood or is moistened with a small amount of saline, it will expand and produce tamponade to halt the bleeding. The packing may remain in place for 48 hours or up to 5 or 6 days if necessary to control bleeding. Antibiotics may be prescribed because of the risk of iatrogenic sinusitis and toxic shock syndrome.

A nurse is caring for a young adult patient whose medical history includes an alpha1 -antitrypsin deficiency. This deficiency predisposes the patient to what health problem? A) Pulmonary edema B) Lobular emphysema C) Cystic fibrosis (CF) D) Empyema

Ans: B Feedback: A host risk factor for COPD is a deficiency of alpha1 -antitrypsin, an enzyme inhibitor that protects the lung parenchyma from injury. This deficiency predisposes young patients to rapid development of lobular emphysema even in the absence of smoking. This deficiency does not influence the patients risk of pulmonary edema, CF, or empyema.

What would the critical care nurse recognize as a condition that may indicate a patients need to have a tracheostomy? A) A patient has a respiratory rate of 10 breaths per minute. B) A patient requires permanent ventilation. C) A patient exhibits symptoms of dyspnea. D) A patient has respiratory acidosis.

Ans: B Feedback: A tracheostomy permits long-term use of mechanical ventilation to prevent aspiration of oral and gastric secretions in the unconscious or paralyzed patient. Indications for a tracheostomy do not include a respiratory rate of 10 breaths per minute, symptoms of dyspnea, or respiratory acidosis.

A firefighter was trapped in a fire and is admitted to the ICU for smoke inhalation. After 12 hours, the firefighter is exhibiting signs of ARDS and is intubated. What other supportive measures are initiated in a patient with ARDS? A) Psychological counseling B) Nutritional support C) High-protein oral diet D) Occupational therapy

Ans: B Feedback: Aggressive, supportive care must be provided to compensate for the severe respiratory dysfunction. This supportive therapy almost always includes intubation and mechanical ventilation. In addition, circulatory support, adequate fluid volume, and nutritional support are important. Oral intake is contraindicated by intubation. Counseling and occupational therapy would not be priorities during the acute stage of ARDS.

A patient has been scheduled for cardiovascular computed tomography (CT) with contrast. To prepare the patient for this test, what action should the nurse perform? A) Keep the patient NPO for at least 6 hours prior to the test. B) Establish peripheral IV access. C) Limit the patients activity for 2 hours before the test. D) Teach the patient to perform incentive spirometry.

Ans: B Feedback: An IV is necessary if contrast is to be used to enhance the images of the CT. The patient does not need to fast or limit his or her activity. Incentive spirometry is not relevant to this diagnostic test.

The nurse is doing discharge teaching with a patient who has coronary artery disease. The patient asks why he has to take an aspirin every day if he doesnt have any pain. What would be the nurses best response? A) Taking an aspirin every day is an easy way to help restore the normal function of your heart. B) An aspirin a day can help prevent some of the blockages that can cause chest pain or heart attacks. C) Taking an aspirin every day is a simple way to make your blood penetrate your heart more freely. D) An aspirin a day eventually helps your blood carry more oxygen that it would otherwise.

Ans: B Feedback: An aspirin a day is a common nonprescription medication that improves outcomes in patients with CAD due to its antiplatelet action. It does not affect oxygen carrying capacity or perfusion. Aspirin does not restore cardiac function.

The nurse is performing the health interview of a patient with chronic rhinosinusitis who experiences frequent nose bleeds. The nurse asks the patient about her current medication regimen. Which medication would put the patient at a higher risk for recurrent epistaxis? A) Afrin B) Beconase C) Sinustop Pro D) Singulair

Ans: B Feedback: Beconase should be avoided in patients with recurrent epistaxis, glaucoma, and cataracts. Sinustop Pro and Afrin are pseudoephedrine and do not have a side effect of epistaxis. Singulair is a bronchodilator and does not have epistaxis as a side effect.

The acute medical nurse is preparing to wean a patient from the ventilator. Which assessment parameter is most important for the nurse to assess? A) Fluid intake for the last 24 hours B) Baseline arterial blood gas (ABG) levels C) Prior outcomes of weaning D) Electrocardiogram (ECG) results

Ans: B Feedback: Before weaning a patient from mechanical ventilation, it is most important to have baseline ABG levels. During the weaning process, ABG levels will be checked to assess how the patient is tolerating the procedure. Other assessment parameters are relevant, but less critical. Measuring fluid volume intake and output is always important when a patient is being mechanically ventilated. Prior attempts at weaning and ECG results are documented on the patients record, and the nurse can refer to them before the weaning process begins.

The critical care nurse is caring for a patient with a central venous pressure (CVP) monitoring system. The nurse notes that the patients CVP is increasing. Of what may this indicate? A) Psychosocial stress B) Hypervolemia C) Dislodgment of the catheter D) Hypomagnesemia

Ans: B Feedback: CVP is a useful hemodynamic parameter to observe when managing an unstable patients fluid volume status. An increasing pressure may be caused by hypervolemia or by a condition, such as heart failure, that results in decreased myocardial contractility. Stress, dislodgement of the catheter, and low magnesium levels would not typically result in increased CVP.

The nurse is caring for a patient who has a history of heart disease. What factor should the nurse identify as possibly contributing to a decrease in cardiac output? A) A change in position from standing to sitting B) A heart rate of 54 bpm C) A pulse oximetry reading of 94% D) An increase in preload related to ambulation

Ans: B Feedback: Cardiac output is computed by multiplying the stroke volume by the heart rate. Cardiac output can be affected by changes in either stroke volume or heart rate, such as a rate of 54 bpm. An increase in preload will lead to an increase in stroke volume. A pulse oximetry reading of 94% does not indicate hypoxemia, as hypoxia can decrease contractility. Transitioning from standing to sitting would more likely increase rather than decrease cardiac output.

A triage nurse in the ED is on shift when a grandfather carries his 4-year-old grandson into the ED. The child is not breathing, and the grandfather states the boy was stung by a bee in a nearby park while they were waiting for the boys mother to get off work. Which of the following would lead the nurse to suspect that the boy is experiencing anaphylactic shock? A) Rapid onset of acute hypertension B) Rapid onset of respiratory distress C) Rapid onset of neurologic compensation D) Rapid onset of cardiac arrest

Ans: B Feedback: Characteristics of severe anaphylaxis usually include rapid onset of hypotension, neurologic compromise, and respiratory distress. Cardiac arrest can occur if prompt treatment is not provided.

A resident of a long-term care facility has complained to the nurse of chest pain. What aspect of the residents pain would be most suggestive of angina as the cause? A) The pain is worse when the resident inhales deeply. B) The pain occurs immediately following physical exertion. C) The pain is worse when the resident coughs. D) The pain is most severe when the resident moves his upper body.

Ans: B Feedback: Chest pain associated with angina is often precipitated by physical exertion. The other listed aspects of chest pain are more closely associated with noncardiac etiologies.

A patient states that her family has had several colds during this winter and spring despite their commitment to handwashing. The high communicability of the common cold is attributable to what factor? A) Cold viruses are increasingly resistant to common antibiotics. B) The virus is shed for 2 days prior to the emergence of symptoms. C) A genetic predisposition to viral rhinitis has recently been identified. D) Overuse of OTC cold remedies creates a rebound susceptibility to future colds.

Ans: B Feedback: Colds are highly contagious because virus is shed for about 2 days before the symptoms appear and during the first part of the symptomatic phase. Antibiotic resistance is not relevant to viral illnesses and OTC medications do not have a rebound effect. Genetic factors do not exist.

The nurse is caring for a patient suspected of having ARDS. What is the most likely diagnostic test ordered in the early stages of this disease to differentiate the patients symptoms from those of a cardiac etiology? A) Carboxyhemoglobin level B) Brain natriuretic peptide (BNP) level C) C-reactive protein (CRP) level D) Complete blood count

Ans: B Feedback: Common diagnostic tests performed for patients with potential ARDS include plasma brain natriuretic peptide (BNP) levels, echocardiography, and pulmonary artery catheterization. The BNP level is helpful in distinguishing ARDS from cardiogenic pulmonary edema. The carboxyhemoglobin level will be increased in a client with an inhalation injury, which commonly progresses into ARDS. CRP and CBC levels do not help differentiate from a cardiac problem.

The nurse is caring for a client with an endotracheal tube who is on a ventilator. When assessing the client, the nurse knows to maintain what cuff pressure to maintain appropriate pressure on the tracheal wall? A) Between 10 and 15 mm Hg B) Between 15 and 20 mm Hg C) Between 20 and 25 mm Hg D) Between 25 and 30 mm Hg

Ans: B Feedback: Complications can occur from pressure exerted by the cuff on the tracheal wall. Cuff pressures should be maintained between 15 and 20 mm

The nurse is conducting a presurgical interview for a patient with laryngeal cancer. The patient states that he drinks approximately six to eight shots of vodka per day. It is imperative that the nurse inform the surgical team so the patient can be assessed for what? A) Increased risk for infection B) Delirium tremens C) Depression D) Nonadherence to postoperative care

Ans: B Feedback: Considering the known risk factors for cancer of the larynx, it is essential to assess the patients history of alcohol intake. Infection is a risk in the postoperative period, but not an appropriate answer based on the patients history. Depression and nonadherence are risks in the postoperative phase, but would not be critical short-term assessments.

. A nurse is developing the teaching portion of a care plan for a patient with COPD. What would be the most important component for the nurse to emphasize? A) Smoking up to one-half of a pack of cigarettes weekly is allowable. B) Chronic inhalation of indoor toxins can cause lung damage. C) Minor respiratory infections are considered to be self-limited and are not treated. D) Activities of daily living (ADLs) should be clustered in the early morning hours.

Ans: B Feedback: Environmental risk factors for COPD include prolonged and intense exposure to occupational dusts and chemicals, indoor air pollution, and outdoor air pollution. Smoking cessation should be taught to all patients who are currently smoking. Minor respiratory infections that are of no consequence to the person with normal lungs can produce fatal disturbances in the lungs of the person with emphysema. ADLs should be paced throughout the day to permit patients to perform these without excessive distress.

The nurse is caring for an acutely ill patient who has central venous pressure monitoring in place. What intervention should be included in the care plan of a patient with CVP in place? A) Apply antibiotic ointment to the insertion site twice daily. B) Change the site dressing whenever it becomes visibly soiled. C) Perform passive range-of-motion exercises to prevent venous stasis. D) Aspirate blood from the device once daily to test pH.

Ans: B Feedback: Gauze dressings should be changed every 2 days or transparent dressings at least every 7 days and whenever dressings become damp, loosened, or visibly soiled. Passive ROM exercise is not indicated and it is unnecessary and inappropriate to aspirate blood to test it for pH. Antibiotic ointments are contraindicated.

The perioperative nurse has admitted a patient who has just underwent a tonsillectomy. The nurses postoperative assessment should prioritize which of the following potential complications of this surgery? A) Difficulty ambulating B) Hemorrhage C) Infrequent swallowing D) Bradycardia

Ans: B Feedback: Hemorrhage is a potential complication of a tonsillectomy. Increased pulse, fever, and restlessness may indicate a postoperative hemorrhage. Difficulty ambulating and bradycardia are not common complications in a patient after a tonsillectomy. Infrequent swallowing does not indicate hemorrhage; frequent swallowing does.

The physician has ordered a high-sensitivity C-reactive protein (hs-CRP) drawn on a patient. The results of this test will allow the nurse to evaluate the role of what process that is implicated in the development of atherosclerosis? A) Immunosuppression B) Inflammation C) Infection D) Hemostasis

Ans: B Feedback: High-sensitivity CRP is a protein produced by the liver in response to systemic inflammation. Inflammation is thought to play a role in the development and progression of atherosclerosis.

A patient with thoracic trauma is admitted to the ICU. The nurse notes the patients chest and neck are swollen and there is a crackling sensation when palpated. The nurse consequently identifies the presence of subcutaneous emphysema. If this condition becomes severe and threatens airway patency, what intervention is indicated? A) A chest tube B) A tracheostomy C) An endotracheal tube D) A feeding tube

Ans: B Feedback: In severe cases in which there is widespread subcutaneous emphysema, a tracheostomy is indicated if airway patency is threatened by pressure of the trapped air on the trachea. The other listed tubes would neither resolve the subcutaneous emphysema nor the consequent airway constriction.

A patient is being admitted to the preoperative holding area for a thoracotomy. Preoperative teaching includes what? A) Correct use of a ventilator B) Correct use of incentive spirometry C) Correct use of a mini-nebulizer D) Correct technique for rhythmic breathing

Ans: B Feedback: Instruction in the use of incentive spirometry begins before surgery to familiarize the patient with its correct use. You do not teach a patient the use of a ventilator; you explain that he may be on a ventilator to help him breathe. Rhythmic breathing and mini-nebulizers are unnecessary.

While caring for a patient with an endotracheal tube, the nurses recognizes that suctioning is required how often? A) Every 2 hours when the patient is awake B) When adventitious breath sounds are auscultated C) When there is a need to prevent the patient from coughing D) When the nurse needs to stimulate the cough reflex

Ans: B Feedback: It is usually necessary to suction the patients secretions because of the decreased effectiveness of the cough mechanism. Tracheal suctioning is performed when adventitious breath sounds are detected or whenever secretions are present. Unnecessary suctioning, such as scheduling every 2 hours, can initiate bronchospasm and cause trauma to the tracheal mucosa.

A nurse in the ICU receives report from the nurse in the ED about a new patient being admitted with a neck injury he received while diving into a lake. The ED nurse reports that his blood pressure is 85/54, heart rate is 53 beats per minute, and his skin is warm and dry. What does the ICU nurse recognize that that patient is probably experiencing? A) Anaphylactic shock B) Neurogenic shock C) Septic shock D) Hypovolemic shock

Ans: B Feedback: Neurogenic shock can be caused by spinal cord injury. The patient will present with a low blood pressure; bradycardia; and warm, dry skin due to the loss of sympathetic muscle tone and increased parasympathetic stimulation. Anaphylactic shock is caused by an identifiable offending agent, such as a bee sting. Septic shock is caused by bacteremia in the blood and presents with a tachycardia. Hypovolemic shock presents with tachycardia and a probable source of blood loss.

The nurse is caring for a patient with chronic obstructive pulmonary disease (COPD). The patient has been receiving high-flow oxygen therapy for an extended time. What symptoms should the nurse anticipate if the patient were experiencing oxygen toxicity? A) Bradycardia and frontal headache B) Dyspnea and substernal pain C) Peripheral cyanosis and restlessness D) Hypotension and tachycardia

Ans: B Feedback: Oxygen toxicity can occur when patients receive too high a concentration of oxygen for an extended period. Symptoms of oxygen toxicity include dyspnea, substernal pain, restlessness, fatigue, and progressive respiratory difficulty. Bradycardia, frontal headache, cyanosis, hypotension, and tachycardia are not symptoms of oxygen toxicity.

The physician has ordered continuous positive airway pressure (CPAP) with the delivery of a patients high-flow oxygen therapy. The patient asks the nurse what the benefit of CPAP is. What would be the nurses best response? A) CPAP allows a higher percentage of oxygen to be safely used. B) CPAP allows a lower percentage of oxygen to be used with a similar effect. C) CPAP allows for greater humidification of the oxygen that is administered. D) CPAP allows for the elimination of bacterial growth in oxygen delivery systems

Ans: B Feedback: Prevention of oxygen toxicity is achieved by using oxygen only as prescribed. Often, positive end- expiratory pressure (PEEP) or CPAP is used with oxygen therapy to reverse or prevent microatelectasis, thus allowing a lower percentage of oxygen to be used. Oxygen is moistened by passing through a humidification system. Changing the tubing on the oxygen therapy equipment is the best technique for controlling bacterial growth.

The nurse is caring for a patient who is scheduled to have a thoracotomy. When planning preoperative teaching, what information should the nurse communicate to the patient? A) How to milk the chest tubing B) How to splint the incision when coughing C) How to take prophylactic antibiotics correctly D) How to manage the need for fluid restriction

Ans: B Feedback: Prior to thoracotomy, the nurse educates the patient about how to splint the incision with the hands, a pillow, or a folded towel. The patient is not taught how to milk the chest tubing because this is performed by the nurse. Prophylactic antibiotics are not normally used and fluid restriction is not indicated following thoracotomy

A nurse in the ICU is planning the care of a patient who is being treated for shock. Which of the following statements best describes the pathophysiology of this patients health problem? A) Blood is shunted from vital organs to peripheral areas of the body. B) Cells lack an adequate blood supply and are deprived of oxygen and nutrients. C) Circulating blood volume is decreased with a resulting change in the osmotic pressure gradient. D) Hemorrhage occurs as a result of trauma, depriving vital organs of adequate perfusion.

Ans: B Feedback: Shock is a life-threatening condition with a variety of underlying causes. Shock is caused when the cells have a lack of adequate blood supply and are deprived of oxygen and nutrients. In cases of shock, blood is shunted from peripheral areas of the body to the vital organs. Hemorrhage and decreased blood volume are associated with some, but not all, types of shock

A nurse is reviewing the pathophysiology of cystic fibrosis (CF) in anticipation of a new admission. The nurse should identify what characteristic aspects of CF? A) Alveolar mucus plugging, infection, and eventual bronchiectasis B) Bronchial mucus plugging, inflammation, and eventual bronchiectasis C) Atelectasis, infection, and eventual COPD D) Bronchial mucus plugging, infection, and eventual COPD

Ans: B Feedback: The hallmark pathology of CF is bronchial mucus plugging, inflammation, and eventual bronchiectasis. Commonly, the bronchiectasis begins in the upper lobes and progresses to involve all lobes. Infection, atelectasis, and COPD are not hallmark pathologies of CF.

An adult patient has survived an episode of shock and will be discharged home to finish the recovery phase of his disease process. The home health nurse plays an integral part in monitoring this patient. What aspect of his care should be prioritized by the home health nurse? A) Providing supervision to home health aides in providing necessary patient care B) Assisting the patient and family to identify and mobilize community resources C) Providing ongoing medical care during the familys rehabilitation phase D) Reinforcing the importance of continuous assessment with the family

Ans: B Feedback: The home care nurse reinforces the importance of continuing medical care and helps the patient and family identify and mobilize community resources. The home health nurse is part of a team that provides patient care in the home. The nurse does not directly supervise home health aides. The nurse provides nursing care to both the patient and family, not just the family. The nurse performs continuous and ongoing assessment of the patient; he or she does not just reinforce the importance of that assessment.

During a shift assessment, the nurse is identifying the clients point of maximum impulse (PMI). Where will the nurse best palpate the PMI? A) Left midclavicular line of the chest at the level of the nipple B) Left midclavicular line of the chest at the fifth intercostal space C) Midline between the xiphoid process and the left nipple D) Two to three centimeters to the left of the sternum

Ans: B Feedback: The left ventricle is responsible for the apical beat or the point of maximum impulse, which is normally palpated in the left midclavicular line of the chest wall at the fifth intercostal space.

A brain (B-type) natriuretic peptide (BNP) sample has been drawn from an older adult patient who has been experienced vital fatigue and shortness of breath. This test will allow the care team to investigate the possibility of what diagnosis? A) Pleurisy B) Heart failure C) Valve dysfunction D) Cardiomyopathy

Ans: B Feedback: The level of BNP in the blood increases as the ventricular walls expand from increased pressure, making it a helpful diagnostic, monitoring, and prognostic tool in the setting of HF. It is not specific to cardiomyopathy, pleurisy, or valve dysfunction.

The nurse is doing discharge teaching in the ED with a patient who had a nosebleed. What should the nurse include in the discharge teaching of this patient? A) Avoid blowing the nose for the next 45 minutes. B) In case of recurrence, apply direct pressure for 15 minutes. C) Do not take aspirin for the next 2 weeks. D) Seek immediate medical attention if the nosebleed recurs

Ans: B Feedback: The nurse explains how to apply direct pressure to the nose with the thumb and the index finger for 15 minutes in case of a recurrent nosebleed. If recurrent bleeding cannot be stopped, the patient is instructed to seek additional medical attention. ASA is not contraindicated in most cases and the patient should avoiding blowing the nose for an extended period of time, not just 45 minutes.

A patient who involved in a workplace accident suffered a penetrating wound of the chest that led to acute respiratory failure. What goal of treatment should the care team prioritize when planning this patients care? A) Facilitation of long-term intubation B) Restoration of adequate gas exchange C) Attainment of effective coping D) Self-management of oxygen therapy

Ans: B Feedback: The objectives of treatment are to correct the underlying cause of respiratory failure and to restore adequate gas exchange in the lung. This is priority over coping and self-care. Long-term ventilation may or may not be indicated.

The nurse is providing care for a patient who has just been admitted to the postsurgical unit following a laryngectomy. What assessment should the nurse prioritize? A) The patients swallowing ability B) The patients airway patency C) The patients carotid pulses D) Signs and symptoms of infection

Ans: B Feedback: The patient with a laryngectomy is a risk for airway occlusion and respiratory distress. As in all nursing situations, assessment of the airway is a priority over other potential complications and assessment parameters.

The nurse has explained to the patient that after his thoracotomy, it will be important to adhere to a coughing schedule. The patient is concerned about being in too much pain to be able to cough. What would be an appropriate nursing intervention for this client? A) Teach him postural drainage. B) Teach him how to perform huffing. C) Teach him to use a mini-nebulizer. D) Teach him how to use a metered dose inhaler.

Ans: B Feedback: The technique of huffing may be helpful for the patient with diminished expiratory flow rates or for the patient who refuses to cough because of severe pain. Huffing is the expulsion of air through an open glottis. Inhalers, nebulizers, and postural drainage are not substitutes for performing coughing exercises.

An admitting nurse is assessing a patient with COPD. The nurse auscultates diminished breath sounds, which signify changes in the airway. These changes indicate to the nurse to monitor the patient for what? A) Kyphosis and clubbing of the fingers B) Dyspnea and hypoxemia C) Sepsis and pneumothorax D) Bradypnea and pursed lip breathing

Ans: B Feedback: These changes in the airway require that the nurse monitor the patient for dyspnea and hypoxemia. Kyphosis is a musculoskeletal problem. Sepsis and pneumothorax are atypical complications. Tachypnea is much more likely than bradypnea. Pursed lip breathing can relieve dyspnea

The nurse is caring for a patient who has just been diagnosed with chronic rhinosinusitis. While being admitted to the clinic, the patient asks, Will this chronic infection hurt my new kidney? What should the nurse know about chronic rhinosinusitis in patients who have had a transplant? A) The patient will have exaggerated symptoms of rhinosinusitis due to immunosuppression. B) Taking immunosuppressive drugs can contribute to chronic rhinosinusitis. C) Chronic rhinosinusitis can damage the transplanted organ. D) Immunosuppressive drugs can cause organ rejection

Ans: B Feedback: URIs, specifically chronic rhinosinusitis and recurrent acute rhinosinusitis, may be linked to primary or secondary immune deficiency or treatment with immunosuppressive therapy (i.e., for cancer or organ transplantation). Typical symptoms may be blunted or absent due to immunosuppression. No evidence indicates damage to the transplanted organ due to chronic rhinosinusitis. Immunosuppressive drugs do not cause organ rejection.

The physical therapist notifies the nurse that a patient with coronary artery disease (CAD) experiences a much greater-than-average increase in heart rate during physical therapy. The nurse recognizes that an increase in heart rate in a patient with CAD may result in what? A) Development of an atrial-septal defect B) Myocardial ischemia C) Formation of a pulmonary embolism D) Release of potassium ions from cardiac cells

Ans: B Feedback: Unlike other arteries, the coronary arteries are perfused during diastole. An increase in heart rate shortens diastole and can decrease myocardial perfusion. Patients, particularly those with CAD, can develop myocardial ischemia. An increase in heart rate will not usually result in a pulmonary embolism or create electrolyte imbalances. Atrial-septal defects are congenital

A nurses assessment reveals that a client with COPD may be experiencing bronchospasm. What assessment finding would suggest that the patient is experiencing bronchospasm? A) Fine or coarse crackles on auscultation B) Wheezes or diminished breath sounds on auscultation C) Reduced respiratory rate or lethargy D) Slow, deliberate respirations

Ans: B Feedback: Wheezing and diminished breath sounds are consistent with bronchospasm. Crackles are usually attributable to other respiratory or cardiac pathologies. Bronchospasm usually results in rapid, inefficient breathing and agitation.

A nurse is assessing a patient who is suspected of having bronchiectasis. The nurse should consider which of the following potential causes? Select all that apply. A) Pulmonary hypertension B) Airway obstruction C) Pulmonary infections D) Genetic disorders E) Atelectasis

Ans: B, C, D Feedback: Bronchiectasis is a chronic, irreversible dilation of the bronchi and bronchioles. Under the new definition of COPD, it is considered a disease process separate from COPD. Bronchiectasis may be caused by a variety of conditions, including airway obstruction, diffuse airway injury, pulmonary infections and obstruction of the bronchus or complications of long-term pulmonary infections, or genetic disorders such as cystic fibrosis. Bronchiectasis is not caused by pulmonary hypertension or atelectasis.

A patient is brought to the emergency department by the paramedics. The patient is a type 2 diabetic and is experiencing HHS. The nurse should identify what components of HHS? Select all that apply. A) Leukocytosis B) Glycosuria C) Dehydration D) Hypernatremia E) Hyperglycemia

Ans: B, C, D, E In HHS, persistent hyperglycemia causes osmotic diuresis, which results in losses of water and electrolytes. To maintain osmotic equilibrium, water shifts from the intracellular fluid space to the extracellular fluid space. With glycosuria and dehydration, hypernatremia and increased osmolarity occur. Leukocytosis does not take place.

An interdisciplinary team is planning the care of a patient with bronchiectasis. What aspects of care should the nurse anticipate? Select all that apply. A) Occupational therapy B) Antimicrobial therapy C) Positive pressure isolation D) Chest physiotherapy E) Smoking cessation

Ans: B, D, E Feedback: Chest physiotherapy, antibiotics, and smoking cessation are cornerstones of the care of patients with bronchiectasis. Occupational therapy and isolation are not normally indicated.

The nurse is preparing to insert a peripheral IV catheter into a patient who will require fluids and IV antibiotics. How should the nurse always start the process of insertion? A) Leave one hand ungloved to assess the site. B) Cleanse the skin with normal saline. C) Ask the patient about allergies to latex or iodine. D) Remove excessive hair from the selected site.

Ans: C

A school nurse is caring for a 10-year-old girl who is having an asthma attack. What is the preferred intervention to alleviate this clients airflow obstruction? A) Administer corticosteroids by metered dose inhaler B) Administer inhaled anticholinergics C) Administer an inhaled beta-adrenergic agonist D) Utilize a peak flow monitoring device

Ans: C Feedback: Asthma exacerbations are best managed by early treatment and education of the patient. Quick-acting beta-adrenergic medications are the first used for prompt relief of airflow obstruction. Systemic corticosteroids may be necessary to decrease airway inflammation in patients who fail to respond to inhaled beta-adrenergic medication. A peak flow device will not resolve short-term shortness of breath.

A gerontologic nurse is teaching a group of medical nurses about the high incidence and mortality of pneumonia in older adults. What is a contributing factor to this that the nurse should describe? A) Older adults have less compliant lung tissue than younger adults. B) Older adults are not normally candidates for pneumococcal vaccination. C) Older adults often lack the classic signs and symptoms of pneumonia. D) Older adults often cannot tolerate the most common antibiotics used to treat pneumonia.

Ans: C Feedback: The diagnosis of pneumonia may be missed because the classic symptoms of cough, chest pain, sputum production, and fever may be absent or masked in older adult patients. Mortality from pneumonia in the elderly is not a result of limited antibiotic options or lower lung compliance. The pneumococcal vaccine is appropriate for older adults.

The nurse is preparing to suction a patient with an endotracheal tube. What should be the nurses first step in the suctioning process? A) Explain the suctioning procedure to the patient and reposition the patient. B) Turn on suction source at a pressure not exceeding 120 mm Hg. C) Assess the patients lung sounds and SAO2 via pulse oximeter. D) Perform hand hygiene and don nonsterile gloves, goggles, gown, and mask

Ans: C Feedback: Assessment data indicate the need for suctioning and allow the nurse to monitor the effect of suction on the patients level of oxygenation. Explaining the procedure would be the second step; performing hand hygiene is the third step, and turning on the suction source is the fourth step.

A patients severe asthma has necessitated the use of a long-acting beta2 -agonist (LABA). Which of the patients statements suggests a need for further education? A) I know that these drugs can sometimes make my heart beat faster. B) Ive heard that this drug is particularly good at preventing asthma attacks during exercise. C) Ill make sure to use this each time I feel an asthma attack coming on. D) Ive heard that this drug sometimes gets less effective over time.

Ans: C LABAs are not used for management of acute asthma symptoms. Tachycardia is a potential adverse effect and decreased protection against exercise-induced bronchospasm may occur with regular use.

A patient is being treated for bacterial pharyngitis. Which of the following should the nurse recommend when promoting the patients nutrition during treatment? A) A 1.5 L/day fluid restriction B) A high-potassium, low-sodium diet C) A liquid or soft diet D) A high-protein diet

Ans: C Feedback: A liquid or soft diet is provided during the acute stage of the disease, depending on the patients appetite and the degree of discomfort that occurs with swallowing. The patient is encouraged to drink as much fluid as possible (at least 2 to 3 L/day). There is no need for increased potassium or protein intake.

A student nurse is developing a teaching plan for an adult patient with asthma. Which teaching point should have the highest priority in the plan of care that the student is developing? A) Gradually increase levels of physical exertion. B) Change filters on heaters and air conditioners frequently. C) Take prescribed medications as scheduled. D) Avoid goose-down pillows

Ans: C Feedback: Although all of the measures are appropriate for a client with asthma, taking prescribed medications on time is the most important measure in preventing asthma attacks.

A lipid profile has been ordered for a patient who has been experiencing cardiac symptoms. When should a lipid profile be drawn in order to maximize the accuracy of results? A) As close to the end of the day as possible B) After a meal high in fat C) After a 12-hour fast D) Thirty minutes after a normal meal

Ans: C Feedback: Although cholesterol levels remain relatively constant over 24 hours, the blood specimen for the lipid profile should be obtained after a 12-hour fast

The nurse is providing discharge teaching for a patient who developed a pulmonary embolism after total knee surgery. The patient has been converted from heparin to sodium warfarin (Coumadin) anticoagulant therapy. What should the nurse teach the client? A) Coumadin will continue to break up the clot over a period of weeks B) Coumadin must be taken concurrent with ASA to achieve anticoagulation. C) Anticoagulant therapy usually lasts between 3 and 6 months. D) He should take a vitamin supplement containing vitamin K

Ans: C Feedback: Anticoagulant therapy prevents further clot formation, but cannot be used to dissolve a clot. The therapy continues for approximately 3 to 6 months and is not combined with ASA. Vitamin K reverses the effect of anticoagulant therapy and normally should not be taken.

A nurse is teaching a patient with asthma about Azmacort, an inhaled corticosteroid. Which adverse effects should the nurse be sure to address in patient teaching? A) Dyspnea and increased respiratory secretions B) Nausea and vomiting C) Cough and oral thrush D) Fatigue and decreased level of consciousness

Ans: C Feedback: Azmacort has possible adverse effects of cough, dysphonia, oral thrush (candidiasis), and headache. In high doses, systemic effects may occur (e.g., adrenal suppression, osteoporosis, skin thinning, and easy bruising). The other listed adverse effects are not associated with this drug.

A student nurse is preparing to care for a patient with bronchiectasis. The student nurse should recognize that this patient is likely to experience respiratory difficulties related to what pathophysiologic process? A) Intermittent episodes of acute bronchospasm B) Alveolar distention and impaired diffusion C) Dilation of bronchi and bronchioles D) Excessive gas exchange in the bronchioles

Ans: C Feedback: Bronchiectasis is a chronic, irreversible dilation of the bronchi and bronchioles that results from destruction of muscles and elastic connective tissue. It is not characterized by acute bronchospasm, alveolar distention, or excessive gas exchange.

The critical care nurse is precepting a new nurse on the unit. Together they are caring for a patient who has a tracheostomy tube and is receiving mechanical ventilation. What action should the critical care nurse recommend when caring for the cuff? A) Deflate the cuff overnight to prevent tracheal tissue trauma. B) Inflate the cuff to the highest possible pressure in order to prevent aspiration. C) Monitor the pressure in the cuff at least every 8 hours D) Keep the tracheostomy tube plugged at all times

Ans: C Feedback: Cuff pressure must be monitored by the respiratory therapist or nurse at least every 8 hours by attaching a handheld pressure gauge to the pilot balloon of the tube or by using the minimal leak volume or minimal occlusion volume technique. Plugging is only used when weaning the patient from tracheal support. Deflating the cuff overnight would be unsafe and inappropriate. High cuff pressure can cause tissue trauma.

The nurse is caring for a patient in the ED for epistaxis. What information should the nurse include in patient discharge teaching as a way to prevent epistaxis? A) Keep nasal passages clear. B) Use decongestants regularly. C) Humidify the indoor environment. D) Use a tissue when blowing the nose.

Ans: C Feedback: Discharge teaching for prevention of epistaxis should include the following: avoid forceful nose bleeding, straining, high altitudes, and nasal trauma (nose picking). Adequate humidification may prevent drying of the nasal passages. Keeping nasal passages clear and using a tissue when blowing the nose are not included in discharge teaching for the prevention of epistaxis. Decongestants are not indicated.

The acute care nurse is providing care for an adult patient who is in hypovolemic shock. The nurse recognizes that antidiuretic hormone (ADH) plays a significant role in this health problem. What assessment finding will the nurse likely observe related to the role of the ADH during hypovolemic shock? A) Increased hunger B) Decreased thirst C) Decreased urinary output D) Increased capillary perfusion

Ans: C Feedback: During hypovolemic shock, a state of hypernatremia occurs. Hypernatremia stimulates the release of ADH by the pituitary gland. ADH causes the kidneys to retain water further in an effort to raise blood volume and blood pressure. In a hypovolemic state the body shifts blood away from anything that is not a vital organ, so hunger is not an issue; thirst is increased as the body tries to increase fluid volume; and capillary profusion decreases as the body shunts blood away from the periphery and to the vital organs.

The nurse is performing an intake assessment on a patient with a new diagnosis of coronary artery disease. What would be the most important determination to make during this intake assessment? A) Whether the patient and involved family members understand the role of genetics in the etiology of the disease B) Whether the patient and involved family members understand dietary changes and the role of nutrition C) Whether the patient and involved family members are able to recognize symptoms of an acute cardiac problem and respond appropriately D) Whether the patient and involved family members understand the importance of social support and community agencies

Ans: C Feedback: During the health history, the nurse needs to determine if the patient and involved family members are able to recognize symptoms of an acute cardiac problem, such as acute coronary syndrome (ACS) or HF, and seek timely treatment for these symptoms. Each of the other listed topics is valid, but the timely and appropriate response to a cardiac emergency is paramount.

The nurse is planning the care of a patient who is scheduled for a laryngectomy. The nurse should assign the highest priority to which postoperative nursing diagnosis? A) Anxiety related to diagnosis of cancer B) Altered nutrition related to swallowing difficulties C) Ineffective airway clearance related to airway alterations D) Impaired verbal communication related to removal of the larynx

Ans: C Feedback: Each of the listed diagnoses is valid, but ineffective airway clearance is the priority nursing diagnosis for all conditions.

A patient presents to the ED stating she was in a boating accident about 3 hours ago. Now the patient has complaints of headache, fatigue, and the feeling that he just cant breathe enough. The nurse notes that the patient is restless and tachycardic with an elevated blood pressure. This patient may be in the early stages of what respiratory problem? A) Pneumoconiosis B) Pleural effusion C) Acute respiratory failure D) Pneumonia

Ans: C Feedback: Early signs of acute respiratory failure are those associated with impaired oxygenation and may include restlessness, fatigue, headache, dyspnea, air hunger, tachycardia, and increased blood pressure. As the hypoxemia progresses, more obvious signs may be present, including confusion, lethargy, tachycardia, tachypnea, central cyanosis, diaphoresis, and, finally, respiratory arrest. Pneumonia is infectious and would not result from trauma. Pneumoconiosis results from exposure to occupational toxins. A pleural effusion does not cause this constellation of symptoms

A patient in the ICU has had an endotracheal tube in place for 3 weeks. The physician has ordered that a tracheostomy tube be placed. The patients family wants to know why the endotracheal tube cannot be left in place. What would be the nurses best response? A) The physician may feel that mechanical ventilation will have to be used long-term. B) Long-term use of an endotracheal tube diminishes the normal breathing reflex. C) When an endotracheal tube is left in too long it can damage the lining of the windpipe. D) It is much harder to breathe through an endotracheal tube than a tracheostomy.

Ans: C Feedback: Endotracheal intubation may be used for no longer than 2 to 3 weeks, by which time a tracheostomy must be considered to decrease irritation of and, trauma to, the tracheal lining, to reduce the incidence of vocal cord paralysis (secondary to laryngeal nerve damage), and to decrease the work of breathing. The need for long-term ventilation would not be the primary rationale for this change in treatment. Endotracheal tubes do not diminish the breathing reflex.

The nurse is writing a plan of care for a patient with a cardiac dysrhythmia. What would be the most appropriate goal for the patient? A) Maintain a resting heart rate below 70 bpm. B) Maintain adequate control of chest pain. C) Maintain adequate cardiac output D) Maintain normal cardiac structure.

Ans: C Feedback: For patient safety, the most appropriate goal is to maintain cardiac output to prevent worsening complications as a result of decreased cardiac output. A resting rate of less than 70 bpm is not appropriate for every patient. Chest pain is more closely associated with acute coronary syndrome than with dysrhythmias. Nursing actions cannot normally influence the physical structure of the heart.

The occupational health nurse is obtaining a patient history during a pre-employment physical. During the history, the patient states that he has hereditary angioedema. The nurse should identify what implication of this health condition? A) It will result in increased loss of work days. B) It may cause episodes of weakness due to reduced cardiac output. C) It can cause life-threatening airway obstruction. D) It is unlikely to interfere with the individuals health.

Ans: C Feedback: Hereditary angioedema is an inherited condition that is characterized by episodes of life-threatening laryngeal edema. No information supports lost days of work or reduced cardiac function

The nurse is performing nasotracheal suctioning on a medical patient and obtains copious amounts of secretions from the patients airway, even after inserting and withdrawing the catheter several times. How should the nurse proceed? A) Continue suctioning the patient until no more secretions are obtained. B) Perform chest physiotherapy rather than nasotracheal suctioning. C) Wait several minutes and then repeat suctioning. D) Perform postural drainage and then repeat suctioning.

Ans: C Feedback: If additional suctioning is needed, the nurse should withdraw the catheter to the back of the pharynx, reassure the patient, and oxygenate for several minutes before resuming suctioning. Chest physiotherapy and postural drainage are not necessarily indicated.

The nurse is assessing a patient whose respiratory disease in characterized by chronic hyperinflation of the lungs. What would the nurse most likely assess in this patient? A) Signs of oxygen toxicity B) Chronic chest pain C) A barrel chest D) Long, thin fingers

Ans: C Feedback: In COPD patients with a primary emphysematous component, chronic hyperinflation leads to the barrel chest thorax configuration. The nurse most likely would not assess chest pain or long, thin fingers; these are not characteristic of emphysema. The patient would not show signs of oxygen toxicity unless he or she received excess supplementary oxygen.

The ICU nurse is caring for a patient in neurogenic shock following an overdose of antianxiety medication. When assessing this patient, the nurse should recognize what characteristic of neurogenic shock? A) Hypertension B) Cool, moist skin C) Bradycardia D) Signs of sympathetic stimulation

Ans: C Feedback: In neurogenic shock, the sympathetic system is not able to respond to body stressors. Therefore, the clinical characteristics of neurogenic shock are signs of parasympathetic stimulation. It is characterized by dry, warm skin rather than the cool, moist skin seen in hypovolemic shock. Another characteristic is hypotension with bradycardia, rather than the tachycardia that characterizes other forms of shock.

The nurse is performing preoperative teaching with a patient who has cancer of the larynx. After completing patient teaching, what would be most important for the nurse to do? A) Give the patient his or her cell phone number. B) Refer the patient to a social worker or psychologist. C) Provide the patient with audiovisual materials about the surgery. D) Reassure the patient and family that everything will be alright.

Ans: C Feedback: Informational materials (written and audiovisual) about the surgery are given to the patient and family for review and reinforcement. The nurse never gives personal contact information to the patient. Nothing in the scenario indicates that a referral to a social worker or psychologist is necessary. False reassurance must always be avoided.

A client presents to the walk-in clinic complaining of a dry, irritating cough and production of a minute amount of mucus-like sputum. The patient complains of soreness in her chest in the sternal area. The nurse should suspect that the primary care provider will assess the patient for what health problem? A) Pleural effusion B) Pulmonary embolism C) Tracheobronchitis D) Tuberculosis

Ans: C Feedback: Initially, the patient with tracheobronchitis has a dry, irritating cough and expectorates a scant amount of mucoid sputum. The patient may report sternal soreness from coughing and have fever or chills, night sweats, headache, and general malaise. Pleural effusion and pulmonary embolism do not normally cause sputum production and would likely cause acute shortness of breath. Hemoptysis is characteristic of TB

A nurse is providing discharge teaching for a client with COPD. When teaching the client about breathing exercises, what should the nurse include in the teaching? A) Lie supine to facilitate air entry B) Avoid pursed lip breathing C) Use diaphragmatic breathing D) Use chest breathing

Ans: C Feedback: Inspiratory muscle training and breathing retraining may help improve breathing patterns in patients with COPD. Training in diaphragmatic breathing reduces the respiratory rate, increases alveolar ventilation, and, sometimes, helps expel as much air as possible during expiration. Pursed-lip breathing helps slow expiration, prevents collapse of small airways, and controls the rate and depth of respiration. Diaphragmatic breathing, not chest breathing, increases lung expansion. Supine positioning does not aid breathing.

The nurse is creating a plan of car for a patient diagnosed with acute laryngitis. What intervention should be included in the patients plan of care? A) Place warm cloths on the patients throat, as needed. B) Have the patient inhale warm steam three times daily. C) Encourage the patient to limit speech whenever possible. D) Limit the patients fluid intake to 1.5 L/day.

Ans: C Feedback: Management of acute laryngitis includes resting the voice, avoiding irritants (including smoking), resting, and inhaling cool steam or an aerosol. Fluid intake should be increased. Warm cloths on the throat will not help relieve the symptoms of acute laryngitis.

The nurse is reviewing the electronic health record of a patient with an empyema. What health problem in the patients history is most likely to have caused the empyema? A) Smoking B) Asbestosis C) Pneumonia D) Lung cancer

Ans: C Feedback: Most empyemas occur as complications of bacterial pneumonia or lung abscess. Cancer, smoking, and asbestosis are not noted to be common causes.

A nursing student is discussing a patient with viral pharyngitis with the preceptor at the walk-in clinic. What should the preceptor tell the student about nursing care for patients with viral pharyngitis? A) Teaching focuses on safe and effective use of antibiotics. B) The patient should be preliminarily screened for surgery. C) Symptom management is the main focus of medical and nursing care. D) The focus of care is resting the voice to prevent chronic hoarseness

Ans: C Feedback: Nursing care for patients with viral pharyngitis focuses on symptomatic management. Antibiotics are not prescribed for viral etiologies. Surgery is not indicated in the treatment of viral pharyngitis. Chronic hoarseness is not a common sequela of viral pharyngitis, so teaching ways to prevent it would be of no use in this instance

The intensive care nurse caring for a patient in shock is planning assessments and interventions related to the patients nutritional needs. What physiologic process contributes to these increased nutritional needs? A) The use of albumin as an energy source by the body because of the need for increased adenosine triphosphate (ATP) B) The loss of fluids due to decreased skin integrity and decreased stomach acids due to increased parasympathetic activity C) The release of catecholamines that creates an increase in metabolic rate and caloric requirements D) The increase in GI peristalsis during shock and the resulting diarrhea

Ans: C Feedback: Nutritional support is an important aspect of care for patients in shock. Patients in shock may require 3,000 calories daily. This caloric need is directly related to the release of catecholamines and the resulting increase in metabolic rate and caloric requirements. Albumin is not primarily metabolized as an energy source. The special nutritional needs of shock are not related to increased parasympathetic activity, but are instead related to increased sympathetic activity. GI function does not increase during shock.

A critical care nurse is aware of similarities and differences between the treatments for different types of shock. Which of the following interventions is used in all types of shock? A) Aggressive hypoglycemic control B) Administration of hypertonic IV fluids C) Early provision of nutritional support D) Aggressive antibiotic therapy

Ans: C Feedback: Nutritional support is necessary for all patients who are experiencing shock. Hyperglycemic (not hypoglycemic) control is needed for many patients. Hypertonic IV fluids are not normally utilized and antibiotics are necessary only in patients with septic shock.

A 45-year-old obese man arrives in a clinic with complaints of daytime sleepiness, difficulty going to sleep at night, and snoring. The nurse should recognize the manifestations of what health problem? A) Adenoiditis B) Chronic tonsillitis C) Obstructive sleep apnea D) Laryngeal cancer

Ans: C Feedback: Obstructive sleep apnea occurs in men, especially those who are older and overweight. Symptoms include excessive daytime sleepiness, insomnia, and snoring. Daytime sleepiness and difficulty going to sleep at night are not indications of tonsillitis or adenoiditis. This patients symptoms are not suggestive of laryngeal cancer

The nurse is teaching a patient with allergic rhinitis about the safe and effective use of his medications. What would be the most essential information to give this patient about preventing possible drug interactions? A) Prescription medications can be safely supplemented with OTC medications. B) Use only one pharmacy so the pharmacist can check drug interactions. C) Read drug labels carefully before taking OTC medications. D) Consult the Internet before selecting an OTC medication.

Ans: C Feedback: Patient education is essential when assisting the patient in the use of all medications. To prevent possible drug interactions, the patient is cautioned to read drug labels before taking any OTC medications. Some Web sites are reliable and valid information sources, but this is not always the case. Patients do not necessarily need to limit themselves to one pharmacy, though checking for potential interactions is important. Not all OTC medications are safe additions to prescription medication regimens

A patient arrives in the emergency department with an attack of acute bronchiectasis. Chest auscultation reveals the presence of copious secretions. What intervention should the nurse prioritize in this patients care? A) Oral administration of diuretics B) Intravenous fluids to reduce the viscosity of secretions C) Postural chest drainage D) Pulmonary function testing

Ans: C Feedback: Postural drainage is part of all treatment plans for bronchiectasis, because draining of the bronchiectatic areas by gravity reduces the amount of secretions and the degree of infection. Diuretics and IV fluids will not aid in the mobilization of secretions. Lung function testing may be indicated, but this assessment will not relieve the patients symptoms.

You are caring for a 65-year-old male patient admitted to your medical unit 72 hours ago with pyloric stenosis. A nasogastric tube placed upon admission has been on low intermittent suction ever since. Upon review of the mornings blood work, you notice that the patients potassium is below reference range. You should recognize that the patient may be at risk for what imbalance? A) Hypercalcemia B) Metabolic acidosis C) Metabolic alkalosis D) Respiratory acidosis

Ans: C Feedback: Probably the most common cause of metabolic alkalosis is vomiting or gastric suction with loss of hydrogen and chloride ions. The disorder also occurs in pyloric stenosis in which only gastric fluid is lost. Vomiting, gastric suction, and pyloric stenosis all remove potassium and can cause hypokalemia. This patient would not be at risk for hypercalcemia; hyperparathyroidism and cancer account for almost all cases of hypercalcemia. The nasogastric tube is removing stomach acid and will likely raise pH. Respiratory acidosis is unlikely since no change was reported in the patients respiratory status.

The nurse is caring for a patient in the ICU who has been diagnosed with multiple organ dysfunction syndrome (MODS). The nurses plan of care should include which of the following interventions? A) Encouraging the family to stay hopeful and educating them to the fact that, in nearly all cases, the prognosis is good B) Encouraging the family to leave the hospital and to take time for themselves as acute care of MODS patients may last for several months C) Promoting communication with the patient and family along with addressing end-of-life issues D) Discussing organ donation on a number of different occasions to allow the family time to adjust to the idea

Ans: C Feedback: Promoting communication with the patient and family is a critical role of the nurse with a patient in progressive shock. It is also important that the health care team address end-of-life decisions to ensure that supportive therapies are congruent with the patients wishes. Many cases of MODS result in death and the life expectancy of patients with MODS is usually measured in hours and possibly days, but not in months. Organ donation should be offered as an option on one occasion, and then allow the family time to discuss and return to the health care providers with an answer following the death of the patient.

The nurse is preparing to discharge a patient after thoracotomy. The patient is going home on oxygen therapy and requires wound care. As a result, he will receive home care nursing. What should the nurse include in discharge teaching for this patient? A) Safe technique for self-suctioning of secretions B) Technique for performing postural drainage C) Correct and safe use of oxygen therapy equipment D) How to provide safe and effective tracheostomy care

Ans: C Feedback: Respiratory care and other treatment modalities (oxygen, incentive spirometry, chest physiotherapy [CPT], and oral, inhaled, or IV medications) may be continued at home. Therefore, the nurse needs to instruct the patient and family in their correct and safe use. The scenario does not indicate the patient needs help with suctioning, postural drainage, or tracheostomy care.

The nurse is caring for a 46-year-old patient recently diagnosed with the early stages of lung cancer. The nurse is aware that the preferred method of treating patients with nonsmall cell tumors is what? A) Chemotherapy B) Radiation C) Surgical resection D) Bronchoscopic opening of the airway

Ans: C Feedback: Surgical resection is the preferred method of treating patients with localized nonsmall cell tumors with no evidence of metastatic spread and adequate cardiopulmonary function. The other listed treatment options may be considered, but surgery is preferred.

The nurse is providing care for a patient who is in shock after massive blood loss from a workplace injury. The nurse recognizes that many of the findings from the most recent assessment are due to compensatory mechanisms. What is a compensatory mechanism to increase cardiac output during hypovolemic states? A) Third spacing of fluid B) Dysrhythmias C) Tachycardia D) Gastric hypermotility

Ans: C Feedback: Tachycardia is a primary compensatory mechanism to increase cardiac output during hypovolemic states. The third spacing of fluid takes fluid out of the vascular space. Gastric hypermotility and dysrhythmias would not increase cardiac output and are not considered to be compensatory mechanisms.

A patient with a severe exacerbation of COPD requires reliable and precise oxygen delivery. Which mask will the nurse expect the physician to order? A) Non-rebreather air mask B) Tracheostomy collar C) Venturi mask D) Face tent

Ans: C Feedback: The Venturi mask provides the most accurate method of oxygen delivery. Other methods of oxygen delivery include the aerosol mask, tracheostomy collar, and face tents, but these do not match the precision of a Venturi mask.

The nurse in a rural nursing outpost has just been notified that she will be receiving a patient in hypovolemic shock due to a massive postpartum hemorrhage after her home birth. You know that the best choice for fluid replacement for this patient is what? A) 5% albumin because it is inexpensive and is always readily available B) Dextran because it increases intravascular volume and counteracts coagulopathy C) Whatever fluid is most readily available in the clinic, due to the nature of the emergency D) Lactated Ringers solution because it increases volume, buffers acidosis, and is the best choice for patients with liver failure

Ans: C Feedback: The best fluid to treat shock remains controversial. In emergencies, the best fluid is often the fluid that is readily available. Fluid resuscitation should be initiated early in shock to maximize intravascular volume. Both crystalloids and colloids can be administered to restore intravascular volume. There is no consensus regarding whether crystalloids or colloids, such as dextran and albumin, should be used; however, with crystalloids, more fluid is necessary to restore intravascular volume. Albumin is very expensive and is a blood product so it is not always readily available for use. Dextran does increase intravascular volume, but it increases the risk for coagulopathy. Lactated Ringers is a good solution choice because it increases volume and buffers acidosis, but it should not be used in patients with liver failure because the liver is unable to covert lactate to bicarbonate.

The nurse is caring for a patient who is receiving oxygen therapy for pneumonia. How should the nurse best assess whether the patient is hypoxemic? A) Assess the patients level of consciousness (LOC). B) Assess the patients extremities for signs of cyanosis. C) Assess the patients oxygen saturation level. D) Review the patients hemoglobin, hematocrit, and red blood cell levels.

Ans: C Feedback: The effectiveness of the patients oxygen therapy is assessed by the ABG analysis or pulse oximetry. ABG results may not be readily available. Presence or absence of cyanosis is not an accurate indicator of oxygen effectiveness. The patients LOC may be affected by hypoxia, but not every change in LOC is related to oxygenation. Hemoglobin, hematocrit, and red blood cell levels do not directly reflect current oxygenation status.

The nurse is caring for an 82-year-old patient with a diagnosis of tracheobronchitis. The patient begins complaining of right-sided chest pain that gets worse when he coughs or breathes deeply. Vital signs are within normal limits. What would you suspect this patient is experiencing? A) Traumatic pneumothorax B) Empyema C) Pleuritic pain D) Myocardial infarction

Ans: C Feedback: The key characteristic of pleuritic pain is its relationship to respiratory movement. Taking a deep breath, coughing, or sneezing worsens the pain. Pleuritic pain is limited in distribution rather than diffuse; it usually occurs only on one side. The pain may become minimal or absent when the breath is held. It may be localized or radiate to the shoulder or abdomen. Later, as pleural fluid develops, the pain decreases. The scenario does not indicate any trauma to the patient, so a traumatic pneumothorax is implausible. Empyema is unlikely as there is no fever indicative of infection. Myocardial infarction would affect the patients vital signs profoundly

The nurse is caring for a patient who has central venous pressure (CVP) monitoring in place. The nurses most recent assessment reveals that CVP is 7 mm Hg. What is the nurses most appropriate action? A) Arrange for continuous cardiac monitoring and reposition the patient. B) Remove the CVP catheter and apply an occlusive dressing. C) Assess the patient for fluid overload and inform the physician. D) Raise the head of the patients bed and have the patient perform deep breathing exercise, if possible.

Ans: C Feedback: The normal CVP is 2 to 6 mm Hg. Many problems can cause an elevated CVP, but the most common is due to hypervolemia. Assessing the patient and informing the physician are the most prudent actions. Repositioning the patient is ineffective and removing the device is inappropriate

The nurse has noted the emergence of a significant amount of fresh blood at the drain site of a patient who is postoperative day 1 following total laryngectomy. How should the nurse respond to this development? A) Remove the patients drain and apply pressure with a sterile gauze. B) Assess the patient, reposition the patient supine, and apply wall suction to the drain. C) Rapidly assess the patient and notify the surgeon about the patients bleeding. D) Administer a STAT dose of vitamin K to aid coagulation.

Ans: C Feedback: The nurse promptly notifies the surgeon of any active bleeding, which can occur at a variety of sites, including the surgical site, drains, and trachea. The drain should not be removed or connected to suction. Supine positioning would exacerbate the bleeding. Vitamin K would not be administered without an order.

A nurse has performed tracheal suctioning on a patient who experienced increasing dyspnea prior to a procedure. When applying the nursing process, how can the nurse best evaluate the outcomes of this intervention? A) Determine whether the patient can now perform forced expiratory technique (FET). B) Percuss the patients lungs and thorax. C) Measure the patients oxygen saturation. D) Have the patient perform incentive spirometry.

Ans: C Feedback: The patients response to suctioning is usually determined by performing chest auscultation and by measuring the patients oxygen saturation. FET, incentive spirometry, and percussion are not normally used as evaluative techniques.

A nurse is completing a focused respiratory assessment of a child with asthma. What assessment finding is most closely associated with the characteristic signs and symptoms of asthma? A) Shallow respirations B) Increased anterior-posterior (A-P) diameter C) Bilateral wheezes D) Bradypnea

Ans: C Feedback: The three most common symptoms of asthma are cough, dyspnea, and wheezing. There may be generalized wheezing (the sound of airflow through narrowed airways), first on expiration and then, possibly, during inspiration as well. Respirations are not usually slow and the childs A-P diameter does not normally change

The home care nurse is assessing a patient who requires home oxygen therapy. What criterion indicates that an oxygen concentrator will best meet the needs of the patient in the home environment? A) The patient desires a low-maintenance oxygen delivery system that delivers oxygen flow rates up to 6 L/min. B) The patient requires a high-flow system for use with a tracheostomy collar. C) The patient desires a portable oxygen delivery system that can deliver 2 L/min. D) The patients respiratory status requires a system that provides an FiO2 of 65%.

Ans: C Feedback: The use of oxygen concentrators is another means of providing varying amounts of oxygen, especially in the home setting. They can deliver oxygen flows from 1 to 10 L/min and provide an FiO2 of about 40%. They require regular maintenance and are not used for high-flow applications. The patient desiring a portable oxygen delivery system of 2L/min will benefit from the use of an oxygen concentrator.

The nurse is creating a care plan for a patient who is status post-total laryngectomy. Much of the plan consists of a long-term postoperative communication plan for alaryngeal communication. What form of alaryngeal communication will likely be chosen? A) Esophageal speech B) Electric larynx C) Tracheoesophageal puncture D) American sign language (ASL)

Ans: C Feedback: Tracheoesophageal puncture is simple and has few complications. It is associated with high phonation success, good phonation quality, and steady long-term results. As a result, it is preferred over esophageal speech, and electric larynx or ASL.

A patient has just been diagnosed with lung cancer. After the physician discusses treatment options and leaves the room, the patient asks the nurse how the treatment is decided upon. What would be the nurses best response? A) The type of treatment depends on the patients age and health status. B) The type of treatment depends on what the patient wants when given the options. C) The type of treatment depends on the cell type of the cancer, the stage of the cancer, and the patients health status. D) The type of treatment depends on the discussion between the patient and the physician of which treatment is best.

Ans: C Feedback: Treatment of lung cancer depends on the cell type, the stage of the disease, and the patients physiologic status (particularly cardiac and pulmonary status). Treatment does not depend solely on the patients age or the patients preference between the different treatment modes. The decision about treatment does not primarily depend on a discussion between the patient and the physician of which treatment is best, though this discussion will take place.

The nurse is caring for a patient admitted with unstable angina. The laboratory result for the initial troponin I is elevated in this patient. The nurse should recognize what implication of this assessment finding? A) This is only an accurate indicator of myocardial damage when it reaches its peak in 24 hours. B) Because the patient has a history of unstable angina, this is a poor indicator of myocardial injury. C) This is an accurate indicator of myocardial injury. D) This result indicates muscle injury, but does not specify the source.

Ans: C Feedback: Troponin I, which is specific to cardiac muscle, is elevated within hours after myocardial injury. Even with a diagnosis of unstable angina, this is an accurate indicator of myocardial injury.

You are an emergency-room nurse caring for a trauma patient. Your patient has the following arterial blood gas results: pH 7.26, PaCO2 28, HCO3 11 mEq/L. How would you interpret these results? A) Respiratory acidosis with no compensation B) Metabolic alkalosis with a compensatory alkalosis C) Metabolic acidosis with no compensation D) Metabolic acidosis with a compensatory respiratory alkalosis

Ans: D Feedback: A low pH indicates acidosis (normal pH is 7.35 to 7.45). The PaCO3 is also low, which causes alkalosis. The bicarbonate is low, which causes acidosis. The pH bicarbonate more closely corresponds with a decrease in pH, making the metabolic component the primary problem.

You are caring for a patient who has a diagnosis of syndrome of inappropriate antidiuretic hormone secretion (SIADH). Your patients plan of care includes assessment of specific gravity every 4 hours. The results of this test will allow the nurse to assess what aspect of the patients health? A) Nutritional status B) Potassium balance C) Calcium balance D) Fluid volume status

Ans: D Feedback: A specific gravity will detect if the patient has a fluid volume deficit or fluid volume excess. Nutrition, potassium, and calcium levels are not directly indicated.

A patient in the ICU is status post embolectomy after a pulmonary embolus. What assessment parameter does the nurse monitor most closely on a patient who is postoperative following an embolectomy? A) Pupillary response B) Pressure in the vena cava C) White blood cell differential D) Pulmonary arterial pressure

Ans: D Feedback: If the patient has undergone surgical embolectomy, the nurse measures the patients pulmonary arterial pressure and urinary output. Pressure is not monitored in a patients vena cava. White cell levels and pupillary responses would be monitored, but not to the extent of the patients pulmonary arterial pressure.

You are working on a burns unit and one of your acutely ill patients is exhibiting signs and symptoms of third spacing. Based on this change in status, you should expect the patient to exhibit signs and symptoms of what imbalance? A) Metabolic alkalosis B) Hypermagnesemia C) Hypercalcemia D) Hypovolemia

Ans: D Feedback: Third-spacing fluid shift, which occurs when fluid moves out of the intravascular space but not into the intracellular space, can cause hypovolemia. Increased calcium and magnesium levels are not indicators of third-spacing fluid shift. Burns typically cause acidosis, not alkalosis.

The nurse is caring for a patient who is postoperative day 2 following a total laryngectomy for supraglottic cancer. The nurse should prioritize what assessment? A) Assessment of body image B) Assessment of jugular venous pressure C) Assessment of carotid pulse D) Assessment of swallowing ability

Ans: D Feedback: A common postoperative complication from this type of surgery is difficulty in swallowing, which creates a potential for aspiration. Cardiovascular complications are less likely at this stage of recovery. The patients body image should be assessed, but dysphagia has the potential to affect the patients airway, and is a consequent priority

The campus nurse at a university is assessing a 21-year-old student who presents with a severe nosebleed. The site of bleeding appears to be the anterior portion of the nasal septum. The nurse instructs the student to tilt her head forward and the nurse applies pressure to the nose, but the students nose continues to bleed. Which intervention should the nurse next implement? A) Apply ice to the bridge of her nose B) Lay the patient down on a cot C) Arrange for transfer to the local ED D) Insert a tampon in the affected nare

Ans: D Feedback: A cotton tampon may be used to try to stop the bleeding. The use of ice on the bridge of the nose has no scientific rationale for care. Laying the client down on the cot could block the clients airway. Hospital admission is necessary only if the bleeding becomes serious.

A nurse is providing health education to the family of a patient with bronchiectasis. What should the nurse teach the patients family members? A) The correct technique for chest palpation and auscultation B) Techniques for assessing the patients fluid balance C) The technique for providing deep nasotracheal suctioning D) The correct technique for providing postural drainage

Ans: D Feedback: A focus of the care of bronchiectasis is helping patients clear pulmonary secretions; consequently, patients and families are taught to perform postural drainage. Chest palpation and auscultation and assessment of fluid balance are not prioritized over postural drainage. Nasotracheal suctioning is not normally necessary.

A nursing is planning the care of a patient with emphysema who will soon be discharged. What teaching should the nurse prioritize in the plan of care? A) Taking prophylactic antibiotics as ordered B) Adhering to the treatment regimen in order to cure the disease C) Avoiding airplanes, buses, and other crowded public places D) Setting realistic short-term and long-range goals

Ans: D Feedback: A major area of teaching involves setting and accepting realistic short-term and long-range goals. Emphysema is not considered curable and antibiotics are not used on a preventative basis. The patient does not normally need to avoid public places.

Sepsis is an evolving process, with neither clearly definable clinical signs and symptoms nor predictable progression. As the ICU nurse caring for a patient with sepsis, the nurse knows that tissue perfusion declines during sepsis and the patient begins to show signs of organ dysfunction. What sign would indicate to the nurse that end-organ damage may be occurring? A) Urinary output increases B) Skin becomes warm and dry C) Adventitious lung sounds occur in the upper airway D) Heart and respiratory rates are elevated

Ans: D Feedback: As sepsis progresses, tissues become less perfused and acidotic, compensation begins to fail, and the patient begins to show signs of organ dysfunction. The cardiovascular system also begins to fail, the blood pressure does not respond to fluid resuscitation and vasoactive agents, and signs of end-organ damage are evident (e.g., renal failure, pulmonary failure, hepatic failure). As sepsis progresses to septic shock, the blood pressure drops, and the skin becomes cool, pale, and mottled. Temperature may be normal or below normal. Heart and respiratory rates remain rapid. Urine production ceases, and multiple organ dysfunction progressing to death occurs. Adventitious lung sounds occur throughout the lung fields, not just in the upper fields of the lungs.

The home care nurse is planning to begin breathing retraining exercises with a client newly admitted to the home health service. The home care nurse knows that breathing retraining is especially indicated if the patient has what diagnosis? A) Asthma B) Pneumonia C) Lung cancer D) COPD

Ans: D Feedback: Breathing retraining is especially indicated in patients with COPD and dyspnea. Breathing retraining may be indicated in patients with other lung pathologies, but not to the extent indicated in patients with COPD

The nurse is caring for an 82-year-old patient. The nurse knows that changes in cardiac structure and function occur in older adults. What is a normal change expected in the aging heart of an older adult? A) Decreased left ventricular ejection time B) Decreased connective tissue in the SA and AV nodes and bundle branches C) Thinning and flaccidity of the cardiac values D) Widening of the aorta

Ans: D Feedback: Changes in cardiac structure and function are clearly observable in the aging heart. Aging results in decreased elasticity and widening of the aorta, thickening and rigidity of the cardiac valves, increased connective tissue in the SA and AV nodes and bundle branches, and an increased left ventricular ejection time (prolonged systole)

A nurse is documenting the results of assessment of a patient with bronchiectasis. What would the nurse most likely include in documentation? A) Sudden onset of pleuritic chest pain B) Wheezes on auscultation C) Increased anterior-posterior (A-P) diameter D) Clubbing of the fingers

Ans: D Feedback: Characteristic symptoms of bronchiectasis include chronic cough and production of purulent sputum in copious amounts. Clubbing of the fingers also is common because of respiratory insufficiency. Sudden pleuritic chest pain is a common manifestation of a pulmonary embolism. Wheezes on auscultation are common in patients with asthma. An increased A-P diameter is noted in patients with COPD

A nurse is educating a patient in anticipation of a procedure that will require a water-sealed chest drainage system. What should the nurse tell the patient and the family that this drainage system is used for? A) Maintaining positive chest-wall pressure B) Monitoring pleural fluid osmolarity C) Providing positive intrathoracic pressure D) Removing excess air and fluid

Ans: D Feedback: Chest tubes and closed drainage systems are used to re-expand the lung involved and to remove excess air, fluid, and blood. They are not used to maintain positive chest-wall pressure, monitor pleural fluid, or provide positive intrathoracic pressure.

A nurse is providing health education to a patient scheduled for cryoablation therapy. The nurse should describe what aspect of this treatment? A) Peeling away the area of endocardium responsible for the dysrhythmia B) Using electrical shocks directly to the endocarduim to eliminate the source of dysrhythmia C) Using high-frequency sound waves to eliminate the source of dysrhythmia D) Using a cooled probe to eliminate the source of dysrhythmia

Ans: D Feedback: Cryoablation therapy involves using a cooled probe to create a small scar on the endocardium to eliminate the source of the dysrhythmias. Endocardium resection involves peeling away a specified area of the endocardium. Electrical ablation involves using shocks to eliminate the area causing the dysrhythmias. Radio frequency ablation uses high-frequency sound waves to destroy the area causing the dysrhythmias

The nurse is performing patient education for a patient who is being discharged on mini-nebulizer treatments. What information should the nurse prioritize in the patients discharge teaching? A) How to count her respirations accurately B) How to collect serial sputum samples C) How to independently wean herself from treatment D) How to perform diaphragmatic breathing

Ans: D Feedback: Diaphragmatic breathing is a helpful technique to prepare for proper use of the small-volume nebulizer. Patient teaching would not include counting respirations and the patient should not wean herself from treatment without the involvement of her primary care provider. Serial sputum samples are not normally necessary.

An immunocompromised older adult has developed a urinary tract infection and the care team recognizes the need to prevent an exacerbation of the patients infection that could result in urosepsis and septic shock. What action should the nurse perform to reduce the patients risk of septic shock? A) Apply an antibiotic ointment to the patients mucous membranes, as ordered. B) Perform passive range-of-motion exercises unless contraindicated C) Initiate total parenteral nutrition (TPN) D) Remove invasive devices as soon as they are no longer needed

Ans: D Feedback: Early removal of invasive devices can reduce the incidence of infections. Broad application of antibiotic ointments is not performed. TPN may be needed, but this does not directly reduce the risk of further infection. Range-of-motion exercises are not a relevant intervention.

The nurse is assessing a patient who has a chest tube in place for the treatment of a pneumothorax. The nurse observes that the water level in the water seal rises and falls in rhythm with the patients respirations. How should the nurse best respond to this assessment finding? A) Gently reinsert the chest tube 1 to 2 cm and observe if the water level stabilizes. B) Inform the physician promptly that there is in imminent leak in the drainage system. C) Encourage the patient to do deep breathing and coughing exercises. D) Document that the chest drainage system is operating as it is intended.

Ans: D Feedback: Fluctuation of the water level in the water seal shows effective connection between the pleural cavity and the drainage chamber and indicates that the drainage system remains patent. No further action is needed.

A patient visiting the clinic is diagnosed with acute sinusitis. To promote sinus drainage, the nurse should instruct the patient to perform which of the following? A) Apply a cold pack to the affected area. B) Apply a mustard poultice to the forehead. C) Perform postural drainage. D) Increase fluid intake.

Ans: D Feedback: For a patient diagnosed with acute sinusitis, the nurse should instruct the patient that hot packs, increasing fluid intake, and elevating the head of the bed can promote drainage. Applying a mustard poultice will not promote sinus drainage. Postural drainage is used to remove bronchial secretions.

The nurse is providing patient teaching to a young mother who has brought her 3-month-old infant to the clinic for a well-baby checkup. What action should the nurse recommend to the woman to prevent the transmission of organisms to her infant during the cold season? A) Take preventative antibiotics, as ordered. B) Gargle with warm salt water regularly. C) Dress herself and her infant warmly. D) Wash her hands frequently.

Ans: D Feedback: Handwashing remains the most effective preventive measure to reduce the transmission of organisms. Taking prescribed antibiotics, using warm salt-water gargles, and dressing warmly do not suppress transmission. Antibiotics are not prescribed for a cold.

A patient comes to the ED and is admitted with epistaxis. Pressure has been applied to the patients midline septum for 10 minutes, but the bleeding continues. The nurse should anticipate using what treatment to control the bleeding? A) Irrigation with a hypertonic solution B) Nasopharyngeal suction C) Normal saline application D) Silver nitrate application

Ans: D Feedback: If pressure to the midline septum does not stop the bleeding for epistaxis, additional treatment of silver nitrate application, Gelfoam, electrocautery, or vasoconstrictors may be used. Suction may be used to visualize the nasal septum, but it does not alleviate the bleeding. Irrigation with a hypertonic solution is not used to treat epistaxis.

A patient recovering from thoracic surgery is on long-term mechanical ventilation and becomes very frustrated when he tries to communicate. What intervention should the nurse perform to assist the patient? A) Assure the patient that everything will be all right and that remaining calm is the best strategy. B) Ask a family member to interpret what the patient is trying to communicate. C) Ask the physician to wean the patient off the mechanical ventilator to allow the patient to speak freely. D) Express empathy and then encourage the patient to write, use a picture board, or spell words with an alphabet board.

Ans: D Feedback: If the patient uses an alternative method of communication, he will feel in better control and likely be less frustrated. Assuring the patient that everything will be all right offers false reassurance, and telling him not to be upset minimizes his feelings. Neither of these methods helps the patient to communicate. In a patient with an endotracheal or tracheostomy tube, the family members are also likely to encounter difficulty interpreting the patients wishes. Making them responsible for interpreting the patients gestures may frustrate the family. The patient may be weaned off a mechanical ventilator only when the physiologic parameters for weaning have been met.

A 42-year-old patient is admitted to the ED after an assault. The patient received blunt trauma to the face and has a suspected nasal fracture. Which of the following interventions should the nurse perform? A) Administer nasal spray and apply an occlusive dressing to the patients face. B) Position the patients head in a dependent position. C) Irrigate the patients nose with warm tap water. D) Apply ice and keep the patients head elevated.

Ans: D Feedback: Immediately after the fracture, the nurse applies ice and encourages the patient to keep the head elevated. The nurse instructs the patient to apply ice packs to the nose to decrease swelling. Dependent positioning would exacerbate bleeding and the nose is not irrigated. Occlusive dressings are not used.

The nurse caring for a patient recently diagnosed with lung disease encourages the patient not to smoke. What is the primary rationale behind this nursing action? A) Smoking decreases the amount of mucus production. B) Smoke particles compete for binding sites on hemoglobin C) Smoking causes atrophy of the alveoli. D) Smoking damages the ciliary cleansing mechanism

Ans: D Feedback: In addition to irritating the mucous cells of the bronchi and inhibiting the function of alveolar macrophage (scavenger) cells, smoking damages the ciliary cleansing mechanism of the respiratory tract. Smoking also increases the amount of mucus production and distends the alveoli in the lungs. It reduces the oxygen-carrying capacity of hemoglobin, but not by directly competing for binding sites

A nurse is preparing to perform an admission assessment on a patient with COPD. It is most important for the nurse to review which of the following? A) Social work assessment B) Insurance coverage C) Chloride levels D) Available diagnostic tests

Ans: D Feedback: In addition to the patients history, the nurse reviews the results of available diagnostic tests. Social work assessment is not a priority for the majority of patients. Chloride levels are relevant to CF, not COPD. Insurance coverage is not normally the domain of the nurse.

A patients total laryngectomy has created a need for alaryngeal speech which will be achieved through the use of tracheoesophageal puncture. What action should the nurse describe to the patient when teaching him about this process? A) Training on how to perform controlled belching B) Use of an electronically enhanced artificial pharynx C) Insertion of a specialized nasogastric tube D) Fitting for a voice prosthesis

Ans: D Feedback: In patients receiving transesophageal puncture, a valve is placed in the tracheal stoma to divert air into the esophagus and out the mouth. Once the puncture is surgically created and has healed, a voice prosthesis (Blom-Singer) is fitted over the puncture site. A nasogastric tube and belching are not required. An artificial pharynx is not used.

The emergency nurse is admitting a patient experiencing a GI bleed who is believed to be in the compensatory stage of shock. What assessment finding would be most consistent with the early stage of compensation? A) Increased urine output B) Decreased heart rate C) Hyperactive bowel sounds D) Cool, clammy skin

Ans: D Feedback: In the compensatory stage of shock, the body shunts blood from the organs, such as the skin and kidneys, to the brain and heart to ensure adequate blood supply. As a result, the patients skin is cool and clammy. Also in this compensatory stage, blood vessels vasoconstrict, the heart rate increases, bowel sounds are hypoactive, and the urine output decreases

A critical care nurse is caring for a patient with a pulmonary artery catheter in place. What does this catheter measure that is particularly important in critically ill patients? A) Pulmonary artery systolic pressure B) Right ventricular afterload C) Pulmonary artery pressure D) Left ventricular preload

Ans: D Feedback: Monitoring of the pulmonary artery diastolic and pulmonary artery wedge pressures is particularly important in critically ill patients because it is used to evaluate left ventricular filling pressures (i.e., left ventricular preload). This device does not directly measure the other listed aspects of cardiac function.

In all types of shock, nutritional demands increase rapidly as the body depletes its stores of glycogen. Enteral nutrition is the preferred method of meeting these increasing energy demands. What is the basis for enteral nutrition being the preferred method of meeting the bodys needs? A) It slows the proliferation of bacteria and viruses during shock. B) It decreases the energy expended through the functioning of the GI system. C) It assists in expanding the intravascular volume of the body. D) It promotes GI function through direct exposure to nutrients.

Ans: D Feedback: Parenteral or enteral nutritional support should be initiated as soon as possible. Enteral nutrition is preferred, promoting GI function through direct exposure to nutrients and limiting infectious complications associated with parenteral feeding. Enteral feeding does not decrease the proliferation of microorganisms or the amount of energy expended through the functioning of the GI system and it does not assist in expanding the intravascular volume of the body.

A patients plan of care specifies postural drainage. What action should the nurse perform when providing this noninvasive therapy? A) Administer the treatment with the patient in a high Fowlers or semi-Fowlers position. B) Perform the procedure immediately following the patients meals. C) Apply percussion firmly to bare skin to facilitate drainage. D) Assist the patient into a position that will allow gravity to move secretions.

Ans: D Feedback: Postural drainage is usually performed two to four times per day. The patient uses gravity to facilitate postural draining. The skin should be covered with a cloth or a towel during percussion to protect the skin. Postural drainage is not administered in an upright position or directly following a meal.

A nurse is explaining to a patient with asthma what her new prescription for prednisone is used for. What would be the most accurate explanation that the nurse could give? A) To ensure long-term prevention of asthma exacerbations B) To cure any systemic infection underlying asthma attacks C) To prevent recurrent pulmonary infections D) To gain prompt control of inadequately controlled, persistent asthma

Ans: D Feedback: Prednisone is used for a short-term (310 days) burst to gain prompt control of inadequately controlled, persistent asthma. It is not used to treat infection or to prevent exacerbations in the long term.

When caring for a patient in shock, one of the major nursing goals is to reduce the risk that the patient will develop complications of shock. How can the nurse best achieve this goal? A) Provide a detailed diagnosis and plan of care in order to promote the patients and familys coping. B) Keep the physician updated with the most accurate information because in cases of shock the nurse often cannot provide relevant interventions. C) Monitor for significant changes and evaluate patient outcomes on a scheduled basis focusing on blood pressure and skin temperature. D) Understand the underlying mechanisms of shock, recognize the subtle and more obvious signs, and then provide rapid assessment.

Ans: D Feedback: Shock is a life-threatening condition with a variety of underlying causes. It is critical that the nurse apply the nursing process as the guide for care. Shock is unpredictable and rapidly changing so the nurse must understand the underlying mechanisms of shock. The nurse must also be able to recognize the subtle as well as more obvious signs and then provide rapid assessment and response to provide the patient with the best chance for recovery. Coping skills are important, but not the ultimate priority. Keeping the physician updated with the most accurate information is important, but the nurse is in the best position to provide rapid assessment and response, which gives the patient the best chance for survival. Monitoring for significant changes is critical, and evaluating patient outcomes is always a part of the nursing process, but the subtle signs and symptoms of shock are as important as the more obvious signs, such as blood pressure and skin temperature. Assessment must lead to diagnosis and interventions.

The nurse is caring for a patient who is undergoing an exercise stress test. Prior to reaching the target heart rate, the patient develops chest pain. What is the nurses most appropriate response? A) Administer sublingual nitroglycerin to allow the patient to finish the test. B) Initiate cardiopulmonary resuscitation. C) Administer analgesia and slow the test. D) Stop the test and monitor the patient closely.

Ans: D Feedback: Signs of myocardial ischemia would necessitate stopping the test. CPR would only be necessary if signs of cardiac or respiratory arrest were evident.

A patient is having pulmonary-function studies performed. The patient performs a spirometry test, revealing an FEV1 /FVC ratio of 60%. How should the nurse interpret this assessment finding? A) Strong exercise tolerance B) Exhalation volume is normal C) Respiratory infection D) Obstructive lung disease

Ans: D Feedback: Spirometry is used to evaluate airflow obstruction, which is determined by the ratio of forced expiration volume in 1 second to forced vital capacity. Obstructive lung disease is apparent when an FEV1 /FVC ratio is less than 70%

An 87-year-old patient has been hospitalized with pneumonia. Which nursing action would be a priority in this patients plan of care? A) Nasogastric intubation B) Administration of probiotic supplements C) Bedrest D) Cautious hydration

Ans: D Feedback: Supportive treatment of pneumonia in the elderly includes hydration (with caution and with frequent assessment because of the risk of fluid overload in the elderly); supplemental oxygen therapy; and assistance with deep breathing, coughing, frequent position changes, and early ambulation. Mobility is not normally discouraged and an NG tube is not necessary in most cases. Probiotics may or may not be prescribed for the patient.

A 54-year-old man has just been diagnosed with small cell lung cancer. The patient asks the nurse why the doctor is not offering surgery as a treatment for his cancer. What fact about lung cancer treatment should inform the nurses response? A) The cells in small cell cancer of the lung are not large enough to visualize in surgery. B) Small cell lung cancer is self-limiting in many patients and surgery should be delayed. C) Patients with small cell lung cancer are not normally stable enough to survive surgery. D) Small cell cancer of the lung grows rapidly and metastasizes early and extensively.

Ans: D Feedback: Surgery is primarily used for NSCLCs, because small cell cancer of the lung grows rapidly and metastasizes early and extensively. Difficult visualization and a patients medical instability are not the limiting factors. Lung cancer is not a self-limiting disease

A nurse who works in the specialty of palliative care frequently encounters issues and situations that constitute ethical dilemmas. What issue has most often presented challenging ethical issues, especially in the context of palliative care? A) The increase in cultural diversity in the United States B) Staffing shortages in health care and questions concerning quality of care C) Increased costs of health care coupled with inequalities in access D) Ability of technology to prolong life beyond meaningful quality of life

Ans: D Feedback: The application of technology to prolong life has raised several ethical issues. The major question is, Because we can prolong life through increasingly sophisticated technology, does it necessarily follow that we must do so? The increase in cultural diversity has not raised ethical issues in health care. Similarly, costs and staffing issues are relevant, but not central to the most common ethical issues surrounding palliative care.

The medical nurse is creating the care plan of an adult patient requiring mechanical ventilation. What nursing action is most appropriate? A) Keep the patient in a low Fowlers position. B) Perform tracheostomy care at least once per day. C) Maintain continuous bedrest. D) Monitor cuff pressure every 8 hours

Ans: D Feedback: The cuff pressure should be monitored every 8 hours. It is important to perform tracheostomy care at least every 8 hours because of the risk of infection. The patient should be encouraged to ambulate, if possible, and a low Fowlers position is not indicated.

In the past three to four decades, nursing has moved into the forefront in providing care for the dying. Which phenomenon has most contributed to this increased focus of care of the dying? A) Increased incidence of infections and acute illnesses B) Increased focus of health care providers on disease prevention C) Larger numbers of people dying in hospital settings D) Demographic changes in the population

Ans: D Feedback: The focus on care of the dying has been motivated by the aging of the population, the prevalence of, and publicity surrounding, life-threatening illnesses (e.g., cancer and AIDS), and the increasing likelihood of a prolonged period of chronic illness prior to death. The salience of acute infections, prevention measures, and death in hospital settings are not noted to have had a major influence on this phenomenon.

The home care nurse is monitoring a patient discharged home after resolution of a pulmonary embolus. For what potential complication would the home care nurse be most closely monitoring this patient? A) Signs and symptoms of pulmonary infection B) Swallowing ability and signs of aspiration C) Activity level and role performance D) Residual effects of compromised oxygenation

Ans: D Feedback: The home care nurse should monitor the patient for residual effects of the PE, which involved a severe disruption in respiration and oxygenation. PE has a noninfectious etiology; pneumonia is not impossible, but it is a less likely sequela. Swallowing ability is unlikely to be affected; activity level is important, but secondary to the effects of deoxygenation.

The nurse is caring for a patient who has been in a motor vehicle accident and the care team suspects that the patient has developed pleurisy. Which of the nurses assessment findings would best corroborate this diagnosis? A) The patient is experiencing painless hemoptysis. B) The patients arterial blood gases (ABGs) are normal, but he demonstrates increased work of breathing. C) The patients oxygen saturation level is below 88%, but he denies shortness of breath. D) The patients pain intensifies when he coughs or takes a deep breath.

Ans: D Feedback: The key characteristic of pleuritic pain is its relationship to respiratory movement. Taking a deep breath, coughing, or sneezing worsens the pain. The patients ABGs would most likely be abnormal and shortness of breath would be expected.

A clinic nurse is caring for a patient who has just been diagnosed with chronic obstructive pulmonary disease (COPD). The patient asks the nurse what he could have done to minimize the risk of contracting this disease. What would be the nurses best answer? A) The most important risk factor for COPD is exposure to occupational toxins. B) The most important risk factor for COPD is inadequate exercise. C) The most important risk factor for COPD is exposure to dust and pollen. D) The most important risk factor for COPD is cigarette smoking.

Ans: D Feedback: The most important risk factor for COPD is cigarette smoking. Lack of exercise and exposure to dust and pollen are not risk factors for COPD. Occupational risks are significant but are far exceeded by smoking

The home care nurse is visiting a patient newly discharged home after a lobectomy. What would be most important for the home care nurse to assess? A) Resumption of the patients ADLs B) The familys willingness to care for the patient C) Nutritional status and fluid balance D) Signs and symptoms of respiratory complications

Ans: D Feedback: The nurse assesses the patients adherence to the postoperative treatment plan and identifies acute or late postoperative complications. All options presented need assessment, but respiratory complications are the highest priority because they affect the patients airway and breathing.

The nurse is discussing activity management with a patient who is postoperative following thoracotomy. What instructions should the nurse give to the patient regarding activity immediately following discharge? A) Walk 1 mile 3 to 4 times a week. B) Use weights daily to increase arm strength. C) Walk on a treadmill 30 minutes daily. D) Perform shoulder exercises five times daily.

Ans: D Feedback: The nurse emphasizes the importance of progressively increased activity. The nurse also instructs the patient on the importance of performing shoulder exercises five times daily. The patient should ambulate with limits and realize that the return of strength will likely be gradual and likely will not include weight lifting or lengthy walks.

As a clinic nurse, you are caring for a patient who has been prescribed an antibiotic for tonsillitis and has been instructed to take the antibiotic for 10 days. When you do a follow-up call with this patient, you are informed that the patient is feeling better and is stopping the medication after taking it for 4 days. What information should you provide to this patient? A) Keep the remaining tablets for an infection at a later time. B) Discontinue the medications if the fever is gone. C) Dispose of the remaining medication in a biohazard receptacle. D) Finish all the antibiotics to eliminate the organism completely.

Ans: D Feedback: The nurse informs the patient about the need to take the full course of any prescribed antibiotic. Antibiotics should be taken for the entire 10-day period to eliminate the microorganisms. A patient should never be instructed to keep leftover antibiotics for use at a later time. Even if the fever or other symptoms are gone, the medications should be continued. Antibiotics do not need to be disposed of in a biohazard receptacle, though they should be discarded appropriately.

A nurse is teaching a patient how to perform flow type incentive spirometry prior to his scheduled thoracic surgery. What instruction should the nurse provide to the patient? A) Hold the spirometer at your lips and breathe in and out like you normally would. B) When youre ready, blow hard into the spirometer for as long as you can. C) Take a deep breath and then blow short, forceful breaths into the spirometer. D) Breathe in deeply through the spirometer, hold your breath briefly, and then exhale.

Ans: D Feedback: The patient should be taught to lace the mouthpiece of the spirometer firmly in the mouth, breathe air in through the mouth, and hold the breath at the end of inspiration for about 3 seconds. The patient should then exhale slowly through the mouthpiece.

A critically ill patient is admitted to the ICU. The physician decides to use intra-arterial pressure monitoring. After this intervention is performed, what assessment should the nurse prioritize in the plan of care? A) Fluctuations in core body temperature B) Signs and symptoms of esophageal varices C) Signs and symptoms of compartment syndrome D) Perfusion distal to the insertion site

Ans: D Feedback: The radial artery is the usual site selected. However, placement of a catheter into the radial artery can further impede perfusion to an area that has poor circulation. As a result, the tissue distal to the cannulated artery can become ischemic or necrotic. Vigilant assessment is thus necessary. Alterations in temperature and the development of esophageal varices or compartment syndrome are not high risks.

The nurse is explaining the safe and effective administration of nasal spray to a patient with seasonal allergies. What information is most important to include in this teaching? A) Finish the bottle of nasal spray to clear the infection effectively. B) Nasal spray can only be shared between immediate family members. C) Nasal spray should be administered in a prone position. D) Overuse of nasal spray may cause rebound congestion.

Ans: D Feedback: The use of topical decongestants is controversial because of the potential for a rebound effect. The patient should hold his or her head back for maximal distribution of the spray. Only the patient should use the bottle.

The nurse caring for a patient with an endotracheal tube recognizes several disadvantages of an endotracheal tube. What would the nurse recognize as a disadvantage of endotracheal tubes? A) Cognition is decreased. B) Daily arterial blood gases (ABGs) are necessary. C) Slight tracheal bleeding is anticipated. D) The cough reflex is depressed.

Ans: D Feedback: There are several disadvantages of an endotracheal tube. Disadvantages include suppression of the patients cough reflex, thickening of secretions, and depressed swallowing reflexes. Ulceration and stricture of the larynx or trachea may develop, but bleeding is not an expected finding. The tube should not influence cognition and daily ABGs are not always required.

A patient is brought into the ED by family members who tell the nurse the patient grabbed his chest and complained of substernal chest pain. The care team recognizes the need to monitor the patients cardiac function closely while interventions are performed. What form of monitoring should the nurse anticipate? A) Left-sided heart catheterization B) Cardiac telemetry C) Transesophageal echocardiography D) Hardwire continuous ECG monitoring

Ans: D Feedback: Two types of continuous ECG monitoring techniques are used in health care settings: hardwire cardiac monitoring, found in EDs, critical care units, and progressive care units; and telemetry, found in general nursing care units or outpatient cardiac rehabilitation programs. Cardiac catheterization and transesophageal echocardiography would not be used in emergent situations to monitor cardiac function.

The critical care nurse is preparing to initiate an infusion of a vasoactive medication to a patient in shock. The nurse knows that vasoactive medications are given in all forms of shock. What is the primary goal of this aspect of treatment? A) To prevent the formation of infarcts of emboli B) To limit stroke volume and cardiac output C) To prevent pulmonary and peripheral edema D) To maintain adequate mean arterial pressure

Ans: D Feedback: Vasoactive medications can be administered in all forms of shock to improve the patients hemodynamic stability when fluid therapy alone cannot maintain adequate MAP. Specific medications are selected to correct the particular hemodynamic alteration that is impeding cardiac output. These medications help increase the strength of myocardial contractility, regulate the heart rate, reduce myocardial resistance, and initiate vasoconstriction. They are not specifically used to prevent emboli, edema, or infarc

The case manager for a group of patients with COPD is providing health education. What is most important for the nurse to assess when providing instructions on self-management to these patients? A) Knowledge of alternative treatment modalities B) Family awareness of functional ability and activities of daily living (ADLs) C) Knowledge of the pathophysiology of the disease process D) Knowledge about self-care and their therapeutic regimen

Ans: D Feedback: When providing instructions about self-management, it is important for the nurse to assess the knowledge of patients and family members about self-care and the therapeutic regimen. This supersedes knowledge of alternative treatments or the pathophysiology of the disease, neither of which is absolutely necessary for patients to know. The patients own knowledge is more important than that of the family

A nurse is caring for a patient with COPD. The patients medication regimen has been recently changed and the nurse is assessing for therapeutic effect of a new bronchodilator. What assessment parameters suggest a consequent improvement in respiratory status? Select all that apply. A) Negative sputum culture B) Increased viscosity of lung secretions C) Increased respiratory rate D) Increased expiratory flow rate E) Relief of dyspnea

Ans: D, E Feedback: The relief of bronchospasm is confirmed by measuring improvement in expiratory flow rates and volumes (the force of expiration, how long it takes to exhale, and the amount of air exhaled) as well as by assessing the dyspnea and making sure that it has lessened. Increased respiratory rate and viscosity of secretions would suggest a worsening of the patients respiratory status. Bronchodilators would not have a direct result on the patients infectious process.

A 54-year-old has a diagnosis of breast cancer and is tearfully discussing her diagnosis with the nurse. The patient states, They tell me my cancer is malignant, while my coworkers breast tumor was benign. I just dont understand at all. When preparing a response to this patient, the nurse should be cognizant of what characteristic that distinguishes malignant cells from benign cells of the same tissue type? A) Slow rate of mitosis of cancer cells B) Different proteins in the cell membrane C) Differing size of the cells D) Different molecular structure in the cells

B

A medical nurse educator is reviewing a patients recent episode of metabolic acidosis with members of the nursing staff. What should the educator describe about the role of the kidneys in metabolic acidosis? A) The kidneys retain hydrogen ions and excrete bicarbonate ions to help restore balance. B) The kidneys excrete hydrogen ions and conserve bicarbonate ions to help restore balance. C) The kidneys react rapidly to compensate for imbalances in the body. D) The kidneys regulate the bicarbonate level in the intracellular fluid.

B

A nurse is creating a plan of care for an oncology patient and one of the identified nursing diagnoses is risk for infection related to myelosuppression. What intervention addresses the leading cause of infection-related death in oncology patients? A) Encourage several small meals daily. B) Provide skin care to maintain skin integrity. C) Assist the patient with hygiene, as needed. D) Assess the integrity of the patients oral mucosa regularly.

B

A patient with a diagnosis of gastric cancer has been unable to tolerate oral food and fluid intake and her tumor location precludes the use of enteral feeding. What intervention should the nurse identify as best meeting this patients nutritional needs? A) Administration of parenteral feeds via a peripheral IV B) TPN administered via a peripherally inserted central catheter C) Insertion of an NG tube for administration of feeds D) Maintaining NPO status and IV hydration until treatment completion

B

The clinic nurse is caring for a patient whose grandmother and sister have both had breast cancer. She requested a screening test to determine her risk of developing breast cancer and it has come back positive. The patient asks you what she can do to help prevent breast cancer from occurring. What would be your best response? A) Research has shown that eating a healthy diet can provide all the protection you need against breast cancer. B) Research has shown that taking the drug tamoxifen can reduce your chance of breast cancer. C) Research has shown that exercising at least 30 minutes every day can reduce your chance of breast cancer. D) Research has shown that there is little you can do to reduce your risk of breast cancer if you have a genetic predisposition.

B

The community health nurse is performing a home visit to an 84-year-old woman recovering from hip surgery. The nurse notes that the woman seems uncharacteristically confused and has dry mucous membranes. When asked about her fluid intake, the patient states, I stop drinking water early in the day because it is just too difficult to get up during the night to go to the bathroom. What would be the nurses best response? A) I will need to have your medications adjusted so you will need to be readmitted to the hospital for a complete workup. B) Limiting your fluids can create imbalances in your body that can result in confusion. Maybe we need to adjust the timing of your fluids. C) It is normal to be a little confused following surgery, and it is safe not to urinate at night. D) If you build up too much urine in your bladder, it can cause you to get confused, especially when your body is under stress.

B

The nurse caring for a patient post colon resection is assessing the patient on the second postoperative day. The nasogastric tube (NG) remains patent and continues at low intermittent wall suction. The IV is patent and infusing at 125 mL/hr. The patient reports pain at the incision site rated at a 3 on a 0-to-10 rating scale. During your initial shift assessment, the patient complains of cramps in her legs and a tingling sensation in her feet. Your assessment indicates decreased deep tendon reflexes (DTRs) and you suspect the patient has hypokalemia. What other sign or symptom would you expect this patient to exhibit? A) Diarrhea B) Dilute urine C) Increased muscle tone D) Joint pain

B

The nurse is caring for a patient with an advanced stage of breast cancer and the patient has recently learned that her cancer has metastasized. The nurse enters the room and finds the patient struggling to breath and the nurses rapid assessment reveals that the patients jugular veins are distended. The nurse should suspect the development of what oncologic emergency? A) Increased intracranial pressure B) Superior vena cava syndrome (SVCS) C) Spinal cord compression D) Metastatic tumor of the neck

B

You are caring for a patient with a diagnosis of pancreatitis. The patient was admitted from a homeless shelter and is a vague historian. The patient appears malnourished and on day 3 of the patients admission total parenteral nutrition (TPN) has been started. Why would you know to start the infusion of TPN slowly? A) Patients receiving TPN are at risk for hypercalcemia if calories are started too rapidly. B) Malnourished patients receiving parenteral nutrition are at risk for hypophosphatemia if calories are started too aggressively. C) Malnourished patients who receive fluids too rapidly are at risk for hypernatremia. D) Patients receiving TPN need a slow initiation of treatment in order to allow digestive enzymes to accumulate

B

A patient has undergone the creation of an Indiana pouch for the treatment of bladder cancer. The nurse identified the nursing diagnosis of disturbed body image. How can the nurse best address the effects of this urinary diversion on the patients body image? A) Emphasize that the diversion is an integral part of successful cancer treatment. B) Encourage the patient to speak openly and frankly about the diversion. C) Allow the patient to initiate the process of providing care for the diversion. D) Provide the patient with detailed written materials about the diversion at the time of discharge.

B Allowing the patient to express concerns and anxious feelings can help with body image, especially in adjusting to the changes in toileting habits. The nurse may have to initiate dialogue about the management of the diversion, especially if the patient is hesitant. Provision of educational materials is rarely sufficient to address a sudden change and profound change in body image. Emphasizing the role of the diversion in cancer treatment does not directly address the patients body image.

A participant in a health fair has asked the nurse about the role of drugs in liver disease. What health promotion teaching has the most potential to prevent drug-induced hepatitis? A) Finish all prescribed courses of antibiotics, regardless of symptom resolution. B) Adhere to dosing recommendations of OTC analgesics. C) Ensure that expired medications are disposed of safely. D) Ensure that pharmacists regularly review drug regimens for potential interactions.

B Although any medication can affect liver function, use of acetaminophen (found in many over-the- counter medications used to treat fever and pain) has been identified as the leading cause of acute liver failure. Finishing prescribed antibiotics and avoiding expired medications are unrelated to this disease. Drug interactions are rarely the cause of drug-induced hepatitis

A patient with an occluded coronary artery is admitted and has an emergency percutaneous transluminal coronary angioplasty (PTCA). The patient is admitted to the cardiac critical care unit after the PTCA. For what complication should the nurse most closely monitor the patient? A) Hyperlipidemia B) Bleeding at insertion site C) Left ventricular hypertrophy D) Congestive heart failure

B Feedback: Complications of PTCA may include bleeding at the insertion site, abrupt closure of the artery, arterial thrombosis, and perforation of the artery. Complications do not include hyperlipidemia, left ventricular hypertrophy, or congestive heart failure; each of these problems takes an extended time to develop and none is emergent.

The nurse caring for a patient whose sudden onset of sinus bradycardia is not responding adequately to atropine. What might be the treatment of choice for this patient? A) Implanted pacemaker B) Trancutaneous pacemaker C) ICD D) Asynchronous defibrillator

B Feedback: If a patient suddenly develops a bradycardia, is symptomatic but has a pulse, and is unresponsive to atropine, emergency pacing may be started with transcutaneous pacing, which most defibrillators are now equipped to perform. An implanted pacemaker is not a time-appropriate option. An asynchronous defibrillator or ICD would not provide relief.

A rehabilitation nurse caring for a patient who has had a stroke is approached by the patients family and asked why the patient has to do so much for herself when she is obviously struggling. What would be the nurses best answer? A) We are trying to help her be as useful as she possibly can. B) The focus on care in a rehabilitation facility is to help the patient to resume as much self-care as possible. C) We arent here to care for her the way the hospital staff did; we are here to help her get better so she can go home. D) Rehabilitation means helping patients do exactly what they did before their stroke.

B Feedback: In both acute care and rehabilitation facilities, the focus is on teaching the patient to resume as much self-care as possible. The goal of rehabilitation is not to be useful, nor is it to return patients to their prestroke level of functioning, which may be unrealistic.

As a staff member in a local hospice, a nurse deals with death and dying on a frequent basis. Where would be the safe venue for the nurse to express her feelings of frustration and grief about a patient who has recently died? A) In the cafeteria B) At a staff meeting C) At a social gathering D) At a memorial service

B Feedback: In hospice settings, where death, grief, and loss are expected outcomes of patient care, interdisciplinary colleagues rely on each other for support, using meeting time to express frustration, sadness, anger, and other emotions; to learn coping skills from each other; and to speak about how they were affected by the lives of those patients who have died since the last meeting. Public settings are inappropriate places to express frustration about the death of a patient.

The nurse in the ICU is caring for a 47-year-old, obese male patient who is in shock following a motor vehicle accident. The nurse is aware that patients in shock possess excess energy requirements. What would be the main challenge in meeting this patients elevated energy requirements during prolonged rehabilitation? A) Loss of adipose tissue B) Loss of skeletal muscle C) Inability to convert adipose tissue to energy D) Inability to maintain normal body mass

B Feedback: Nutritional energy requirements are met by breaking down lean body mass. In this catabolic process, skeletal muscle mass is broken down even when the patient has large stores of fat or adipose tissue. Loss of skeletal muscle greatly prolongs the patients recovery time. Loss of adipose tissue, the inability to convert adipose tissue to energy, and the inability to maintain normal body mass are not main concerns in meeting nutritional energy requirements for this patient.

A patient with a diagnosis of syndrome of inappropriate antidiuretic hormone secretion (SIADH) is being cared for on the critical care unit. The priority nursing diagnosis for a patient with this condition is what? A) Risk for peripheral neurovascular dysfunction B) Excess fluid volume C) Hypothermia D) Ineffective airway clearanc

B Feedback: The priority nursing diagnosis for a patient with SIADH is excess fluid volume, as the patient retains fluids and develops a sodium deficiency. Restricting fluid intake is a typical intervention for managing this syndrome. Temperature imbalances are not associated with SIADH. The patient is not at risk for neurovascular dysfunction or a compromised airway.

The nurses assessment of an older adult client reveals the following data: Lying BP 144/82 mm Hg; sitting BP 121/69 mm Hg; standing BP 98/56 mm Hg. The nurse should consequently identify what nursing diagnosis in the patients plan of care? A) Risk for ineffective breathing pattern related to hypotension B) Risk for falls related to orthostatic hypotension C) Risk for ineffective role performance related to hypotension D) Risk for imbalanced fluid balance related to hemodynamic variability

B Orthostatic hypotension creates a significant risk for falls due to the dizziness and lightheadedness that accompanies it. It does not normally affect breathing or fluid balance. The patients ability to perform normal roles may be affected, but the risk for falls is the most significant threat to safety.

A patient converts from normal sinus rhythm at 80 bpm to atrial fibrillation with a ventricular response at 166 bpm. Blood pressure is 162/74 mm Hg. Respiratory rate is 20 breaths per minute with normal chest expansion and clear lungs bilaterally. IV heparin and Cardizem are given. The nurse caring for the patient understands that the main goal of treatment is what? A) Decrease SA node conduction B) Control ventricular heart rate C) Improve oxygenation D) Maintain anticoagulation

B Treatment for atrial fibrillation is to terminate the rhythm or to control ventricular rate. This is a priority because it directly affects cardiac output. A rapid ventricular response reduces the time for ventricular filling, resulting in a smaller stroke volume. Control of rhythm is the initial treatment of choice, followed by anticoagulation with heparin and then Coumadin.

A patient with mitral valve stenosis is receiving health education at an outpatient clinic. To minimize the patients symptoms, the nurse should teach the patient to do which of the following? A) Eat a high-protein, low-carbohydrate diet. B) Avoid activities that cause an increased heart rate. C) Avoid large crowds and public events. D) Perform deep breathing and coughing exercises.

B atients with mitral stenosis are advised to avoid strenuous activities, competitive sports, and pregnancy, all of which increase heart rate. Infection prevention is important, but avoiding crowds is not usually necessary. Deep breathing and coughing are not likely to prevent exacerbations of symptoms and increased protein intake is not necessary.

A patient returns to the unit after a neck dissection. The surgeon placed a Jackson Pratt drain in the wound. When assessing the wound drainage over the first 24 postoperative hours the nurse would notify the physician immediately for what? A) Presence of small blood clots in the drainage B) 60 mL of milky or cloudy drainage C) Spots of drainage on the dressings surrounding the drain D) 120 mL of serosanguinous drainage

B Between 80 and 120 mL of serosanguineous secretions may drain over the first 24 hours. Milky drainage is indicative of a chyle fistula, which requires prompt treatment

The nurse is auscultating the breath sounds of a patient with pericarditis. What finding is most consistent with this diagnosis? A) Wheezes B) Friction rub C) Fine crackles D) Coarse crackles

B Feedback: A pericardial friction rub is diagnostic of pericarditis. Crackles are associated with pulmonary edema and fluid accumulation, whereas wheezes signal airway constriction; neither of these occurs with pericarditis.

A patients burns have required a homograft. During the nurses most recent assessment, the nurse observes that the graft is newly covered with purulent exudate. What is the nurses most appropriate response? A) Perform mechanical debridement to remove the exudate and prevent further infection. B) Inform the primary care provider promptly because the graft may need to be removed. C) Perform range of motion exercises to increase perfusion to the graft site and facilitate healing. D) Document this finding as an expected phase of graft healing.

B Feedback: An infected graft may need to be removed, thus the care provider should be promptly informed. ROM exercises will not resolve this problem and the nurse would not independently perform debridement.

A nurse practitioner is assessing a patient who has a fever, malaise, and a white blood cell count that is elevated. Which of the following principles should guide the nurses management of the patients care? A) There is a need for the patient to be assessed for lymphoma. B) Infection is the most likely cause of the patients change in health status. C) The patient is exhibiting signs and symptoms of leukemia. D) The patient should undergo diagnostic testing for multiple myeloma.

B Leukocytosis is most often the result of infection. It is only considered pathologic (and suggestive of leukemia) if it is persistent and extreme. Multiple myeloma and lymphoma are not likely causes of this constellation of symptoms.

A nurse in the neurologic ICU has orders to infuse a hypertonic solution into a patient with increased intracranial pressure. This solution will increase the number of dissolved particles in the patients blood, creating pressure for fluids in the tissues to shift into the capillaries and increase the blood volume. This process is best described as which of the following? A) Hydrostatic pressure B) Osmosis and osmolality C) Diffusion D) Active transport

B Osmosis is the movement of fluid from a region of low solute concentration to a region of high solute concentration across a semipermeable membrane. Hydrostatic pressure refers to changes in water or volume related to water pressure. Diffusion is the movement of solutes from an area of greater concentration to lesser concentration; the solutes in an intact vascular system are unable to move so diffusion normally should not be taking place. Active transport is the movement of molecules against the concentration gradient and requires adenosine triphosphate (ATP) as an energy source; this process typically takes place at the cellular level and is not involved in vascular volume changes

A patient has just returned to the unit from the PACU after surgery for a tumor within the spine. The patient complains of pain. When positioning the patient for comfort and to reduce injury to the surgical site, the nurse will position to patient in what position? A) In the high Fowlers position B) In a flat side-lying position C) In the Trendelenberg position D) In the reverse Trendelenberg positio

B After spinal surgery, the bed is usually kept flat initially. The side-lying position is usually the most comfortable because this position imposes the least pressure on the surgical site. The Fowlers position, Trendelenberg position, and reverse Trendelenberg position are inappropriate for this patient because they would result in increased pain and complications.

A patient with severe environmental allergies is scheduled for an immunotherapy injection. What should be included in teaching the patient about this treatment? A) The patient will be given a low dose of epinephrine before the treatment. B) The patient will remain in the clinic to be monitored for 30 minutes following the injection. C) Therapeutic failure occurs if the symptoms to the allergen do not decrease after 3 months. D) The allergen will be administered by the peripheral intravenous route.

B Although severe systemic reactions are rare, the risk of systemic and potentially fatal anaphylaxis exists. Because of this risk, the patient must remain in the office or clinic for at least 30 minutes after the injection and is observed for possible systemic symptoms. Therapeutic failure is evident when a patient does not experience a decrease in symptoms within 12 to 24 months. Epinephrine is not given prior to treatment and the IV route is not used.

A nurse is caring for a patient who has undergone neck resection with a radial forearm free flap. The nurses most recent assessment of the graft reveals that it has a bluish color and that mottling is visible. What is the nurses most appropriate action? A) Document the findings as being consistent with a viable graft. B) Promptly report these indications of venous congestion. C) Closely monitor the patient and reassess in 30 minutes. D) Reposition the patient to promote peripheral circulation.

B Feedback: A graft that is blue with mottling may indicate venous congestion. This finding constitutes a risk for tissue ischemia and necrosis; prompt referral is necessary.

A 6-year-old is admitted to the ED after being rescued from a pond after falling through the ice while ice skating. What action should the nurse perform while rewarming the patient? A) Assessing the patients oral temperature frequently B) Ensuring continuous ECG monitoring C) Massaging the patients skin surfaces to promote circulation D) Administering bronchodilators by nebulizer

B Feedback: A hypothermic patient requires continuous ECG monitoring and assessment of core temperatures with an esophageal probe, bladder, or rectal thermometer. Massage is not performed and bronchodilators would normally be insufficient to meet the patients respiratory needs.

A patient on the critical care unit is postoperative day 1 following kidney transplantation from a living donor. The nurses most recent assessments indicate that the patient is producing copious quantities of dilute urine. What is the nurses most appropriate response? A) Assess the patient for further signs or symptoms of rejection. B) Recognize this as an expected finding. C) Inform the primary care provider of this finding. D) Administer exogenous antidiuretic hormone as ordered.

B Feedback: A kidney from a living donor related to the patient usually begins to function immediately after surgery and may produce large quantities of dilute urine. This is not suggestive of rejection and treatment is not warranted. There is no obvious need to report this finding.

The nurse is participating in the care of a patient with increased ICP. What diagnostic test is contraindicated in this patients treatment? A) Computed tomography (CT) scan B) Lumbar puncture C) Magnetic resonance imaging (MRI) D) Venous Doppler studies

B Feedback: A lumbar puncture in a patient with increased ICP may cause the brain to herniate from the withdrawal of fluid and change in pressure during the lumbar puncture. Herniation of the brain is a dire and frequently fatal event. CT, MRI, and venous Doppler are considered noninvasive procedures and they would not affect the ICP itself

A nurse is caring for a patient who experiences debilitating cluster headaches. The patient should be taught to take appropriate medications at what point in the course of the onset of a new headache? A) As soon as the patients pain becomes unbearable B) As soon as the patient senses the onset of symptoms C) Twenty to 30 minutes after the onset of symptoms D) When the patient senses his or her symptoms peaking

B Feedback: A migraine or a cluster headache in the early phase requires abortive medication therapy instituted as soon as possible. Delaying medication administration would lead to unnecessary pain.

A patient is receiving a transfusion of packed red blood cells. Shortly after initiation of the transfusion, the patient begins to exhibit signs and symptoms of a transfusion reaction. The patient is suffering from which type of hypersensitivity? A) Anaphylactic (type 1) B) Cytotoxic (type II) C) Immunecomplex (type III) D) Delayed type (type IV)

B Feedback: A type II hypersensitivity reaction resulting in red blood cell destruction is associated with blood transfusions. This type of reaction does not result from types I, III, or IV reactions.

A patient with a diagnosis of esophageal varices has undergone endoscopy to gauge the progression of this complication of liver disease. Following the completion of this diagnostic test, what nursing intervention should the nurse perform? A) Keep patient NPO until the results of test are known. B) Keep patient NPO until the patients gag reflex returns. C) Administer analgesia until post-procedure tenderness is relieved. D) Give the patient a cold beverage to promote swallowing ability.

B Feedback: After the examination, fluids are not given until the patients gag reflex returns. Lozenges and gargles may be used to relieve throat discomfort if the patients physical condition and mental status permit. The result of the test is known immediately. Food and fluids are contraindicated until the gag reflex returns.

A clinic nurse is caring for a patient diagnosed with migraine headaches. During the patient teaching session, the patient questions the nurse regarding alcohol consumption. What would the nurse be correct in telling the patient about the effects of alcohol? A) Alcohol causes hormone fluctuations. B) Alcohol causes vasodilation of the blood vessels. C) Alcohol has an excitatory effect on the CNS. D) Alcohol diminishes endorphins in the brain.

B Feedback: Alcohol causes vasodilation of the blood vessels and may exacerbate migraine headaches. Alcohol has a depressant effect on the CNS. Alcohol does not cause hormone fluctuations, nor does it decrease endorphins (morphine-like substances produced by the body) in the brain.

A child has been diagnosed with a severe walnut allergy after suffering an anaphylactic reaction. What is a priority for health education? A) The need to begin immunotherapy as soon as possible B) The need for the parents to carry an epinephrine pen C) The need to vigilantly maintain the childs immunization status D) The need for the child to avoid all foods that have a high potential for allergies

B Feedback: All patients with food allergies, especially seafood and nuts, should have an EpiPen device prescribed. The child does not necessarily need to avoid all common food allergens. Immunotherapy is not indicated in the treatment of childhood food allergies. Immunizations are important, but do not address food allergies.

When caring for a patient who had a hemorrhagic stroke, close monitoring of vital signs and neurologic changes is imperative. What is the earliest sign of deterioration in a patient with a hemorrhagic stroke of which the nurse should be aware? A) Generalized pain B) Alteration in level of consciousness (LOC) C) Tonicclonic seizures D) Shortness of breath

B Feedback: Alteration in LOC is the earliest sign of deterioration in a patient after a hemorrhagic stroke, such as mild drowsiness, slight slurring of speech, and sluggish papillary reaction. Sudden headache may occur, but generalized pain is less common. Seizures and shortness of breath are not identified as early signs of hemorrhagic stroke.

A 71-year-old patient with ESKD has been told by the physician that it is time to consider hemodialysis until a transplant can be found. The patient tells the nurse she is not sure she wants to undergo a kidney transplant. What would be an appropriate response for the nurse to make? A) The decision is certainly yours to make, but be sure not to make a mistake. B) Kidney transplants in patients your age are as successful as they are in younger patients. C) I understand your hesitancy to commit to a transplant surgery. Success is comparatively rare. D) Have you talked this over with your family?

B Feedback: Although there is no specific age limitation for renal transplantation, concomitant disorders (e.g., coronary artery disease, peripheral vascular disease) have made it a less common treatment for the elderly. However, the outcome is comparable to that of younger patients. The other listed options either belittle the patient or give the patient misinformation.

A patient is brought to the emergency department (ED) in a state of anaphylaxis. What is the ED nurses priority for care? A) Monitor the patients level of consciousness. B) Protect the patients airway. C) Provide psychosocial support. D) Administer medications as ordered.

B Feedback: Anaphylaxis severely threatens a patients airway; the nurses priority is preserving airway patency and breathing pattern. This is a higher priority than other valid aspects of care, including medication administration, psychosocial support, and assessment of LOC.

A patients electronic health record notes that he has previously undergone treatment for secondary polycythemia. How should this aspect of the patients history guide the nurses subsequent assessment? A) The nurse should assess for recent blood donation. B) The nurse should assess for evidence of lung disease. C) The nurse should assess for a history of venous thromboembolism. D) The nurse should assess the patient for impaired renal function.

B Feedback: Any reduction in oxygenation, such as lung disease, can cause secondary polycythemia. Blood donation does not precipitate this problem and impaired renal function typically causes anemia, not polycythemia. A history of VTE is not a likely contributor.

The staff educator is presenting a workshop on valvular disorders. When discussing the pathophysiology of aortic regurgitation the educator points out the need to emphasize that aortic regurgitation causes what? A) Cardiac tamponade B) Left ventricular hypertrophy C) Right-sided heart failure D) Ventricular insufficiency

B Feedback: Aortic regurgitation eventually causes left ventricular hypertrophy. In aortic regurgitation, blood from the aorta returns to the left ventricle during diastole in addition to the blood normally delivered by the left atrium. The left ventricle dilates, trying to accommodate the increased volume of blood. Aortic regurgitation does not cause cardiac tamponade, right-sided heart failure, or ventricular insufficiency.

A nurse is caring for a critically ill patient with autonomic dysreflexia. What clinical manifestations would the nurse expect in this patient? A) Respiratory distress and projectile vomiting B) Bradycardia and hypertension C) Tachycardia and agitation D) Third-spacing and hyperthermia

B Feedback: Autonomic dysreflexia is characterized by a pounding headache, profuse sweating, nasal congestion, piloerection (goose bumps), bradycardia, and hypertension. It occurs in cord lesions above T6 after spinal shock has resolved; it does not result in vomiting, tachycardia, or third-spacing.

Patients who are enrolled in hospice care through Medicare are often felt to suffer unnecessarily because they do not receive adequate attention for their symptoms of the underlying illness. What factor most contributes to this phenomenon? A) Unwillingness to overmedicate the dying patient B) Rules concerning completion of all cure-focused medical treatment C) Unwillingness of patients and families to acknowledge the patient is terminal D) Lack of knowledge of patients and families regarding availability of care

B Feedback: Because of Medicare rules concerning completion of all cure-focused medical treatment before the Medicare hospice benefit may be accessed, many patients delay enrollment in hospice programs until very close to the end of life. Hospice care does not include an unwillingness to medicate the patient to keep him or her from suffering. Patients must accept that they are terminal before being admitted to hospice care. Lack of knowledge is common; however, this is not why some Medicare patients do not receive adequate attention for the symptoms of their underlying illness.

A patient presents to the ED complaining of increasing shortness of breath. The nurse assessing the patient notes a history of left-sided HF. The patient is agitated and occasionally coughing up pink-tinged, foamy sputum. The nurse should recognize the signs and symptoms of what health problem? A) Right-sided heart failure B) Acute pulmonary edema C) Pneumonia D) Cardiogenic shock

B Feedback: Because of decreased contractility and increased fluid volume and pressure in patients with HF, fluid may be driven from the pulmonary capillary beds into the alveoli, causing pulmonary edema and signs and symptoms described. In right-sided heart failure, the patient exhibits hepatomegaly, jugular vein distention, and peripheral edema. In pneumonia, the patient would have a temperature spike, and sputum that varies in color. Cardiogenic shock would show signs of hypotension and tachycardia.

A nurse is preparing health education for a patient who has received a diagnosis of myelodysplastic syndrome (MDS). Which of the following topics should the nurse prioritize? A) Techniques for energy conservation and activity management B) Emergency management of bleeding episodes C) Technique for the administration of bronchodilators by metered-dose inhaler D) Techniques for self-palpation of the lymph nodes

B Feedback: Because of patients risks of hemorrhage, patients with MDS should be taught techniques for managing emergent bleeding episodes. Bronchodilators are not indicated for the treatment of MDS and lymphedema is not normally associated with the disease. Energy conservation techniques are likely to be useful, but management of hemorrhage is a priority because of the potential consequences.

The nurse is caring for a patient recovering from an ischemic stroke. What intervention best addresses a potential complication after an ischemic stroke? A) Providing frequent small meals rather than three larger meals B) Teaching the patient to perform deep breathing and coughing exercises C) Keeping a urinary catheter in situ for the full duration of recovery D) Limiting intake of insoluble fiber

B Feedback: Because pneumonia is a potential complication of stroke, deep breathing and coughing exercises should be encouraged unless contraindicated. No particular need exists to provide frequent meals and normally fiber intake should not be restricted. Urinary catheters should be discontinued as soon as possible.

A patient has returned to the cardiac care unit after having a permanent pacemaker implantation. For which potential complication should the nurse most closely assess this patient? A) Chest pain B) Bleeding at the implantation site C) Malignant hyperthermia D) Bradycardia

B Feedback: Bleeding, hematomas, local infections, perforation of the myocardium, and tachycardia are complications of pacemaker implantations. The nurse should monitor for chest pain and bradycardia, but bleeding is a more common immediate complication. Malignant hyperthermia is unlikely because it is a response to anesthesia administration.

A 13-year-old is being admitted to the ED after falling from a roof and sustaining blunt abdominal injuries. To assess for internal injury in the patients peritoneum, the nurse should anticipate what diagnostic test? A) Radiograph B) Computed tomography (CT) scan C) Complete blood count (CBC) D) Barium swallow

B Feedback: CT scan of the abdomen, diagnostic peritoneal lavage, and abdominal ultrasound are appropriate diagnostic tools to assess intra-abdominal injuries. X-rays do not yield sufficient data and a CBC would not reveal the presence of intraperitoneal injury.

A preceptor is discussing stroke with a new nurse on the unit. The preceptor would tell the new nurse which cardiac dysrhythmia is associated with cardiogenic embolic strokes? A) Ventricular tachycardia B) Atrial fibrillation C) Supraventricular tachycardia D) Bundle branch block

B Feedback: Cardiogenic embolic strokes are associated with cardiac dysrhythmias, usually atrial fibrillation. The other listed dysrhythmias are less commonly associated with this type of stroke.

The nurse overseeing care in the ICU reviews the shift report on four patients. The nurse recognizes which patient to be at greatest risk for the development of cardiogenic shock? A) The patient admitted with acute renal failure B) The patient admitted following an MI C) The patient admitted with malignant hypertension D) The patient admitted following a strok

B Feedback: Cardiogenic shock may occur following an MI when a large area of the myocardium becomes ischemic, necrotic, and hypokinetic. It also can occur as a result of end-stage heart failure, cardiac tamponade, pulmonary embolism, cardiomyopathy, and dysrhythmias. While patients with acute renal failure are at risk for dysrhythmias and patients experiencing a stroke are at risk for thrombus formation, the patient admitted following an MI is at the greatest risk for development of cardiogenic shock when compared with the other listed diagnoses.

A patient is scheduled for catheter ablation therapy. When describing this procedure to the patients family, the nurse should address what aspect of the treatment? A) Resetting of the hearts contractility B) Destruction of specific cardiac cells C) Correction of structural cardiac abnormalities D) Clearance of partially occluded coronary arteries

B Feedback: Catheter ablation destroys specific cells that are the cause or central conduction route of a tachydysrhythmia. It does not reset the hearts contractility and it does not address structural or vascular abnormalities.

A patient has been admitted to the ICU after being recently diagnosed with an aneurysm and the patients admission orders include specific aneurysm precautions. What nursing action will the nurse incorporate into the patients plan of care? A) Elevate the head of the bed to 45 degrees. B) Maintain the patient on complete bed rest. C) Administer enemas when the patient is constipated. D) Avoid use of thigh-high elastic compression stockings.

B Feedback: Cerebral aneurysm precautions are implemented for the patient with a diagnosis of aneurysm to provide a nonstimulating environment, prevent increases in ICP, and prevent further bleeding. The patient is placed on immediate and absolute bed rest in a quiet, nonstressful environment because activity, pain, and anxiety elevate BP, which increases the risk for bleeding. Visitors, except for family, are restricted. The head of the bed is elevated 15 to 30 degrees to promote venous drainage and decrease ICP. Some neurologists, however, prefer that the patient remains flat to increase cerebral perfusion. No enemas are permitted, but stool softeners and mild laxatives are prescribed. Thigh-high elastic compression stockings or sequential compression boots may be ordered to decrease the patients risk for deep vein thrombosis (DVT).

The nurse is reviewing the echocardiography results of a patient who has just been diagnosed with dilated cardiomyopathy (DCM). What changes in heart structure characterize DCM? A) Dilated ventricles with atrophy of the ventricles B) Dilated ventricles without hypertrophy of the ventricles C) Dilation and hypertrophy of all four heart chambers D) Dilation of the atria and hypertrophy of the ventricles

B Feedback: DCM is characterized by significant dilation of the ventricles without significant concomitant hypertrophy and systolic dysfunction. The ventricles do not atrophy in patients with DCM.

A 44-year-old male patient has been exposed to severe amount of radiation after a leak in a reactor plant. When planning this patients care, the nurse should implement what action? A) The patient should be scrubbed with alcohol and iodine. B) The patient should be carefully protected from infection. C) The patients immunization status should be promptly assessed. D) The patients body hair should be removed to prevent secondary contamination.

B Feedback: Damage to the hematopoietic system following radiation exposure creates a serious risk for infection. There is no need to remove the patients hair and the patients immunization status is not significant. Alcohol and iodine are ineffective against radiation

Which assessment would be most appropriate for a patient who is receiving a loop diuretic for HF? A) Monitor liver function studies B) Monitor for hypotension C) Assess the patients vitamin D intake D) Assess the patient for hyperkalemia

B Feedback: Diuretic therapy increases urine output and decreases blood volume, which places the patient at risk of hypotension. Patients are at risk of losing potassium with loop diuretic therapy and need to continue with potassium in their diet; hypokalemia is a consequent risk. Liver function is rarely compromised by diuretic therapy and vitamin D intake is not relevant.

A patient has recently begun mobilizing during the recovery from an ischemic stroke. To protect the patients safety during mobilization, the nurse should perform what action? A) Support the patients full body weight with a waist belt during ambulation. B) Have a colleague follow the patient closely with a wheelchair. C) Avoid mobilizing the patient in the early morning or late evening. D) Ensure that the patients family members do not participate in mobilization.

B Feedback: During mobilization, a chair or wheelchair should be readily available in case the patient suddenly becomes fatigued or feels dizzy. The family should be encouraged to participate, as appropriate, and the nurse should not have to support the patients full body weight. Morning and evening activity are not necessarily problematic.

A nurse is caring for a patient diagnosed with a hemorrhagic stroke. When creating this patients plan of care, what goal should be prioritized? A) Prevent complications of immobility. B) Maintain and improve cerebral tissue perfusion. C) Relieve anxiety and pain. D) Relieve sensory deprivation.

B Feedback: Each of the listed goals is appropriate in the care of a patient recovering from a stroke. However, promoting cerebral perfusion is a priority physiologic need, on which the patients survival depends

The nurse is coordinating the care of victims who arrive at the ED after a radiation leak at a nearby nuclear plant. What would be the first intervention initiated when victims arrive at the hospital? A) Administer prophylactic antibiotics. B) Survey the victims using a radiation survey meter. C) Irrigate victims open wounds. D) Perform soap and water decontamination.

B Feedback: Each patient arriving at the hospital should first be surveyed with the radiation survey meter for external contamination and then directed toward the decontamination area as needed. This survey should precede decontamination efforts or irrigation of wounds. Antibiotics are not indicated.

When caring for a patient who has had a stroke, a priority is reduction of ICP. What patient position is most consistent with this goal? A) Head turned slightly to the right side B) Elevation of the head of the bed C) Position changes every 15 minutes while awake D) Extension of the neck

B Feedback: Elevation of the head of the bed promotes venous drainage and lowers ICP; the nurse should avoid flexing or extending the neck or turning the head side to side. The head should be in a neutral midline position. Excessively frequent position changes are unnecessary.

A nurse is caring for a patient who has been admitted for the treatment of advanced cirrhosis. What assessment should the nurse prioritize in this patients plan of care? A) Measurement of abdominal girth and body weight B) Assessment for variceal bleeding C) Assessment for signs and symptoms of jaundice D) Monitoring of results of liver function testing

B Feedback: Esophageal varices are a major cause of mortality in patients with uncompensated cirrhosis. Consequently, this should be a focus of the nurses assessments and should be prioritized over the other listed assessments, even though each should be performed.

A patient with a history of major depression is brought to the ED by her parents. Which of the following nursing actions is most appropriate? A) Noting that symptoms of physical illness are not relevant to the current diagnosis B) Asking the patient if she has ever thought about taking her own life C) Conducting interviews in a brief and direct manner D) Arranging for the patient to spend time alone to consider her feelings

B Feedback: Establishing if the patient has suicidal thoughts or intents helps identify the level of depression and intervention. Physical symptoms are relevant and should be explored. Allow the patient to express feelings, and conduct the interview at a comfortable pace for the patient. Never leave the patient alone, because suicide is usually committed in soli

A patient with a documented history of glucose-6-phosphate dehydrogenase deficiency has presented to the emergency department with signs and symptoms including pallor, jaundice, and malaise. Which of the nurses assessment questions relates most directly to this patients hematologic disorder? A) When did you last have a blood transfusion? B) What medications have taken recently? C) Have you been under significant stress lately? D) Have you suffered any recent injuries?

B Feedback: Exacerbations of glucose-6-phosphate dehydrogenase deficiency are nearly always precipitated by medications. Blood transfusions, stress, and injury are less common triggers.

The nurse is caring for a patient who has just returned from the ERCP removal of gallstones. The nurse should monitor the patient for signs of what complications? A) Pain and peritonitis B) Bleeding and perforation C) Acidosis and hypoglycemia D) Gangrene of the gallbladder and hyperglycemia

B Feedback: Following ERCP removal of gallstones, the patient is observed closely for bleeding, perforation, and the development of pancreatitis or sepsis. Blood sugar alterations, gangrene, peritonitis, and acidosis are less likely complications.

When planning the care of a patient with an implanted pacemaker, what assessment should the nurse prioritize? A) Core body temperature B) Heart rate and rhythm C) Blood pressure D) Oxygen saturation level

B Feedback: For patients with pacemakers, close monitoring of the heart rate and rhythm is a priority, even though each of the other listed vital signs must be assessed.

A patient diagnosed with a pituitary adenoma has arrived on the neurologic unit. When planning the patients care, the nurse should be aware that the effects of the tumor will primarily depend on what variable? A) Whether the tumor utilizes aerobic or anaerobic respiration B) The specific hormones secreted by the tumor C) The patients pre-existing health status D) Whether the tumor is primary or the result of metastasis

B Feedback: Functioning pituitary tumors can produce one or more hormones normally produced by the anterior pituitary and the effects of the tumor depend largely on the identity of these hormones. This variable is more significant than the patients health status or whether the tumor is primary versus secondary. Anaerobic and aerobic respiration is not relevant.

A patient with angina has been prescribed nitroglycerin. Before administering the drug, the nurse should inform the patient about what potential adverse effects? A) Nervousness or paresthesia B) Throbbing headache or dizziness C) Drowsiness or blurred vision D) Tinnitus or diplopia

B Feedback: Headache and dizziness commonly occur when nitroglycerin is taken at the beginning of therapy. Nervousness, paresthesia, drowsiness, blurred vision, tinnitus, and diplopia do not typically occur as a result of nitroglycerin therapy.

The nursing educator is reviewing the signs and symptoms of heat stroke with a group of nurses who provide care in a desert region. The educator should describe what sign or symptom? A) Hypertension with a wide pulse pressure B) Anhidrosis C) Copious diuresis D) Cheyne-Stokes respirations

B Feedback: Heat stroke is manifested by anhidrosis confusion, bizarre behavior, coma, elevated body temperature, hot dry skin, tachypnea, hypotension, and tachycardia. This health problem is not associated with anhidrosis or Cheyne-Stokes respirations.

The nurses comprehensive assessment of a patient who has HF includes evaluation of the patients hepatojugular reflux. What action should the nurse perform during this assessment? A) Elevate the patients head to 90 degrees. B) Press the right upper abdomen. C) Press above the patients symphysis pubis. D) Lay the patient flat in bed.

B Feedback: Hepatojugular reflux, a sign of right-sided heart failure, is assessed with the head of the bed at a 45- degree angle. As the right upper abdomen (the area over the liver) is compressed for 30 to 40 seconds, the nurse observes the internal jugular vein. If the internal jugular vein becomes distended, a patient has positive hepatojugular reflux.

The nurse in an allergy clinic is educating a new patient about the pathology of the patients health problem. What response should the nurse describe as a possible consequence of histamine release? A) Constriction of small venules B) Contraction of bronchial smooth muscle C) Dilation of large blood vessels D) Decreased secretions from gastric and mucosal cells

B Feedback: Histamines effects during the immune response include contraction of bronchial smooth muscle, resulting in wheezing and bronchospasm, dilation of small venules, constriction of large blood vessels, and an increase in secretion of gastric and mucosal cell

An 11-year-old boy has been brought to the ED by his teacher, who reports that the boy may be having a really bad allergic reaction to peanuts after trading lunches with a peer. The triage nurses rapid assessment reveals the presence of respiratory and cardiac arrest. What interventions should the nurse prioritize? A) Establishing central venous access and beginning fluid resuscitation B) Establishing a patent airway and beginning cardiopulmonary resuscitation C) Establishing peripheral IV access and administering IV epinephrine D) Performing a comprehensive assessment and initiating rapid fluid replacement

B Feedback: If cardiac arrest and respiratory arrest are imminent or have occurred, CPR is performed. As well, a patent airway is an immediate priority. Epinephrine is not withheld pending IV access and fluid resuscitation is not a priority.

A patient is brought to the trauma center by ambulance after sustaining a high cervical spinal cord injury 1 hours ago. Endotracheal intubation has been deemed necessary and the nurse is preparing to assist. What nursing diagnosis should the nurse associate with this procedure? A) Risk for impaired skin integrity B) Risk for injury C) Risk for autonomic dysreflexia D) Risk for suffocation

B Feedback: If endotracheal intubation is necessary, extreme care is taken to avoid flexing or extending the patients neck, which can result in extension of a cervical injury. Intubation does not directly cause autonomic dysreflexia and the threat to skin integrity is a not a primary concern. Intubation does not carry the potential to cause suffocation.

Following a spinal cord injury a patient is placed in halo traction. While performing pin site care, the nurse notes that one of the traction pins has become detached. The nurse would be correct in implementing what priority nursing action? A) Complete the pin site care to decrease risk of infection. B) Notify the neurosurgeon of the occurrence. C) Stabilize the head in a lateral position. D) Reattach the pin to prevent further head trauma.

B Feedback: If one of the pins became detached, the head is stabilized in neutral position by one person while another notifies the neurosurgeon. Reattaching the pin as a nursing intervention would not be done due to risk of increased injury. Pin site care would not be a priority in this instance. Prevention of neurologic injury is the priority.

A patient has had a laparoscopic cholecystectomy. The patient is now complaining of right shoulder pain. What should the nurse suggest to relieve the pain? A) Aspirin every 4 to 6 hours as ordered B) Application of heat 15 to 20 minutes each hour C) Application of an ice pack for no more than 15 minutes D) Application of liniment rub to affected area

B Feedback: If pain occurs in the right shoulder or scapular area (from migration of the CO2 used to insufflate the abdominal cavity during the procedure), the nurse may recommend use of a heating pad for 15 to 20 minutes hourly, walking, and sitting up when in bed. Aspirin would constitute a risk for bleeding.

A nurse is caring for a patient with a blocked bile duct from a tumor. What manifestation of obstructive jaundice should the nurse anticipate? A) Watery, blood-streaked diarrhea B) Orange and foamy urine C) Increased abdominal girth D) Decreased cognition

B Feedback: If the bile duct is obstructed, the bile will be reabsorbed into the blood and carried throughout the entire body. It is excreted in the urine, which becomes deep orange and foamy. Bloody diarrhea, ascites, and cognitive changes are not associated with obstructive jaundice.

A patient is being treated for AKI and the patient daily weights have been ordered. The nurse notes a weight gain of 3 pounds over the past 48 hours. What nursing diagnosis is suggested by this assessment finding? A) Imbalanced nutrition: More than body requirements B) Excess fluid volume C) Sedentary lifestyle D) Adult failure to thrive

B Feedback: If the patient with AKI gains or does not lose weight, fluid retention should be suspected. Short-term weight gain is not associated with excessive caloric intake or a sedentary lifestyle. Failure to thrive is not associated with weight gain.

A patient with chronic kidney disease is completing an exchange during peritoneal dialysis. The nurse observes that the peritoneal fluid is draining slowly and that the patients abdomen is increasing in girth. What is the nurses most appropriate action? A) Advance the catheter 2 to 4 cm further into the peritoneal cavity. B) Reposition the patient to facilitate drainage. C) Aspirate from the catheter using a 60-mL syringe. D) Infuse 50 mL of additional dialysate.

B Feedback: If the peritoneal fluid does not drain properly, the nurse can facilitate drainage by turning the patient from side to side or raising the head of the bed. The catheter should never be pushed further into the peritoneal cavity. It would be unsafe to aspirate or to infuse more dialysate.

The nurse in an extended care facility is planning the daily activities of a patient with postpolio syndrome. The nurse recognizes the patient will best benefit from physical therapy when it is scheduled at what time? A) Immediately after meals B) In the morning C) Before bedtime D) In the early evening

B Feedback: Important activities for patients with postpolio syndrome should be planned for the morning, as fatigue often increases in the afternoon and evening.

You are caring for a patient, a 42-year-old mother of two children, with a diagnosis of ovarian cancer. She has just been told that her ovarian cancer is terminal. When you admitted this patient, you did a spiritual assessment. What question would it have been most important for you to evaluate during this assessment? A) Is she able to tell her family of negative test results? B) Does she have a sense of peace of mind and a purpose to her life? C) Can she let go of her husband so he can make a new life? D) Does she need time and space to bargain with God for a cure?

B Feedback: In addition to assessment of the role of religious faith and practices, important religious rituals, and connection to a religious community, you should further explore the presence or absence of a sense of peace of mind and purpose in life; other sources of meaning, hope, and comfort; and spiritual or religious beliefs about illness, medical treatment, and care of the sick. Telling her family and letting her husband go are not parts of a spiritual assessment. Bargaining is a stage of death and dying, not part of a spiritual assessment.

A patient with a history of type 1 diabetes has just been admitted to the critical care unit (CCU) for diabetic ketoacidosis. The CCU nurse should prioritize what assessment during the patients initial phase of treatment? A) Monitoring the patient for dysrhythmias B) Maintaining and monitoring the patients fluid balance C) Assessing the patients level of consciousness D) Assessing the patient for signs and symptoms of venous thromboembolism

B Feedback: In addition to treating hyperglycemia, management of DKA is aimed at correcting dehydration, electrolyte loss, and acidosis before correcting the hyperglycemia with insulin. The nurse should monitor the patient for dysrhythmias, decreased LOC and VTE, but restoration and maintenance of fluid balance is the highest priority.

A nurse is undergoing debriefing with the critical incident stress management (CISM) team after participating in the response to a disaster. During this process, the nurse will do which of the following? A) Evaluate the care that he or she provided during the disaster. B) Discuss own emotional responses to the disaster. C) Explore the ethics of the care provided during the disaster. D) Provide suggestions for improving the emergency operations plan.

B Feedback: In debriefing, participants are asked about their emotional reactions to the incident, what symptoms they may be experiencing (e.g., flashbacks, difficulty sleeping, intrusive thoughts), and other psychological ramifications. The EOP and the care the nurse provided are not evaluated.

The ED nurse is caring for a patient who has gone into cardiac arrest. During external defibrillation, what action should the nurse perform? A) Place gel pads over the apex and posterior chest for better conduction. B) Ensure no one is touching the patient at the time shock is delivered. C) Continue to ventilate the patient via endotracheal tube during the procedure. D) Allow at least 3 minutes between shocks.

B Feedback: In external defibrillation, both paddles may be placed on the front of the chest, which is the standard paddle placement. Whether using pads, or paddles, the nurse must observe two safety measures. First, maintain good contact between the pads or paddles and the patients skin to prevent leaking. Second, ensure that no one is in contact with the patient or with anything that is touching the patient when the defibrillator is discharged, to minimize the chance that electrical current will be conducted to anyone other than the patient. Ventilation should be stopped during defibrillation.

The nurse is caring for a patient with mitral stenosis who is scheduled for a balloon valvuloplasty. The patient tells the nurse that he is unsure why the surgeon did not opt to replace his damaged valve rather than repairing it. What is an advantage of valvuloplasty that the nurse should cite? A) The procedure can be performed on an outpatient basis in a physicians office. B) Repaired valves tend to function longer than replaced valves. C) The procedure is not associated with a risk for infection. D) Lower doses of antirejection drugs are required than with valve replacement.

B Feedback: In general, valves that undergo valvuloplasty function longer than prosthetic valve replacements and patients do not require continuous anticoagulation. Valvuloplasty carries a risk of infection, like all surgical procedures, and it is not performed in a physicians office. Antirejection drugs are unnecessary because foreign tissue is not introduced.

A nurse is triaging patients after a chemical leak at a nearby fertilizer factory. The guiding principle of this activity is what? A) Assigning a high priority to the most critical injuries B) Doing the greatest good for the greatest number of people C) Allocating resources to the youngest and most critical D) Allocating resources on a first come, first served basis

B Feedback: In nondisaster situations, health care workers assign a high priority and allocate the most resources to those who are the most critically ill. However, in a disaster, when health care providers are faced with a large number of casualties, the fundamental principle guiding resource allocation is to do the greatest good for the greatest number of people. A first come, first served approach is unethical.

While completing a health history on a patient who has recently experienced a seizure, the nurse would assess for what characteristic associated with the postictal state? A) Epileptic cry B) Confusion C) Urinary incontinence D) Body rigidity

B Feedback: In the postictal state (after the seizure), the patient is often confused and hard to arouse and may sleep for hours. The epileptic cry occurs from the simultaneous contractions of the diaphragm and chest muscles that occur during the seizure. Urinary incontinence and intense rigidity of the entire body are followed by alternating muscle relaxation and contraction (generalized tonicclonic contraction) during the seizure

A nursing student is caring for a patient with acute myeloid leukemia who is preparing to undergo induction therapy. In preparing a plan of care for this patient, the student should assign the highest priority to which nursing diagnoses? A) Activity Intolerance B) Risk for Infection C) Acute Confusion D) Risk for Spiritual Distress

B Feedback: Induction therapy places the patient at risk for infection, thus this is the priority nursing diagnosis. During the time of induction therapy, the patient is very ill, with bacterial, fungal, and occasional viral infections; bleeding and severe mucositis, which causes diarrhea; and marked decline in the ability to maintain adequate nutrition. Supportive care consists of administering blood products and promptly treating infections. Immobility, confusion, and spiritual distress are possible, but infection is the patients most acute physiologic threat.

A night nurse is reviewing the next days medication administration record (MAR) of a patient who has hemophilia. The nurse notes that the MAR specifies both oral and subcutaneous options for the administration of a PRN antiemetic. What is the nurses best action? A) Ensure that the day nurse knows not to give the antiemetic. B) Contact the prescriber to have the subcutaneous option discontinued. C) Reassess the patients need for antiemetics. D) Remove the subcutaneous route from the patients MAR.

B Feedback: Injections must be avoided in patients with hemophilia. Consequently, the nurse should ensure that the prescriber makes the necessary change. The nurse cannot independently make a change to a patients MAR in most cases. Facilitating the necessary change is preferable to deferring to the day nurse.

A patient with spinal cord injury has a nursing diagnosis of altered mobility and the nurse recognizes the increased the risk of deep vein thrombosis (DVT). Which of the following would be included as an appropriate nursing intervention to prevent a DVT from occurring? A) Placing the patient on a fluid restriction as ordered B) Applying thigh-high elastic stockings C) Administering an antifibrinolyic agent D) Assisting the patient with passive range of motion (PROM) exercises

B Feedback: It is important to promote venous return to the heart and prevent venous stasis in a patient with altered mobility. Applying elastic stockings will aid in the prevention of a DVT. The patient should not be placed on fluid restriction because a dehydrated state will increase the risk of clotting throughout the body. Antifibrinolytic agents cause the blood to clot, which is absolutely contraindicated in this situation. PROM exercises are not an effective protection against the development of DVT.

A patient exhibiting an altered level of consciousness (LOC) due to blunt-force trauma to the head is admitted to the ED. The physician determines the patients injury is causing increased intracranial pressure (ICP). The nurse should gauge the patients LOC on the results of what diagnostic tool? A) Monro-Kellie hypothesis B) Glasgow Coma Scale C) Cranial nerve function D) Mental status examination

B Feedback: LOC, a sensitive indicator of neurologic function, is assessed based on the criteria in the Glasgow Coma Scale: eye opening, verbal response, and motor response. The Monro-Kellie hypothesis states that because of the limited space for expansion within the skull, an increase in any one of the components (blood, brain tissue, cerebrospinal fluid) causes a change in the volume of the others. Cranial nerve function and the mental status examination would be part of the neurologic examination for this patient, but would not be the priority in evaluating LOC.

A nurse is caring for patients exposed to a terrorist attack involving chemicals. The nurse has been advised that personal protective equipment must be worn in order to give the highest level of respiratory protection with a lesser level of skin and eye protection. What level protection is this considered? A) Level A B) Level B C) Level C D) Level

B Feedback: Level B personal protective equipment provides the highest level of respiratory protection, with a lesser level of skin and eye protection. Level A provides the highest level of respiratory, mucous membrane, skin, and eye protection. Level C incorporates the use of an air-purified respirator, a chemical resistant coverall with splash hood, chemical resistant gloves, and boots. Level D is the same as a work uniform.

The nurse manager in the ED receives information that a local chemical plant has had a chemical leak. This disaster is assigned a status of level II. What does this classification indicate? A) First responders can manage the situation. B) Regional efforts and aid from surrounding communities can manage the situation. C) Statewide or federal assistance is required. D) The area must be evacuated immediately.

B Feedback: Level II disasters indicate that regional efforts and aid from the surrounding communities will be able to manage the situation. Local efforts are likely to be overwhelmed, while state and federal assistance are not likely necessary. The disaster level does not indicate the necessity of evacuation.

A patient has experienced an electrical burn and has developed thick eschar over the burn site. Which of the following topical antibacterial agents will the nurse expect the physician to order for the wound? A) Silver sulfadiazine 1% (Silvadene) water-soluble cream B) Mafenide acetate 10% (Sulfamylon) hydrophilic-based cream C) Silver nitrate 0.5% aqueous solution D) Acticoat

B Feedback: Mafenide acetate 10% hydrophilic-based cream is the agent of choice when there is a need to penetrate thick eschar. Silver products do not penetrate eschar; Acticoat is a type of silver dressing.

The nurse is providing care for a patient who is unconscious. What nursing intervention takes highest priority? A) Maintaining accurate records of intake and output B) Maintaining a patent airway C) Inserting a nasogastric (NG) tube as ordered D) Providing appropriate pain control

B Feedback: Maintaining a patent airway always takes top priority, even though each of the other listed actions is necessary and appropriate.

The nurses comprehensive assessment of a patient includes inspection for signs of oral cancer. What assessment finding is most characteristic of oral cancer in its early stages? A) Dull pain radiating to the ears and teeth B) Presence of a painless sore with raised edges C) Areas of tenderness that make chewing difficult D) Diffuse inflammation of the buccal mucosa

B Feedback: Malignant lesions of the oral cavity are most often painless lumps or sores with raised borders. Because they do not bother the patient, delay in seeking treatment occurs frequently, and negatively affects prognosis. Dull pain radiating to the ears and teeth is characteristic of malocclusion. Inflammation of the buccal mucosa causes discomfort and often occurs as a side effect of chemotherapy. Tenderness resulting in pain on chewing may be associated with gingivitis, abscess, irritation from dentures, and other causes. Pain related to oral cancer is a late symptom

The staff educator is precepting a nurse new to the critical care unit when a patient with a T2 spinal cord injury is admitted. The patient is soon exhibiting manifestations of neurogenic shock. In addition to monitoring the patient closely, what would be the nurses most appropriate action? A) Prepare to transfuse packed red blood cells. B) Prepare for interventions to increase the patients BP. C) Place the patient in the Trendelenberg position. D) Prepare an ice bath to lower core body temperature.

B Feedback: Manifestations of neurogenic shock include decreased BP and heart rate. Cardiac markers would be expected to rise in cardiogenic shock. Transfusion, repositioning, and ice baths are not indicated interventions.

While assisting with the surgical removal of an adrenal tumor, the OR nurse is aware that the patients vital signs may change upon manipulation of the tumor. What vital sign changes would the nurse expect to see? A) Hyperthermia and tachypnea B) Hypertension and heart rate changes C) Hypotension and hypothermia D) Hyperthermia and bradycardia

B Feedback: Manipulation of the tumor during surgical excision may cause release of stored epinephrine and norepinephrine, with marked increases in BP and changes in heart rate. The use of sodium nitroprusside and alpha-adrenergic blocking agents may be required during and after surgery. While other vital sign changes may occur related to surgical complications, the most common changes are related to hypertension and changes in the heart rate.

The nurse is preparing the patient for mechanical dbridement and informs the patient that this will involve which of the following procedures? A) A spontaneous separation of dead tissue from the viable tissue B) Removal of eschar until the point of pain and bleeding occurs C) Shaving of burned skin layers until bleeding, viable tissue is revealed D) Early closure of the woun

B Feedback: Mechanical dbridementcan be achieved through the use of surgical scissors, scalpels, or forceps to remove the eschar until the point of pain and bleeding occurs. Mechanical dbridement can also be accomplished through the use of topical enzymatic dbridement agents. The spontaneous separation of dead tissue from the viable tissue is an example of natural dbridement. Shaving the burned skin layers and early wound closure are examples of surgical dbridement.

The nurse is caring for a patient who is in status epilepticus. What medication does the nurse know may be given to halt the seizure immediately? A) Intravenous phenobarbital (Luminal) B) Intravenous diazepam (Valium) C) Oral lorazepam (Ativan) D) Oral phenytoin (Dilantin)

B Feedback: Medical management of status epilepticus includes IV diazepam (Valium) and IV lorazepam (Ativan) given slowly in an attempt to halt seizures immediately. Other medications (phenytoin, phenobarbital) are given later to maintain a seizure-free state. Oral medications are not given during status epilepticus.

A nurse is caring for a patient who has been scheduled for endoscopic retrograde cholangiopancreatography (ERCP) the following day. When providing anticipatory guidance for this patient, the nurse should describe what aspect of this diagnostic procedure? A) The need to protect the incision postprocedure B) The use of moderate sedation C) The need to infuse 50% dextrose during the procedure D) The use of general anesthesia

B Feedback: Moderate sedation, not general anesthesia, is used during ERCP. D50 is not administered and the procedure does not involve the creation of an incision.

A nurse is caring for a patient with cirrhosis secondary to heavy alcohol use. The nurses most recent assessment reveals subtle changes in the patients cognition and behavior. What is the nurses most appropriate response? A) Ensure that the patients sodium intake does not exceed recommended levels. B) Report this finding to the primary care provider due to the possibility of hepatic encephalopathy. C) Inform the primary care provider that the patient should be assessed for alcoholic hepatitis. D) Implement interventions aimed at ensuring a calm and therapeutic care environment.

B Feedback: Monitoring is an essential nursing function to identify early deterioration in mental status. The nurse monitors the patients mental status closely and reports changes so that treatment of encephalopathy can be initiated promptly. This change in status is likely unrelated to sodium intake and would not signal the onset of hepatitis. A supportive care environment is beneficial, but does not address the patients physiologic deterioration.

A patient who has been taking corticosteroids for several months has been experiencing muscle wasting. The patient has asked the nurse for suggestions to address this adverse effect. What should the nurse recommend? A) Activity limitation to conserve energy B) Consumption of a high-protein diet C) Use of OTC vitamin D and calcium supplements D) Passive range-of-motion exercises

B Feedback: Muscle wasting can be partly addressed through increased protein intake. Passive ROM exercises maintain flexibility, but do not build muscle mass. Vitamin D and calcium supplements do not decrease muscle wasting. Activity limitation would exacerbate the problem.

A patient is admitted to the ED with an apparent overdose of IV heroin. After stabilizing the patients cardiopulmonary status, the nurse should prepare to perform what intervention? A) Administer a bolus of lactated Ringers. B) Administer naloxone hydrochloride (Narcan). C) Insert an indwelling urinary catheter. D) Perform a focused neurologic assessment.

B Feedback: Narcan is an opioid antagonist that is administered for the treatment of narcotic overdoses. There is no definitive need for a urinary catheter or for a bolus of lactated Ringers. The patients basic neurologic status should be ascertained during the rapid assessment, but a detailed examination would be take precedence over administration of an antidote.

A patient who has sustained a nondepressed skull fracture is admitted to the acute medical unit. Nursing care should include which of the following? A) Preparation for emergency craniotomy B) Watchful waiting and close monitoring C) Administration of inotropic drugs D) Fluid resuscitation

B Feedback: Nondepressed skull fractures generally do not require surgical treatment; however, close observation of the patient is essential. A craniotomy would not likely be needed if the fracture is nondepressed. Even if treatment is warranted, it is unlikely to include inotropes or fluid resuscitation.

A patients physician has determined that for the next 3 to 4 weeks the patient will require parenteral nutrition (PN). The nurse should anticipate the placement of what type of venous access device? A) Peripheral catheter B) Nontunneled central catheter C) Implantable port D) Tunneled central catheter

B Feedback: Nontunneled central catheters are used for short-term (less than 6 weeks) IV therapy. A peripheral catheter can be used for the administration of peripheral parenteral nutrition for 5 to 7 days. Implantable ports and tunneled central catheters are for long-term use and may remain in place for many years. Peripherally inserted central catheters (PICCs) are another potential option

During a CPR class, a participant asks about the difference between cardioversion and defibrillation. What would be the instructors best response? A) Cardioversion is done on a beating heart; defibrillation is not. B) The difference is the timing of the delivery of the electric current. C) Defibrillation is synchronized with the electrical activity of the heart, but cardioversion is not. D) Cardioversion is always attempted before defibrillation because it has fewer risks.

B Feedback: One major difference between cardioversion and defibrillation is the timing of the delivery of electrical current. In cardioversion, the delivery of the electrical current is synchronized with the patients electrical events; in defibrillation, the delivery of the current is immediate and unsynchronized. Both can be done on beating heart (i.e., in a dysrhythmia). Cardioversion is not necessarily attempted first.

The nurse is educating an 80-year-old patient diagnosed with HF about his medication regimen. What should the nurse to teach this patient about the use of oral diuretics? A) Avoid drinking fluids for 2 hours after taking the diuretic. B) Take the diuretic in the morning to avoid interfering with sleep. C) Avoid taking the medication within 2 hours consuming dairy products. D) Take the diuretic only on days when experiencing shortness of breath.

B Feedback: Oral diuretics should be administered early in the morning so that diuresis does not interfere with the patients nighttime rest. Discussing the timing of medication administration is especially important for elderly patients who may have urinary urgency or incontinence. The nurse would not teach the patient about the timing of fluid intake. Fluid intake does not need to be adjusted and dairy products are not contraindicated.

The nurse is creating a plan of care for a patient with acute coronary syndrome. What nursing action should be included in the patients care plan? A) Facilitate daily arterial blood gas (ABG) sampling. B) Administer supplementary oxygen, as needed. C) Have patient maintain supine positioning when in bed D) Perform chest physiotherapy, as indicated.

B Feedback: Oxygen should be administered along with medication therapy to assist with symptom relief. Administration of oxygen raises the circulating level of oxygen to reduce pain associated with low levels of myocardial oxygen. Physical rest in bed with the head of the bed elevated or in a supportive chair helps decrease chest discomfort and dyspnea. ABGs are diagnostic, not therapeutic, and they are rarely needed on a daily basis. Chest physiotherapy is not used in the treatment of ACS.

A patient on the medical unit has a documented history of polycystic kidney disease (PKD). What principle should guide the nurses care of this patient? A) The disease is self-limiting and cysts usually resolve spontaneously in the fifth or sixth decade of life. B) The patients disease is incurable and the nurses interventions will be supportive. C) The patient will eventually require surgical removal of his or her renal cysts. D) The patient is likely to respond favorably to lithotripsy treatment of the cysts

B Feedback: PKD is incurable and care focuses on support and symptom control. It is not self-limiting and is not treated surgically or with lithotripsy.

A nurse is initiating parenteral nutrition (PN) to a postoperative patient who has developed complications. The nurse should initiate therapy by performing which of the following actions? A) Starting with a rapid infusion rate to meet the patients nutritional needs as quickly as possible B) Initiating the infusion slowly and monitoring the patients fluid and glucose tolerance C) Changing the rate of administration every 2 hours based on serum electrolyte values D) Increasing the rate of infusion at mealtimes to mimic the circadian rhythm of the body

B Feedback: PN solutions are initiated slowly and advanced gradually each day to the desired rate as the patients fluid and glucose tolerance permits. The formulation of the PN solutions is calculated carefully each day to meet the complete nutritional needs of the individual patient based on clinical findings and laboratory data. It is not infused more quickly at mealtimes.

A patient with chronic pancreatitis had a pancreaticojejunostomy created 3 months ago for relief of pain and to restore drainage of pancreatic secretions. The patient has come to the office for a routine postsurgical appointment. The patient is frustrated that the pain has not decreased. What is the most appropriate initial response by the nurse? A) The majority of patients who have a pancreaticojejunostomy have their normal digestion restored but do not achieve pain relief. B) Pain relief occurs by 6 months in most patients who undergo this procedure, but some people experience a recurrence of their pain. C) Your physician will likely want to discuss the removal of your gallbladder to achieve pain relief. D) You are probably not appropriately taking the medications for your pancreatitis and pain, so we will need to discuss your medication regimen in detail.

B Feedback: Pain relief from a pancreaticojejunostomy often occurs by 6 months in more than 85% of the patients who undergo this procedure, but pain returns in a substantial number of patients as the disease progresses. This patient had surgery 3 months ago; the patient has 3 months before optimal benefits of the procedure may be experienced. There is no obvious indication for gallbladder removal and nonadherence is not the most likely factor underlying the pain.

A patients rapid cancer metastases have prompted a shift from active treatment to palliative care. When planning this patients care, the nurse should identify what primary aim? A) To prioritize emotional needs B) To prevent and relieve suffering C) To bridge between curative care and hospice care D) To provide care while there is still hope

B Feedback: Palliative care, which is conceptually broader than hospice care, is both an approach to care and a structured system for care delivery that aims to prevent and relieve suffering and to support the best possible quality of life for patients and their families, regardless of the stage of the disease or the need for other therapies. Palliative care goes beyond simple prioritization of emotional needs; these are always considered and addressed. Palliative care is considered a bridge, but it is not limited to just hospice care. Hope is something patients and families have even while the patient is actively dying.

A patient who was burned in a workplace accident has completed the acute phase of treatment and the plan of care has been altered to prioritize rehabilitation. What nursing action should be prioritized during this phase of treatment? A) Monitoring fluid and electrolyte imbalances B) Providing education to the patient and family C) Treating infection D) Promoting thermoregulatio

B Feedback: Patient and family education is a priority during rehabilitation. There should be no fluid and electrolyte imbalances in the rehabilitation phase. The presence of impaired thermoregulation or infection would suggest that the patient is still in the acute phase of burn recovery.

A nurse who is taking care of a patient with burns is asked by a family member why the patient is losing so much weight. The patient is currently in the intermediate phase of recovery. What would be the nurses most appropriate response to the family member? A) Hes on a calorie-restricted diet in order to divert energy to wound healing. B) His body has consumed his fat deposits for fuel because his calorie intake is lower than normal. C) He actually hasnt lost weight. Instead, theres been a change in the distribution of his body fat. D) He lost many fluids while he was being treated in the emergency phase of burn care.

B Feedback: Patients lose a great deal of weight during recovery from severe burns. Reserve fat deposits are catabolized as a result of hypermetabolism. Patients are not placed on a calorie restriction during recovery and fluid losses would not account for weight loss later in the recovery period. Changes in the overall distribution of body fat do not occur.

A nurse is providing discharge education to a patient who has recently been diagnosed with a bleeding disorder. What topic should the nurse prioritize when teaching this patient? A) Avoiding buses, subways, and other crowded, public sites B) Avoiding activities that carry a risk for injury C) Keeping immunizations current D) Avoiding foods high in vitamin K

B Feedback: Patients with bleeding disorders need to understand the importance of avoiding activities that increase the risk of bleeding, such as contact sports. Immunizations involve injections and may be contraindicated for some patients. Patients with bleeding disorders do not need to normally avoid crowds. Foods high in vitamin K may beneficial, not detrimental

A 25-year-old female patient with brain metastases is considering her life expectancy after her most recent meeting with her oncologist. Based on the fact that the patient is not receiving treatment for her brain metastases, what is the nurses most appropriate action? A) Promoting the patients functional status and ADLs B) Ensuring that the patient receives adequate palliative care C) Ensuring that the family does not tell the patient that her condition is terminal D) Promoting adherence to the prescribed medication regimen

B Feedback: Patients with intracerebral metastases who are not treated have a steady downhill course with a limited survival time, whereas those who are treated may survive for slightly longer periods, but for most cure is not possible. Palliative care is thus necessary. This is a priority over promotion of function and the family should not normally withhold information from the patient. Adherence to medications such as analgesics is important, but palliative care is a high priority.

A patient has been admitted to the neurologic unit for the treatment of a newly diagnosed brain tumor. The patient has just exhibited seizure activity for the first time. What is the nurses priority response to this event? A) Identify the triggers that precipitated the seizure. B) Implement precautions to ensure the patients safety. C) Teach the patients family about the relationship between brain tumors and seizure activity. D) Ensure that the patient is housed in a private room.

B Feedback: Patients with seizures are carefully monitored and protected from injury. Patient safety is a priority over health education, even though this is appropriate and necessary. Specific triggers may or may not be evident; identifying these is not the highest priority. A private room is preferable, but not absolutely necessary.

A patient is brought to the ED by ambulance with a gunshot wound to the abdomen. The nurse knows that the most common hollow organ injured in this type of injury is what? A) Liver B) Small bowel C) Stomach D) Large bowel

B Feedback: Penetrating abdominal wounds have a high incidence of injury to hollow organs, especially the small bowel. The liver is also injured frequently, but it is a solid organ.

A patient is admitted to the hospital with pernicious anemia. The nurse should prepare to administer which of the following medications? A) Folic acid B) Vitamin B12 C) Lactulose D) Magnesium sulfate

B Feedback: Pernicious anemia is characterized by vitamin B12 deficiency. Magnesium sulfate, lactulose, and folic acid do not address the pathology of this type of anemia.

The nurse is caring for a boy who has muscular dystrophy. When planning assistance with the patients ADLs, what goal should the nurse prioritize? A) Promoting the patients recovery from the disease B) Maximizing the patients level of function C) Ensuring the patients adherence to treatment D) Fostering the familys participation in care

B Feedback: Priority for the care of the child with muscular dystrophy is the need to maximize the patients level of function. Family participation is also important, but should be guided by this goal. Adherence is not a central goal, even though it is highly beneficial, and the disease is not curable.

You are caring for a patient who has just been told that his illness is progressing and nothing more can be done for him. After the physician leaves, the patient asks you to stay with him for a while. The patient becomes tearful and tries several times to say something, but cannot get the words out. What would be an appropriate response for you to make at this time? A) Can I give you some advice? B) Do you need more time to think about this? C) Is there anything you want to say? D) I have cared for lots of patients in your position. It will get easier.

B Feedback: Prompt gently: Do you need more time to think about this? Giving advice is inappropriate and it is obvious from the scenario that the patient has something to say. Referring to other patients negates the patients feelings at this time.

A patient newly diagnosed with a cervical disk herniation is receiving health education from the clinic nurse. What conservative management measures should the nurse teach the patient to implement? A) Perform active ROM exercises three times daily. B) Sleep on a firm mattress. C) Apply cool compresses to the back of the neck daily. D) Wear the cervical collar for at least 2 hours at a time.

B Feedback: Proper positioning on a firm mattress and bed rest for 1 to 2 days may bring dramatic relief from pain. The patient may need to wear a cervical collar 24 hours a day during the acute phase of pain from a cervical disk herniation. Hot, moist compresses applied to the back of the neck will increase blood flow to the muscles and help relax the spastic muscles.

A patient is scheduled for a CT scan of the abdomen with contrast. The patient has a baseline creatinine level of 2.3 mg/dL. In preparing this patient for the procedure, the nurse anticipates what orders? A) Monitor the patients electrolyte values every hour before the procedure. B) Preprocedure hydration and administration of acetylcysteine C) Hemodialysis immediately prior to the CT scan D) Obtain a creatinine clearance by collecting a 24-hour urine specimen.

B Feedback: Radiocontrast-induced nephropathy is a major cause of hospital-acquired acute kidney injury. Baseline levels of creatinine greater than 2 mg/dL identify the patient as being high risk. Preprocedure hydration and prescription of acetylcysteine (Mucomyst) the day prior to the test is effective in prevention. The nurse would not monitor the patients electrolytes every hour preprocedure. Nothing in the scenario indicates the need for hemodialysis. A creatinine clearance is not necessary prior to a CT scan with contrast.

A patient has been admitted to the critical care unit with a diagnosis of toxic hepatitis. When planning the patients care, the nurse should be aware of what potential clinical course of this health problem? Place the following events in the correct sequence. 1. Fever rises. 2. Hematemesis. 3. Clotting abnormalities. 4. Vascular collapse. 5. Coma. A) 1, 2, 5, 4, 3 B) 1, 2, 3, 4, 5 C) 2, 3, 1, 4, 5 D) 3, 1, 2, 5, 4

B Feedback: Recovery from acute toxic hepatitis is rapid if the hepatotoxin is identified early and removed or if exposure to the agent has been limited. Recovery is unlikely if there is a prolonged period between exposure and onset of symptoms. There are no effective antidotes. The fever rises; the patient becomes toxic and prostrated. Vomiting may be persistent, with the emesis containing blood. Clotting abnormalities may be severe, and hemorrhages may appear under the skin. The severe GI symptoms may lead to vascular collapse. Delirium, coma, and seizures develop, and within a few days the patient may die of fulminant hepatic failure unless he or she receives a liver transplant.

As the American population ages, nurses expect see more patients admitted to long-term care facilities in need of palliative care. Regulations now in place that govern how the care in these facilities is both organized and reimbursed emphasize what aspect of care? A) Ongoing acute care B) Restorative measures C) Mobility and socialization D) Incentives to palliative care

B Feedback: Regulations that govern how care in these facilities is organized and reimbursed tend to emphasize restorative measures and serve as a disincentive to palliative care. Long-term care facilities do not normally provide acute care for their patients. Regulations for long-term care facilities do not primarily emphasize mobility and socialization.

The nurse observes that the family members of a patient who was injured in an accident are blaming each other for the circumstances leading up to the accident. The nurse appropriately lets the family members express their feelings of responsibility, while explaining that there was probably little they could do to prevent the injury. In what stage of crisis is this family? A) Anxiety and denial B) Remorse and guilt C) Anger D) Grief

B Feedback: Remorse and guilt are natural processes of the stages of a crisis and should be facilitated for the family members to process the crisis. The familys sense of blame and responsibility are more suggestive of guilt than anger, grief, or anxiety.

A patient has a diagnosis of multiple myeloma and the nurse is preparing health education in preparation for discharge from the hospital. What action should the nurse promote? A) Daily performance of weight-bearing exercise to prevent muscle atrophy B) Close monitoring of urine output and kidney function C) Daily administration of warfarin (Coumadin) as ordered D) Safe use of supplementary oxygen in the home setting

B Feedback: Renal function must be monitored closely in the patient with multiple myeloma. Excessive weight- bearing can cause pathologic fractures. There is no direct indication for anticoagulation or supplementary oxygen.

A patient who is undergoing consolidation therapy for the treatment of leukemia has been experiencing debilitating fatigue. How can the nurse best meet this patients needs for physical activity? A) Teach the patient about the risks of immobility and the benefits of exercise. B) Assist the patient to a chair during awake times, as tolerated. C) Collaborate with the physical therapist to arrange for stair exercises. D) Teach the patient to perform deep breathing and coughing exercises.

B Feedback: Sitting is a chair is preferable to bed rest, even if a patient is experiencing severe fatigue. A patient who has debilitating fatigue would not likely be able to perform stair exercises. Teaching about mobility may be necessary, but education must be followed by interventions that actually involve mobility. Deep breathing and coughing reduce the risk of respiratory complications but are not substitutes for physical mobility in preventing deconditioning.

The nurse coming on shift on the medical unit is taking a report on four patients. What patient does the nurse know is at the greatest risk of developing ESKD? A) A patient with a history of polycystic kidney disease B) A patient with diabetes mellitus and poorly controlled hypertension C) A patient who is morbidly obese with a history of vascular disorders D) A patient with severe chronic obstructive pulmonary disease

B Feedback: Systemic diseases, such as diabetes mellitus (leading cause); hypertension; chronic glomerulonephritis; pyelonephritis; obstruction of the urinary tract; hereditary lesions, such as in polycystic kidney disease; vascular disorders; infections; medications; or toxic agents may cause ESKD. A patient with more than one of these risk factors is at the greatest risk for developing ESKD. Therefore, the patient with diabetes and hypertension is likely at highest risk for ESKD.

The nurse is caring for a patient who is believed to have just experienced an MI. The nurse notes changes in the ECG of the patient. What change on an ECG most strongly suggests to the nurse that ischemia is occurring? A) P wave inversion B) T wave inversion C) Q wave changes with no change in ST or T wave D) P wave enlargement

B Feedback: T-wave inversion is an indicator of ischemic damage to myocardium. Typically, few changes to P waves occur during or after an MI, whereas Q-wave changes with no change in the ST or T wave indicate an old MI

A patients daughter has asked the nurse about helping him end his terrible suffering. The nurse is aware of the ANA Position Statement on Assisted Suicide, which clearly states that nursing participation in assisted suicide is a violation of the Code for Nurses. What does the Position Statement further stress? A) Educating families about the moral implications of assisted suicide B) Identifying patient and family concerns and fears C) Identifying resources that meet the patients desire to die D) Supporting effective means to honor the patients desire to die

B Feedback: The ANA Position Statement further stresses the important role of the nurse in supporting effective symptom management, contributing to the creation of environments for care that honor the patients and familys wishes, as well as identifying their concerns and fears. Discussion of moral implications would normally be beyond the purview of the nurse.

A nurse is aware of the high incidence of catheter-related bloodstream infections in patients receiving parenteral nutrition. What nursing action has the greatest potential to reduce catheter-related bloodstream infections? A) Use clean technique and wear a mask during dressing changes. B) Change the dressing no more than weekly. C) Apply antibiotic ointment around the site with each dressing change. D) Irrigate the insertion site with sterile water during each dressing change.

B Feedback: The CDC (2011) recommends changing CVAD dressings not more than every 7 days unless the dressing is damp, bloody, loose, or soiled. Sterile technique (not clean technique) is used. Irrigation and antibiotic ointments are not used.

A nurse is reviewing the trend of a patients scores on the Glasgow Coma Scale (GCS). This allows the nurse to gauge what aspect of the patients status? A) Reflex activity B) Level of consciousness C) Cognitive ability D) Sensory involvement

B Feedback: The Glasgow Coma Scale (GCS) examines three responses related to LOC: eye opening, best verbal response, and best motor response.

The nurse is analyzing a rhythm strip. What component of the ECG corresponds to the resting state of the patients heart? A) P wave B) T wave C) U wave D) QRS complex

B Feedback: The T wave specifically represents ventricular muscle depolarization, also referred to as the resting state. Ventricular muscle depolarization does not result in the P wave, U wave, or QRS comple

A patient with poorly controlled diabetes has developed end-stage renal failure and consequent anemia. When reviewing this patients treatment plan, the nurse should anticipate the use of what drug? A) Magnesium sulfate B) Epoetin alfa C) Low-molecular weight heparin D) Vitamin K

B Feedback: The availability of recombinant erythropoietin (epoetin alfa [Epogen, Procrit], darbepoetin alfa [Aranesp]) has dramatically altered the management of anemia in end-stage renal disease. Heparin, vitamin K, and magnesium are not indicated in the treatment of renal failure or the consequent anemia.

A patient with renal failure has decreased erythropoietin production. Upon analysis of the patients complete blood count, the nurse will expect which of the following results? A) An increased hemoglobin and decreased hematocrit B) A decreased hemoglobin and hematocrit C) A decreased mean corpuscular volume (MCV) and red cell distribution width (RDW) D) An increased MCV and RDW

B Feedback: The decreased production of erythropoietin will result in a decreased hemoglobin and hematocrit. The patient will have normal MCV and RDW because the erythrocytes are normal in appearance.

A patient has been diagnosed with advanced stage breast cancer and will soon begin aggressive treatment. What assessment findings would most strongly suggest that the patient may have developed liver metastases? A) Persistent fever and cognitive changes B) Abdominal pain and hepatomegaly C) Peripheral edema unresponsive to diuresis D) Spontaneous bleeding and jaundice

B Feedback: The early manifestations of malignancy of the liver include paina continuous dull ache in the right upper quadrant, epigastrium, or back. Weight loss, loss of strength, anorexia, and anemia may also occur. The liver may be enlarged and irregular on palpation. Jaundice is present only if the larger bile ducts are occluded by the pressure of malignant nodules in the hilum of the liver. Fever, cognitive changes, peripheral edema, and bleeding are atypical signs.

New nurses on the telemetry unit have been paired with preceptors. One new nurse asks her preceptor to explain depolarization. What would be the best answer by the preceptor? A) Depolarization is the mechanical contraction of the heart muscles. B) Depolarization is the electrical stimulation of the heart muscles. C) Depolarization is the electrical relaxation of the heart muscles. D) Depolarization is the mechanical relaxation of the heart muscles.

B Feedback: The electrical stimulation of the heart is called depolarization, and the mechanical contraction is called systole. Electrical relaxation is called repolarization, and mechanical relaxation is called diastole.

A patient newly admitted to the telemetry unit is experiencing progressive fatigue, hemoptysis, and dyspnea. Diagnostic testing has revealed that these signs and symptoms are attributable to pulmonary venous hypertension. What valvular disorder should the nurse anticipate being diagnosed in this patient? A) Aortic regurgitation B) Mitral stenosis C) Mitral valve prolapse D) Aortic stenosis

B Feedback: The first symptom of mitral stenosis is often dyspnea on exertion as a result of pulmonary venous hypertension. Symptoms usually develop after the valve opening is reduced by one-third to one-half its usual size. Patients are likely to show progressive fatigue as a result of low cardiac output. The enlarged left atrium may create pressure on the left bronchial tree, resulting in a dry cough or wheezing. Patients may expectorate blood (i.e., hemoptysis) or experience palpitations, orthopnea, paroxysmal nocturnal dyspnea (PND), and repeated respiratory infections. Pulmonary venous hypertension is not typically caused by aortic regurgitation, mitral valve prolapse, or aortic stenosis.

The physician has ordered a fluid deprivation test for a patient suspected of having diabetes insipidus. During the test, the nurse should prioritize what assessments? A) Temperature and oxygen saturation B) Heart rate and BP C) Breath sounds and bowel sounds D) Color, warmth, movement, and sensation of extremities

B Feedback: The fluid deprivation test is carried out by withholding fluids for 8 to 12 hours or until 3% to 5% of the body weight is lost. The patients condition needs to be monitored frequently during the test, and the test is terminated if tachycardia, excessive weight loss, or hypotension develops. Consequently, BP and heart rate monitoring are priorities over the other listed assessments.

A nurse is caring for a patient with a subclavian central line who is receiving parenteral nutrition (PN). In preparing a care plan for this patient, what nursing diagnosis should the nurse prioritize? A) Risk for Activity Intolerance Related to the Presence of a Subclavian Catheter B) Risk for Infection Related to the Presence of a Subclavian Catheter C) Risk for Functional Urinary Incontinence Related to the Presence of a Subclavian Catheter D) Risk for Sleep Deprivation Related to the presence of a Subclavian Catheter

B Feedback: The high glucose content of PN solutions makes the solutions an idea culture media for bacterial and fungal growth, and the central venous access devices provide a port of entry. Prevention of infection is consequently a high priority. The patient will experience some inconveniences with regard to toileting, activity, and sleep, but the infection risk is a priority over each of these.

A group of nurses are learning about the high incidence and prevalence of anemia among different populations. Which of the following individuals is most likely to have anemia? A) A 50-year-old African-American woman who is going through menopause B) An 81-year-old woman who has chronic heart failure C) A 48-year-old man who travels extensively and has a high-stress job D) A 13-year-old girl who has just experienced menarche

B Feedback: The incidence and prevalence of anemia are exceptionally high among older adults, and the risk of anemia is compounded by the presence of heart disease. None of the other listed individuals exhibits high-risk factors for anemia, though exceptionally heavy menstrual flow can result in anemia.

An adult patients abnormal complete blood count (CBC) and physical assessment have prompted the primary care provider to order a diagnostic workup for Hodgkin lymphoma. The presence of what assessment finding is considered diagnostic of the disease? A) Schwann cells B) Reed-Sternberg cells C) Lewy bodies D) Loops of Henle

B Feedback: The malignant cell of Hodgkin lymphoma is the Reed-Sternberg cell, a gigantic tumor cell that is morphologically unique and thought to be of immature lymphoid origin. It is the pathologic hallmark and essential diagnostic criterion. Schwann cells exist in the peripheral n

The nurse is caring for a patient with refractory atrial fibrillation who underwent the maze procedure several months ago. The nurse reviews the result of the patients most recent cardiac imaging, which notes the presence of scarring on the atria. How should the nurse best respond to this finding? A) Recognize that the procedure was unsuccessful. B) Recognize this as a therapeutic goal of the procedure. C) Liaise with the care team in preparation for repeating the maze procedure. D) Prepare the patient for pacemaker implantation.

B Feedback: The maze procedure is an open heart surgical procedure for refractory atrial fibrillation. Small transmural incisions are made throughout the atria. The resulting formation of scar tissue prevents reentry conduction of the electrical impulse. Consequently, scar formation would constitute a successful procedure. There is no indication for repeating the procedure or implanting a pacemaker.

A patient with pericarditis has just been admitted to the CCU. The nurse planning the patients care should prioritize what nursing diagnosis? A) Anxiety related to pericarditis B) Acute pain related to pericarditis C) Ineffective tissue perfusion related to pericarditis D) Ineffective breathing pattern related to pericarditis

B Feedback: The most characteristic symptom of pericarditis is chest pain, although pain also may be located beneath the clavicle, in the neck, or in the left trapezius (scapula) region. The pain or discomfort usually remains fairly constant, but it may worsen with deep inspiration and when lying down or turning. Anxiety is highly plausible and should be addressed, but chest pain is a nearly certain accompaniment to the disease. Breathing and tissue perfusion are likely to be at risk, but pain is certain, especially in the early stages of treatment.

When assessing a patient diagnosed with angina pectoris it is most important for the nurse to gather what information? A) The patients activities limitations and level of consciousness after the attacks B) The patients symptoms and the activities that precipitate attacks C) The patients understanding of the pathology of angina D) The patients coping strategies surrounding the attacks

B Feedback: The nurse must gather information about the patients symptoms and activities, especially those that precede and precipitate attacks of angina pectoris. The patients coping, understanding of the disease, and status following attacks are all important to know, but causative factors are a primary focus of the assessment interview.

The ED nurse admitting a patient with a history of depression is screening the patient for suicide risk. What assessment question should the nurse ask when screening the patient? A) How would you describe your mood over the past few days? B) Have you ever thought about taking your own life? C) How do you think that your life is most likely to end? D) How would you rate the severity of your depression right now on a 10-point scale?

B Feedback: The nurse should address the patients possible plans for suicide in a direct yet empathic manner. The nurse should avoid oblique or indirect references to suicide and should not limit questions to the patients depression.

A male patient with multiple injuries is brought to the ED by ambulance. He has had his airway stabilized and is breathing on his own. The ED nurse does not see any active bleeding, but should suspect internal hemorrhage based on what finding? A) Absence of bruising at contusion sites B) Rapid pulse and decreased capillary refill C) Increased BP with narrowed pulse pressure D) Sudden diaphoresis

B Feedback: The nurse would anticipate that the pulse would increase and BP would decrease. Urine output would also decrease. An absence of bruising and the presence of diaphoresis would not suggest internal hemorrhage.

A patient has been diagnosed with acute pancreatitis. The nurse is addressing the diagnosis of Acute Pain Related to Pancreatitis. What pharmacologic intervention is most likely to be ordered for this patient? A) Oral oxycodone B) IV hydromorphone (Dilaudid) C) IM meperidine (Demerol) D) Oral naproxen (Aleve)

B Feedback: The pain of acute pancreatitis is often very severe and pain relief may require parenteral opioids such as morphine, fentanyl (Sublimaze), or hydromorphone (Dilaudid). There is no clinical evidence to support the use of meperidine for pain relief in pancreatitis. Opioids are preferred over NSAIDs.

What should the nurse teach a patient on corticosteroid therapy in order to reduce the patients risk of adrenal insufficiency? A) Take the medication late in the day to mimic the bodys natural rhythms. B) Always have enough medication on hand to avoid running out. C) Skip up to 2 doses in cases of illness involving nausea. D) Take up to 1 extra dose per day during times of stress

B Feedback: The patient and family should be informed that acute adrenal insufficiency and underlying symptoms will recur if corticosteroid therapy is stopped abruptly without medical supervision. The patient should be instructed to have an adequate supply of the corticosteroid medication always available to avoid running out. Doses should not be skipped or added without explicit instructions to do so. Corticosteroids should normally be taken in the morning to mimic natural rhythms.

Resection of a patients bladder tumor has been incomplete and the patient is preparing for the administration of the first ordered instillation of topical chemotherapy. When preparing the patient, the nurse should emphasize the need to do which of the following? A) Remain NPO for 12 hours prior to the treatment. B) Hold the solution in the bladder for 2 hours before voiding. C) Drink the intravesical solution quickly and on an empty stomach. D) Avoid acidic foods and beverages until the full cycle of treatment is complete.

B Feedback: The patient is allowed to eat and drink before the instillation procedure. Once the bladder is full, the patient must retain the intravesical solution for 2 hours before voiding. The solution is instilled through the meatus; it is not consumed orally. There is no need to avoid acidic foods and beverages during treatment.

A nurse is caring for a teenage girl who has had an anaphylactic reaction after a bee sting. The nurse is providing patient teaching prior to the patients discharge. In the event of an anaphylactic reaction, the nurse informs the patient that she should self-administer epinephrine in what site? A) Forearm B) Thigh C) Deltoid muscle D) Abdomen

B Feedback: The patient is taught to position the device at the middle portion of the thigh and push the device into the thigh as far as possible. The device will autoinject a premeasured dose of epinephrine into the subcutaneous tissue.

A patient is brought to the renal unit from the PACU status post resection of a renal tumor. Which of the following nursing actions should the nurse prioritize in the care of this patient? A) Increasing oral intake B) Managing postoperative pain C) Managing dialysis D) Increasing mobility

B Feedback: The patient requires frequent analgesia during the postoperative period and assistance with turning, coughing, use of incentive spirometry, and deep breathing to prevent atelectasis and other pulmonary complications. Increasing oral intake and mobility are not priority nursing actions in the immediate postoperative care of this patient. Dialysis is not necessary following kidney surgery.

A nurse is caring for a patient who has sickle cell anemia and the nurses assessment reveals the possibility of substance abuse. What is the nurses most appropriate action? A) Encourage the patient to rely on complementary and alternative therapies. B) Encourage the patient to seek care from a single provider for pain relief. C) Teach the patient to accept chronic pain as an inevitable aspect of the disease. D) Limit the reporting of emergency department visits to the primary health care provider.

B Feedback: The patient should be encouraged to use a single primary health care provider to address health care concerns. Emergency department visits should be reported to the primary health care provider to achieve optimal management of the disease. It would inappropriate to teach the patient to simply accept his or her pain. Complementary therapies are usually insufficient to fully address pain in sickle cell disease.

The neurologic ICU nurse is admitting a patient following a craniotomy using the supratentorial approach. How should the nurse best position the patient? A) Position the patient supine. B) Maintain head of bed (HOB) elevated at 30 to 45 degrees. C) Position patient in prone position. D) Maintain bed in Trendelenberg position.

B Feedback: The patient undergoing a craniotomy with a supratentorial (above the tentorium) approach should be placed with the HOB elevated 30 to 45 degrees, with the neck in neutral alignment. Each of the other listed positions would cause a dangerous elevation in ICP.

A 52-year-old patient is scheduled to undergo ileal conduit surgery. When planning this patients discharge education, what is the most plausible nursing diagnosis that the nurse should address? A) Impaired mobility related to limitations posed by the ileal conduit B) Deficient knowledge related to care of the ileal conduit C) Risk for deficient fluid volume related to urinary diversion D) Risk for autonomic dysreflexia related to disruption of the sacral plexus

B Feedback: The patient will most likely require extensive teaching about the care and maintenance of a new urinary diversion. A diversion does not create a serious risk of fluid volume deficit. Mobility is unlikely to be impaired after the immediate postsurgical recovery. The sacral plexus is not threatened by the creation of a urinary diversion.

An adult patient is admitted to the ED with chest pain. The patient states that he had developed unrelieved chest pain that was present for approximately 20 minutes before coming to the hospital. To minimize cardiac damage, the nurse should expect to administer which of the following interventions? A) Thrombolytics, oxygen administration, and nonsteroidal anti-inflammatories B) Morphine sulphate, oxygen, and bed rest C) Oxygen and beta-adrenergic blockers D) Bed rest, albuterol nebulizer treatments, and oxygen

B Feedback: The patient with suspected MI should immediately receive supplemental oxygen, aspirin, nitroglycerin, and morphine. Morphine sulphate reduces preload and decreases workload of the heart, along with increased oxygen from oxygen therapy and bed rest. With decreased cardiac demand, this provides the best chance of decreasing cardiac damage. NSAIDs and beta-blockers are not normally indicated. Albuterol, which is a medication used to manage asthma and respiratory conditions, will increase the heart rate.

A patient with cirrhosis has experienced a progressive decline in his health; and liver transplantation is being considered by the interdisciplinary team. How will the patients prioritization for receiving a donor liver be determined? A) By considering the patients age and prognosis B) By objectively determining the patients medical need C) By objectively assessing the patients willingness to adhere to post-transplantation care D) By systematically ruling out alternative treatment options

B Feedback: The patient would undergo a classification of the degree of medical need through an objective determination known as the Model of End-Stage Liver Disease (MELD) classification, which stratifies the level of illness of those awaiting a liver transplant. This algorithm considers multiple variables, not solely age, prognosis, potential for adherence, and the rejection of alternative options.

A patient with HF is placed on a low-sodium diet. Which statement by the patient indicates that the nurses nutritional teaching plan has been effective? A) I will have a ham and cheese sandwich for lunch. B) I will have a baked potato with broiled chicken for dinner. C) I will have a tossed salad with cheese and croutons for lunch. D) I will have chicken noodle soup with crackers and an apple for lunch.

B Feedback: The patients choice of a baked potato with broiled chicken indicates that the teaching plan has been effective. Potatoes and chicken are relatively low in sodium. Ham, cheese, and soup are often high in sodium.

A nurse is caring for a patient who has been diagnosed with leukemia. The nurses most recent assessment reveals the presence of ecchymoseson the patients sacral area and petechiae in her forearms. In addition to informing the patients primary care provider, the nurse should perform what action? A) Initiate measures to prevent venous thromboembolism (VTE). B) Check the patients most recent platelet level. C) Place the patient on protective isolation. D) Ambulate the patient to promote circulatory function.

B Feedback: The patients signs are suggestive of thrombocytopenia, thus the nurse should check the patients most recent platelet level. VTE is not a risk and this does not constitute a need for isolation. Ambulation and activity may be contraindicated due to the risk of bleeding.

The nurse is caring for a patient who is postoperative following a craniotomy. When writing the plan of care, the nurse identifies a diagnosis of deficient fluid volume related to fluid restriction and osmotic diuretic use. What would be an appropriate intervention for this diagnosis? A) Change the patients position as indicated. B) Monitor serum electrolytes. C) Maintain NPO status. D) Monitor arterial blood gas (ABG) values.

B Feedback: The postoperative fluid regimen depends on the type of neurosurgical procedure and is determined on an individual basis. The volume and composition of fluids are adjusted based on daily serum electrolyte values, along with fluid intake and output. Fluids may have to be restricted in patients with cerebral edema. Changing the patients position, maintaining an NPO status, and monitoring ABG values do not relate to the nursing diagnosis of deficient fluid volume.

The nurse is caring for a 68-year-old patient the nurse suspects has digoxin toxicity. In addition to physical assessment, the nurse should collect what assessment datum? A) Skin turgor B) Potassium level C) White blood cell count D) Peripheral pulses

B Feedback: The serum potassium level is monitored because the effect of digoxin is enhanced in the presence of hypokalemia and digoxin toxicity may occur. Skin turgor, white cell levels, and peripheral pulses are not normally affected in cases of digitalis toxicity.

A nurse who is a member of the local disaster response team is learning about blast injuries. The nurse should plan for what event that occurs in the tertiary phase of the blast injury? A) Victims pre-existing medical conditions are exacerbated. B) Victims are thrown by the pressure wave. C) Victims experience burns from the blast. D) Victims suffer injuries caused by debris or shrapnel from the blast.

B Feedback: The tertiary phase of the blast injury results from the pressure wave that causes the victims to be thrown, resulting in traumatic injury. None of the other listed events occurs in this specific phase of a blast.

A patient with a history of injection drug use has been diagnosed with hepatitis C. When collaborating with the care team to plan this patients treatment, the nurse should anticipate what intervention? A) Administration of immune globulins B) A regimen of antiviral medications C) Rest and watchful waiting D) Administration of fresh-frozen plasma (FFP)

B Feedback: There is no benefit from rest, diet, or vitamin supplements in HCV treatment. Studies have demonstrated that a combination of two antiviral agents, Peg-interferon and ribavirin (Rebetol), is effective in producing improvement in patients with hepatitis C and in treating relapses. Immune globulins and FFP are not indicated.

A patient newly diagnosed with thrombocytopenia is admitted to the medical unit. After the admission assessment, the patient asks the nurse to explain the disease. What should the nurse explain to this patient? A) There could be an attack on the platelets by antibodies. B) There could be decreased production of platelets. C) There could be impaired communication between platelets. D) There could be an autoimmune process causing platelet malfunction.

B Feedback: Thrombocytopenia can result from a decreased platelet production, increased platelet destruction, or increased consumption of platelets. Impaired platelet communication, antibodies, and autoimmune processes are not typical pathologies.

A patient has been admitted to the medical unit with signs and symptoms suggestive of endocarditis. The physicians choice of antibiotics would be primarily based on what diagnostic test? A) Echocardiography B) Blood cultures C) Cardiac aspiration D) Complete blood count

B Feedback: To help determine the causative organisms and the most effective antibiotic treatment for the patient, blood cultures are taken. A CBC can help establish the degree and stage of infection, but not the causative microorganism. Echocardiography cannot indicate the microorganisms causing the infection. Cardiac aspiration is not a diagnostic test.

A patients decline in respiratory and renal function has been attributed to Goodpasture syndrome, which is a type II hypersensitivity reaction. What pathologic process underlies the patients health problem? A) Antigens have bound to antibodies and formed inappropriate immune complexes. B) The patients body has mistakenly identified a normal constituent of the body as foreign. C) Sensitized T cells have caused cell and tissue damage. D) Mast cells have released histamines that directly cause cell lysis.

B Feedback: Type II reactions, or cytotoxic hypersensitivity, occur when the system mistakenly identifies a normal constituent of the body as foreign. An example of this type of reaction is Goodpasture syndrome. Type III, or immune complex, hypersensitivity involves immune complexes that are formed when antigens bind to antibodies. Type IV hypersensitivity is mediated by sensitized T cells that cause cell and tissue damage. Histamine does not directly cause cell lysis.

A nurse is caring for a patient with gallstones who has been prescribed ursodeoxycholic acid (UDCA). The patient askshow this medicine is going to help his symptoms. The nurse should be aware of what aspect of this drugs pharmacodynamics? A) It inhibits the synthesis of bile. B) It inhibits the synthesis and secretion of cholesterol. C) It inhibits the secretion of bile. D) It inhibits the synthesis and secretion of amylase.

B Feedback: UDCA acts by inhibiting the synthesis and secretion of cholesterol, thereby desaturating bile. UDCA does not directly inhibit either the synthesis or secretion of bile or amylase.

A patient has just died following urosepsis that progressed to septic shock. The patients spouse says, I knew this was coming, but I feel so numb and hollow inside. The nurse should know that these statements are characteristic of what? A) Complicated grief and mourning B) Uncomplicated grief and mourning C) Depression stage of dying D) Acceptance stage of dying

B Feedback: Uncomplicated grief and mourning are characterized by emotional feelings of sadness, anger, guilt, and numbness; physical sensations, such as hollowness in the stomach and tightness in the chest, weakness, and lack of energy; cognitions that include preoccupation with the loss and a sense of the deceased as still present; and behaviors such as crying, visiting places that are reminders of the deceased, social withdrawal, and restless overactivity. Complicated grief and mourning occur at a prolonged time after the death. The spouses statement does not clearly suggest depression or acceptance.

The nurse is performing stroke risk screenings at a hospital open house. The nurse has identified four patients who might be at risk for a stroke. Which patient is likely at the highest risk for a hemorrhagic stroke? A) White female, age 60, with history of excessive alcohol intake B) White male, age 60, with history of uncontrolled hypertension C) Black male, age 60, with history of diabetes D) Black male, age 50, with history of smoking

B Feedback: Uncontrolled hypertension is the primary cause of a hemorrhagic stroke. Control of hypertension, especially in individuals over 55 years of age, clearly reduces the risk for hemorrhagic stroke. Additional risk factors are increased age, male gender, and excessive alcohol intake. Another high-risk group includes African Americans, where the incidence of first stroke is almost twice that as in Caucasians.

A patient is admitted to the unit with acute cholecystitis. The physician has noted that surgery will be scheduled in 4 days. The patient asks why the surgery is being put off for a week when he has a sick gallbladder. What rationale would underlie the nurses response? A) Surgery is delayed until the patient can eat a regular diet without vomiting. B) Surgery is delayed until the acute symptoms subside. C) The patient requires aggressive nutritional support prior to surgery. D) Time is needed to determine whether a laparoscopic procedure can be used.

B Feedback: Unless the patients condition deteriorates, surgical intervention is delayed just until the acute symptoms subside (usually within a few days). There is no need to delay surgery pending an improvement in nutritional status, and deciding on a laparoscopic approach is not a lengthy process.

The critical care nurse is monitoring the patients urine output and drains following renal surgery. What should the nurse promptly report to the physician? A) Increased pain on movement B) Absence of drain output C) Increased urine output D) Blood-tinged serosanguineous drain output

B Feedback: Urine output and drainage from tubes inserted during surgery are monitored for amount, color, and type or characteristics. Decreased or absent drainage is promptly reported to the physician because it may indicate obstruction that could cause pain, infection, and disruption of the suture lines. Reporting increased pain on movement has nothing to do with the scenario described. Increased urine output and serosanguineous drainage are expected.

A patient with gallstones has been prescribed ursodeoxycholic acid (UDCA). The nurse understands that additional teaching is needed regarding this medication when the patient states: A) It is important that I see my physician for scheduled follow-up appointments while taking this medication. B) I will take this medication for 2 weeks and then gradually stop taking it. C) If I lose weight, the dose of the medication may need to be changed. D) This medication will help dissolve small gallstones made of cholesterol.

B Feedback: Ursodeoxycholic acid (UDCA) has been used to dissolve small, radiolucent gallstones composed primarily of cholesterol. This drug can reduce the size of existing stones, dissolve small stones, and prevent new stones from forming. Six to 12 months of therapy is required in many patients to dissolve stones, and monitoring of the patient is required during this time. The effective dose of medication depends on body weight.

The staff educator is teaching a CPR class. Which of the following aspects of defibrillation should the educator stress to the class? A) Apply the paddles directly to the patients skin. B) Use a conducting medium between the paddles and the skin. C) Always use a petroleum-based gel between the paddles and the skin. D) Any available liquid can be used between the paddles and the skin.

B Feedback: Use multifunction conductor pads or paddles with a conducting medium between the paddles and the skin (the conducting medium is available as a sheet, gel, or paste). Do not use gels or pastes with poor electrical conductivity.

A nurse is preparing a plan of care for a patient with pancreatic cysts that have necessitated drainage through the abdominal wall. What nursing diagnosis should the nurse prioritize? A) Disturbed Body Image B) Impaired Skin Integrity C) Nausea D) Risk for Deficient Fluid Volume

B Feedback: While each of the diagnoses may be applicable to a patient with pancreatic drainage, the priority nursing diagnosis is Impaired Skin Integrity. The drainage is often perfuse and destructive to tissue because of the enzyme contents. Nursing measures must focus on steps to protect the skin near the drainage site from excoriation. The application of ointments or the use of a suction apparatus protects the skin from excoriation.

An ED nurse is triaging patients according to the Emergency Severity Index (ESI). When assigning patients to a triage level, the nurse will consider the patients acuity as well as what other variable? A) The likelihood of a repeat visit to the ED in the next 7 days B) The resources that the patient is likely to require C) The patients or insurers ability to pay for care D) Whether the patient is known to ED staff from previous visits

B Feedback: With the ESI, patients are assigned to triage levels based on both their acuity and their anticipated resource needs. Ability to pay, the likelihood of repeat visits, and the history of prior visits are not explicitly considered.

The clinic nurse caring for a patient with Parkinsons disease notes that the patient has been taking levodopa and carbidopa (Sinemet) for 7 years. For what common side effect of Sinemet would the nurse assesses this patient? A) Pruritus B) Dyskinesia C) Lactose intolerance D) Diarrhea

B Feedback: Within 5 to 10 years of taking levodopa, most patients develop a response to the medication characterized by dyskinesia (abnormal involuntary movements). Another potential complication of longterm dopaminergic medication use is neuroleptic malignant syndrome characterized by severe rigidity, stupor, and hyperthermia. Side effects of long-term Sinemet therapy are not pruritus, lactose intolerance, or diarrhea.

A patient has undergone a successful heart transplant and has been discharged home with a medication regimen that includes cyclosporine and tacrolimus. In light of this patients medication regimen, what nursing diagnosis should be prioritized? A) Risk for injury B) Risk for infection C) Risk for peripheral neurovascular dysfunction D) Risk for unstable blood glucose

B Immunosuppressants decrease the bodys ability to resist infections, and a satisfactory balance must be achieved between suppressing rejection and avoiding infection. These drugs do not create a heightened risk of injury, neurovascular dysfunction, or unstable blood glucose levels.

A 67-year-old woman experienced the death of her husband from a sudden myocardial infarction 5 weeks ago. The nurse recognizes that the woman will be going through the process of mourning for an extended period of time. What processes of mourning will allow the woman to accommodate the loss in a healthy way? Select all that apply. A) Reiterating her anger at her husbands care team B) Reinvesting in new relationships at the appropriate time C) Reminiscing about the relationship she had with her husband D) Relinquishing old attachments to her husband at the appropriate time E) Renewing her lifelong commitment to her husband

B, C, D Feedback: Six key processes of mourning allow people to accommodate to the loss in a healthy way: 1.) Recognition of the loss 2.) Reaction to the separation, and experiencing and expressing the pain of the loss 3.) Recollection and re-experiencing the deceased, the relationship, and the associated feelings 4.) Relinquishing old attachments to the deceased 5.) Readjustment to adapt to the new world without forgetting the old 6.) Reinvestment Reiterating her anger and renewing her lifelong commitment may be counterproductive to the mourning process.

The nurse is caring for a patient who is rapidly progressing toward brain death. The nurse should be aware of what cardinal signs of brain death? Select all that apply. A) Absence of pain response B) Apnea C) Coma D) Absence of brain stem reflexes E) Absence of deep tendon reflexes

B, C, D Feedback: The three cardinal signs of brain death upon clinical examination are coma, the absence of brain stem reflexes, and apnea. Absences of pain response and deep tendon reflexes are not necessarily indicative of brain death.

The nurse is relating the deficits in a patients synchronization of the atrial and ventricular events to his diagnosis. What are the physiologic characteristics of the nodal and Purkinje cells that provide this synchronization? Select all that apply. A) Loop connectivity B) Excitability C) Automaticity D) Conductivity E) Independence

B, C, D Feedback: Three physiologic characteristics of two types of specialized electrical cells, the nodal cells and the Purkinje cells, provide this synchronization: automaticity, or the ability to initiate an electrical impulse; excitability, or the ability to respond to an electrical impulse; and conductivity, the ability to transmit an electrical impulse from one cell to another. Loop connectivity is a distracter for this question. Independence of the cells has nothing to do with the synchronization described in the scenario.

A nurse is creating a care plan for a patient who is receiving parenteral nutrition. The patients care plan should include nursing actions relevant to what potential complications? Select all that apply. A) Dumping syndrome B) Clotted or displaced catheter C) Pneumothorax D) Hyperglycemia E) Line sepsis

B, C, D, E Feedback: Common complications of PN include a clotted or displaced catheter, pneumothorax, hyperglycemia, and infection from the venous access device (line sepsis). Dumping syndrome applies to enteral nutrition, not PN

A child has been transported to the emergency department (ED) after a severe allergic reaction. The ED nurse is evaluating the patients respiratory status. How should the nurse evaluate the patients respiratory status? Select all that apply. A) Facilitate lung function testing. B) Assess breath sounds. C) Measure the childs oxygen saturation by oximeter. D) Monitor the childs respiratory pattern. E) Assess the childs respiratory rate.

B, C, D, E Feedback: The respiratory status is evaluated by monitoring the respiratory rate and pattern and by assessing for breathing difficulties, low oxygen saturation, or abnormal lung sounds such

The nurse is caring for a patient with acute glomerular inflammation. When assessing for the characteristic signs and symptoms of this health problem, the nurse should include which assessments? Select all that apply. A) Percuss for pain in the right lower abdominal quadrant. B) Assess for the presence of peripheral edema. C) Auscultate the patients apical heart rate for dysrhythmias. D) Assess the patients BP. E) Assess the patients orientation and judgment.

B, D Feedback: Most patients with acute glomerular inflammation have some degree of edema and hypertension. Dysrhythmias, RLQ pain, and changes in mental status are not among the most common manifestations of acute glomerular inflammation.

As a member of the stroke team, the nurse knows that thrombolytic therapy carries the potential for benefit and for harm. The nurse should be cognizant of what contraindications for thrombolytic therapy? Select all that apply. A) INR above 1.0 B) Recent intracranial pathology C) Sudden symptom onset D) Current anticoagulation therapy E) Symptom onset greater than 3 hours prior to admission

B, D, E Feedback: Some of the absolute contraindications for thrombolytic therapy include symptom onset greater than 3 hours before admission, a patient who is anticoagulated (with an INR above 1.7), or a patient who has recently had any type of intracranial pathology (e.g., previous stroke, head injury, trauma).

A nurse educator is reviewing peripheral IV insertion with a group of novice nurses. How should these nurses be encouraged to deal with excess hair at the intended site? A) Leave the hair intact. B) Shave the area. C) Clip the hair in the area. D) Remove the hair with a depilatory.

C

A patient on the oncology unit is receiving carmustine, a chemotherapy agent, and the nurse is aware that a significant side effect of this medication is thrombocytopenia. Which symptom should the nurse assess for in patients at risk for thrombocytopenia? A) Interrupted sleep pattern B) Hot flashes C) Epistaxis (nose bleed) D) Increased weight

C

A public health nurse has formed an interdisciplinary team that is developing an educational program entitled Cancer: The Risks and What You Can Do About Them. Participants will receive information, but the major focus will be screening for relevant cancers. This program is an example of what type of health promotion activity? A) Disease prophylaxis B) Risk reduction C) Secondary prevention D) Tertiary prevention

C

An oncology patient has just returned from the postanesthesia care unit after an open hemicolectomy. This patients plan of nursing care should prioritize which of the following? A) Assess the patient hourly for signs of compartment syndrome. B) Assess the patients fine motor skills once per shift. C) Assess the patients wound for dehiscence every 4 hours. D) Maintain the patients head of bed at 45 degrees or more at all times.

C

One day after a patient is admitted to the medical unit, you note that the patient is oliguric. You notify the acute-care nurse practitioner who orders a fluid challenge of 200 mL of normal saline solution over 15 minutes. This intervention will achieve which of the following? A) Help distinguish hyponatremia from hypernatremia B) Help evaluate pituitary gland function C) Help distinguish reduced renal blood flow from decreased renal function D) Help provide an effective treatment for hypertension-induced oliguria

C

The hospice nurse is caring for a patient with cancer in her home. The nurse has explained to the patient and the family that the patient is at risk for hypercalcemia and has educated them on that signs and symptoms of this health problem. What else should the nurse teach this patient and family to do to reduce the patients risk of hypercalcemia? A) Stool softeners are contraindicated. B) Laxatives should be taken daily. C) Consume 2 to 4 L of fluid daily. D) Restrict calcium intake.

C

The nurse in the medical ICU is caring for a patient who is in respiratory acidosis due to inadequate ventilation. What diagnosis could the patient have that could cause inadequate ventilation? A) Endocarditis B) Multiple myeloma C) Guillain-Barr syndrome D) Overdose of amphetamines

C

The nurse is assessing the patient for the presence of a Chvosteks sign. What electrolyte imbalance would a positive Chvosteks sign indicate? A) Hypermagnesemia B) Hyponatremia C) Hypocalcemia D) Hyperkalemia

C

The nurse is caring for a 39-year-old woman with a family history of breast cancer. She requested a breast tumor marking test and the results have come back positive. As a result, the patient is requesting a bilateral mastectomy. This surgery is an example of what type of oncologic surgery? A) Salvage surgery B) Palliative surgery C) Prophylactic surgery D) Reconstructive surgery

C

The nurse on a bone marrow transplant unit is caring for a patient with cancer who is preparing for HSCT. What is a priority nursing diagnosis for this patient? A) Fatigue related to altered metabolic processes B) Altered nutrition: less than body requirements related to anorexia C) Risk for infection related to altered immunologic response D) Body image disturbance related to weight loss and anorexia

C

Traditionally, nurses have been involved with tertiary cancer prevention. However, an increasing emphasis is being placed on both primary and secondary prevention. What would be an example of primary prevention? A) Yearly Pap tests B) Testicular self-examination C) Teaching patients to wear sunscreen D) Screening mammograms

C

A nurse who provides care on a burn unit is preparing to apply a patients ordered topical antibiotic ointment. What action should the nurse perform when administering this medication? A) Apply the new ointment without disturbing the existing layer of ointment. B) Apply the ointment using a sterile tongue depressor. C) Apply a layer of ointment approximately 1/16 inch thick. D) Gently irrigate the wound bed after applying the antibiotic ointment.

C After removing the old ointment from the wound bed, the nurse should apply a layer of ointment 1/16- inch thick using clean gloves. The wound would not be irrigated after application of new ointment.

A community health nurse is giving an educational presentation about stroke and heart disease at the local senior citizens center. What nonmodifiable risk factor for stroke should the nurse cite? A) Female gender B) Asian American race C) Advanced age D) Smoking

C Feedback: Advanced age, male gender, and race are well-known nonmodifiable risk factors for stroke. High-risk groups include people older than 55 years of age; the incidence of stroke more than doubles in each successive decade. Men have a higher rate of stroke than that of women. Another high-risk group is African Americans; the incidence of first stroke in African Americans is almost twice that as in Caucasian Americans; Asian American race is not a risk factor. Smoking is a modifiable risk.

A nurse is caring for an 87-year-old Mexican-American female patient who is in end-stage renal disease. The physician has just been in to see the patient and her family to tell them that nothing more can be done for the patient and that death is not far. The physician offers to discharge the patient home to hospice care, but the patient and family refuse. After the physician leaves, the patients daughter approaches you and asks what hospice care is. What would this lack of knowledge about hospice care be perceived as? A) Lack of an American education of the patient and her family B) A language barrier to hospice care for this patient C) A barrier to hospice care for this patient D) Inability to grasp American concepts of health care

C Feedback: Historical mistrust of the health care system and unequal access to even basic medical care may underlie the beliefs and attitudes among ethnically diverse populations. In addition, lack of education or knowledge about end-of-life care treatment options and language barriers influence decisions among many socioeconomically disadvantaged groups. The scenario does not indicate whether the patients family has an American education, whether they are unable to grasp American concepts of health care, or whether they can speak or understand English.

A previously healthy adults sudden and precipitous decline in health has been attributed to fulminant hepatic failure, and the patient has been admitted to the intensive care unit. The nurse should be aware that the treatment of choice for this patient is what? A) IV administration of immune globulins B) Transfusion of packed red blood cells and fresh-frozen plasma (FFP) C) Liver transplantation D) Lobectomy

C Feedback: Liver transplantation carries the highest potential for the resolution of fulminant hepatic failure. This is preferred over other interventions, such as pharmacologic treatments, transfusions, and surgery.

A nurse is caring for a patient with severe hemolytic jaundice. Laboratory tests show free bilirubin to be 24 mg/dL. For what complication is this patient at risk? A) Chronic jaundice B) Pigment stones in portal circulation C) Central nervous system damage D) Hepatomegaly

C Feedback: Prolonged jaundice, even if mild, predisposes to the formation of pigment stones in the gallbladder, and extremely severe jaundice (levels of free bilirubin exceeding 20 to 25 mg/dL) poses a risk for CNS damage. There are not specific risks of hepatomegaly or chronic jaundice resulting from high bilirubin.

The nurse is caring for a patient in acute kidney injury. Which of the following complications would most clearly warrant the administration of polystyrene sulfonate (Kayexalate)? A) Hypernatremia B) Hypomagnesemia C) Hyperkalemia D) Hypercalcemia

C Hyperkalemia, a common complication of acute kidney injury, is life-threatening if immediate action is not taken to reverse it. The administration of polystyrene sulfonate reduces serum potassium levels.

Paramedics have brought an intubated patient to the RD following a head injury due to acceleration- deceleration motor vehicle accident. Increased ICP is suspected. Appropriate nursing interventions would include which of the following? A) Keep the head of the bed (HOB) flat at all times. B) Teach the patient to perform the Valsalva maneuver. C) Administer benzodiazepines on a PRN basis. D) Perform endotracheal suctioning every hour.

C If the patient with a brain injury is very agitated, benzodiazepines are the most commonly used sedatives and do not affect cerebral blood flow or ICP. The HOB should be elevated 30 degrees. Suctioning should be done a limited basis, due to increasing the pressure in the cranium. The Valsalva maneuver is to be avoided. This also causes increased ICP.

A nurse is planning care for a nephrology patient with a new nursing graduate. The nurse states, A patient in renal failure partially loses the ability to regulate changes in pH. What is the cause of this partial inability? A) The kidneys regulate and reabsorb carbonic acid to change and maintain pH. B) The kidneys buffer acids through electrolyte changes. C) The kidneys regenerate and reabsorb bicarbonate to maintain a stable pH. D) The kidneys combine carbonic acid and bicarbonate to maintain a stable pH

C The kidneys regulate the bicarbonate level in the ECF; they can regenerate bicarbonate ions as well as reabsorb them from the renal tubular cells. In respiratory acidosis and most cases of metabolic acidosis, the kidneys excrete hydrogen ions and conserve bicarbonate ions to help restore balance. The lungs regulate and reabsorb carbonic acid to change and maintain pH. The kidneys do not buffer acids through electrolyte changes; buffering occurs in reaction to changes in pH. Carbonic acid works as the chemical medium to exchange O2 and CO2 in the lungs to maintain a stable pH whereas the kidneys use bicarbonate as the chemical medium to maintain a stable pH by moving and eliminating H+.

A patient is prescribed corticosteroid therapy. What would be priority information for the nurse to give the patient who is prescribed long-term corticosteroid therapy? A) The patients diet should be low protein with ample fat. B) The patient may experience short-term changes in cognition. C) The patient is at an increased risk for developing infection. D) The patient is at a decreased risk for development of thrombophlebitis and thromboembolism.

C The patient is at increased risk of infection and masking of signs of infection. The cardiovascular effects of corticosteroid therapy may result in development of thrombophlebitis or thromboembolism. Diet should be high in protein with limited fat. Changes in appearance usually disappear when therapy is no longer necessary. Cognitive changes are not common adverse effects.

The nurse is caring for a patient diagnosed with an ischemic stroke and knows that effective positioning of the patient is important. Which of the following should be integrated into the patients plan of care? A) The patients hip joint should be maintained in a flexed position. B) The patient should be in a supine position unless ambulating. C) The patient should be placed in a prone position for 15 to 30 minutes several times a day. D) The patient should be placed in a Trendelenberg position two to three times daily to promote cerebral perfusion

C Feedback: If possible, the patient is placed in a prone position for 15 to 30 minutes several times a day. A small pillow or a support is placed under the pelvis, extending from the level of the umbilicus to the upper third of the thigh. This helps to promote hyperextension of the hip joints, which is essential for normal gait, and helps prevent knee and hip flexion contractures. The hip joints should not be maintained in flexion and the Trendelenberg position is not indicated.

You are performing an admission assessment on an older adult patient newly admitted for end-stage liver disease. What principle should guide your assessment of the patients skin turgor? A) Overhydration is common among healthy older adults. B) Dehydration causes the skin to appear spongy. C) Inelastic skin turgor is a normal part of aging. D) Skin turgor cannot be assessed in patients over 70.

C Feedback: Inelastic skin is a normal change of aging. However, this does not mean that skin turgor cannot be assessed in older patients. Dehydration, not overhydration, causes inelastic skin with tenting. Overhydration, not dehydration, causes the skin to appear edematous and spongy

A nurse is caring for a patient who is being treated for leukemia in the hospital. The patient was able to maintain her nutritional status for the first few weeks following her diagnosis but is now exhibiting early signs and symptoms of malnutrition. In collaboration with the dietitian, the nurse should implement what intervention? A) Arrange for total parenteral nutrition (TPN). B) Facilitate placement of a percutaneous endoscopic gastrostomy (PEG) tube. C) Provide the patient with several small, soft-textured meals each day. D) Assign responsibility for the patients nutrition to the patients friends and family.

C For patients experiencing difficulties with oral intake, the provision of small, easily chewed meals may be beneficial. This option would be trialed before resorting to tube feeding or TPN. The family should be encouraged to participate in care, but should not be assigned full responsibility.

A patient suffering from blast lung has been admitted to the hospital and is exhibiting signs and symptoms of an air embolus. What is the nurses most appropriate action? A) Place the patient in the Trendelenberg position. B) Assess the patients airway and begin chest compressions. C) Position the patient in the prone, left lateral position. D) Encourage the patient to perform deep breathing and coughing exercises.

C In the event of an air embolus, the patient should be placed immediately in the prone left lateral position to prevent migration of the embolus and will require emergent treatment in a hyperbaric chamber. Chest compressions, deep breathing, and coughing would exacerbate the patients condition. Trendelenberg positioning is not recommended.

A patient with end-stage heart failure has participated in a family meeting with the interdisciplinary team and opted for hospice care. On what belief should the patients care in this setting be based? A) Meaningful living during terminal illness requires technologic interventions. B) Meaningful living during terminal illness is best supported in designated facilities. C) Meaningful living during terminal illness is best supported in the home. D) Meaningful living during terminal illness is best achieved by prolonging physiologic dying.

C The hospice movement in the United States is based on the belief that meaningful living is achievable during terminal illness and that it is best supported in the home, free from technologic interventions to prolong physiologic dying.

The nurse is participating in the care conference for a patient with ACS. What goal should guide the care teams selection of assessments, interventions, and treatments? A) Maximizing cardiac output while minimizing heart rate B) Decreasing energy expenditure of the myocardium C) Balancing myocardial oxygen supply with demand D) Increasing the size of the myocardial muscle

C Balancing myocardial oxygen supply with demand (e.g., as evidenced by the relief of chest pain) is the top priority in the care of the patient with ACS. Treatment is not aimed directly at minimizing heart rate because some patients experience bradycardia. Increasing the size of the myocardium is never a goal. Reducing the myocardiums energy expenditure is often beneficial, but this must be balanced with productivity.

The nurse is caring for a patient with systolic HF whose previous adverse reactions preclude the safe use of ACE inhibitors. The nurse should anticipate that the prescriber may choose what combination of drugs? A) Loop diuretic and antiplatelet aggregator B) Loop diuretic and calcium channel blocker C) Combination of hydralazine and isosorbide dinitrate D) Combination of digoxin and normal saline

C Feedback: A combination of hydralazine and isosorbide dinitrate may be an alternative for patients who cannot take ACE inhibitors. Antiplatelet aggregators, calcium channel blockers, and normal saline are not typically prescribed.

A nurse has entered the room of a patient with cirrhosis and found the patient on the floor. The patient states that she fell when transferring to the commode. The patients vital signs are within reference ranges and the nurse observes no apparent injuries. What is the nurses most appropriate action? A) Remove the patients commode and supply a bedpan. B) Complete an incident report and submit it to the unit supervisor. C) Have the patient assessed by the physician due to the risk of internal bleeding. D) Perform a focused abdominal assessment in order to rule out injury.

C Feedback: A fall would necessitate thorough medical assessment due to the patients risk of bleeding. The nurses abdominal assessment is an appropriate action, but is not wholly sufficient to rule out internal injury. Medical assessment is a priority over removing the commode or filling out an incident report, even though these actions are appropriate.

The nurse is caring for a patient who has undergone creation of a urinary diversion. Forty-eight hours postoperatively, the nurses assessment reveals that the stoma is a dark purplish color. What is the nurses most appropriate response? A) Document the presence of a healthy stoma. B) Assess the patient for further signs and symptoms of infection. C) Inform the primary care provider that the vascular supply may be compromised. D) Liaise with the wound-ostomy-continence (WOC) nurse because the ostomy appliance around the stoma may be too loose.

C Feedback: A healthy stoma is pink or red. A change from this normal color to a dark purplish color suggests that the vascular supply may be compromised. A loose ostomy appliance and infections do not cause a dark purplish stoma.

After a radiation exposure, a patient has been assessed and determined to be a possible survivor. Following the resolution of the patients initial symptoms, the care team should anticipate what event? A) A return to full health B) Internal bleeding C) A latent phase D) Massive tissue necrosis

C Feedback: A latent phase commonly follows the prodromal phase of radiation exposure. The patient is deemed a possible survivor, not a probable survivor, so an immediate return to health is unlikely. However, internal bleeding and massive tissue necrosis would not be expected in a patient categorized as a possible survivor.

A patient is admitted to the neurologic ICU with a C4 spinal cord injury. When writing the plan of care for this patient, which of the following nursing diagnoses would the nurse prioritize in the immediate care of this patient? A) Risk for impaired skin integrity related to immobility and sensory loss B) Impaired physical mobility related to loss of motor function C) Ineffective breathing patterns related to weakness of the intercostal muscles D) Urinary retention related to inability to void spontaneously

C Feedback: A nursing diagnosis related to breathing pattern would be the priority for this patient. A C4 spinal cord injury will require ventilatory support, due to the diaphragm and intercostals being affected. The other nursing diagnoses would be used in the care plan, but not designated as a higher priority than ineffective breathing patterns.

A group of medical nurses are being certified in their response to potential bioterrorism. The nurses learn that if a patient is exposed to the smallpox virus he or she becomes contagious at what time? A) 6 to 12 hours after exposure B) When pustules form C) After a rash appears D) When the patient becomes febrile

C Feedback: A patient is contagious after a rash develops, which initially develops on the face, mouth, pharynx, and forearms. The patient exposed to the smallpox virus is not contagious immediately after exposure; only when pustules form, or with a body temperature of 38C.

A patient is admitted to the cardiac care unit for an electrophysiology (EP) study. What goal should guide the planning and execution of the patients care? A) Ablate the area causing the dysrhythmia. B) Freeze hypersensitive cells. C) Diagnose the dysrhythmia. D) Determine the nursing plan of care.

C Feedback: A patient may undergo an EP study in which electrodes are placed inside the heart to obtain an intracardiac ECG. This is used not only to diagnose the dysrhythmia but also to determine the most effective treatment plan. However, because an EP study is invasive, it is performed in the hospital and may require that the patient be admitted.

A patient is brought to the ED by ambulance after swallowing highly acidic toilet bowl cleaner 2 hours earlier. The patient is alert and oriented. What is the care teams most appropriate treatment? A) Administering syrup of ipecac B) Performing a gastric lavage C) Giving milk to drink D) Referring to psychiatry

C Feedback: A patient who has swallowed an acidic substance, such as toilet bowl cleaner, may be given milk or water to drink for dilution. Gastric lavage must be performed within 1 hour of ingestion. A psychiatric consult may be considered once the patient is physically stable and it is deemed appropriate by the physician. Syrup of ipecac is no longer used in clinical settings.

A patient has had an ischemic stroke and has been admitted to the medical unit. What action should the nurse perform to best prevent joint deformities? A) Place the patient in the prone position for 30 minutes/day. B) Assist the patient in acutely flexing the thigh to promote movement. C) Place a pillow in the axilla when there is limited external rotation. D) Place patients hand in pronation.

C Feedback: A pillow in the axilla prevents adduction of the affected shoulder and keeps the arm away from the chest. The prone position with a pillow under the pelvis, not flat, promotes hyperextension of the hip joints, essential for normal gait. To promote venous return and prevent edema, the upper thigh should not be flexed acutely. The hand is placed in slight supination, not pronation, which is its most functional position.

While developing an emergency operations plan (EOP), the committee is discussing the components of the EOP. During the post-incident response of an emergency operations plan, what activity will take place? A) Deciding when the facility will go from disaster response to daily activities B) Conducting practice drills for the community and facility C) Conducting a critique and debriefing for all involved in the incident D) Replacing the resources in the facility

C Feedback: A post-incident response includes critiquing and debriefing all parties involved immediately and at later dates. It does not include the decision to go from disaster response to daily activities; it does not include practice drills; and it does not include replacement of resources in the facility.

A patient with Parkinsons disease is undergoing a swallowing assessment because she has recently developed adventitious lung sounds. The patients nutritional needs should be met by what method? A) Total parenteral nutrition (TPN) B) Provision of a low-residue diet C) Semisolid food with thick liquids D) Minced foods and a fluid restriction

C Feedback: A semisolid diet with thick liquids is easier for a patient with swallowing difficulties to consume than is a solid diet. Low-residue foods and fluid restriction are unnecessary and counterproductive to the patients nutritional status. The patients status does not warrant TPN.

A nurse is working with a patient who has been scheduled for a percutaneous coronary intervention (PCI) later in the week. What anticipatory guidance should the nurse provide to the patient? A) He will remain on bed rest for 48 to 72 hours after the procedure. B) He will be given vitamin K infusions to prevent bleeding following PCI. C) A sheath will be placed over the insertion site after the procedure is finished. D) The procedure will likely be repeated in 6 to 8 weeks to ensure success.

C Feedback: A sheath is placed over the PCI access site and kept in place until adequate coagulation is achieved. Patients resume activity a few hours after PCI and repeated treatments may or may not be necessary. Anticoagulants, not vitamin K, are administered during PCI.

A patient who underwent surgery for esophageal cancer is admitted to the critical care unit following postanesthetic recovery. Which of the following should be included in the patients immediate postoperative plan of care? A) Teaching the patient to self-suction B) Performing chest physiotherapy to promote oxygenation C) Positioning the patient to prevent gastric reflux D) Providing a regular diet as tolerated

C Feedback: After recovering from the effects of anesthesia, the patient is placed in a low Fowlers position, and later in a Fowlers position, to help prevent reflux of gastric secretions. The patient is observed carefully for regurgitation and dyspnea because a common postoperative complication is aspiration pneumonia. In this period of recovery, self-suctioning is also not likely realistic or safe. Chest physiotherapy is contraindicated because of the risk of aspiration. Nutrition is prioritized, but a regular diet is contraindicated in the immediate recovery from esophageal surgery.

A patient has undergone surgery for oral cancer and has just been extubated in postanesthetic recovery. What nursing action best promotes comfort and facilitates spontaneous breathing for this patient? A) Placing the patient in a left lateral position B) Administering opioids as ordered C) Placing the patient in Fowlers position D) Teaching the patient to use the patient-controlled analgesia (PCA) syste

C Feedback: After the endotracheal tube or airway has been removed and the effects of the anesthesia have worn off, the patient may be placed in Fowlers position to facilitate breathing and promote comfort. Lateral positioning does not facilitate oxygenation or comfort. Medications do not facilitate spontaneous breathing.

A triage nurse in the emergency department (ED) receives a phone call from a frantic father who saw his 4-year-old child tip a pot of boiling water onto her chest. The father has called an ambulance. What would the nurse in the ED receiving the call instruct the father to do? A) Cover the burn with ice and secure with a towel. B) Apply butter to the area that is burned. C) Immerse the child in a cool bath. D) Avoid touching the burned area under any circumstances.

C Feedback: After the flames or heat source have been removed or extinguished, the burned area and adherent clothing are soaked with cool water briefly to cool the wound and halt the burning process. Cool water is the best first-aid measure. Ice and butter are contraindicated. Appropriate first aid necessitates touching the burn.

A nurse has had contact with a patient who developed smallpox and became febrile after a terrorist attack. This nurse will require what treatment? A) Watchful waiting B) Treatment with colony-stimulating factors (CSFs) C) Vaccination D) Treatment with ceftriaxone

C Feedback: All people who have had household or face-to-face contact with a patient with small pox after the fever begins should be vaccinated within 4 days to prevent infection and death. Watchful waiting would be inappropriate and CSFs are not used for treatment. Vaccination, rather than antibiotics, is the treatment of choice.

A patient is brought to the ED by friends. The friends tell the nurse that the patient was using cocaine at a party. On arrival to the ED the patient is in visible distress with an axillary temperature of 40.1C (104.2F). What would be the priority nursing action for this patient? A) Monitor cardiovascular effects. B) Administer antipyretics. C) Ensure airway and ventilation. D) Prevent seizure activity.

C Feedback: Although all of the listed actions may be necessary for this patients care, the priority is to establish a patent airway and adequate ventilation.

You are writing a care plan for a patient who has been diagnosed with angina pectoris. The patient describes herself as being distressed and shocked by her new diagnosis. What nursing diagnosis is most clearly suggested by the womans statement? A) Spiritual distress related to change in health status B) Acute confusion related to prognosis for recovery C) Anxiety related to cardiac symptoms D) Deficient knowledge related to treatment of angina pectoris

C Feedback: Although further assessment is warranted, it is not unlikely that the patient is experiencing anxiety. In patients with CAD, this often relates to the threat of sudden death. There is no evidence of confusion (i.e., delirium or dementia) and there may or may not be a spiritual element to her concerns. Similarly, it is not clear that a lack of knowledge or information is the root of her anxiety.

The ED nurse is caring for a patient who has been brought in by ambulance after sustaining a fall at home. What physical assessment finding is suggestive of a basilar skull fracture? A) Epistaxis B) Periorbital edema C) Bruising over the mastoid D) Unilateral facial numbness

C Feedback: An area of ecchymosis (bruising) may be seen over the mastoid (Battles sign) in a basilar skull fracture. Numbness, edema, and epistaxis are not directly associated with a basilar skull f

A patient is brought to the ED by her family after falling off the roof. A family member tells the nurse that when the patient fell she was knocked out, but came to and seemed okay. Now she is complaining of a severe headache and not feeling well. The care team suspects an epidural hematoma, prompting the nurse to prepare for which priority intervention? A) Insertion of an intracranial monitoring device B) Treatment with antihypertensives C) Emergency craniotomy D) Administration of anticoagulant therapy

C Feedback: An epidural hematoma is considered an extreme emergency. Marked neurologic deficit or respiratory arrest can occur within minutes. Treatment consists of making an opening through the skull to decrease ICP emergently, remove the clot, and control the bleeding. Antihypertensive medications would not be a priority. Anticoagulant therapy should not be ordered for a patient who has a cranial bleed. This could further increase bleeding activity. Insertion of an intracranial monitoring device may be done during the surgery, but is not priority for this patient.

Diagnostic imaging reveals that the quantity of fluid in a clients pericardial sac is dangerously increased. The nurse should collaborate with the other members of the care team to prevent the development of what complication? A) Pulmonary edema B) Pericardiocentesis C) Cardiac tamponade D) Pericarditis

C Feedback: An increase in pericardial fluid raises the pressure within the pericardial sac and compresses the heart, eventually causing cardiac tamponade. Pericardiocentesis is the treatment for this complication. Pericarditis and pulmonary edema do not result from this pathophysiological pro

Family members bring a patient to the ED with pale cool skin, sudden midsternal chest pain unrelieved with rest, and a history of CAD. How should the nurse best interpret these initial data? A) The symptoms indicate angina and should be treated as such. B) The symptoms indicate a pulmonary etiology rather than a cardiac etiology. C) The symptoms indicate an acute coronary episode and should be treated as such. D) Treatment should be determined pending the results of an exercise stress test.

C Feedback: Angina and MI have similar symptoms and are considered the same process, but are on different points along a continuum. That the patients symptoms are unrelieved by rest suggests an acute coronary episode rather than angina. Pale cool skin and sudden onset are inconsistent with a pulmonary etiology. Treatment should be initiated immediately regardless of diagnosis.

A patient has been admitted to the medical unit with signs and symptoms that are suggestive of anthrax infection. The nurse should anticipate what intervention? A) Administration of acyclovir B) Hematopoietic stem cell transplantation (HSCT) C) Administration of penicillin D) Hemodialysis

C Feedback: Anthrax infection is treated with penicillin. Acyclovir is ineffective because anthrax is a bacterium. Dialysis and HSCT are not indicated.

The nurse notes that a patient has developed a cough productive for mucoid sputum, is short of breath, has cyanotic hands, and has noisy, moist-sounding, rapid breathing. These symptoms and signs are suggestive of what health problem? A) Pericarditis B) Cardiomyopathy C) Pulmonary edema D) Right ventricular hypertrophy

C Feedback: As a result of decreased cerebral oxygenation, the patient with pulmonary edema becomes increasingly restless and anxious. Along with a sudden onset of breathlessness and a sense of suffocation, the patients hands become cold and moist, the nail beds become cyanotic (bluish), and the skin turns ashen (gray). The pulse is weak and rapid, and the neck veins are distended. Incessant coughing may occur, producing increasing quantities of foamy sputum. Pericarditis, ventricular hypertrophy, and cardiomyopathy do not involve wet breath sounds or mucus production.

A patient experienced a 33% TBSA burn 72 hours ago. The nurse observes that the patients hourly urine output has been steadily increasing over the past 24 hours. How should the nurse best respond to this finding? A) Obtain an order to reduce the rate of the patients IV fluid infusion. B) Report the patients early signs of acute kidney injury (AKI). C) Recognize that the patient is experiencing an expected onset of diuresis. D) Administer sodium chloride as ordered to compensate for this fluid loss.

C Feedback: As capillaries regain integrity, 48 or more hours after the burn, fluid moves from the interstitial to the intravascular compartment and diuresis begins. This is an expected development and does not require a reduction in the IV infusion rate or the administration of NaCl. Diuresis is not suggestive of AKI.

A patient with severe burns is admitted to the intensive care unit to stabilize and begin fluid resuscitation before transport to the burn center. The nurse should monitor the patient closely for what signs of the onset of burn shock? A) Confusion B) High fever C) Decreased blood pressure D) Sudden agitation

C Feedback: As fluid loss continues and vascular volume decreases, cardiac output continues to decrease and the blood pressure drops, marking the onset of burn shock. Shock and the accompanying hemodynamic changes are not normally accompanied by confusion, fever, or agitation.

A patient with a partial-thickness burn injury had Biobrane applied 2 weeks ago. The nurse notices that the Biobrane is separating from the burn wound. What is the nurses most appropriate intervention? A) Reinforce the Biobrane dressing with another piece of Biobrane. B) Remove the Biobrane dressing and apply a new dressing. C) Trim away the separated Biobrane. D) Notify the physician for further emergency-related orders.

C Feedback: As the Biobrane gradually separates, it is trimmed, leaving a healed wound. When the Biobrane dressing adheres to the wound, the wound remains stable and the Biobrane can remain in place for 3 to 4 weeks. There is no need to reinforce the Biobrane nor to remove it and apply a new dressing. There is not likely any need to notify the physician for further orders.

client with several chronic health problems has been newly diagnosed with a qualitative platelet defect. What component of the patients previous medication regimen may have contributed to the development of this disorder? A) Calcium carbonate B) Vitamin B12 C) Aspirin D) Vitamin

C Feedback: Aspirin may induce a platelet disorder. Even small amounts of aspirin reduce normal platelet aggregation, and the prolonged bleeding time lasts for several days after aspirin ingestion. Calcium, vitamin D, and vitamin B12 do not have the potential to induce a platelet def

An emergency department nurse is triaging a 77-year-old man who presents with uncharacteristic fatigue as well as back and rib pain. The patient denies any recent injuries. The nurse should recognize the need for this patient to be assessed for what health problem? A) Hodgkin disease B) Non-Hodgkin lymphoma C) Multiple myeloma D) Acute thrombocythemia

C Feedback: Back pain, which is often a presenting symptom in multiple myeloma, should be closely investigated in older patients. The lymphomas and bleeding disorders do not typically present with the primary symptom of back pain or rib pain.

A patient has undergone a laparoscopic cholecystectomy and is being prepared for discharge home. When providing health education, the nurse should prioritize which of the following topics? A) Management of fluid balance in the home setting B) The need for blood glucose monitoring for the next week C) Signs and symptoms of intra-abdominal complications D) Appropriate use of prescribed pancreatic enzymes

C Feedback: Because of the early discharge following laparoscopic cholecystectomy, the patient needs thorough education in the signs and symptoms of complications. Fluid balance is not typically a problem in the recovery period after laparoscopic cholecystectomy. There is no need for blood glucose monitoring or pancreatic enzymes.

The nurse is caring for a patient who has developed obvious signs of pulmonary edema. What is the priority nursing action? A) Lay the patient flat. B) Notify the family of the patients critical state. C) Stay with the patient. D) Update the physician.

C Feedback: Because the patient has an unstable condition, the nurse must remain with the patient. The physician must be updated promptly, but the patient should not be left alone in order for this to happen. Supine positioning is unlikely to relieve dyspnea. The family should be informed, but this is not the priority action.

A patient with a myelodysplastic syndrome is being treated on the medical unit. What assessment finding should prompt the nurse to contact the patients primary care provider? A) The patient is experiencing a frontal lobe headache. B) The patient has an episode of urinary incontinence. C) The patient has an oral temperature of 37.5C (99.5F). D) The patients SpO2 is 91% on room air

C Feedback: Because the patient with MDS is at a high risk for infection, any early signs of infection must be reported promptly. The nurse should address each of the listed assessment findings, but none is as direct a threat to the patients immediate health as an infection.

The nurse is caring for an 84-year-old man who has just returned from the OR after inguinal hernia repair. The OR report indicates that the patient received large volumes of IV fluids during surgery and the nurse recognizes that the patient is at risk for left-sided heart failure. What signs and symptoms would indicate left-sided heart failure? A) Jugular vein distention B) Right upper quadrant pain C) Bibasilar fine crackles D) Dependent edema

C Feedback: Bibasilar fine crackles are a sign of alveolar fluid, a sequela of left ventricular fluid, or pressure overload. Jugular vein distention, right upper quadrant pain (hepatomegaly), and dependent edema are caused by right-sided heart failure, usually a chronic condition.

The nurse is caring for a patient after kidney surgery. The nurse is aware that bleeding is a major complication of kidney surgery and that if it goes undetected and untreated can result in hypovolemia and hemorrhagic shock in the patient. When assessing for bleeding, what assessment parameter should the nurse evaluate? A) Oral intake B) Pain intensity C) Level of consciousness D) Radiaion of pain

C Feedback: Bleeding is a major complication of kidney surgery. If undetected and untreated, this can result in hypovolemia and hemorrhagic shock. The nurses role is to observe for these complications, to report their signs and symptoms, and to administer prescribed parenteral fluids and blood and blood components. Monitoring of vital signs, skin condition, the urinary drainage system, the surgical incision, and the level of consciousness is necessary to detect evidence of bleeding, decreased circulating blood, and fluid volume and cardiac output. Bleeding is not normally evidenced by changes in pain or oral intake.

A patient with a new diagnosis of ischemic stroke is deemed to be a candidate for treatment with tissue plasminogen activator (t-PA) and has been admitted to the ICU. In addition to closely monitoring the patients cardiac and neurologic status, the nurse monitors the patient for signs of what complication? A) Acute pain B) Septicemia C) Bleeding D) Seizures

C Feedback: Bleeding is the most common side effect of t-PA administration, and the patient is closely monitored for any bleeding. Septicemia, pain, and seizures are much less likely to result from thrombolytic therapy.

A patient has just been diagnosed with chronic pancreatitis. The patient is underweight and in severe pain and diagnostic testing indicates that over 80% of the patients pancreas has been destroyed. The patient asks the nurse why the diagnosis was not made earlier in the disease process. What would be the nurses best response? A) The symptoms of pancreatitis mimic those of much less serious illnesses. B) Your body doesnt require pancreatic function until it is under great stress, so it is easy to go unnoticed. C) Chronic pancreatitis often goes undetected until a large majority of pancreatic function is lost. D) Its likely that your other organs were compensating for your decreased pancreatic function.

C Feedback: By the time symptoms occur in chronic pancreatitis, approximately 90% of normal acinar cell function (exocrine function) has been lost. Late detection is not usually attributable to the vagueness of symptoms. The pancreas contributes continually to homeostasis and other organs are unable to perform its physiologic functions.

A patient is admitted to the ICU after a motor vehicle accident. On the second day of the hospital admission, the patient develops acute kidney injury. The patient is hemodynamically unstable, but renal replacement therapy is needed to manage the patients hypervolemia and hyperkalemia. Which of the following therapies will the patients hemodynamic status best tolerate? A) Hemodialysis B) Peritoneal dialysis C) Continuous venovenous hemodialysis (CVVHD) D) Plasmapheresis

C Feedback: CVVHD facilitates the removal of uremic toxins and fluid. The hemodynamic effects of CVVHD are usually mild in comparison to hemodialysis, so CVVHD is best tolerated by an unstable patient. Peritoneal dialysis is not the best choice, as the patient may have sustained abdominal injuries during the accident and catheter placement would be risky. Plasmapheresis does not achieve fluid removal and electrolyte balance.

During a patients care conference, the team is discussing whether the patient is a candidate for cardiac conduction surgery. What would be the most important criterion for a patient to have this surgery? A) Angina pectoris not responsive to other treatments B) Decreased activity tolerance related to decreased cardiac output C) Atrial and ventricular tachycardias not responsive to other treatments D) Ventricular fibrillation not responsive to other treatments

C Feedback: Cardiac conduction surgery is considered in patients who do not respond to medications and antitachycardia pacing. Angina, reduced activity tolerance, and ventricular fibrillation are not criteria.

A community health nurse is caring for a patient whose multiple health problems include chronic pancreatitis. During the most recent visit, the nurse notes that the patient is experiencing severe abdominal pain and has vomited 3 times in the past several hours. What is the nurses most appropriate action? A) Administer a PRN dose of pancreatic enzymes as ordered. B) Teach the patient about the importance of abstaining from alcohol. C) Arrange for the patient to be transported to the hospital. D) Insert an NG tube, if available, and stay with the patient.

C Feedback: Chronic pancreatitis is characterized by recurring attacks of severe upper abdominal and back pain, accompanied by vomiting. The onset of these acute symptoms warrants hospital treatment. Pancreatic enzymes are not indicated and an NG tube would not be inserted in the home setting. Patient education is a later priority that may or may not be relevant.

A patient is recovering in the hospital from cardiac surgery. The nurse has identified the diagnosis of risk for ineffective airway clearance related to pulmonary secretions. What intervention best addresses this risk? A) Administration of bronchodilators by nebulizer B) Administration of inhaled corticosteroids by metered dose inhaler (MDI) C) Patients consistent performance of deep breathing and coughing exercises D) Patients active participation in the cardiac rehabilitation program

C Feedback: Clearance of pulmonary secretions is accomplished by frequent repositioning of the patient, suctioning, and chest physical therapy, as well as educating and encouraging the patient to breathe deeply and cough. Medications are not normally used to achieve this goal. Rehabilitation is important, but will not necessarily aid the mobilization of respiratory secretions.

A nurse is planning the care of a patient who has been admitted to the medical unit with a diagnosis of multiple myeloma. In the patients care plan, the nurse has identified a diagnosis of Risk for Injury. What pathophysiologic effect of multiple myeloma most contributes to this risk? A) Labyrinthitis B) Left ventricular hypertrophy C) Decreased bone density D) Hypercoagulation

C Feedback: Clients with multiple myeloma are at risk for pathologic bone fractures secondary to diffuse osteoporosis and osteolytic lesions. Labyrinthitis is uncharacteristic, and patients do not normally experience hypercoagulation or cardiac hypertrophy.

Following a traumatic brain injury, a patient has been in a coma for several days. Which of the following statements is true of this patients current LOC? A) The patient occasionally makes incomprehensible sounds. B) The patients current LOC will likely become a permanent state. C) The patient may occasionally make nonpurposeful movements. D) The patient is incapable of spontaneous respirations.

C Feedback: Coma is a clinical state of unarousable unresponsiveness in which no purposeful responses to internal or external stimuli occur, although nonpurposeful responses to painful stimuli and brain stem reflexes may be present. Verbal sounds, however, are atypical. Ventilator support may or may not be necessary. Comas are not permanent states.

The nurse is discharging home a patient who suffered a stroke. He has a flaccid right arm and leg and is experiencing problems with urinary incontinence. The nurse makes a referral to a home health nurse because of an awareness of what common patient response to a change in body image? A) Denial B) Fear C) Depression D) Disassociation

C Feedback: Depression is a common and serious problem in the patient who has had a stroke. It can result from a profound disruption in his or her life and changes in total function, leaving the patient with a loss of independence. The nurse needs to encourage the patient to verbalize feelings to assess the effect of the stroke on self-esteem. Denial, fear, and disassociation are not the most common patient response to a change in body image, although each can occur in some patients.

A 23-year-old woman is brought to the ED complaining of stomach cramps, nausea, vomiting, and diarrhea. The care team suspects food poisoning. What is the key to treatment in food poisoning? A) Administering IV antibiotics B) Assessing immunization status C) Determining the source and type of food poisoning D) Determining if anyone else in the family is ill

C Feedback: Determining the source and type of food poisoning is essential to treatment, and is more important than determining other sick family members. Antibiotics are not normally indicated and immunizations are not relevant to diagnosis or treatment of food poisoning.

A patient has experienced burns to his upper thighs and knees. Following the application of new wound dressings, the nurse should perform what nursing action? A) Instruct the patient to keep the wound site in a dependent position. B) Administer PRN analgesia as ordered. C) Assess the patients peripheral pulses distal to the dressing. D) Assist with passive range of motion exercises to set the new dressing.

C Feedback: Dressings can impede circulation if they are wrapped too tightly. The peripheral pulses must be checked frequently and burned extremities elevated. Dependent positioning does not need to be maintained. PRN analgesics should be administered prior to the dressing change. ROM exercises do not normally follow a dressing change.

A patient with multiple trauma is brought to the ED by ambulance after a fall while rock climbing. What is a responsibility of the ED nurse in this patients care? A) Intubating the patient B) Notifying family members C) Ensuring IV access D) Delivering specimens to the laboratory

C Feedback: ED nursing responsibilities include ensuring airway and IV access. Nurses are not normally responsible for notifying family members. Nurses collect specimens, but are not responsible for their delivery. Physicians or other team members with specialized training intubate the patie

A patient is admitted to the ICU with acute pancreatitis. The patients family asks what causes acute pancreatitis. The critical care nurse knows that a majority of patients with acute pancreatitis have what? A) Type 1 diabetes B) An impaired immune system C) Undiagnosed chronic pancreatitis D) An amylase deficiency

C Feedback: Eighty percent of patients with acute pancreatitis have biliary tract disease or a history of long-term alcohol abuse. These patients usually have had undiagnosed chronic pancreatitis before their first episode of acute pancreatitis. Diabetes, an impaired immune function, and amylase deficiency are not specific precursors to acute pancreatitis.

A nurse is teaching a patient with a partial-thickness wound how to wear his elastic pressure garment. How would the nurse instruct the patient to wear this garment? A) 4 to 6 hours a day for 6 months B) During waking hours for 2 to 3 months after the injury C) Continuously D) At night while sleeping for a year after the injury

C Feedback: Elastic pressure garments are worn continuously (i.e., 23 hours a day).

A patient is experiencing respiratory insufficiency and cannot maintain spontaneous respirations. The nurse suspects that the physician will perform which of the following actions? A) Insert an oropharyngeal airway. B) Perform the jaw thrust maneuver. C) Perform endotracheal intubation. D) Perform a cricothyroidotomy.

C Feedback: Endotracheal tubes are used in cases when the patient cannot be ventilated with an oropharyngeal airway, which is used in patients who are breathing spontaneously. The jaw thrust maneuver does not establish an airway and cricothyroidotomy would be performed as a last resort.

A nurse who provides care in a walk-in clinic assesses a wide range of individuals. The nurse should identify which of the following patients as having the highest risk for chronic pancreatitis? A) A 45-year-old obese woman with a high-fat diet B) An 18-year-old man who is a weekend binge drinker C) A 39-year-old man with chronic alcoholism D) A 51-year-old woman who smokes one-and-a-half packs of cigarettes per day

C Feedback: Excessive and prolonged consumption of alcohol accounts for approximately 70% to 80% of all cases of chronic pancreatitis.

A patient who has undergone a valve replacement with a mechanical valve prosthesis is due to be discharged home. During discharge teaching, the nurse should discuss the importance of antibiotic prophylaxis prior to which of the following? A) Exposure to immunocompromised individuals B) Future hospital admissions C) Dental procedures D) Live vaccinations

C Feedback: Following mechanical valve replacement, antibiotic prophylaxis is necessary before dental procedures involving manipulation of gingival tissue, the periapical area of the teeth or perforation of the oral mucosa (not including routine anesthetic injections, placement of orthodontic brackets, or loss of deciduous teeth). There are no current recommendations around antibiotic prophylaxis prior to vaccination, future hospital admissions, or exposure to people who are immunosuppressed.

A woman who is in her third trimester of pregnancy has been experiencing an exacerbation of iron- deficiency anemia in recent weeks. When providing the patient with nutritional guidelines and meal suggestions, what foods would be most likely to increase the womans iron stores? A) Salmon accompanied by whole milk B) Mixed vegetables and brown rice C) Beef liver accompanied by orange juice D) Yogurt, almonds, and whole grain oats

C Feedback: Food sources high in iron include organ meats, other meats, beans (e.g., black, pinto, and garbanzo), leafy green vegetables, raisins, and molasses. Taking iron-rich foods with a source of vitamin C (e.g., orange juice) enhances the absorption of iron. All of the listed foods are nutritious, but liver and orange juice are most likely to be of benefit.

A patient diagnosed with a hemorrhagic stroke has been admitted to the neurologic ICU. The nurse knows that teaching for the patient and family needs to begin as soon as the patient is settled on the unit and will continue until the patient is discharged. What will family education need to include? A) How to differentiate between hemorrhagic and ischemic stroke B) Risk factors for ischemic stroke C) How to correctly modify the home environment D) Techniques for adjusting the patients medication dosages at home

C Feedback: For a patient with a hemorrhagic stroke, teaching addresses the use of assistive devices or modification of the home environment to help the patient live with the disability. This is more important to the patients needs than knowing about risk factors for ischemic stroke. It is not necessary for the family to differentiate between different types of strokes. Medication regimens should never be altered without consultation.

A patient who has been on long-term phenytoin (Dilantin) therapy is admitted to the unit. In light of the adverse of effects of this medication, the nurse should prioritize which of the following in the patients plan of care? A) Monitoring of pulse oximetry B) Administration of a low-protein diet C) Administration of thorough oral hygiene D) Fluid restriction as ordered

C Feedback: Gingival hyperplasia (swollen and tender gums) can be associated with long-term phenytoin (Dilantin) use. Thorough oral hygiene should be provided consistently and encouraged after discharge. Fluid and protein restriction are contraindicated and there is no particular need for constant oxygen saturation monitoring.

The nurse is planning discharge teaching for a patient with a newly inserted permanent pacemaker. What is the priority teaching point for this patient? A) Start lifting the arm above the shoulder right away to prevent chest wall adhesion. B) Avoid cooking with a microwave oven. C) Avoid exposure to high-voltage electrical generators. D) Avoid walking through store and library antitheft devices.

C Feedback: High-output electrical generators can reprogram pacemakers and should be avoided. Recent pacemaker technology allows patients to safely use most household electronic appliances and devices (e.g., microwave ovens). The affected arm should not be raised above the shoulder for 1 week following placement of the pacemaker. Antitheft alarms may be triggered so patients should be taught to walk through them quickly and avoid standing in or near these devices. These alarms generally do not interfere with pacemaker function.

A hospice nurse is caring for a 22-year-old with a terminal diagnosis of leukemia. When updating this patients plan of nursing care, what should the nurse prioritize? A) Interventions aimed at maximizing quantity of life B) Providing financial advice to pay for care C) Providing realistic emotional preparation for death D) Making suggestions to maximize family social interactions after the patients death

C Feedback: Hospice care focuses on quality of life, but, by necessity, it usually includes realistic emotional, social, spiritual, and financial preparation for death. Financial advice and actions aimed at post-death interaction would not be appropriate priorities.

A nurse at a long-term care facility is amending the care plan of a resident who has just been diagnosed with essential thrombocythemia (ET). The nurse should anticipate the administration of what medication? A) Dalteparin B) Allopurinol C) Hydroxyurea D) Hydrochlorothiazide

C Feedback: Hydroxyurea is effective in lowering the platelet count for patients with ET. Dalteparin, allopurinol, and HCTZ do not have this therapeutic effect.

The nurse is caring for a patient with a brain tumor. What drug would the nurse expect to be ordered to reduce the edema surrounding the tumor? A) Solumedrol B) Dextromethorphan C) Dexamethasone D) Furosemide

C Feedback: If a brain tumor is the cause of the increased ICP, corticosteroids (e.g., dexamethasone) help reduce the edema surrounding the tumor. Solumedrol, a steroid, and furosemide, a loop diuretic, are not the drugs of choice in this instance. Dextromethorphan is used in cough medicines.

The nurse is assessing a patient who was admitted to the critical care unit 3 hours ago following cardiac surgery. The nurses most recent assessment reveals that the patients left pedal pulses are not palpable and that the right pedal pulses are rated at +2. What is the nurses best response? A) Document this expected assessment finding during the initial postoperative period. B) Reposition the patient with his left leg in a dependent position. C) Inform the patients physician of this assessment finding. D) Administer an ordered dose of subcutaneous heparin.

C Feedback: If a pulse is absent in any extremity, the cause may be prior catheterization of that extremity, chronic peripheral vascular disease, or a thromboembolic obstruction. The nurse immediately reports newly identified absence of any pulse.

A junior nursing student is having an observation day in the operating room. Early in the day, the student tells the OR nurse that her eyes are swelling and she is having trouble breathing. What should the nurse suspect? A) Cytotoxic reaction due to contact with the powder in the gloves B) Immune complex reaction due to contact with anesthetic gases C) Anaphylaxis due to a latex allergy D) Delayed reaction due to exposure to cleaning products

C Feedback: Immediate hypersensitivity to latex, a type I allergic reaction, is mediated by the IgE mast cell system. Symptoms can include rhinitis, conjunctivitis, asthma, and anaphylaxis. The term latex allergy is usually used to describe the type I reaction. The rapid onset is not consistent with a cytotoxic reaction, an immune complex reaction, or a delayed reaction.

A patient with amyotrophic lateral sclerosis (ALS) is being visited by the home health nurse who is creating a care plan. What nursing diagnosis is most likely for a patient with this condition? A) Chronic confusion B) Impaired urinary elimination C) Impaired verbal communication D) Bowel incontinence

C Feedback: Impaired communication is an appropriate nursing diagnosis; the voice in patients with ALS assumes a nasal sound and articulation becomes so disrupted that speech is unintelligible. Intellectual function is marginally impaired in patients with late ALS. Usually, the anal and bladder sphincters are intact because the spinal nerves that control muscles of the rectum and urinary bladder are not affected.

A student nurse is caring for a patient who has a diagnosis of acute pancreatitis and who is receiving parenteral nutrition. The student should prioritize which of the following assessments? A) Fluid output B) Oral intake C) Blood glucose levels D) BUN and creatinine levels

C Feedback: In addition to administering enteral or parenteral nutrition, the nurse monitors serum glucose levels every 4 to 6 hours. Output should be monitored but in most cases it is not more important than serum glucose levels. A patient on parenteral nutrition would have no oral intake to monitor. Blood sugar levels are more likely to be unstable than indicators of renal function.

A group of disaster survivors is working with the critical incident stress management (CISM) team. Members of this team should be guided by what goal? A) Determining whether the incident was managed effectively B) Educating survivors on potential coping strategies for future disasters C) Providing individuals with education about recognizing stress reactions D) Determining if individuals responded appropriately during the incident

C Feedback: In defusing, patients are given information about recognizing stress reactions and how to deal with handling the stress they may experience. Debriefing involves asking patients about their current emotional coping and symptoms, following up, and identifying patients who require further assessment and assistance in dealing with the stress experienced. The CISM team does not focus primarily on the management of the incident or on providing skills for future incidents.

After receiving a diagnosis of acute lymphocytic leukemia, a patient is visibly distraught, stating, I have no idea where to go from here. How should the nurse prepare to meet this patients psychosocial needs? A) Assess the patients previous experience with the health care system. B) Reassure the patient that treatment will be challenging but successful. C) Assess the patients specific needs for education and support. D) Identify the patients plan of medical care.

C Feedback: In order to meets the patients needs, the nurse must first identify the specific nature of these needs. According to the nursing process, assessment must precede interventions. The plan of medical care is important, but not central to the provision of support. The patients previous health care is not a primary consideration, and the nurse cannot assure the patient of successful treatment.

A patient is admitted to the neurologic ICU with a spinal cord injury. When assessing the patient the nurse notes there is a sudden depression of reflex activity in the spinal cord below the level of injury. What should the nurse suspect? A) Epidural hemorrhage B) Hypertensive emergency C) Spinal shock D) Hypovolemia

C Feedback: In spinal shock, the reflexes are absent, BP and heart rate fall, and respiratory failure can occur. Hypovolemia, hemorrhage, and hypertension do not cause this sudden change in neurologic function.

An adult patient with third-degree AV block is admitted to the cardiac care unit and placed on continuous cardiac monitoring. What rhythm characteristic will the ECG most likely show? A) PP interval and RR interval are irregular. B) PP interval is equal to RR interval. C) Fewer QRS complexes than P waves D) PR interval is constant.

C Feedback: In third-degree AV block, no atrial impulse is conducted through the AV node into the ventricles. As a result, there are impulses stimulating the atria and impulses stimulating the ventricles. Therefore, there are more P waves than QRS complexes due to the difference in the natural pacemaker (nodes) rates of the heart. The other listed ECG changes are not consistent with this diagnosis.

The nurse has observed that an older adult patient with a diagnosis of end-stage renal failure seems to prefer to have his eldest son make all of his health care decisions. While the family is visiting, the patient explains to you that this is a cultural practice and very important to him. How should you respond? A) Privately ask the son to allow the patient to make his own health care decisions. B) Explain to the patient that he is responsible for his own decisions. C) Work with the team to negotiate informed consent. D) Avoid divulging information to the eldest son.

C Feedback: In this case of a patient who wishes to defer decisions to his son, the nurse can work with the team to negotiate informed consent, respecting the patients right not to participate in decision making and honoring his familys cultural practices.

The nurse is caring for an adult patient who had symptoms of unstable angina upon admission to the hospital. What nursing diagnosis underlies the discomfort associated with angina? A) Ineffective breathing pattern related to decreased cardiac output B) Anxiety related to fear of death C) Ineffective cardiopulmonary tissue perfusion related to coronary artery disease (CAD) D) Impaired skin integrity related to CAD

C Feedback: Ineffective cardiopulmonary tissue perfusion directly results in the symptoms of discomfort associated with angina. Anxiety and ineffective breathing may result from angina chest pain, but they are not the causes. Skin integrity is not impaired by the effects of angina.

A young man with a diagnosis of hemophilia A has been brought to emergency department after suffering a workplace accident resulting in bleeding. Rapid assessment has revealed the source of the patients bleeding and established that his vital signs are stable. What should be the nurses next action? A) Position the patient in a prone position to minimize bleeding B) Establish IV access for the administration of vitamin K. C) Prepare for the administration of factor VIII. D) Administer a normal saline bolus to increase circulatory volume.

C Feedback: Injuries in patients with hemophilia necessitate prompt administration of clotting factors. Vitamin K is not a treatment modality and a prone position will not be appropriate for all types and locations of wounds. A normal saline bolus is not indicated.

A patient has returned to the floor after having a thyroidectomy for thyroid cancer. The nurse knows that sometimes during thyroid surgery the parathyroid glands can be injured or removed. What laboratory finding may be an early indication of parathyroid gland injury or removal? A) Hyponatremia B) Hypophosphatemia C) Hypocalcemia D) Hypokalemia

C Feedback: Injury or removal of the parathyroid glands may produce a disturbance in calcium metabolism and result in a decline of calcium levels (hypocalcemia). As the blood calcium levels fall, hyperirritability of the nerves occurs, with spasms of the hands and feet and muscle twitching. This group of symptoms is known as tetany and must be reported to the physician immediately, because laryngospasm may occur and obstruct the airway. Hypophosphatemia, hyponatremia, and hypokalemia are not expected responses to parathyroid injury or removal. In fact, parathyroid removal or injury that results in hypocalcemia may lead to hyperphosphatemia.

The organization of a patients care on the palliative care unit is based on interdisciplinary collaboration. How does interdisciplinary collaboration differ from multidisciplinary practice? A) It is based on the participation of clinicians without a team leader. B) It is based on clinicians of varied backgrounds integrating their separate plans of care. C) It is based on communication and cooperation between disciplines. D) It is based on medical expertise and patient preference with the support of nursing.

C Feedback: Interdisciplinary collaboration, which is different from multidisciplinary practice, is based on communication and cooperation among the various disciplines, each member of the team contributing to a single integrated care plan that addresses the needs of the patient and family. Multidisciplinary care refers to participation of clinicians with varied backgrounds and skill sets, but without coordination and integration. Interdisciplinary collaboration is not based on patient preference and should not prioritize medical expertise over other disciplines.

A patient has been admitted with an aortic valve stenosis and has been scheduled for a balloon valvuloplasty in the cardiac catheterization lab later today. During the admission assessment, the patient tells the nurse he has thoracolumbar scoliosis and is concerned about lying down for any extended period of time. What is a priority action for the nurse? A) Arrange for an alternative bed. B) Measure the degree of the curvature. C) Notify the surgeon immediately. D) Note the scoliosis on the intake assessment.

C Feedback: Most often used for mitral and aortic valve stenosis, balloon valvuloplasty is contraindicated for patients with left atrial or ventricular thrombus, severe aortic root dilation, significant mitral valve regurgitation, thoracolumbar scoliosis, rotation of the great vessels, and other cardiac conditions that require open heart surgery. Therefore notifying the physician would be the priority over further physical assessment. An alternative bed would be unnecessary and documentation is not a sufficient response.

The nurse on the pediatric unit is caring for a 10-year-old boy with a diagnosis of hemophilia. The nurse knows that a priority nursing diagnosis for a patient with hemophilia is what? A) Hypothermia B) Diarrhea C) Ineffective coping D) Imbalanced nutrition: Less than body requirements

C Feedback: Most patients with hemophilia are diagnosed as children. They often require assistance in coping with the condition because it is chronic, places restrictions on their lives, and is an inherited disorder that can be passed to future generations. Children with hemophilia are not at risk of hypothermia, diarrhea, or imbalanced nutrition.

A patient who is postoperative day 1 following a CABG has produced 20 mL of urine in the past 3 hours and the nurse has confirmed the patency of the urinary catheter. What is the nurses most appropriate action? A) Document the patients low urine output and monitor closely for the next several hours. B) Contact the dietitian and suggest the need for increased oral fluid intake. C) Contact the patients physician and suggest assessment of fluid balance and renal function. D) Increase the infusion rate of the patients IV fluid to prompt an increase in renal function.

C Feedback: Nursing management includes accurate measurement of urine output. An output of less than 1 mL/kg/h may indicate hypovolemia or renal insufficiency. Prompt referral is necessary. IV fluid replacement may be indicated, but is beyond the independent scope of the dietitian or nurse.

A patient with portal hypertension has been admitted to the medical floor. The nurse should prioritize which of the following assessments related to the manifestations of this health problem? A) Assessment of blood pressure and assessment for headaches and visual changes B) Assessments for signs and symptoms of venous thromboembolism C) Daily weights and abdominal girth measurement D) Blood glucose monitoring q4h

C Feedback: Obstruction to blood flow through the damaged liver results in increased blood pressure (portal hypertension) throughout the portal venous system. This can result in varices and ascites in the abdominal cavity. Assessments related to ascites are daily weights and abdominal girths. Portal hypertension is not synonymous with cardiovascular hypertension and does not create a risk for unstable blood glucose or VTE.

The nurse is caring for acutely ill patient. What assessment finding should prompt the nurse to inform the physician that the patient may be exhibiting signs of acute kidney injury (AKI)? A) The patient is complains of an inability to initiate voiding. B) The patients urine is cloudy with a foul odor. C) The patients average urine output has been 10 mL/hr for several hours. D) The patient complains of acute flank pain.

C Feedback: Oliguria (<500 mL/d of urine) is the most common clinical situation seen in AKI. Flank pain and inability to initiate voiding are not characteristic of AKI. Cloudy, foul-smelling urine is suggestive of a urinary tract infection.

A patient is receiving care in the intensive care unit for acute pancreatitis. The nurse is aware that pancreatic necrosis is a major cause of morbidity and mortality in patients with acute pancreatitis. Consequently, the nurse should assess for what signs or symptoms of this complication? A) Sudden increase in random blood glucose readings B) Increased abdominal girth accompanied by decreased level of consciousness C) Fever, increased heart rate and decreased blood pressure D) Abdominal pain unresponsive to analgesics

C Feedback: Pancreatic necrosis is a major cause of morbidity and mortality in patients with acute pancreatitis because of resulting hemorrhage, septic shock, and multiple organ dysfunction syndrome (MODS). Signs of shock would include hypotension, tachycardia and fever. Each of the other listed changes in status warrants intervention, but none is clearly suggestive of an onset of pancreatic necrosis.

The nurse recognizes that a patient with a SCI is at risk for muscle spasticity. How can the nurse best prevent this complication of an SCI? A) Position the patient in a high Fowlers position when in bed. B) Support the knees with a pillow when the patient is in bed. C) Perform passive ROM exercises as ordered. D) Administer NSAIDs as ordered.

C Feedback: Passive ROM exercises can prevent muscle spasticity following SCI. NSAIDs are not used for this purpose. Pillows and sitting upright do not directly address the patients risk of muscle spasticity.

A patient is admitted to the neurologic ICU with a spinal cord injury. In writing the patients care plan, the nurse specifies that contractures can best be prevented by what action? A) Repositioning the patient every 2 hours B) Initiating range-of-motion exercises (ROM) as soon as the patient initiates C) Initiating (ROM) exercises as soon as possible after the injury D) Performing ROM exercises once a da

C Feedback: Passive ROM exercises should be implemented as soon as possible after injury. It would be inappropriate to wait for the patient to first initiate exercises. Toes, metatarsals, ankles, knees, and hips should be put through a full ROM at least four, and ideally five, times daily. Repositioning alone will not prevent contractures.

A nurse is caring for a patient with hepatic encephalopathy. The nurses assessment reveals that the patient exhibits episodes of confusion, is difficult to arouse from sleep and has rigid extremities on these clinical findings, the nurse should document what stage of hepatic encephalopathy? A) Stage 1 B) Stage 2 C) Stage 3 D) Stage 4

C Feedback: Patients in the third stage of hepatic encephalopathy exhibit the following symptoms: stuporous, difficult to arouse, sleeps most of the time, exhibits marked confusion, incoherent in speech, asterixis, increased deep tendon reflexes, rigidity of extremities, marked EEG abnormalities. Patients in stages 1 and 2 exhibit clinical symptoms that are not as advanced as found in stage 3, and patients in stage 4 are comatose. In stage 4, there is an absence of asterixis, absence of deep tendon reflexes, flaccidity of extremities, and EEG abnormalities.

The nurse is writing a care plan for a patient with brain metastases. The nurse decides that an appropriate nursing diagnosis is anxiety related to lack of control over the health circumstances. In establishing this plan of care for the patient, the nurse should include what intervention? A) The patient will receive antianxiety medications every 4 hours. B) The patients family will be instructed on planning the patients care. C) The patient will be encouraged to verbalize concerns related to the disease and its treatment. D) The patient will begin intensive therapy with the goal of distraction.

C Feedback: Patients need the opportunity to exercise some control over their situation. A sense of mastery can be gained as they learn to understand the disease and its treatment and how to deal with their feelings. Distraction and administering medications will not allow the patient to gain control over anxiety. Delegating planning to the family will not help the patient gain a sense of control a

A nurse caring for a patient with diabetes insipidus is reviewing laboratory results. What is an expected urinalysis finding? A) Glucose in the urine B) Albumin in the urine C) Highly dilute urine D) Leukocytes in the urine

C Feedback: Patients with diabetes insipidus produce an enormous daily output of very dilute, water-like urine with a specific gravity of 1.001 to 1.005. The urine contains no abnormal substances such as glucose or albumin. Leukocytes in the urine are not related to the condition of diabetes insipidus, but would indicate a urinary tract infection, if present in the urine.

A nurse is assessing a patient who has just been admitted to the postsurgical unit following surgical resection for the treatment of oropharyngeal cancer. What assessment should the nurse prioritize? A) Assess ability to clear oral secretions. B) Assess for signs of infection. C) Assess for a patent airway. D) Assess for ability to communicate.

C Feedback: Postoperatively, the nurse assesses for a patent airway. The patients ability to manage secretions has a direct bearing on airway patency. However, airway patency is the overarching goal. This immediate physiologic need is prioritized over communication, though this is an important consideration. Infection is not normally a threat in the immediate postoperative period.

A patient with cancer of the tongue has had a radical neck dissection. What nursing assessment would be a priority for this patient? A) Presence of acute pain and anxiety B) Tissue integrity and color of the operative site C) Respiratory status and airway clearance D) Self-esteem and body image

C Feedback: Postoperatively, the patient is assessed for complications such as altered respiratory status, wound infection, and hemorrhage. The other assessments are part of the plan of care for a patient who has had a radical neck dissection, but are not the nurses chief priority.

The nurse is caring for a patient newly diagnosed with a primary brain tumor. The patient asks the nurse where his tumor came from. What would be the nurses best response? A) Your tumor originated from somewhere outside the CNS. B) Your tumor likely started out in one of your glands. C) Your tumor originated from cells within your brain itself. D) Your tumor is from nerve tissue somewhere in your body.

C Feedback: Primary brain tumors originate from cells and structures within the brain. Secondary brain tumors are metastatic tumors that originate somewhere else in the body. The scenario does not indicate that the patients tumor is a pituitary tumor or a neuroma.

A patient with advanced leukemia is responding poorly to treatment. The nurse finds the patient tearful and trying to express his feelings, but he is clearly having difficulty. What is the nurses most appropriate action? A) Tell him that you will give him privacy and leave the room. B) Offer to call pastoral care. C) Ask if he would like you to sit with him while he collects his thoughts. D) Tell him that you can understand how hes feeling.

C Feedback: Providing emotional support and discussing the uncertain future are crucial. Leaving is incorrect because leaving the patient doesnt show acceptance of his feelings. Offering to call pastoral care may be helpful for some patients but should be done after the nurse has spent time with the patient. Telling the patient that you understand how hes feeling is inappropriate because it doesnt help him express his feelings.

A nurse is aware of the need to assess patients risks for anaphylaxis. What health care procedure constitutes the highest risk for anaphylaxis? A) Administration of the measles-mumps-rubella (MMR) vaccine B) Rapid administration of intravenous fluids C) Computed tomography with contrast solution D) Administration of nebulized bronchodilators

C Feedback: Radiocontrast agents present a significant threat of anaphylaxis in the hospital setting. Vaccinations less often cause anaphylaxis. Bronchodilators and IV fluids are not implicated in hypersensitivity reactions.

A patient who has had a radical neck dissection is being prepared for discharge. The discharge plan includes referral to an outpatient rehabilitation center for physical therapy. What would the goals of physical therapy for this patient include? A) Muscle training to relieve dysphagia B) Relieving nerve paralysis in the cervical plexus C) Promoting maximum shoulder function D) Alleviating achalasia by decreasing esophageal peristalsis

C Feedback: Shoulder drop occurs as a result of radical neck dissection. Shoulder function can be improved by rehabilitation exercises. Rehabilitation would not be initiated until the patients neck incision and graft, if present, were sufficiently healed. Nerve paralysis in the cervical plexus and other variables affecting swallowing would be managed by a speech therapist rather than a physical therapist.

A patient has a glomerular filtration rate (GFR) of 43 mL/min/1.73 m2 . Based on this GFR, the nurse interprets that the patients chronic kidney disease is at what stage? A) Stage 1 B) Stage 2 C) Stage 3 D) Stage 4

C Feedback: Stages of chronic renal failure are based on the GFR. Stage 3 is defined by a GFR in the range of 30 to 59 mL/min/1.73 m2 . This is considered a moderate decrease in GFR.

Diagnostic testing has revealed that a patients hepatocellular carcinoma (HCC) is limited to one lobe. The nurse should anticipate that this patients plan of care will focus on what intervention? A) Cryosurgery B) Liver transplantation C) Lobectomy D) Laser hyperthermia

C Feedback: Surgical resection is the treatment of choice when HCC is confined to one lobe of the liver and the function of the remaining liver is considered adequate for postoperative recovery. Removal of a lobe of the liver (lobectomy) is the most common surgical procedure for excising a liver tumor. While cryosurgery and liver transplantation are other surgical options for management of liver cancer, these procedures are not performed at the same frequency as a lobectomy. Laser hyperthermia is a nonsurgical treatment for liver cancer.

A young adult patient has received the news that her treatment for Hodgkin lymphoma has been deemed successful and that no further treatment is necessary at this time. The care team should ensure that the patient receives regular health assessments in the future due to the risk of what complication? A) Iron-deficiency anemia B) Hemophilia C) Hematologic cancers D) Genitourinary cancers

C Feedback: Survivors of Hodgkin lymphoma have a high risk of second cancers, with hematologic cancers being the most common. There is no consequent risk of anemia or hemophilia, and hematologic cancers are much more common than GU cancers.

A nurse is participating in a patients care conference and the team is deciding between parenteral nutrition (PN) and a total nutritional admixture (TNA). What advantages are associated with providing TNA rather than PN? A) TNA can be mixed by a certified registered nurse. B) TNA can be administered over 8 hours, while PN requires 24-hour administration. C) TNA is less costly than PN. D) TNA does not require the use of a micron filter.

C Feedback: TNA is mixed in one container and administered to the patient over a 24-hour period. A 1.5-micron filter is used with the TNA solution. Advantages of the TNA over PN include cost savings. Pharmacy staff must prepare both solutions.

Splints have been ordered for a patient who is at risk of developing footdrop following a spinal cord injury. The nurse caring for this patient knows that the splints are removed and reapplied when? A) At the patients request B) Each morning and evening C) Every 2 hours D) One hour prior to mobility exercises

C Feedback: The feet are prone to footdrop; therefore, various types of splints are used to prevent footdrop. When used, the splints are removed and reapplied every 2 hours.

A patient is undergoing diagnostic testing for mitral stenosis. What statement by the patient during the nurses interview is most suggestive of this valvular disorder? A) I get chest pain from time to time, but it usually resolves when I rest. B) Sometimes when Im resting, I can feel my heart skip a beat. C) Whenever I do any form of exercise I get terribly short of breath. D) My feet and ankles have gotten terribly puffy the last few weeks.

C Feedback: The first symptom of mitral stenosis is often breathing difficulty (dyspnea) on exertion as a result of pulmonary venous hypertension. Patients with mitral stenosis are likely to show progressive fatigue as a result of low cardiac output. Palpitations occur in some patients, but dyspnea is a characteristic early symptom. Peripheral edema and chest pain are atypical.

A critical care nurse is caring for a patient diagnosed with acute pancreatitis. The nurse knows that the indications for starting parenteral nutrition (PN) for this patient are what? A) 5% deficit in body weight compared to preillness weight and increased caloric need B) Calorie deficit and muscle wasting combined with low electrolyte levels C) Inability to take in adequate oral food or fluids within 7 days D) Significant risk of aspiration coupled with decreased level of consciousness

C Feedback: The indications for PN include an inability to ingest adequate oral food or fluids within 7 days. Weight loss, muscle wasting combined with electrolyte imbalances, and aspiration indicate a need for nutritional support, but this does not necessary have to be parenteral.

The pathophysiology of an ischemic stroke involves the ischemic cascade, which includes the following steps: 1. Change in pH 2. Blood flow decreases 3. A switch to anaerobic respiration 4. Membrane pumps fail 5. Cells cease to function 6. Lactic acid is generated Put these steps in order in which they occur. A) 635241 B) 352416 C) 236145 D) 162534

C Feedback: The ischemic cascade begins when cerebral blood flow decreases to less than 25 mL per 100 g of blood per minute. At this point, neurons are no longer able to maintain aerobic respiration. The mitochondria must then switch to anaerobic respiration, which generates large amounts of lactic acid, causing a change in the pH. This switch to the less efficient anaerobic respiration also renders the neuron incapable of producing sufficient quantities of adenosine triphosphate (ATP) to fuel the depolarization processes. The membrane pumps that maintain electrolyte balances begin to fail, and the cells cease to f

A patient who had surgery for gallbladder disease has just returned to the postsurgical unit from postanesthetic recovery. The nurse caring for this patient knows to immediately report what assessment finding to the physician? A) Decreased breath sounds B) Drainage of bile-colored fluid onto the abdominal dressing C) Rigidity of the abdomen D) Acute pain with movement

C Feedback: The location of the subcostal incision will likely cause the patient to take shallow breaths to prevent pain, which may result in decreased breath sounds. The nurse should remind patients to take deep breaths and cough to expand the lungs fully and prevent atelectasis. Acute pain is an expected assessment finding following surgery; analgesics should be administered for pain relief. Abdominal splinting or application of an abdominal binder may assist in reducing the pain. Bile may continue to drain from the drainage tract after surgery, which will require frequent changes of the abdominal dressing. Increased abdominal tenderness and rigidity should be reported immediately to the physician, as it may indicate bleeding from an inadvertent puncture or nicking of a major blood vessel during the surgical procedure.

A patient diagnosed with transient ischemic attacks (TIAs) is scheduled for a carotid endarterectomy. The nurse explains that this procedure will be done for what purpose? A) To decrease cerebral edema B) To prevent seizure activity that is common following a TIA C) To remove atherosclerotic plaques blocking cerebral flow D) To determine the cause of the TIA

C Feedback: The main surgical procedure for select patients with TIAs is carotid endarterectomy, the removal of an atherosclerotic plaque or thrombus from the carotid artery to prevent stroke in patients with occlusive disease of the extracranial arteries. An endarterectomy does not decrease cerebral edema, prevent seizure activity, or determine the cause of a TIA.

A child is undergoing testing for food allergies after experiencing unexplained signs and symptoms of hypersensitivity. What food items would the nurse inform the parents are common allergens? A) Citrus fruits and rice B) Root vegetables and tomatoes C) Eggs and wheat D) Hard cheeses and vegetable oils

C Feedback: The most common causes of food allergies are seafood (lobster, shrimp, crab, clams, fish), legumes (peanuts, peas, beans, licorice), seeds (sesame, cottonseed, caraway, mustard, flaxseed, sunflower seeds), tree nuts, berries, egg white, buckwheat, milk, and chocolate.

A patient has just been told that her illness is terminal. The patient tearfully states, I cant believe I am going to die. Why me? What is your best response? A) I know how you are feeling. B) You have lived a long life. C) This must be very difficult for you. D) Life can be so unfair.

C Feedback: The most important intervention the nurse can provide is listening empathetically. To communicate effectively, the nurse should ask open-ended questions and acknowledge the patients fears. Deflecting the statement or providing false sympathy must be avoided.

The nurse is caring for a patient who has had a dysrhythmic event. The nurse is aware of the need to assess for signs of diminished cardiac output (CO). What change in status may signal to the nurse a decrease in cardiac output? A) Increased blood pressure B) Bounding peripheral pulses C) Changes in level of consciousness D) Skin flushing

C Feedback: The nurse conducts a physical assessment to confirm the data obtained from the history and to observe for signs of diminished cardiac output (CO) during the dysrhythmic event, especially changes in level of consciousness. Blood pressure tends to decrease with lowered CO and bounding peripheral pulses are inconsistent with this problem. Pallor, not skin flushing, is expected.

The nurse is creating an education plan for a patient who underwent a nephrectomy for the treatment of a renal tumor. What should the nurse include in the teaching plan? A) The importance of increased fluid intake B) Signs and symptoms of rejection C) Inspection and care of the incision D) Techniques for preventing metastasis

C Feedback: The nurse teaches the patient to inspect and care for the incision and perform other general postoperative care, including activity and lifting restrictions, driving, and pain management. There would be no need to teach the signs or symptoms of rejection as there has been no transplant. Increased fluid intake is not normally recommended and the patient has minimal control on the future risk for metastasis.

A nurse is providing education to a patient with iron deficiency anemia who has been prescribed iron supplements. What should the nurse include in health education? A) Take the iron with dairy products to enhance absorption. B) Increase the intake of vitamin E to enhance absorption. C) Iron will cause the stools to darken in color. D) Limit foods high in fiber due to the risk for diarrhea.

C Feedback: The nurse will inform the patient that iron will cause the stools to become dark in color. Iron should be taken on an empty stomach, as its absorption is affected by food, especially dairy products. Patients should be instructed to increase their intake of vitamin C to enhance iron absorption. Foods high in fiber should be consumed to minimize problems with constipation, a common side effect associated with iron therapy.

The nurse is preparing health education for a patient who is being discharged after hospitalization for a hemorrhagic stroke. What content should the nurse include in this education? A) Mild, intermittent seizures can be expected. B) Take ibuprofen for complaints of a serious headache. C) Take antihypertensive medication as ordered. D) Drowsiness is normal for the first week after discharge.

C Feedback: The patient and family are provided with information that will enable them to cooperate with the care and restrictions required during the acute phase of hemorrhagic stroke and to prepare the patient to return home. Patient and family teaching includes information about the causes of hemorrhagic stroke and its possible consequences. Symptoms of hydrocephalus include gradual onset of drowsiness and behavioral changes. Hypertension is the most serious risk factor, suggesting that appropriate antihypertensive treatment is essential for a patient being discharged. Seizure activity is not normal; complaints of a serious headache should be reported to the physician before any medication is taken. Drowsiness is not normal or expected.

A patient who has undergone liver transplantation is ready to be discharged home. Which outcome of health education should the nurse prioritize? A) The patient will obtain measurement of drainage from the T-tube. B) The patient will exercise three times a week. C) The patient will take immunosuppressive agents as required. D) The patient will monitor for signs of liver dysfunction.

C Feedback: The patient is given written and verbal instructions about immunosuppressive agent doses and dosing schedules. The patient is also instructed on steps to follow to ensure that an adequate supply of medication is available so that there is no chance of running out of the medication or skipping a dose. Failure to take medications as instructed may precipitate rejection. The nurse would not teach the patient to measure drainage from a T-tube as the patient wouldnt go home with a T-tube. The nurse may teach the patient about the need to exercise or what the signs of liver dysfunction are, but the nurse would not stress these topics over the immunosuppressive drug regimen.

A patient has been admitted to the hospital for the treatment of chronic pancreatitis. The patient has been stabilized and the nurse is now planning health promotion and educational interventions. Which of the following should the nurse prioritize? A) Educating the patient about expectations and care following surgery B) Educating the patient about the management of blood glucose after discharge C) Educating the patient about postdischarge lifestyle modifications D) Educating the patient about the potential benefits of pancreatic transplantation

C Feedback: The patients lifestyle (especially regarding alcohol use) is a major determinant of the course of chronic pancreatitis. The disease is not often managed by surgery and blood sugar monitoring is not necessarily indicated for every patient after hospital treatment. Transplantation is not an option.

A patient with a fractured femur presenting to the ED exhibits cool, moist skin, increased heart rate, and falling BP. The care team should consider the possibility of what complication of the patients injuries? A) Myocardial infarction B) Hypoglycemia C) Hemorrhage D) Peritonitis

C Feedback: The signs and symptoms the patient is experiencing suggest a volume deficit from an internal bleed. That the symptoms follow an acute injury suggests hemorrhage rather than myocardial infarction or hypoglycemia. Peritonitis would be an unlikely result of a femoral fracture.

A patient is recovering from intracranial surgery that was performed using the transsphenoidal approach. The nurse should be aware that the patient may have required surgery on what neurologic structure? A) Cerebellum B) Hypothalamus C) Pituitary gland D) Pineal gland

C Feedback: The transsphenoidal approach (through the mouth and nasal sinuses) is often used to gain access to the pituitary gland. This surgical approach would not allow for access to the pineal gland, cerebellum, or hypothalamus.

A nurse is planning the care of a patient with a diagnosis of sickle cell disease who has been admitted for the treatment of an acute vaso-occlusive crisis. What nursing diagnosis should the nurse prioritize in the patients plan of care? A) Risk for disuse syndrome related to ineffective peripheral circulation B) Functional urinary incontinence related to urethral occlusion C) Ineffective tissue perfusion related to thrombosis D) Ineffective thermoregulation related to hypothalamic dysfunction

C Feedback: There are multiple potential complications of sickle cell disease and sickle cell crises. Central among these, however, is the risk of thrombosis and consequent lack of tissue perfusion. Sickle cell crises are not normally accompanied by impaired thermoregulation or genitourinary complications. Risk for disuse syndrome is not associated with the effects of acute vaso-occlusive crisis.

A nurse is addressing the prevention of esophageal cancer in response to a question posed by a participant in a health promotion workshop. What action has the greatest potential to prevent esophageal cancer? A) Promotion of a nutrient-dense, low-fat diet B) Annual screening endoscopy for patients over 50 with a family history of esophageal cancer C) Early diagnosis and treatment of gastroesophageal reflux disease D) Adequate fluid intake and avoidance of spicy foods

C Feedback: There are numerous risk factors for esophageal cancer but chronic esophageal irritation or GERD is among the most significant. This is a more significant risk factor than dietary habits. Screening endoscopies are not recommended solely on the basis of family history

The nurse is providing discharge education to a patient diagnosed with HF. What should the nurse teach this patient to do to assess her fluid balance in the home setting? A) Monitor her blood pressure daily B) Assess her radial pulses daily C) Monitor her weight daily D) Monitor her bowel movements

C Feedback: To assess fluid balance at home, the patient should monitor daily weights at the same time every day. Assessing radial pulses and monitoring the blood pressure may be done, but these measurements do not provide information about fluid balance. Bowel function is not indicative of fluid balance.

The nurse is caring for a patient with acute pericarditis. What nursing management should be instituted to minimize complications? A) The nurse keeps the patient isolated to prevent nosocomial infections. B) The nurse encourages coughing and deep breathing. C) The nurse helps the patient with activities until the pain and fever subside. D) The nurse encourages increased fluid intake until the infection resolves.

C Feedback: To minimize complications, the nurse helps the patient with activity restrictions until the pain and fever subside. As the patients condition improves, the nurse encourages gradual increases of activity. Actions to minimize complications of acute pericarditis do not include keeping the patient isolated. Due to pain, coughing and deep breathing are not normally encouraged. An increase in fluid intake is not always necessary.

The nurse caring for a patient with a spinal cord injury notes that the patient is exhibiting early signs and symptoms of disuse syndrome. Which of the following is the most appropriate nursing action? A) Limit the amount of assistance provided with ADLs. B) Collaborate with the physical therapist and immobilize the patients extremities temporarily. C) Increase the frequency of ROM exercises. D) Educate the patient about the importance of frequent position changes.

C Feedback: To prevent disuse syndrome, ROM exercises must be provided at least four times a day, and care is taken to stretch the Achilles tendon with exercises. The patient is repositioned frequently and is maintained in proper body alignment whether in bed or in a wheelchair. The patient must be repositioned by caregivers, not just taught about repositioning. It is inappropriate to limit assistance for the sole purpose of preventing disuse syndrome.

A patient with a spinal cord injury has experienced several hypotensive episodes. How can the nurse best address the patients risk for orthostatic hypotension? A) Administer an IV bolus of normal saline prior to repositioning. B) Maintain bed rest until normal BP regulation returns. C) Monitor the patients BP before and during position changes. D) Allow the patient to initiate repositioning.

C Feedback: To prevent hypotensive episodes, close monitoring of vital signs before and during position changes is essential. Prolonged bed rest carries numerous risks and it is not possible to provide a bolus before each position change. Following the patients lead may or may not help regulate BP.

A nurse is developing a care plan for a patient with a partial-thickness burn, and determines that an appropriate goal is to maintain position of joints in alignment. What is the best rationale for this intervention? A) To prevent neuropathies B) To prevent wound breakdown C) To prevent contractures D) To prevent heterotopic ossification

C Feedback: To prevent the complication of contractures, the nurse will establish a goal to maintain position of joints in alignment. Gentle range of motion exercises and a consult to PT and OT for exercises and positioning recommendations are also appropriate interventions for the prevention of contractures. Joint alignment is not maintained specifically for preventing neuropathy, wound breakdown, or heterotopic ossification.

A nurse is collaborating with the interdisciplinary team to help manage a patients recurrent headaches. What aspect of the patients health history should the nurse identify as a potential contributor to the patients headaches? A) The patient leads a sedentary lifestyle. B) The patient takes vitamin D and calcium supplements. C) The patient takes vasodilators for the treatment of angina. D) The patient has a pattern of weight loss followed by weight gain.

C Feedback: Vasodilators are known to contribute to headaches. Weight fluctuations, sedentary lifestyle, and vitamin supplements are not known to have this effect.

A patient with a history of atrial fibrillation has contacted the clinic saying that she has accidentally overdosed on her prescribed warfarin (Coumadin). The nurse should recognize the possible need for what antidote? A) IVIG B) Factor X C) Vitamin K D) Factor VIII

C Feedback: Vitamin K is administered as an antidote for warfarin toxicity.

A patient is brought by friends to the ED after being involved in a motor vehicle accident. The patient sustained blunt trauma to the abdomen. What nursing action would be most appropriate for this patient? A) Ambulate the patient to expel flatus. B) Place the patient in a high Fowlers position. C) Immobilize the patient on a backboard. D) Place the patient in a left lateral position.

C Feedback: When admitted for blunt trauma, patients must be immobilized until spinal injury is ruled out. Ambulation, side-lying, and upright positioning would be contraindicated until spinal injury is ruled out

A patient has been admitted to the post-surgical unit following a thyroidectomy. To promote comfort and safety, how should the nurse best position the patient? A) Side-lying (lateral) with one pillow under the head B) Head of the bed elevated 30 degrees and no pillows placed under the head C) Semi-Fowlers with the head supported on two pillows D) Supine, with a small roll supporting the neck

C Feedback: When moving and turning the patient, the nurse carefully supports the patients head and avoids tension on the sutures. The most comfortable position is the semi-Fowlers position, with the head elevated and supported by pillows.

A 17-year-old boy is being treated in the ICU after going into cardiac arrest during a football practice. Diagnostic testing reveals cardiomyopathy as the cause of the arrest. What type of cardiomyopathy is particularly common among young people who appear otherwise healthy? A) Dilated cardiomyopathy (DCM). B) Arrhythmogenic right ventricular cardiomyopathy (ARVC) C) Hypertrophic cardiomyopathy (HCM) D) Restrictive or constrictive cardiomyopathy (RCM)

C Feedback: With HCM, cardiac arrest (i.e., sudden cardiac death) may be the initial manifestation in young people, including athletes. DCM, ARVC, and RCM are not typically present in younger adults who appear otherwise healthy.

An 84-year-old woman diagnosed with cancer is admitted to the oncology unit for surgical treatment. The patient has been on chemotherapeutic agents to decrease the tumor size prior to the planned surgery. The nurse caring for the patient is aware that what precipitating factors in this patient may contribute to AKI? Select all that apply. A) Anxiety B) Low BMI C) Age-related physiologic changes D) Chronic systemic disease E) NPO status

C, D Feedback: Changes in kidney function with normal aging increase the susceptibility of elderly patients to kidney dysfunction and renal failure. In addition, the presence of chronic, systemic diseases increases the risk of AKI. Low BMI and anxiety are not risk factors for acute renal disease. NPO status is not a risk, provided adequate parenteral hydration is administered.

One of the functions of nursing care of the terminally ill is to support the patient and his or her family as they come to terms with the diagnosis and progression of the disease process. How should nurses support patients and their families during this process? Select all that apply. A) Describe their personal experiences in dealing with end-of-life issues. B) Encourage the patient and family to keep fighting as a cure may come. C) Try to appreciate and understand the illness from the patients perspective. D) Assist patients with performing a life review. E) Provide interventions that facilitate end-of-life closure.

C, D, E Feedback: Nurses are responsible for educating patients about their illness and for supporting them as they adapt to life with the illness. Nurses can assist patients and families with life review, values clarification, treatment decision making, and end-of-life closure. The only way to do this effectively is to try to appreciate and understand the illness from the patients perspective. The nurses personal experiences should not normally be included and a cure is often not a realistic hope.

The nurse educator is discussing neoplasms with a group of recent graduates. The educator explains that the effects of neoplasms are caused by the compression and infiltration of normal tissue. The physiologic changes that result can cause what pathophysiologic events? Select all that apply. A) Intracranial hemorrhage B) Infection of cerebrospinal fluid C) Increased ICP D) Focal neurologic signs E) Altered pituitary function

C, D, E Feedback: The effects of neoplasms are caused by the compression and infiltration of tissue. A variety of physiologic changes result, causing any or all of the following pathophysiologic events: increased ICP and cerebral edema, seizure activity and focal neurologic signs, hydrocephalus, and altered pituitary function.

A newly graduated nurse is admitting a patient with a long history of emphysema. The new nurses preceptor is going over the patients past lab reports with the new nurse. The nurse takes note that the patients PaCO2 has been between 56 and 64 mm Hg for several months. The preceptor asks the new nurse why they will be cautious administering oxygen. What is the new nurses best response? A) The patients calcium will rise dramatically due to pituitary stimulation. B) Oxygen will increase the patients intracranial pressure and create confusion. C) Oxygen may cause the patient to hyperventilate and become acidotic. D) Using oxygen may result in the patient developing carbon dioxide narcosis and hypoxemia

D

A nurse is reviewing the health record of a client who has a malignant brain tumor and notes the client has a postive romberg sign. Which of the following actions should the nurse take to assess for this finding?A. stroke the lateral aspect of the sole of the foot B. ask the client to blink both eyes C. observes for facial droop D. have the client stand erect with eyes closed

D

A nurse provides care on a bone marrow transplant unit and is preparing a female patient for a hematopoietic stem cell transplantation (HSCT) the following day. What information should the nurse emphasize to the patients family and friends? A) Your family should likely gather at the bedside in case theres a negative outcome. B) Make sure she doesnt eat any food in the 24 hours before the procedure. C) Wear a hospital gown when you go into the patients room. D) Do not visit if youve had a recent infection.

D

A patient with a pulmonary embolism is being treated with a heparin infusion. What diagnostic finding suggests to the nurse that treatment is effective? A)The patients PT is within reference ranges. B)Arterial blood sampling tests positive for the presence of factor XIII C)The patients platelet level is below 100,000/mm3. D)The patients activated partial thromboplastin time (aPTT) is 1.5 to 2.5 times the control value.

D

An oncology nurse is caring for a patient who has developed erythema following radiation therapy. What should the nurse instruct the patient to do? A) Periodically apply ice to the area. B) Keep the area cleanly shaven. C) Apply petroleum jelly to the affected area. D) Avoid using soap on the treatment area.

D

An oncology patient will begin a course of chemotherapy and radiation therapy for the treatment of bone metastases. What is one means by which malignant disease processes transfer cells from one place to another? A) Adhering to primary tumor cells B) Inducing mutation of cells of another organ C) Phagocytizing healthy cells D) Invading healthy host tissues

D

The baroreceptors, located in the left atrium and in the carotid and aortic arches, respond to changes in the circulating blood volume and regulate sympathetic and parasympathetic neural activity as well as endocrine activities. Sympathetic stimulation constricts renal arterioles, causing what effect? A) Decrease in the release of aldosterone B) Increase of filtration in the Loop of Henle C) Decrease in the reabsorption of sodium D) Decrease in glomerular filtration

D

The nurse is caring for a patient has just been given a 6-month prognosis following a diagnosis of extensive stage small-cell lung cancer. The patient states that he would like to die at home, but the team believes that the patients care needs are unable to be met in a home environment. What might you suggest as an alternative? A) Discuss a referral for rehabilitation hospital. B) Panel the patient for a personal care home. C) Discuss a referral for acute care. D) Discuss a referral for hospice care.

D

The nurse is orienting a new nurse to the oncology unit. When reviewing the safe administration of antineoplastic agents, what action should the nurse emphasize? A) Adjust the dose to the patients present symptoms. B) Wash hands with an alcohol-based cleanser following administration. C) Use gloves and a lab coat when preparing the medication. D) Dispose of the antineoplastic wastes in the hazardous waste receptacle.

D

When planning the care of a patient with a fluid imbalance, the nurse understands that in the human body, water and electrolytes move from the arterial capillary bed to the interstitial fluid. What causes this to occur? A) Active transport of hydrogen ions across the capillary walls B) Pressure of the blood in the renal capillaries C) Action of the dissolved particles contained in a unit of blood D) Hydrostatic pressure resulting from the pumping action of the heart

D

You are caring for a patient who is being treated on the oncology unit with a diagnosis of lung cancer with bone metastases. During your assessment, you note the patient complains of a new onset of weakness with abdominal pain. Further assessment suggests that the patient likely has a fluid volume deficit. You should recognize that this patient may be experiencing what electrolyte imbalance? A) Hypernatremia B) Hypomagnesemia C) Hypophosphatemia D) Hypercalcemia

D

A patient diagnosed with a cerebral aneurysm reports a severe headache to the nurse. What action is a priority for the nurse? A) Sit with the patient for a few minutes. B) Administer an analgesic. C) Inform the nurse-manager. D) Call the physician immediately.

D A headache may be an indication that the aneurysm is leaking. The nurse should notify the physician immediately. The physician will decide whether administration of an analgesic is indicated. Informing the nurse-manager is not necessary. Sitting with the patient is appropriate, once the physician has been notified of the change in the patients condition.

The cardiac care nurse is reviewing the conduction system of the heart. The nurse is aware that electrical conduction of the heart usually originates in the SA node and then proceeds in what sequence? A) SA node to bundle of His to AV node to Purkinje fibers B) SA node to AV node to Purkinje fibers to bundle of His C) SA node to bundle of His to Purkinje fibers to AV node D) SA node to AV node to bundle of His to Purkinje fibers

D Feedback: The normal electrophysiological conduction route is SA node to AV node to bundle of HIS to Purkinje fibers.

A patient has a poor prognosis after being involved in a motor vehicle accident resulting in a head injury. As the patients ICP increases and condition worsens, the nurse knows to assess for indications of approaching death. These indications include which of the following? A) Hemiplegia B) Dry mucous membranes C) Signs of internal bleeding D) Loss of brain stem reflexes

D Loss of brain stem reflexes, including pupillary, corneal, gag, and swallowing reflexes, is an ominous sign of approaching death. Dry mucous membranes, hemiplegia, and bleeding must be promptly addressed, but none of these is a common sign of impending death.

The hospice nurse is caring for a 45-year-old mother of three young children in the patients home. During the most recent visit, the nurse has observed that the patient has a new onset of altered mental status, likely resulting from recently diagnosed brain metastases. What goal of nursing interventions should the nurse identify? A) Helping the family to understand why the patient needs to be sedated B) Making arrangements to promptly move the patient to an acute-care facility C) Explaining to the family that death is near and the patient needs around-the-clock nursing care D) Teaching family members how to interact with, and ensure safety for, the patient with impaired cognition

D Nursing interventions should be aimed at accommodating the change in the patients status and maintaining her safety. The scenario does not indicate the need either to sedate the patient or to move her to an acute-care facility. If the family has the resources, there is no need to bring in nurses to be with the patient around-the-clock, and the scenario does not indicate that death is imminent.

A client with autoimmune idiopathic thrombocytopenic purpura (ITP) has had a splenectomy and returned to the surgical unit 2 hours ago. The nurse assesses the client and finds the abdominal dressing saturated with blood. What action is most important? a. Preparing to administer a blood transfusion b. Reinforcing the dressing and documenting findings c. Removing the dressing and assessing the surgical site d. Taking a set of vital signs and notifying the surgeon

D While some bloody drainage on a new surgical dressing is expected, a saturated dressing is not. This client is already at high risk of bleeding due to the ITP. The nurse should assess vital signs for shock and notify the surgeon immediately. The client may or may not need a transfusion. Reinforcing the dressing is an appropriate action, but the nurse needs to do more than document afterward. Removing the dressing increases the risk of infection; plus, it is not needed since the nurse knows where the bleeding is coming from.

The nurse is caring for a patient who has undergone supratentorial removal of a pituitary mass. What medication would the nurse expect to administer prophylactically to prevent seizures in this patient? A) Prednisone B) Dexamethasone C) Cafergot D) Phenytoin

D Antiseizure medication (phenytoin, diazepam) is often prescribed prophylactically for patients who have undergone supratentorial craniotomy because of the high risk of seizures after this procedure. Prednisone and dexamethasone are steroids and do not prevent seizures. Cafergot is used in the treatment of migraines.

What should the nurse suspect when hourly assessment of urine output on a patient postcraniotomy exhibits a urine output from a catheter of 1,500 mL for two consecutive hours? A) Cushing syndrome B) Syndrome of inappropriate antidiuretic hormone (SIADH) C) Adrenal crisis D) Diabetes insipidus

D Diabetes insipidus is an abrupt onset of extreme polyuria that commonly occurs in patients after brain surgery. Cushing syndrome is excessive glucocorticoid secretion resulting in sodium and water retention. SIADH is the result of increased secretion of ADH; the patient becomes volume-overloaded, urine output diminishes, and serum sodium concentration becomes dilute. Adrenal crisis is undersecretion of glucocorticoids resulting in profound hypoglycemia, hypovolemia, and hypotension.

The nurse is caring for a patient admitted with cardiogenic shock. The patient is experiencing chest pain and there is an order for the administration of morphine. In addition to pain control, what is the main rationale for administering morphine to this patient? A) It promotes coping and slows catecholamine release. B) It stimulates the patient so he or she is more alert. C) It decreases gastric secretions. D) It dilates the blood vessels.

D Feedback: For patients experiencing chest pain, morphine is the drug of choice because it dilates the blood vessels and controls the patients anxiety. Morphine would not be ordered to promote coping or to stimulate the patient. The rationale behind using morphine would not be to decrease gastric secretions.

A patient who attempted suicide being treated in the ED is accompanied by his mother, father, and brother. When planning the nursing care of this family, the nurse should perform which of the following action? A) Refer the family to psychiatry in order to provide them with support. B) Explore the causes of the patients suicide attempt with the family. C) Encourage the family to participate in the bedside care of the patient. D) Ensure that the family receives appropriate crisis intervention services.

D It is essential that family crisis intervention services are available for families of ED patients. It would be inappropriate and insensitive to explore causes of the patients suicide attempt with the family. Family participation in bedside care is often impractical in the ED setting. Psychiatry is not the normal source of psychosocial support and crisis intervention.

A clinic nurse is working with a patient who has a long-standing diagnosis of polycythemia vera. How can the nurse best gauge the course of the patients disease? A) Document the color of the patients palms and face during each visit. B) Follow the patients erythrocyte sedimentation rate over time. C) Document the patients response to erythropoietin injections. D) Follow the trends of the patients hematocrit.

D The course of polycythemia vera can be best ascertained by monitoring the patients hematocrit, which should remain below 45%. Erythropoietin injections would exacerbate the condition. Skin tone should be observed, but is a subjective assessment finding. The patients ESR is not relevant to the course of the disease.

A patient who suffered a spinal cord injury is experiencing an exaggerated autonomic response. What aspect of the patients current health status is most likely to have precipitated this event? A) The patient received a blood transfusion. B) The patients analgesia regimen was recent changed. C) The patient was not repositioned during the night shift. D) The patients urinary catheter became occluded.

D Feedback: A distended bladder is the most common cause of autonomic dysreflexia. Infrequent positioning is a less likely cause, although pressure ulcers or tactile stimulation can cause it. Changes in mediations or blood transfusions are unlikely causes.

A patient is brought to the emergency department from the site of a chemical fire, where he suffered a burn that involves the epidermis, dermis, and the muscle and bone of the right arm. On inspection, the skin appears charred. Based on these assessment findings, what is the depth of the burn on the patients arm? A) Superficial partial-thickness B) Deep partial-thickness C) Full partial-thickness D) Full-thickness

D Feedback: A full-thickness burn involves total destruction of the epidermis and dermis and, in some cases, underlying tissue as well. Wound color ranges widely from white to red, brown, or black. The burned area is painless because the nerve fibers are destroyed. The wound can appear leathery; hair follicles and sweat glands are destroyed. Edema may also be present. Superficial partial-thickness burns involve the epidermis and possibly a portion of the dermis; the patient will experience pain that is soothed by cooling. Deep partial-thickness burns involve the epidermis, upper dermis, and portion of the deeper dermis; the patient will complain of pain and sensitivity to cold air. Full partial thickness is not a depth of burn

A patient with Von Willebrand disease (vWD) has experienced recent changes in bowel function that suggest the need for a screening colonoscopy. What intervention should be performed in anticipation of this procedure? A) The patient should not undergo the normal bowel cleansing protocol prior to the procedure. B) The patient should receive a unit of fresh-frozen plasma 48 hours before the procedure. C) The patient should be admitted to the surgical unit on the day before the procedure. D) The patient should be given necessary clotting factors before the procedure.

D Feedback: A goal of treating vWD is to replace the deficient protein (e.g., vWF or factor VIII) prior to an invasive procedure to prevent subsequent bleeding. Bowel cleansing is not contraindicated and FFP does not reduce the patients risk of bleeding. There may or may not be a need for preprocedure hospital admission.

A patient has been living with a diagnosis of anemia for several years and has experienced recent declines in her hemoglobin levels despite active treatment. What assessment finding would signal complications of anemia? A) Venous ulcers and visual disturbances B) Fever and signs of hyperkalemia C) Epistaxis and gastroesophageal reflux D) Ascites and peripheral edema

D Feedback: A significant complication of anemia is heart failure from chronic diminished blood volume and the hearts compensatory effort to increase cardiac output. Patients with anemia should be assessed for signs and symptoms of heart failure, including ascites and peripheral edema. None of the other listed signs and symptoms is characteristic of heart failure.

A critical care nurse is caring for a patient with autoimmune hemolytic anemia. The patient is not responding to conservative treatments, and his condition is now becoming life threatening. The nurse is aware that a treatment option in this case may include what? A) Hepatectomy B) Vitamin K administration C) Platelet transfusion D) Splenectomy

D Feedback: A splenectomy may be the course of treatment if autoimmune hemolytic anemia does not respond to conservative treatment. Vitamin K administration is treatment for vitamin K deficiency and does not resolve anemia. Platelet transfusion may be the course of treatment for some bleeding disorders. Hepatectomy would not help the patient.

While assessing the patient at the beginning of the shift, the nurse inspects a surgical dressing covering the operative site after the patients cervical diskectomy. The nurse notes that the drainage is 75% saturated with serosanguineous discharge. What is the nurses most appropriate action? A) Page the physician and report this sign of infection. B) Reinforce the dressing and reassess in 1 to 2 hours. C) Reposition the patient to prevent further hemorrhage. D) Inform the surgeon of the possibility of a dural leak.

D Feedback: After a cervical diskectomy, the nurse will monitor the operative site and dressing covering this site. Serosanguineous drainage may indicate a dural leak. This constitutes a risk for meningitis, but is not a direct sign of infection. This should be reported to the surgeon, not just reinforced and observed.

The nurse has entered a patients room and found the patient unresponsive and not breathing. What is the nurses next appropriate action? A) Palpate the patients carotid pulse. B) Illuminate the patients call light. C) Begin performing chest compressions. D) Activate the Emergency Response System (ERS).

D Feedback: After checking for responsiveness and breathing, the nurse should activate the ERS. Assessment of carotid pulse should follow and chest compressions may be indicated. Illuminating the call light is an insufficient response.

The critical care nurse is caring for a patient who is receiving cyclosporine postoperative heart transplant. The patient asks the nurse to remind him what this medication is for. How should the nurse best respond? A) Azathioprine decreases the risk of thrombus formation. B) Azathioprine ensures adequate cardiac output. C) Azathioprine increases the number of white blood cells. D) Azathioprine minimizes rejection of the transplant.

D Feedback: After heart transplant, patients are constantly balancing the risk of rejection with the risk of infection. Most commonly, patients receive cyclosporine or tacrolimus (FK506, Prograf), azathioprine (Imuran), or mycophenolate mofetil (CellCept), and corticosteroids (prednisone) to minimize rejection. Cyclosporine does not prevent thrombus formation, enhance cardiac output, or increase white cell counts.

A patient has been treated in the hospital for an episode of acute pancreatitis. The patient has acknowledged the role that his alcohol use played in the development of his health problem, but has not expressed specific plans for lifestyle changes after discharge. What is the nurses most appropriate response? A) Educate the patient about the link between alcohol use and pancreatitis. B) Ensure that the patient knows the importance of attending follow-up appointments. C) Refer the patient to social work or spiritual care. D) Encourage the patient to connect with a community-based support group.

D Feedback: After the acute attack has subsided, some patients may be inclined to return to their previous drinking habits. The nurse provides specific information about resources and support groups that may be of assistance in avoiding alcohol in the future. Referral to Alcoholics Anonymous as appropriate or other support groups is essential. The patient already has an understanding of the effects of alcohol, and follow-up appointments will not necessarily result in lifestyle changes. Social work and spiritual care may or may not be beneficial.

A patient is a candidate for percutaneous balloon valvuloplasty, but is concerned about how this procedure will affect her busy work schedule. What guidance should the nurse provide to the patient? A) Patients generally stay in the hospital for 6 to 8 days. B) Patients are kept in the hospital until they are independent with all aspects of their care. C) Patients need to stay in the hospital until they regain normal heart function for their age. D) Patients usually remain at the hospital for 24 to 48 hours.

D Feedback: After undergoing percutaneous balloon valvuloplasty, the patient usually remains in the hospital for 24 to 48 hours. Prediagnosis levels of heart function are not always attainable and the patient does not need to be wholly independent prior to discharge.

The emergency response team is dealing with a radiation leak at the hospital. What action should be performed to prevent the spread of the contaminants? A) Floors must be scrubbed with undiluted bleach. B) Waste must be promptly incinerated. C) The ventilation system should be deactivated. D) Air ducts and vents should be sealed.

D Feedback: All air ducts and vents must be sealed to prevent spread. Waste is controlled through double-bagging and the use of plastic-lined containers outside of the facility rather than incineration. Bleach would be ineffective against radiation and the ventilation system may or may not be deactivated.

The ED is notified that a 6-year-old is in transit with a suspected brain injury after being struck by a car. The child is unresponsive at this time, but vital signs are within acceptable limits. What will be the primary goal of initial therapy? A) Promoting adequate circulation B) Treating the childs increased ICP C) Assessing secondary brain injury D) Preserving brain homeostasis

D Feedback: All therapy is directed toward preserving brain homeostasis and preventing secondary brain injury, which is injury to the brain that occurs after the original traumatic event. The scenario does not indicate the child has increased ICP or a secondary brain injury at this poi

The nurse working on the coronary care unit is caring for a patient with ACS. How can the nurse best meet the patients psychosocial needs? A) Reinforce the fact that treatment will be successful. B) Facilitate a referral to a chaplain or spiritual leader. C) Increase the patients participation in rehabilitation activities. D) Directly address the patients anxieties and fears.

D Feedback: Alleviating anxiety and decreasing fear are important nursing functions that reduce the sympathetic stress response. Referrals to spiritual care may or may not be appropriate, and this does not relieve the nurse of responsibility for addressing the patients psychosocial needs. Treatment is not always successful, and false hope should never be fostered. Participation in rehabilitation may alleviate anxiety for some patients, but it may exacerbate it for others.

A nurse in the ICU is providing care for a patient who has been admitted with a hemorrhagic stroke. The nurse is performing frequent neurologic assessments and observes that the patient is becoming progressively more drowsy over the course of the day. What is the nurses best response to this assessment finding? A) Report this finding to the physician as an indication of decreased metabolism. B) Provide more stimulation to the patient and monitor the patient closely. C) Recognize this as the expected clinical course of a hemorrhagic stroke. D) Report this to the physician as a possible sign of clinical deterioration.

D Feedback: Alteration in LOC often is the earliest sign of deterioration in a patient with a hemorrhagic stroke. Drowsiness and slight slurring of speech may be early signs that the LOC is deteriorating. This finding is unlikely to be the result of metabolic changes and it is not expected. Stimulating a patient with an acute stroke is usually contraindicated.

A nurse has made a referral to a grief support group, knowing that many individuals find these both comforting and beneficial after the death of a loved one. What is the most important accomplishment available by attending a grief support group? A) Providing a framework for incorporating the old life into the new B) Normalizing adaptation to a continuation of the old life C) Aiding in adjusting to using old, familiar social skills D) Normalization of feelings and experiences

D Feedback: Although many people complete the work of mourning with the informal support of families and friends, many find that talking with others who have had a similar experience, such as in formal support groups, normalizes the feelings and experiences and provides a framework for learning new skills to cope with the loss and create a new life. The other listed options are incorrect because they indicate the need to hold onto the old life and not move on

A patient with a history rheumatic heart disease knows that she is at risk for bacterial endocarditis when undergoing invasive procedures. Prior to a scheduled cystoscopy, the nurse should ensure that the patient knows the importance of taking which of the following drugs? A) Enoxaparin (Lovenox) B) Metoprolol (Lopressor) C) Azathioprine (Imuran) D) Amoxicillin (Amoxil)

D Feedback: Although rare, bacterial endocarditis may be life-threatening. A key strategy is primary prevention in high-risk patients (i.e., those with rheumatic heart disease, mitral valve prolapse, or prosthetic heart valves). Antibiotic prophylaxis is recommended for high-risk patients immediately before and sometimes after certain procedures. Amoxicillin is the drug of choice. None of the other listed drugs is an antibiotic.

A 55-year-old man has been newly diagnosed with acute pancreatitis and admitted to the acute medical unit. How should the nurse most likely explain the pathophysiology of this patients health problem? A) Toxins have accumulated and inflamed your pancreas. B) Bacteria likely migrated from your intestines and became lodged in your pancreas. C) A virus that was likely already present in your body has begun to attack your pancreatic cells. D) The enzymes that your pancreas produces have damaged the pancreas itself.

D Feedback: Although the mechanisms causing pancreatitis are unknown, pancreatitis is commonly described as the autodigestion of the pancreas. Less commonly, toxic substances and microorganisms are implicated as the cause of pancreatitis.

An ECG has been ordered for a newly admitted patient. What should the nurse do prior to electrode placement? A) Clean the skin with providone-iodine solution. B) Ensure that the area for electrode placement is dry. C) Apply tincture of benzoin to the electrode sites and wait for it to become tacky. D) Gently abrade the skin by rubbing the electrode sites with dry gauze or cloth.

D Feedback: An ECG is obtained by slightly abrading the skin with a clean dry gauze pad and placing electrodes on the body at specific areas. The abrading of skin will enhance signal transmission. Disinfecting the skin is unnecessary and conduction gel is used.

A patient with a documented history of seizure disorder experiences a generalized seizure. What nursing action is most appropriate? A) Restrain the patient to prevent injury. B) Open the patients jaws to insert an oral airway. C) Place patient in high Fowlers position. D) Loosen the patients restrictive clothing.

D Feedback: An appropriate nursing intervention would include loosening any restrictive clothing on the patient. No attempt should be made to restrain the patient during the seizure because muscular contractions are strong and restraint can produce injury. Do not attempt to pry open jaws that are clenched in a spasm to insert anything. Broken teeth and injury to the lips and tongue may result from such an action. If possible, place the patient on one side with head flexed forward, which allows the tongue to fall forward and facilitates drainage of saliva and mucus

The nurse is caring for a patient with increased intracranial pressure (ICP). The patient has a nursing diagnosis of ineffective cerebral tissue perfusion. What would be an expected outcome that the nurse would document for this diagnosis? A) Copes with sensory deprivation. B) Registers normal body temperature. C) Pays attention to grooming. D) Obeys commands with appropriate motor responses.

D Feedback: An expected outcome of the diagnosis of ineffective cerebral tissue perfusion in a patient with increased intracranial pressure (ICP) would include obeying commands with appropriate motor responses. Vitals signs and neurologic status are assessed every 15 minutes to every hour. Coping with sensory deprivation would relate to the nursing diagnosis of disturbed sensory perception. The outcome of registers normal body temperature relates to the diagnosis of potential for ineffective thermoregulation. Body image disturbance would have a potential outcome of pays attention to grooming

A patient is admitted to the ED who has been exposed to a nerve agent. The nurse should anticipate the STAT administration of what drug? A) Amyl nitrate B) Dimercaprol C) Erythromycin D) Atropine

D Feedback: Atropine is administered when a patient is exposed to a nerve agent. Exposure to blood agents, such as cyanide, requires treatment with amyl nitrate, sodium nitrite, and sodium thiosulfate. Dimercaprol is administered IV for systemic toxicity and topically for skin lesions when exposed to vesicants. Erythromycin is an antibiotic, which is ineffective against nerve agents.

Diagnostic testing has resulted in a diagnosis of acute myeloid leukemia (AML) in an adult patient who is otherwise healthy. The patient and the care team have collaborated and the patient will soon begin induction therapy. The nurse should prepare the patient for which of the following? A) Daily treatment with targeted therapy medications B) Radiation therapy on a daily basis C) Hematopoietic stem cell transplantation D) An aggressive course of chemotherapy

D Feedback: Attempts are made to achieve remission of AML by the aggressive administration of chemotherapy, called induction therapy, which usually requires hospitalization for several weeks. Induction therapy is not synonymous with radiation, stem cell transplantation, or targeted therapies.

A patient is postoperative day 3 following the creation of an ileal conduit for the treatment of invasive bladder cancer. The patient is quickly learning to self-manage the urinary diversion, but expresses concern about the presence of mucus in the urine. What is the nurses most appropriate response? A) Report this finding promptly to the primary care provider. B) Obtain a sterile urine sample and send it for culture. C) Obtain a urine sample and check it for pH. D) Reassure the patient that this is an expected phenomenon

D Feedback: Because mucous membrane is used in forming the conduit, the patient may excrete a large amount of mucus mixed with urine. This causes anxiety in many patients. To help relieve this anxiety, the nurse reassures the patient that this is a normal occurrence after an ileal conduit procedure. Urine testing for culture or pH is not required.

The nurse is caring for a patient with right ventricular hypertrophy and consequently decreased right ventricular function. What valvular disorder may have contributed to this patients diagnosis? A) Mitral valve regurgitation B) Aortic stenosis C) Aortic regurgitation D) Mitral valve stenosis

D Feedback: Because no valve protects the pulmonary veins from the backward flow of blood from the atrium, the pulmonary circulation becomes congested. As a result, the right ventricle must contract against an abnormally high pulmonary arterial pressure and is subjected to excessive strain. Eventually, the right ventricle fails. None of the other listed valvular disorders has this pathophysiological effect.

A clinic patient is being treated for polycythemia vera and the nurse is providing health education. What practice should the nurse recommend in order to prevent the complications of this health problem? A) Avoiding natural sources of vitamin K B) Avoiding altitudes of 1500 feet (457 meters) C) Performing active range of motion exercises daily D) Avoiding tight and restrictive clothing on the legs

D Feedback: Because of the risk of DVT, patients with polycythemia vera should avoid tight and restrictive clothing. There is no need to avoid foods with vitamin K or to avoid higher altitudes. Activity levels should be maintained, but there is no specific need for ROM exercises.

A patient with esophageal varices is being cared for in the ICU. The varices have begun to bleed and the patient is at risk for hypovolemia. The patient has Ringers lactate at 150 cc/hr infusing. What else might the nurse expect to have ordered to maintain volume for this patient? A) Arterial line B) Diuretics C) Foley catheter D) Volume expanders

D Feedback: Because patients with bleeding esophageal varices have intravascular volume depletion and are subject to electrolyte imbalance, IV fluids with electrolytes and volume expanders are provided to restore fluid volume and replace electrolytes. Diuretics would reduce vascular volume. An arterial line and Foley catheter are likely to be ordered, but neither actively maintains the patients volume.

A nurse is preparing to administer a patients scheduled parenteral nutrition (PN). Upon inspecting the bag, the nurse notices that the presence of small amounts of white precipitate are present in the bag. What is the nurses best action? A) Recognize this as an expected finding. B) Place the bag in a warm environment for 30 minutes. C) Shake the bag vigorously for 10 to 20 seconds. D) Contact the pharmacy to obtain a new bag of PN.

D Feedback: Before PN infusion is administered, the solution must be inspected for separation, oily appearance (also known as a cracked solution), or any precipitate (which appears as white crystals). If any of these are present, it is not used. Warming or shaking the bag is inappropriate and unsafe.

The nurse is providing care for a patient who is withdrawing from heavy alcohol use. The nurse and other members of the care team are present at the bedside when the patient has a seizure. In preparation for documenting this clinical event, the nurse should note which of the following? A) The ability of the patient to follow instructions during the seizure. B) The success or failure of the care team to physically restrain the patient. C) The patients ability to explain his seizure during the postictal period. D) The patients activities immediately prior to the seizure.

D Feedback: Before and during a seizure, the nurse observes the circumstances before the seizure, including visual, auditory, or olfactory stimuli; tactile stimuli; emotional or psychological disturbances; sleep; and hyperventilation. Communication with the patient is not possible during a seizure and physical restraint is not attempted. The patients ability to explain the seizure is not clinically relevant.

A patient who has been diagnosed with cholecystitis is being discharged home from the ED to be scheduled for surgery later. The patient received morphine during the present ED admission and is visibly drowsy. When providing health education to the patient, what would be the most appropriate nursing action? A) Give written instructions to patient. B) Give verbal instructions to one of the patients family members. C) Telephone the patient the next day with verbal instructions D) Give verbal and written instructions to patient and a family member.

D Feedback: Before discharge, verbal and written instructions for continuing care are given to the patient and the family or significant others. Discharge teaching is completed prior to the patient leaving the ED, so phoning the patient the next day in not acceptable.

The nurse is caring for a patient with a history of systemic lupus erythematosus who has been recently diagnosed with end-stage kidney disease (ESKD). The patient has an elevated phosphorus level and has been prescribed calcium acetate to bind the phosphorus. The nurse should teach the patient to take the prescribed phosphorus-binding medication at what time? A) Only when needed B) Daily at bedtime C) First thing in the morning D) With each meal

D Feedback: Both calcium carbonate and calcium acetate are medications that bind with the phosphate and assist in excreting the phosphate from the body, in turn lowering the phosphate levels. Phosphate-binding medications must be administered with food to be effective.

A 66-year-old patient is in a hospice receiving palliative care for lung cancer which has metastasized to the patients liver and bones. For the past several hours, the patient has been experiencing dyspnea. What nursing action is most appropriate to help to relive the dyspnea the patient is experiencing? A) Administer a bolus of normal saline, as ordered. B) Initiate high-flow oxygen therapy. C) Administer high doses of opioids. D) Administer bronchodilators and corticosteroids, as ordered.

D Feedback: Bronchodilators and corticosteroids help to improve lung function as well as low doses of opioids. Lowflow oxygen often provides psychological comfort to the patient and family. A fluid bolus is unlikely to be of benefit.

The cardiac monitor alarm alerts the critical care nurse that the patient is showing no cardiac rhythm on the monitor. The nurses rapid assessment suggests cardiac arrest. In providing cardiac resuscitation documentation, how will the nurse describe this initial absence of cardiac rhythm? A) Pulseless electrical activity (PEA) B) Ventricular fibrillation C) Ventricular tachycardia D) Asystole

D Feedback: Cardiac arrest occurs when the heart ceases to produce an effective pulse and circulate blood. It may be caused by a cardiac electrical event such as ventricular fibrillation, ventricular tachycardia, profound bradycardia, or when there is no heart rhythm at all (asystole). Cardiac arrest may also occur when electrical activity is present, but there is ineffective cardiac contraction or circulating volume, which is PEA. Asystole is the only condition that involves the absolute absence of a heart rhythm.

The OR nurse is explaining to a patient that cardiac surgery requires the absence of blood from the surgical field. At the same time, it is imperative to maintain perfusion of body organs and tissues. What technique for achieving these simultaneous goals should the nurse describe? A) Coronary artery bypass graft (CABG) B) Percutaneous transluminal coronary angioplasty (PTCA) C) Atherectomy D) Cardiopulmonary bypass

D Feedback: Cardiopulmonary bypass is often used to circulate and oxygenate blood mechanically while bypassing the heart and lungs. PTCA, atherectomy, and CABG are all surgical procedures, none of which achieves the two goals listed.

A patient has been diagnosed with a malignancy of the oral cavity and is undergoing oncologic treatment. The oncologic nurse is aware that the prognosis for recovery from head and neck cancers is often poor because of what characteristic of these malignancies? A) Radiation therapy often results in secondary brain tumors. B) Surgical complications are exceedingly common. C) Diagnosis rarely occurs until the cancer is endstage. D) Metastases are common and respond poorly to treatment.

D Feedback: Deaths from malignancies of the head and neck are primarily attributable to local-regional metastasis to the cervical lymph nodes in the neck. This often occurs by way of the lymphatics before the primary lesion has been treated. This local-regional metastasis is not amenable to surgical resection and responds poorly to chemotherapy and radiation therapy. This high mortality rate is not related to surgical complications, late diagnosis, or the development of brain tumors.

A patient with hypofunction of the adrenal cortex has been admitted to the medical unit. What would the nurse most likely find when assessing this patient? A) Increased body temperature B) Jaundice C) Copious urine output D) Decreased BP

D Feedback: Decreased BP may occur with hypofunction of the adrenal cortex. Decreased function of the adrenal cortex does not affect the patients body temperature, urine output, or skin tone.

A nurse is caring for a patient with liver failure and is performing an assessment in the knowledge of the patients increased risk of bleeding. The nurse recognizes that this risk is related to the patients inability to synthesize prothrombin in the liver. What factor most likely contributes to this loss of function? A) Alterations in glucose metabolism B) Retention of bile salts C) Inadequate production of albumin by hepatocytes D) Inability of the liver to use vitamin K

D Feedback: Decreased production of several clotting factors may be partially due to deficient absorption of vitamin K from the GI tract. This probably is caused by the inability of liver cells to use vitamin K to make prothrombin. This bleeding risk is unrelated to the roles of glucose, bile salts, or albumin.

The nurse is assessing a patient who had a pacemaker implanted 4 weeks ago. During the patients most recent follow-up appointment, the nurse identifies data that suggest the patient may be socially isolated and depressed. What nursing diagnosis is suggested by these data? A) Decisional conflict related to pacemaker implantation B) Deficient knowledge related to pacemaker implantation C) Spiritual distress related to pacemaker implantation D) Ineffective coping related to pacemaker implantation

D Feedback: Depression and isolation may be symptoms of ineffective coping with the implantation. These psychosocial symptoms are not necessarily indicative of issues related to knowledge or decisions. Further data would be needed to determine a spiritual component to the patients challeng

The public health nurse is participating in a health fair and interviews a patient with a history of hypertension, who is currently smoking one pack of cigarettes per day. She denies any of the most common manifestations of CAD. Based on these data, the nurse would expect the focuses of CAD treatment most likely to be which of the following? A) Drug therapy and smoking cessation B) Diet and drug therapy C) Diet therapy only D) Diet therapy and smoking cessation

D Feedback: Due to the absence of symptoms, dietary therapy would likely be selected as the first-line treatment for possible CAD. Drug therapy would be determined based on a number of considerations and diagnostics findings, but would not be directly indicated. Smoking cessation is always indicated, regardless of the presence or absence of symptoms.

A patient with liver cancer is being discharged home with a biliary drainage system in place. The nurse should teach the patients family how to safely perform which of the following actions? A) Aspirating bile from the catheter using a syringe B) Removing the catheter when output is 15 mL in 24 hours C) Instilling antibiotics into the catheter D) Assessing the patency of the drainage catheter

D Feedback: Families should be taught to provide basic catheter care, including assessment of patency. Antibiotics are not instilled into the catheter and aspiration using a syringe is contraindicated. The family would not independently remove the catheter; this would be done by a member of the care team when deemed necessary.

An adult patient has been diagnosed with iron-deficiency anemia. What nursing diagnosis is most likely to apply to this patients health status? A) Risk for deficient fluid volume related to impaired erythropoiesis B) Risk for infection related to tissue hypoxia C) Acute pain related to uncontrolled hemolysis D) Fatigue related to decreased oxygen-carrying capacity

D Feedback: Fatigue is the major assessment finding common to all forms of anemia. Anemia does not normally result in acute pain or fluid deficit. The patient may have an increased risk of infection due to impaired immune function, but fatigue is more likely.

A patients burns are estimated at 36% of total body surface area; fluid resuscitation has been ordered in the emergency department. After establishing intravenous access, the nurse should anticipate the administration of what fluid? A) 0.45% NaCl with 20 mEq/L KCl B) 0.45% NaCl with 40 mEq/L KCl C) Normal saline D) Lactated Ringers

D Feedback: Fluid resuscitation with lactated Ringers (LR) should be initiated using the American Burn Associations (ABA) fluid resuscitation formulas. LR is the crystalloid of choice because its composition and osmolality most closely resemble plasma and because use of normal saline is associated with hyperchloremic acidosis. Potassium chloride solutions would exacerbate the hyperkalemia that occurs following burn injuries.

A local public health nurse is informed that a cook in a local restaurant has been diagnosed with hepatitis A. What should the nurse advise individuals to obtain who ate at this restaurant and have never received the hepatitis A vaccine? A) The hepatitis A vaccine B) Albumin infusion C) The hepatitis A and B vaccines D) An immune globulin injection

D Feedback: For people who have not been previously vaccinated, hepatitis A can be prevented by the intramuscular administration of immune globulin during the incubation period, if given within 2 weeks of exposure. Administration of the hepatitis A vaccine will not protect the patient exposed to hepatitis A, as protection will take a few weeks to develop after the first dose of the vaccine. The hepatitis B vaccine provides protection again the hepatitis B virus, but plays no role in protection for the patient exposed to hepatitis A. Albumin confers no therapeutic benefit

A home health nurse is caring for a patient with multiple myeloma. Which of the following interventions should the nurse prioritize when addressing the patients severe bone pain? A) Implementing distraction techniques B) Educating the patient about the effective use of hot and cold packs C) Teaching the patient to use NSAIDs effectively D) Helping the patient manage the opioid analgesic regimen

D Feedback: For severe pain resulting from multiple myeloma, opioids are likely necessary. NSAIDs would likely be ineffective and are associated with significant adverse effects. Hot and cold packs as well as distraction would be insufficient for severe pain.

A medical nurse is aware of the need to screen specific patients for their risk of hyperglycemic hyperosmolar syndrome (HHS). In what patient population does hyperosmolar nonketotic syndrome most often occur? A) Patients who are obese and who have no known history of diabetes B) Patients with type 1 diabetes and poor dietary control C) Adolescents with type 2 diabetes and sporadic use of antihyperglycemics D) Middle-aged or older people with either type 2 diabetes or no known history of diabetes

D Feedback: HHS occurs most often in older people (50 to 70 years of age) who have no known history of diabetes or who have type 2 diabetes.

The nurse has implemented interventions aimed at facilitating family coping in the care of a patient with a traumatic brain injury. How can the nurse best facilitate family coping? A) Help the family understand that the patient could have died. B) Emphasize the importance of accepting the patients new limitations. C) Have the members of the family plan the patients inpatient care. D) Assist the family in setting appropriate short-term goals.

D Feedback: Helpful interventions to facilitate coping include providing family members with accurate and honest information and encouraging them to continue to set well-defined, short-term goals. Stating that patients condition could be worse downplays their concerns. Emphasizing the importance of acceptance may not necessarily help the family accept the patients condition. Family members cannot normally plan a patients hospital care, although they may contribute to the care in some ways

A patient with pheochromocytoma has been admitted for an adrenalectomy to be performed the following day. To prevent complications, the nurse should anticipate preoperative administration of which of the following? A) IV antibiotics B) Oral antihypertensives C) Parenteral nutrition D) IV corticosteroids

D Feedback: IV administration of corticosteroids (methylprednisolone sodium succinate [Solu-Medrol]) may begin on the evening before surgery and continue during the early postoperative period to prevent adrenal insufficiency. Antibiotics, antihypertensives, and parenteral nutrition do not prevent adrenal insufficiency or other common complications of adrenalectomy.

A radial graft is planned in the treatment of a patients oropharyngeal cancer. In order to ensure that the surgery will be successful, the care team must perform what assessment prior to surgery? A) Assessing function of cranial nerves V, VI, and IX B) Assessing for a history of GERD C) Assessing for signs or symptoms of atherosclerosis D) Assessing the patency of the ulnar artery

D Feedback: If a radial graft is to be performed, an Allen test on the donor arm must be performed to ensure that the ulnar artery is patent and can provide blood flow to the hand after removal of the radial artery. The success of this surgery is not primarily dependent on CN function or the absence of GERD and atherosclerosis.

The nurse is caring for a patient on telemetry. The patients ECG shows a shortened PR interval, slurring of the initial QRS deflection, and prolonged QRS duration. What does this ECG show? A) Sinus bradycardia B) Myocardial infarction C) Lupus-like syndrome D) Wolf-Parkinson-White (WPW) syndrome

D Feedback: In WPW syndrome there is a shortened PR interval, slurring (called a delta wave) of the initial QRS deflection, and prolonged QRS duration. These characteristics are not typical of the other listed cardiac anomalies.

A 13-year-old was brought to the ED, unconscious, after being hit in the head by a baseball. When the child regains consciousness, 5 hours after being admitted, he cannot remember the traumatic event. MRI shows no structural sign of injury. What injury would the nurse suspect the patient has? A) Diffuse axonal injury B) Grade 1 concussion with frontal lobe involvement C) Contusion D) Grade 3 concussion with temporal lobe involvement

D Feedback: In a grade 3 concussion there is a loss of consciousness lasting from seconds to minutes. Temporal lobe involvement results in amnesia. Frontal lobe involvement can cause uncharacteristic behavior and a grade 1 concussion does not involve loss of consciousness. Diagnostic studies may show no apparent structural sign of injury, but the duration of unconsciousness is an indicator of the severity of the concussion. Diffuse axonal injury (DAI) results from widespread shearing and rotational forces that produce damage throughout the brainto axons in the cerebral hemispheres, corpus callosum, and brain stem. In cerebral contusion, a moderate to severe head injury, the brain is bruised and damaged in a specific area because of severe acceleration-deceleration force or blunt trauma

A 35-year-old male is admitted to the hospital complaining of severe headaches, vomiting, and testicular pain. His blood work shows reduced numbers of platelets, leukocytes, and erythrocytes, with a high proportion of immature cells. The nurse caring for this patient suspects a diagnosis of what? A) AML B) CML C) MDS D) ALL

D Feedback: In acute lymphocytic leukemia (ALL), manifestations of leukemic cell infiltration into other organs are more common than with other forms of leukemia, and include pain from an enlarged liver or spleen, as well as bone pain. The central nervous system is frequently a site for leukemic cells; thus, patients may exhibit headache and vomiting because of meningeal involvement. Other extranodal sites include the testes and breasts. This particular presentation is not closely associated with acute myeloid leukemia (AML), chronic myeloid leukemia (CML), or myelodysplastic syndromes (MDS).

A patient presents to the emergency department (ED) complaining of severe right upper quadrant pain. The patient states that his family doctor told him he had gallstones. The ED nurse should recognize what possible complication of gallstones? A) Acute pancreatitis B) Atrophy of the gallbladder C) Gallbladder cancer D) Gangrene of the gallbladder

D Feedback: In calculous cholecystitis, a gallbladder stone obstructs bile outflow. Bile remaining in the gallbladder initiates a chemical reaction; autolysis and edema occur; and the blood vessels in the gallbladder are compressed, compromising its vascular supply. Gangrene of the gallbladder with perforation may result. Pancreatitis, atrophy, and cancer of the gallbladder are not plausible complications

A patient comes into the clinic complaining of fatigue. Blood work shows an increased bilirubin concentration and an increased reticulocyte count. What would the nurse suspect the patient has? A) A hypoproliferative anemia B) A leukemia C) Thrombocytopenia D) A hemolytic anemia

D Feedback: In hemolytic anemias, premature destruction of erythrocytes results in the liberation of hemoglobin from the erythrocytes into the plasma; the released hemoglobin is converted in large part to bilirubin, and therefore the bilirubin concentration rises. The increased erythrocyte destruction leads to tissue hypoxia, which in turn stimulates erythropoietin production. This increased production is reflected in an increased reticulocyte count as the bone marrow responds to the loss of erythrocytes. Hypoproliferative anemias, leukemia, and thrombocytopenia lack this pathology and presentation.

A pediatric nurse is emotionally distraught by the death of a 9-year-old girl who received care on the unit over the course of many admissions spanning several years. What action is the most appropriate response to the nurses own grief? A) Take time off from work to mourn the death. B) Post mementos of the patient on the unit. C) Solicit emotional support from the patients family. D) Attend the patients memorial service.

D Feedback: In many settings, staff members organize or attend memorial services to support families and other caregivers who find comfort in joining each other to remember and celebrate the lives of patients. Taking time off should not be necessary and posting mementos would be inappropriate. It would be highly inappropriate to solicit emotional support from the patients family during their time of loss.

A patient presents to the walk-in clinic complaining of intermittent chest pain on exertion, which is eventually attributed to angina. The nurse should inform the patient that angina is most often attributable to what cause? A) Decreased cardiac output B) Decreased cardiac contractility C) Infarction of the myocardium D) Coronary arteriosclerosis

D Feedback: In most cases, angina pectoris is due to arteriosclerosis. The disease is not a result of impaired cardiac output or contractility. Infarction may result from untreated angina, but it is not a cause of the disease.

A patient with liver cancer is being discharged home with a hepatic artery catheter in place. The nurse should be aware that this catheter will facilitate which of the following? A) Continuous monitoring for portal hypertension B) Administration of immunosuppressive drugs during the first weeks after transplantation C) Real-time monitoring of vascular changes in the hepatic system D) Delivery of a continuous chemotherapeutic dose

D Feedback: In most cases, the hepatic artery catheter has been inserted surgically and has a prefilled infusion pump that delivers a continuous chemotherapeutic dose until completed. The hepatic artery catheter does not monitor portal hypertension, deliver immunosuppressive drugs, or monitor vascular changes in the hepatic system.

A nurse is caring for a patient who is postoperative day 1 following neck dissection surgery. The nurse is performing an assessment of the patient and notes the presence of high-pitched adventitious sounds over the patients trachea on auscultation. The patients oxygen saturation is 90% by pulse oximetry with a respiratory rate of 31 breaths per minute. What is the nurses most appropriate action? A) Encourage the patient to perform deep breathing and coughing exercises hourly. B) Reposition the patient into a prone or semi-Fowlers position and apply supplementary oxygen by nasal cannula. C) Activate the emergency response system. D) Report this finding promptly to the physician and remain with the patient.

D Feedback: In the immediate postoperative period, the nurse assesses for stridor (coarse, high-pitched sound on inspiration) by listening frequently over the trachea with a stethoscope. This finding must be reported immediately because it indicates obstruction of the airway. The patients current status does not warrant activation of the emergency response system, and encouraging deep breathing and repositioning the patient are inadequate responses.

A patient has just been diagnosed with Parkinsons disease and the nurse is planning the patients subsequent care for the home setting. What nursing diagnosis should the nurse address when educating the patients family? A) Risk for infection B) Impaired spontaneous ventilation C) Unilateral neglect D) Risk for injury

D Feedback: Individuals with Parkinsons disease face a significant risk for injury related to the effects of dyskinesia. Unilateral neglect is not characteristic of the disease, which affects both sides of the body. Parkinsons disease does not directly constitute a risk for infection or impaired respiration

The medical nurse is aware that patients with sickle cell anemia benefit from understanding what situations can precipitate a sickle cell crisis. When teaching a patient with sickle cell anemia about strategies to prevent crises, what measures should the nurse recommend? A) Using prophylactic antibiotics and performing meticulous hygiene B) Maximizing physical activity and taking OTC iron supplements C) Limiting psychosocial stress and eating a high-protein diet D) Avoiding cold temperatures and ensuring sufficient hydration

D Feedback: Keeping warm and providing adequate hydration can be effective in diminishing the occurrence and severity of attacks. Hygiene, antibiotics, and high protein intake do not prevent crises. Maximizing activity may exacerbate pain and be unrealistic.

Level C personal protective equipment has been deemed necessary in the response to an unknown substance. The nurse is aware that the equipment will include what? A) A self-contained breathing apparatus B) A vapor-tight, chemical-resistant suit C) A uniform only D) An air-purified respirator

D Feedback: Level C incorporates the use of an air-purified respirator, a chemical resistant coverall with splash hood, chemical-resistant gloves, and boots. Level A provides the highest level of respiratory, mucous membrane, skin, and eye protection, incorporating a vapor-tight, chemical-resistant suit and self- contained breathing apparatus (SCBA). Level B personal protective equipment provides the highest level of respiratory protection, with a lesser level of skin and eye protection, incorporating a chemicalresistant suit and SCBA. Level D is the same as a work uniform.

Preoperative education is an important part of the nursing care of patients having coronary artery revascularization. When explaining the pre- and postoperative regimens, the nurse would be sure to include education about which subject? A) Symptoms of hypovolemia B) Symptoms of low blood pressure C) Complications requiring graft removal D) Intubation and mechanical ventilation

D Feedback: Most patients remain intubated and on mechanical ventilation for several hours after surgery. It is important that patients realize that this will prevent them from talking, and the nurse should reassure them that the staff will be able to assist them with other means of communication. Teaching would generally not include symptoms of low blood pressure or hypovolemia, as these are not applicable to most patients. Teaching would also generally not include rare complications that would require graft removal.

A patient was exposed to a dose of more than 5,000 rads of radiation during a terrorist attack. The patients skin will eventually show what manifestation? A) Erythema B) Ecchymosis C) Desquamation D) Necrosis

D Feedback: Necrosis of the skin will become evident within a few days to months at doses of more than 5,000 rads. With 600 to 1,000 rads, erythema will occur; it can disappear within hours and then reappear. At greater than 1,000 rads, desquamation (radiation dermatitis) of the skin will occur. Ecchymosis does not occur.

A patient, diagnosed with cancer of the lung, has just been told he has metastases to the brain. What change in health status would the nurse attribute to the patients metastatic brain disease? A) Chronic pain B) Respiratory distress C) Fixed pupils D) Personality changes

D Feedback: Neurologic signs and symptoms include headache, gait disturbances, visual impairment, personality changes, altered mentation (memory loss and confusion), focal weakness, paralysis, aphasia, and seizures. Pain, respiratory distress, and fixed pupils are not among the more common neurologic signs and symptoms of metastatic brain disease.

The nurse is admitting a 52-year-old father of four into hospice care. The patient has a diagnosis of Parkinsons disease, which is progressing rapidly. The patient has made clear his preference to receive care at home. What interventions should the nurse prioritize in the plan of care? A) Aggressively continuing to fight the disease process B) Moving the patient to a long-term care facility when it becomes necessary C) Including the children in planning their fathers care D) Supporting the patients and familys values and choices

D Feedback: Nurses need to develop skill and comfort in assessing patients and families responses to serious illness and planning interventions that support their values and choices throughout the continuum of care. To be admitted to hospice care, the patient must have come to terms with the fact that he is dying. The scenario states that the patient wants to be cared for at home, not in a long-term setting. The children may be able to participate in their fathers care, but they should not be assigned responsibility for planning

The nurse is caring for a patient with Huntington disease who has been admitted to the hospital for treatment of malnutrition. What independent nursing action should be implemented in the patients plan of care? A) Firmly redirect the patients head when feeding. B) Administer phenothiazines after each meal as ordered. C) Encourage the patient to keep his or her feeding area clean. D) Apply deep, gentle pressure around the patients mouth to aid swallowing.

D Feedback: Nursing interventions for a patient who has inadequate nutritional intake should include the following: Apply deep gentle pressure around the patients mouth to assist with swallowing, and administer phenothiazines prior to the patients meal as ordered. The nurse should disregard the mess of the feeding area and treat the person with dignity. Stiffness and turning away by the patient during feeding are uncontrollable choreiform movements and should not be interrupted.

A nurse is participating in the emergency care of a patient who has just developed variceal bleeding. What intervention should the nurse anticipate? A) Infusion of intravenous heparin B) IV administration of albumin C) STAT administration of vitamin K by the intramuscular route D) IV administration of octreotide (Sandostatin)

D Feedback: Octreotide (Sandostatin)a synthetic analog of the hormone somatostatinis effective in decreasing bleeding from esophageal varices, and lacks the vasoconstrictive effects of vasopressin. Because of this safety and efficacy profile, octreotide is considered the preferred treatment regimen for immediate control of variceal bleeding. Vitamin K and albumin are not administered and heparin would exacerbate, not alleviate, bleeding.

An occupational health nurse is called to the floor of a factory where a worker has sustained a flash burn to the right arm. The nurse arrives and the flames have been extinguished. The next step is to cool the burn. How should the nurse cool the burn? A) Apply ice to the site of the burn for 5 to 10 minutes. B) Wrap the patients affected extremity in ice until help arrives. C) Apply an oil-based substance or butter to the burned area until help arrives. D) Wrap cool towels around the affected extremity intermittently.

D Feedback: Once the burn has been sustained, the application of cool water is the best first-aid measure. Soaking the burn area intermittently in cool water or applying cool towels gives immediate and striking relief from pain, and limits local tissue edema and damage. However, never apply ice directly to the burn, never wrap the person in ice, and never use cold soaks or dressings for longer than several minutes; such procedures may worsen the tissue damage and lead to hypothermia in people with large burns. Butter is contraindicated.

The nurse is reviewing a newly admitted patients electronic health record, which notes a history of orthopnea? What nursing action is most clearly indicated? A) Teach the patient deep breathing and coughing exercises. B) Administer supplemental oxygen at all times. C) Limit the patients activity level. D) Avoid positioning the patient supine.

D Feedback: Orthopnea is defined as difficulty breathing while lying flat. This is a possible complication of HF and, consequently, the nurse should avoid positioning the patient supine. Oxygen supplementation may or may not be necessary and activity does not always need to be curtailed. Deep breathing and coughing exercises do not directly address this symptom.

The nurse performing the health interview of a patient with a new onset of periorbital edema has completed a genogram, noting the health history of the patients siblings, parents, and grandparents. This assessment addresses the patients risk of what kidney disorder? A) Nephritic syndrome B) Acute glomerulonephritis C) Nephrotic syndrome D) Polycystic kidney disease (PKD)

D Feedback: PKD is a genetic disorder characterized by the growth of numerous cysts in the kidneys. Nephritic syndrome, acute glomerulonephritis, and nephrotic syndrome are not genetic disorders.

The nurse is providing care to a patient who has just undergone an electrophysiologic (EP) study. The patient states that she is nervous about things going wrong during the procedure. What is the nurses best response? A) This is basically a risk-free procedure. B) Thousands of patients undergo EP every year. C) Remember that this is a step that will bring you closer to enjoying good health. D) The whole team will be monitoring you very closely for the entire procedure.

D Feedback: Patients who are to undergo an EP study may be anxious about the procedure and its outcome. A detailed discussion involving the patient, the family, and the electrophysiologist usually occurs to ensure that the patient can give informed consent and to reduce the patients anxiety about the procedure. It is inaccurate to state that EP is risk-free and stating that it is common does not necessarily relieve the patients anxiety. Characterizing EP as a step toward good health does not directly address the patients anxiety.

A nurse on the neurologic unit is providing care for a patient who has spinal cord injury at the level of C4. When planning the patients care, what aspect of the patients neurologic and functional status should the nurse consider? A) The patient will be unable to use a wheelchair. B) The patient will be unable to swallow food. C) The patient will be continent of urine, but incontinent of bowel. D) The patient will require full assistance for all aspects of elimination.

D Feedback: Patients with a lesion at C4 are fully dependent for elimination. The patient is dependent for feeding, but is able to swallow. The patient will be capable of using an electric wheelchair.

A nurse is planning discharge health education for a patient who will soon undergo placement of a mechanical valve prosthesis. What aspect of health education should the nurse prioritize in anticipation of discharge? A) The need for long-term antibiotics B) The need for 7 to 10 days of bed rest C) Strategies for preventing atherosclerosis D) Strategies for infection prevention

D Feedback: Patients with a mechanical valve prosthesis (including annuloplasty rings and other prosthetic materials used in valvuloplasty) require education to prevent infective endocarditis. Despite these infections risks, antibiotics are not used long term. Activity management is important, but extended bed rest is unnecessary. Valve replacement does not create a heightened risk for atherosclerosis.

A patient is being treated on the medical unit for a sickle cell crisis. The nurses most recent assessment reveals an oral temperature of 100.5F and a new onset of fine crackles on lung auscultation. What is the nurses most appropriate action? A) Apply supplementary oxygen by nasal cannula. B) Administer bronchodilators by nebulizer. C) Liaise with the respiratory therapist and consider high-flow oxygen. D) Inform the primary care provider that the patient may have an infection.

D Feedback: Patients with sickle cell disease are highly susceptible to infection,thus any early signs of infection should be reported promptly. There is no evidence of respiratory distress, so oxygen therapy and bronchodilators are not indicated.

A patient, 25 years of age, comes to the emergency department complaining of excessive bleeding from a cut sustained when cleaning a knife. Blood work shows a prolonged PT but a vitamin K deficiency is ruled out. When assessing the patient, areas of ecchymosis are noted on other areas of the body. Which of the following is the most plausible cause of the patients signs and symptoms? A) Lymphoma B) Leukemia C) Hemophilia D) Hepatic dysfunction

D Feedback: Prolongation of the PT, unless it is caused by vitamin K deficiency, may indicate severe hepatic dysfunction. The majority of hemophiliacs are diagnosed as children. The scenario does not describe signs or symptoms of lymphoma or leukemia.

A nurse is performing a home visit to a patient who is recovering following a long course of inpatient treatment for burn injuries. When performing this home visit, the nurse should do which of the following? A) Assess the patient for signs of electrolyte imbalances. B) Administer fluids as ordered. C) Assess the risk for injury recurrence. D) Assess the patients psychosocial state.

D Feedback: Recovery from burns can be psychologically challenging; the nurses assessments must address this reality. Fluid and electrolyte imbalances are infrequent during the rehabilitation phase of recovery. Burns are not typically a health problem that tends to recur; the experience of being burned tends to foster vigilance.

A group of nurses are participating in orientation to a telemetry unit. What should the staff educator tell this class about ST segments? A) They are the part of an ECG that reflects systole. B) They are the part of an ECG used to calculate ventricular rate and rhythm. C) They are the part of an ECG that reflects the time from ventricular depolarization through repolarization. D) They are the part of an ECG that represents early ventricular repolarization.

D Feedback: ST segment is the part of an ECG that reflects the end of the QRS complex to the beginning of the T wave. The part of an ECG that reflects repolarization of the ventricles is the T wave. The part of an ECG used to calculate ventricular rate and rhythm is the RR interval. The part of an ECG that reflects the time from ventricular depolarization through repolarization is the QT interval.

An intensive care nurse is aware of the need to identify patients who may be at risk of developing disseminated intravascular coagulation (DIC). Which of the following ICU patients most likely faces the highest risk of DIC? A) A patient with extensive burns B) A patient who has a diagnosis of acute respiratory distress syndrome C) A patient who suffered multiple trauma in a workplace accident D) A patient who is being treated for septic shock

D Feedback: Sepsis is a common cause of DIC. A wide variety of acute illnesses can precipitate DIC, but sepsis is specifically identified as a cause.

A patient has completed the full course of treatment for acute lymphocytic leukemia and has failed to respond appreciably. When preparing for the patients subsequent care, the nurse should perform what action? A) Arrange a meeting between the patients family and the hospital chaplain. B) Assess the factors underlying the patients failure to adhere to the treatment regimen. C) Encourage the patient to vigorously pursue complementary and alternative medicine (CAM). D) Identify the patients specific wishes around end-of-life care.

D Feedback: Should the patient not respond to therapy, it is important to identify and respect the patients choices about treatment, including measures to prolong life and other end-of-life measures. The patient may or may not be open to pursuing CAM. Unsuccessful treatment is not necessarily the result of failure to adhere to the treatment plan. Assessment should precede meetings with a chaplain, which may or may not be beneficial to the patient and congruent with the familys belief system.

The public health nurse is planning a health promotion campaign that reflects current epidemiologic trends. The nurse should know that hemorrhagic stroke currently accounts for what percentage of total strokes in the United States? A) 43% B) 33% C) 23% D) 13%

D Feedback: Strokes can be divided into two major categories: ischemic (87%), in which vascular occlusion and significant hypoperfusion occur, and hemorrhagic (13%), in which there is extravasation of blood into the brain or subarachnoid space.

When assessing patients who are victims of a chemical agent attack, the nurse is aware that assessment findings vary based on the type of chemical agent. The chemical sulfur mustard is an example of what type of chemical warfare agent? A) Nerve agent B) Blood agent C) Pulmonary agent D) Vesicant

D Feedback: Sulfur mustard is a vesicant chemical that causes blistering and results in burning, conjunctivitis, bronchitis, pneumonia, hematopoietic suppression, and death. Nerve agents include sarin, soman, tabun, VX, and organophosphates (pesticides). Hydrogen cyanide is a blood agent that has a direct effect on cellular metabolism, resulting in asphyxiation through alterations in hemoglobin. Chlorine is a pulmonary agent, which destroys the pulmonary membrane that separates the alveolus from the capillary bed.

A patient is brought to the ED by paramedics, who report that the patient has partial-thickness burns on the chest and legs. The patient has also suffered smoke inhalation. What is the priority in the care of patient who has been burned and suffered smoke inhalation? A) Pain B) Fluid balance C) Anxiety and fear D) Airway management

D Feedback: Systemic threats from a burn are the greatest threat to life. The ABCs of all trauma care apply during the early postburn period. While all options should be addressed, pain, fluid balance, and anxiety and fear do not take precedence over airway management.

When assessing the patient with pericardial effusion, the nurse will assess for pulsus paradoxus. Pulsus paradoxus is characterized by what assessment finding? A) A diastolic blood pressure that is lower during exhalation B) A diastolic blood pressure that is higher during inhalation C) A systolic blood pressure that is higher during exhalation D) A systolic blood pressure that is lower during inhalation

D Feedback: Systolic blood pressure that is markedly lower during inhalation is called pulsus paradoxus. The difference in systolic pressure between the point that is heard during exhalation and the point that is heard during inhalation is measured. Pulsus paradoxus exceeding 10 mm Hg is abnormal.

A 37-year-old man is brought to the clinic by his wife because he is experiencing loss of motor function and sensation. The physician suspects the patient has a spinal cord tumor and hospitalizes him for diagnostic testing. In light of the need to diagnose spinal cord compression from a tumor, the nurse will most likely prepare the patient for what test? A) Anterior-posterior x-ray B) Ultrasound C) Lumbar puncture D) MRI

D Feedback: The MRI scan is the most commonly used diagnostic procedure. It is the most sensitive diagnostic tool that is particularly helpful in detecting epidural spinal cord compression and vertebral bone metastases.

What should be included in the patients care plan when establishing an exercise program for a patient affected by a stroke? A) Schedule passive range of motion every other day. B) Keep activity limited, as the patient may be over stimulated. C) Have the patient perform active range-of-motion (ROM) exercises once a day. D) Exercise the affected extremities passively four or five times a day.

D Feedback: The affected extremities are exercised passively and put through a full ROM four or five times a day to maintain joint mobility, regain motor control, prevent development of a contracture in the paralyzed extremity, prevent further deterioration of the neuromuscular system, and enhance circulation. Active ROM exercises should ideally be performed more than once per day.

The nurse is providing care for a patient with high cholesterol and triglyceride values. In teaching the patient about therapeutic lifestyle changes such as diet and exercise, the nurse realizes that the desired goal for cholesterol levels is which of the following? A) High HDL values and high triglyceride values B) Absence of detectable total cholesterol levels C) Elevated blood lipids, fasting glucose less than 100 D) Low LDL values and high HDL values

D Feedback: The desired goal for cholesterol readings is for a patient to have low LDL and high HDL values. LDL exerts a harmful effect on the coronary vasculature because the small LDL particles can be easily transported into the vessel lining. In contrast, HDL promotes the use of total cholesterol by transporting LDL to the liver, where it is excreted. Elevated triglycerides are also a major risk factor for cardiovascular disease. A goal is also to keep triglyceride levels less than 150 mg/dL. All individuals possess detectable levels of total cholesterol.

A nurse is assessing an elderly patient with gallstones. The nurse is aware that the patient may not exhibit typical symptoms, and that particular symptoms that may be exhibited in the elderly patient may include what? A) Fever and pain B) Chills and jaundice C) Nausea and vomiting D) Signs and symptoms of septic shock

D Feedback: The elderly patient may not exhibit the typical symptoms of fever, pain, chills jaundice, and nausea and vomiting. Symptoms of biliary tract disease in the elderly may be accompanied or preceded by those of septic shock, which include oliguria, hypotension, change in mental status, tachycardia, and tachypnea.

A patient in the cardiac step-down unit has begun bleeding from the percutaneous coronary intervention (PCI) access site in her femoral region. What is the nurses most appropriate action? A) Call for assistance and initiate cardiopulmonary resuscitation. B) Reposition the patients leg in a nondependent position. C) Promptly remove the femoral sheath. D) Call for help and apply pressure to the access site.

D Feedback: The femoral sheath produces pressure on the access site. Pressure will temporarily reduce bleeding and allow for subsequent interventions. Removing the sheath would exacerbate bleeding and repositioning would not halt it. CPR is not indicated unless there is evidence of respiratory or cardiac arrest.

An OR nurse is preparing to assist with a coronary artery bypass graft (CABG). The OR nurse knows that the vessel most commonly used as source for a CABG is what? A) Brachial artery B) Brachial vein C) Femoral artery D) Greater saphenous vein

D Feedback: The greater saphenous vein is the most commonly used graft site for CABG. The right and left internal mammary arteries, radial arteries, and gastroepiploic artery are other graft sites used, though not as frequently. The femoral artery, brachial artery, and brachial vein are never harvested.

A patient has presented with signs and symptoms that are characteristic of acute kidney injury, but preliminary assessment reveals no obvious risk factors for this health problem. The nurse should recognize the need to interview the patient about what topic? A) Typical diet B) Allergy status C) Psychosocial stressors D) Current medication use

D Feedback: The kidneys are susceptible to the adverse effects of medications because they are repeatedly exposed to substances in the blood. Nephrotoxic medications are a more likely cause of AKI than diet, allergies, or stress.

A nurse is performing an admission assessment for an 81-year-old patient who generally enjoys good health. When considering normal, age-related changes to hepatic function, the nurse should anticipate what finding? A) Similar liver size and texture as in younger adults B) A nonpalpable liver C) A slightly enlarged liver with palpably hard edges D) A slightly decreased size of the liver

D Feedback: The most common age-related change in the liver is a decrease in size and weight. The liver is usually still palpable, however, and is not expected to have hardened edges.

An ED nurse has just received a call from EMS that they are transporting a 17-year-old man who has just sustained a spinal cord injury (SCI). The nurse recognizes that the most common cause of this type of injury is what? A) Sports-related injuries B) Acts of violence C) Injuries due to a fall D) Motor vehicle accidents

D Feedback: The most common causes of SCIs are motor vehicle crashes (46%), falls (22%), violence (16%), and sports (12%).

A patient who has recently recovered from a systemic viral infection is undergoing diagnostic testing for myocarditis. Which of the nurses assessment findings is most consistent with myocarditis? A) Sudden changes in level of consciousness (LOC) B) Peripheral edema and pulmonary edema C) Pleuritic chest pain D) Flulike symptoms

D Feedback: The most common symptoms of myocarditis are flulike. Chest pain, edema, and changes in LOC are not characteristic of myocarditis.

A patient is being treated on the acute medical unit for acute pancreatitis. The nurse has identified a diagnosis of Ineffective Breathing Pattern Related to Pain. What intervention should the nurse perform in order to best address this diagnosis? A) Position the patient supine to facilitate diaphragm movement. B) Administer corticosteroids by nebulizer as ordered. C) Perform oral suctioning as needed to remove secretions. D) Maintain the patient in a semi-Fowlers position whenever possible.

D Feedback: The nurse maintains the patient in a semi-Fowlers position to decrease pressure on the diaphragm by a distended abdomen and to increase respiratory expansion. A supine position will result in increased pressure on the diaphragm and potentially decreased respiratory expansion. Steroids and oral suctioning are not indicated.

A clinic nurse is providing patient education prior to a patients scheduled palliative radiotherapy to her spine. At the completion of the patient teaching, the patient continues to ask the same questions that the nurse has already addressed. What is the plausible conclusion that the nurse should draw from this? A) The patient is not listening effectively. B) The patient is noncompliant with the plan of care. C) The patient may have a low intelligence quotient or a cognitive deficit. D) The patient has not achieved the desired learning outcomes.

D Feedback: The nurse should be sensitive to patients ongoing needs and may need to repeat previously provided information or simply be present while the patient and family react emotionally. Telling a patient something is not teaching. If a patient continues to ask the same questions, teaching needs to be reinforced. The patients response is not necessarily suggestive of noncompliance, cognitive deficits, or not listening.

A patient with non-Hodgkins lymphoma is receiving information from the oncology nurse. The patient asks the nurse why she should stop drinking and smoking and stay out of the sun. What would be the nurses best response? A) Everyone should do these things because theyre health promotion activities that apply to everyone. B) You dont want to develop a second cancer, do you? C) You need to do this just to be on the safe side. D) Its important to reduce other factors that increase the risk of second cancers.

D Feedback: The nurse should encourage patients to reduce other factors that increase the risk of developing second cancers, such as use of tobacco and alcohol and exposure to environmental carcinogens and excessive sunlight. The other options do not answer the patients question, and also make light of the patients question.

A male patient with a metastatic brain tumor is having a generalized seizure and begins vomiting. What should the nurse do first? A) Perform oral suctioning. B) Page the physician. C) Insert a tongue depressor into the patients mouth. D) Turn the patient on his side.

D Feedback: The nurses first response should be to place the patient on his side to prevent him from aspirating emesis. Inserting something into the seizing patients mouth is no longer part of a seizure protocol. Obtaining supplies to suction the patient would be a delegated task. Paging or calling the physician would only be necessary if this is the patients first seizure

An older adult patient with type 2 diabetes is brought to the emergency department by his daughter. The patient is found to have a blood glucose level of 623 mg/dL. The patients daughter reports that the patient recently had a gastrointestinal virus and has been confused for the last 3 hours. The diagnosis of hyperglycemic hyperosmolar syndrome (HHS) is made. What nursing action would be a priority? A) Administration of antihypertensive medications B) Administering sodium bicarbonate intravenously C) Reversing acidosis by administering insulin D) Fluid and electrolyte replacement

D Feedback: The overall approach to HHS includes fluid replacement, correction of electrolyte imbalances, and insulin administration. Antihypertensive medications are not indicated, as hypotension generally accompanies HHS due to dehydration. Sodium bicarbonate is not administered to patients with HHS, as their plasma bicarbonate level is usually normal. Insulin administration plays a less important role in the treatment of HHS because it is not needed for reversal of acidosis, as in diabetic ketoacidosis (DKA).

The nurse is collaborating with the wound-ostomy-continence (WOC) nurse to teach a patient how to manage her new ileal conduit in the home setting. To prevent leakage or skin breakdown, the nurse should encourage which of the following practices? A) Empty the collection bag when it is between one-half and two-thirds full. B) Limit fluid intake to prevent production of large volumes of dilute urine. C) Reinforce the appliance with tape if small leaks are detected. D) Avoid using moisturizing soaps and body washes when cleaning the peristomal area.

D Feedback: The patient is instructed to avoid moisturizing soaps and body washes when cleaning the area because they interfere with the adhesion of the pouch. To maintain skin integrity, a skin barrier or leaking pouch is never patched with tape to prevent accumulation of urine under the skin barrier or faceplate. Fluids should be encouraged, not limited, and the collection bag should not be allowed to become more than one-third full.

After a subarachnoid hemorrhage, the patients laboratory results indicate a serum sodium level of less than 126 mEq/L. What is the nurses most appropriate action? A) Administer a bolus of normal saline as ordered. B) Prepare the patient for thrombolytic therapy as ordered. C) Facilitate testing for hypothalamic dysfunction. D) Prepare to administer 3% NaCl by IV as ordered.

D Feedback: The patient may be experiencing syndrome of inappropriate antidiuretic hormone (SIADH) or cerebral salt-wasting syndrome. The treatment most often is the use of IV hypertonic 3% saline. A normal saline bolus would exacerbate the problem and there is no indication for tests of hypothalamic function or thrombolytic therapy.

While performing a patients ordered wound care for the treatment of a burn, the patient has made a series of sarcastic remarks to the nurse and criticized her technique. How should the nurse best interpret this patients behavior? A) The patient may be experiencing an adverse drug reaction that is affecting his cognition and behavior. B) The patient may be experiencing neurologic or psychiatric complications of his injuries. C) The patient may be experiencing inconsistencies in the care that he is being provided. D) The patient may be experiencing anger about his circumstances that he is deflecting toward the nurse.

D Feedback: The patient may experience feelings of anger. The anger may be directed outward toward those who escaped unharmed or toward those who are now providing care. While drug reactions, complications, and frustrating inconsistencies in care cannot be automatically ruled out, it is not uncommon for anger to be directed at caregivers

A patient has been witness to a disaster involving a large number of injuries. The patient appears upset, but states that he feels capable of dealing with his emotions. What is the nurses most appropriate intervention? A) Educate the patient about the potential harm in denying his emotions. B) Refer the patient to social work or spiritual care. C) Encourage the patient to take a leave of absence from his job to facilitate emotional healing. D) Encourage the patient to return to normal social roles when appropriate.

D Feedback: The patient should be encouraged to return to normal social roles when appropriate if he is confident and genuine about his ability to cope. The nurse should use active listening to the patients concerns and emotions to enable the patient to process the situation. The patient is not necessarily being unrealistic or dishonest. As a result, social work or spiritual care may not be needed. Time away from work may not be required.

A nursing student is writing a care plan for a newly admitted patient who has been diagnosed with a stroke. What major nursing diagnosis should most likely be included in the patients plan of care? A) Adult failure to thrive B) Post-trauma syndrome C) Hyperthermia D) Disturbed sensory perception

D Feedback: The patient who has experienced a stroke is at a high risk for disturbed sensory perception. Stroke is associated with multiple other nursing diagnoses, but hyperthermia, adult failure to thrive, and posttrauma syndrome are not among these.

A patient is being treated for DIC and the nurse has prioritized the nursing diagnosis of Risk for Deficient Fluid Volume Related to Bleeding. How can the nurse best determine if goals of care relating to this diagnosis are being met? A) Assess for edema. B) Assess skin integrity frequently. C) Assess the patients level of consciousness frequently. D) Closely monitor intake and output.

D Feedback: The patient with DIC is at a high risk of deficient fluid volume. The nurse can best gauge the effectiveness of care by closely monitoring the patients intake and output. Each of the other assessments is a necessary element of care, but none addresses fluid balance as directly as close monitoring of intake and output.

A female patient is diagnosed with a right-sided stroke. The patient is now experiencing hemianopsia. How might the nurse help the patient manage her potential sensory and perceptional difficulties? A) Keep the lighting in the patients room low. B) Place the patients clock on the affected side. C) Approach the patient on the side where vision is impaired. D) Place the patients extremities where she can see them.

D Feedback: The patient with homonymous hemianopsia (loss of half of the visual field) turns away from the affected side of the body and tends to neglect that side and the space on that side; this is called amorphosynthesis. In such instances, the patient cannot see food on half of the tray, and only half of the room is visible. It is important for the nurse to remind the patient constantly of the other side of the body, to maintain alignment of the extremities, and if possible, to place the extremities where the patient can see them. Patients with a decreased field of vision should be approached on the side where visual perception is intact. All visual stimuli (clock, calendar, and television) should be placed on this side. The patient can be taught to turn the head in the direction of the defective visual field to compensate for this loss. Increasing the natural or artificial lighting in the room and providing eyeglasses are important in increasing vision. There is no reason to keep the lights dim.

A patient with a pulmonary embolism is being treated with a heparin infusion. What diagnostic finding suggests to the nurse that treatment is effective? A) The patients PT is within reference ranges. B) Arterial blood sampling tests positive for the presence of factor XIII. C) The patients platelet level is below 100,000/mm3 . D) The patients activated partial thromboplastin time (aPTT) is 1.5 to 2.5 times the control value.

D Feedback: The therapeutic effect of heparin is monitored by serial measurements of the aPTT; the dose is adjusted to maintain the range at 1.5 to 2.5 times the laboratory control. Heparin dosing is not determined on the basis of platelet levels, the presence or absence of clotting factors, or PT levels.

The nurse has been notified that the ED is expecting terrorist attack victims and that level D personal protective equipment is appropriate. What does level D PPE include? A) A chemical-resistant coverall with splash hood, chemical-resistant gloves, and boots B) A self-contained breathing apparatus (SCBA) and a fully encapsulating, vapor-tight, chemicalresistant suit with chemical-resistant gloves and boots. C) The SCBA and a chemical-resistant suit, but the suit is not vapor tight D) The nurses typical work uniform

D Feedback: The typical work uniform is appropriate for Level D protection

The nurse is teaching a patient diagnosed with aortic stenosis appropriate strategies for attempting to relieve the symptom of angina without drugs. What should the nurse teach the patient? A) To eat a small meal before taking nitroglycerin B) To drink a glass of milk before taking nitroglycerin C) To engage in 15 minutes of light exercise before taking nitroglycerin D) To rest and relax before taking nitroglycerin

D Feedback: The venous dilation that results from nitroglycerin decreases blood return to the heart, thus decreasing cardiac output and increasing the risk of syncope and decreased coronary artery blood flow. The nurse teaches the patient about the importance of attempting to relieve the symptoms of angina with rest and relaxation before taking nitroglycerin and to anticipate the potential adverse effects. Exercising, eating, and drinking are not recommended prior to using nitroglycerin.

A patient recovering from a stroke has severe shoulder pain from subluxation of the shoulder and is being cared for on the unit. To prevent further injury and pain, the nurse caring for this patient is aware of what principle of care? A) The patient should be fitted with a cast because use of a sling should be avoided due to adduction of the affected shoulder. B) Elevation of the arm and hand can lead to further complications associated with edema. C) Passively exercising the affected extremity is avoided in order to minimize pain. D) The patient should be taught to interlace fingers, place palms together, and slowly bring scapulae forward to avoid excessive force to shoulder.

D Feedback: To prevent shoulder pain, the nurse should never lift a patient by the flaccid shoulder or pull on the affected arm or shoulder. The patient is taught how to move and exercise the affected arm/shoulder through proper movement and positioning. The patient is instructed to interlace the fingers, place the palms together, and push the clasped hands slowly forward to bring the scapulae forward; he or she then raises both hands above the head. This is repeated throughout the day. The use of a properly worn sling when the patient is out of bed prevents the paralyzed upper extremity from dangling without support. Range-of-motion exercises are still vitally important in preventing a frozen shoulder and ultimately atrophy of subcutaneous tissues, which can cause more pain. Elevation of the arm and hand is also important in preventing dependent edema of the hand.

A patient with a head injury has been increasingly agitated and the nurse has consequently identified a risk for injury. What is the nurses best intervention for preventing injury? A) Restrain the patient as ordered. B) Administer opioids PRN as ordered. C) Arrange for friends and family members to sit with the patient. D) Pad the side rails of the patients bed.

D Feedback: To protect the patient from self-injury, the nurse uses padded side rails. The nurse should avoid restraints, because straining against them can increase ICP or cause other injury. Narcotics used to control restless patients should be avoided because these medications can depress respiration, constrict the pupils, and alter the patients responsiveness. Visitors should be limited if the patient is agitated.

The nurse is preparing to admit patients who have been the victim of a blast injury. The nurse should expect to treat a large number of patients who have experienced what type of injury? A) Chemical burns B) Spinal cord injury C) Meningeal tears D) Tympanic membrane rupture

D Feedback: Tympanic membrane (TM) rupture is the most frequent injury after subjection to a pressure wave resulting from a blast injury because the TM is the bodys most sensitive organ to pressure. In most cases, other injuries such as meningeal tears, spinal cord injury, and chemical injuries are likely to be less common.

A patient has been brought to the ED after suffering genitourinary trauma in an assault. Initial assessment reveals that the patients bladder is distended. What is the nurses most appropriate action? A) Withhold fluids from the patient. B) Perform intermittent urinary catheterization. C) Insert a narrow-gauge indwelling urinary catheter. D) Await orders following the urologists assessment.

D Feedback: Urethral catheter insertion when a possible urethral injury is present is contraindicated; a urology consultation and further evaluation of the urethra are required. The nurse would withhold fluids, but urologic assessment is the priority.

A nurse is amending a patients plan of care in light of the fact that the patient has recently developed ascites. What should the nurse include in this patients care plan? A) Mobilization with assistance at least 4 times daily B) Administration of beta-adrenergic blockers as ordered C) Vitamin B12 injections as ordered D) Administration of diuretics as ordered

D Feedback: Use of diuretics along with sodium restriction is successful in 90% of patients with ascites. Beta- blockers are not used to treat ascites and bed rest is often more beneficial than increased mobility. Vitamin B12 injections are not necessary.

The nurse is caring for patient who tells the nurse that he has an angina attack beginning. What is the nurses most appropriate initial action? A) Have the patient sit down and put his head between his knees. B) Have the patient perform pursed-lip breathing. C) Have the patient stand still and bend over at the waist. D) Place the patient on bed rest in a semi-Fowlers position.

D Feedback: When a patient experiences angina, the patient is directed to stop all activities and sit or rest in bed in a semi-Fowlers position to reduce the oxygen requirements of the ischemic myocardium. Pursed-lip breathing and standing will not reduce workload to the same extent. No need to have the patient put his head between his legs because cerebral perfusion is not lacking.

A patient is brought to the ER in an unconscious state. The physician notes that the patient is in need of emergency surgery. No family members are present, and the patient does not have identification. What action by the nurse is most important regarding consent for treatment? A) Ask the social worker to come and sign the consent. B) Contact the police to obtain the patients identity. C) Obtain a court order to treat the patient. D) Clearly document LOC and health status on the patients chart.

D Feedback: When patients are unconscious and in critical condition, the condition and situation should be documented to administer treatment quickly and timely when no consent can be obtained by usual routes. A social worker is not asked to sign the consent. Finding the patients identity is not a priority. Obtaining a court order would take too long.

An older adult patient with HF is being discharged home on an ACE inhibitor and a loop diuretic. The patients most recent vital signs prior to discharge include oxygen saturation of 93% on room air, heart rate of 81 beats per minute, and blood pressure of 94/59 mm Hg. When planning this patients subsequent care, what nursing diagnosis should be identified? A) Risk for ineffective tissue perfusion related to dysrhythmia B) Risk for fluid volume excess related to medication regimen C) Risk for ineffective breathing pattern related to hypoxia D) Risk for falls related to hypotension

D The combination of low BP, diuretic use, and ACE inhibitor use constitute a risk for falls. There is no evidence, or heightened risk, of dysrhythmia. The patients medications create a risk for fluid deficit, not fluid excess. Hypoxia is a risk for all patients with HF, but this is not in evidence for this patient at this time.

An ED nurse is assessing an adult woman for a suspected MI. When planning the assessment, the nurse should be cognizant of what signs and symptoms of MI that are particularly common in female patients? Select all that apply. A) Shortness of breath B) Chest pain C) Anxiety D) Numbness E) Weakness

D, E Feedback: Although these symptoms are not wholly absent in men, many women have been found to have atypical symptoms of MI, including indigestion, nausea, palpitations, and numbness. Shortness of breath, chest pain, and anxiety are common symptoms of MI among patients of all ages and genders.

A 73-year-old man comes into the emergency department (ED) by ambulance after slipping on a small carpet in his home. The patient fell on his hip with a resultant fracture. He is alert and oriented; his pupils are equal and reactive to light and accommodation. His heart rate is elevated, he is anxious and thirsty, a Foley catheter is placed, and 40 mL of urine is present. What is the nurses most likely explanation for the low urine output? A) The man urinated prior to his arrival to the ED and will probably not need to have the Foley catheter kept in place. B) The man likely has a traumatic brain injury, lacks antidiuretic hormone (ADH), and needs vasopressin. C) The man is experiencing symptoms of heart failure and is releasing atrial natriuretic peptide that results in decreased urine output. D) The man is having a sympathetic reaction, which has stimulated the reninangiotensinaldosterone system that results in diminished urine output.

D.

______ are false sensory perceptions not associated with real external stimuli and may involve any of the five senses

Hallucinations

____________________ personality disorder is characterized by colorful, dramatic, and extroverted behavior in excitable, emotional people.

Histrionic

____________________ is an alteration in mood that is expressed by feelings of elation, inflated self-esteem, grandiosity, hyperactivity, agitation, racing thoughts, and accelerated speech.

MANIA

____________________ are intrusive thoughts that are recurrent and stressful, and even though these thoughts are recognized by the individual as irrational, they continue to be repetitive and cannot be ignored.

Obsessions

The client diagnosed with a pituitary tumor has the pituitary hormone vasopressin (DDAVP) ordered. Which statement by the client indicates the medication is effective? 1. "My headaches are much better since I have been on this medication." 2. "My nasal drainage was initially worse, but now I don't have any." 3. "I am not so thirsty when I take this medication." 4. "My seizures have been eliminated."

Vasopressin is the antidiuretic hormone produced by the pituitary gland that is instrumental in the body's ability to conserve water. It does not affect headaches or any other type of pain. Diabetes insipidus is caused by a lack of vasopressin. 2. DDAVP is given intranasally, and the nurse should be alert to symptoms of rhinitis, but lack of nasal drainage does not indicate that the medication is effective. 3. DDAVP is a synthetic form of the antidiuretic hormone vasopressin. Without vasopressin, the body does not conserve water and a large amount of very dilute urine is excreted. The body will attempt to have the client replace the fluid by producing the symptom of extreme thirst. Lack of thirst indicates the medication is effective. 4. DDAVP will not affect seizure activity.

A nurse is assessing a client with left-sided heart failure. For which clinical manifestations should the nurse assess? (Select all that apply.) a. pulmonary crackles b. confusion, restlessness c. pulmonary hypertension d. dependent edema e. cough that worsens at night

a. Pulmonary crackles b. Confusion, restlessness e. Cough that worsens at night

A patient with liver disease has developed jaundice; the nurse is collaborating with the patient to develop a nutritional plan. The nurse should prioritize which of the following in the patients plan? A) Increased potassium intake B) Fluid restriction to 2 L per day C) Reduction in sodium intake D) High-protein, low-fat diet

c Patients with ascites require a sharp reduction in sodium intake. Potassium intake should not be correspondingly increased. There is no need for fluid restriction or increased protein intake.

The DSM-5 diagnosis of functional neurological symptom disorder can also be identified as ___________________ disorder.

conversion

A patient has been diagnosed with pancreatic cancer and has been admitted for care. Following initial treatment, the nurse should be aware that the patient is most likely to require which of the following? A) Inpatient rehabilitation B) Rehabilitation in the home setting C) Intensive physical therapy D) Hospice care

d Feedback: Pancreatic carcinoma has only a 5% survival rate at 5 years regardless of the stage of disease at diagnosis or treatment. As a result, there is a higher likelihood that the patient will require hospice care than physical therapy and rehabilitation.

A home health nurse is caring for a patient discharged home after pancreatic surgery. The nurse documents the nursing diagnosis Risk for Imbalanced Nutrition: Less than Body Requirements on the care plan based on the potential complications that may occur after surgery. What are the most likely complications for the patient who has had pancreatic surgery? A) Proteinuria and hyperkalemia B) Hemorrhage and hypercalcemia C) Weight loss and hypoglycemia D) Malabsorption and hyperglycemia

d The nurse arrives at this diagnosis based on the complications of malabsorption and hyperglycemia. These complications often lead to the need for dietary modifications. Pancreatic enzyme replacement, a low-fat diet, and vitamin supplementation often are also required to meet the patients nutritional needs and restrictions. Electrolyte imbalances often accompany pancreatic disorders and surgery, but the electrolyte levels are more often deficient than excessive. Hemorrhage is a complication related to surgery, but not specific to the nutritionally based nursing diagnosis. Weight loss is a common complication, but hypoglycemia is less likely.

A client's electrocardiogram reveals an irregular rhythm of 75 bpm with a normal QRS and P wave. The nurse who is caring for the client should anticipate: a. administration of epinephrine. b. a bolus of warmed normal saline. c. administration of a beta-adrenergic blocker .d. no immediate treatment.

d This client's ECG suggests sinus dysrhythmia. Sinus dysrhythmia does not cause any significant hemodynamic effect and therefore is not typically treated.

A nurse assesses a clients ECG and observes that not all QRS complexes are preceded by a P wave. How should the nurse interpret this observation? a. the client has hyperkalemia causing irregular QRS complexes b. Ventricular tachycardia is overriding the normal atrial rhythm c. the clients chest leads are not making sufficient contact with the skin d. Ventricular and atrial depolarizations are initiated from different sites

d. Ventricular and atrial depolarizations are initiated from different sites

In an acute care setting, the nurse is assessing an unstable patient. When prioritizing the patients care, the nurse should recognize that the patient is at risk for hypovolemic shock in which of the following circumstances? A) Fluid volume circulating in the blood vessels decreases. B) There is an uncontrolled increase in cardiac output. C) Blood pressure regulation becomes irregular. D) The patient experiences tachycardia and a bounding pulse.

ns: A Feedback: Hypovolemic shock is characterized by a decrease in intravascular volume. Cardiac output is decreased, blood pressure decreases, and pulse is fast, but weak.

The critical care nurse is caring for a patient with a pulmonary artery pressure monitoring system. The nurse is aware that pulmonary artery pressure monitoring is used to assess left ventricular function. What is an additional function of pulmonary artery pressure monitoring systems? A) To assess the patients response to fluid and drug administration B) To obtain specimens for arterial blood gas measurements C) To dislodge pulmonary emboli D) To diagnose the etiology of chronic obstructive pulmonary disease

ns: A Feedback: Pulmonary artery pressure monitoring is an important tool used in critical care for assessing left ventricular function (cardiac output), diagnosing the etiology of shock, and evaluating the patients response to medical interventions, such as fluid administration and vasoactive medications. Pulmonary artery monitoring is preferred for the patient with heart failure over central venous pressure monitoring. Arterial catheters are useful when arterial blood gas measurements and blood samples need to be obtained frequently. Neither intervention is used to clear pulmonary emboli.


Set pelajaran terkait

ENG101 - Module 1 - The Writing Process

View Set

Lewis: MED-SURG: Chapter 25: Burns

View Set